Vous êtes sur la page 1sur 311

PY

Precalculus

C
O

Teachers Guide

EP
E

This Teachers Guide was developed by the Commission


on Higher Education (CHED) in collaboration with content and
teaching experts from universities, colleges, basic education,
and other academic institutions and was donated to the
Department of Education (DepEd) for use in senior high school
in support of the implementation of the enhanced basic
education curriculum (K to 12).

We encourage teachers and other education stakeholders


to email their feedback, comments, and recommendations to the
Department of Education at action@deped.gov.ph.
We value your feedback and recommendations.

Department of Education
Republic of the Philippines

All rights reserved. No part of this material may be reproduced or transmitted in any form or by any means electronic or mechanical including photocopying without written permission from the DepEd Central Office. First Edition, 2016.

Precalculus
Teachers Guide
First Edition 2016
Republic Act 8293. Section 176 states that: No copyright shall subsist in any work
of the Government of the Philippines. However, prior approval of the government agency or
office wherein the work is created shall be necessary for exploitation of such work for profit.
Such agency or office may, among other things, impose as a condition the payment of
royalties.

PY

Borrowed materials (i.e., songs, stories, poems, pictures, photos, brand names,
trademarks, etc.) included in this learning resource are owned by their respective copyright
holders. DepEd is represented by the Filipinas Copyright Licensing Society (FILCOLS), Inc.
in seeking permission to use these materials from their respective copyright owners. All
means have been exhausted in seeking permission to use these materials. The publisher
and authors do not represent nor claim ownership over them.

C
O

Only institutions and companies which have entered an agreement with FILCOLS
and only within the agreed framework may copy from this Teachers Guide. Those who
have not entered in an agreement with FILCOLS must, if they wish to copy, contact the
publishers and authors directly.
Authors and publishers may email or contact FILCOLS at filcols@gmail.com or (02)
435-5258, respectively.

Published by the Department of Education


Secretary: Br. Armin A. Luistro FSC
Undersecretary: Dina S. Ocampo, PhD

Development Team of the Precalculus Teachers Guide


Jerico B. Bacani, Ph.D.
Richard B. Eden, Ph.D.
Glenn Rey A. Estrada
Flordeliza F. Francisco, Ph.D.
Mark Anthony J. Vidallo
Technical Reviewers:
Maria Alva Q. Aberin, Ph.D.
Flordeliza F. Francisco, Ph.D.
Reginaldo M. Marcelo, Ph.D.
Language / Pedagogy Reviewers: Carly Mae Casteloy
Angela Dianne Agustin
Cover Art Illustrator:
Quincy D. Gonzales
Team Leader:
Ian June L. Garces, Ph.D.

EP
E

Writers:

Printed in the Philippines by Sunshine Interlinks Publishing House, Inc.


3F Maine City Tower, 236 Tomas Morato Avenue,
Brgy. South Triangle, Quezon City
Department of Education-Bureau of Learning Resources (DepEd-BLR)
Office Address: Ground Floor Bonifacio Building, DepEd Complex
Meralco Avenue, Pasig City, Philippines 1600
Telefax:
(02) 634-1054 or 634-1072
E-mail Address: blr.lrqad@deped.gov.ph / blr.lrpd@deped.gov.ph
Copyright of this donated manuscript is transferred to DepEd, including the right to
modify and determine the use of its content. Modifications made to the original manuscript were
the direct result of DepEd's quality assurance process, including compliance with social content
guidelines, and as such are the sole responsibility of the DepEd.

All rights reserved. No part of this material may be reproduced or transmitted in any form or by any means electronic or mechanical including photocopying without written permission from the DepEd Central Office. First Edition, 2016.

Table of Contents
1

DepEd Curriculum Guide for Precalculus

Unit 1:

PY

About This Teaching Guide

Analytic Geometry (19 one-hour sessions)

6
7

1.1.1: An Overview of Conic Sections . . . . . . . . . . . . . . . . . . . . . . . . . . .

C
O

Lesson 1.1: Introduction to Conic Sections and Circles . . . . . . . .


1.1.2: Definition and Equation of a Circle . . . . . . . . . . . . . . . . . . . . . . .

1.1.3: More Properties of Circles . . . . . . . . . . . . . . . . . . . . . . . . . . . . . . . . 11


1.1.4: Situational Problems Involving Circles. . . . . . . . . . . . . . . . . . . . 13

Lesson 1.2: Parabolas . . . . . . . . . . . . . . . . . . . . . . . . . . . . . . . . . . . . . . . . . 20


1.2.1: Definition and Equation of a Parabola . . . . . . . . . . . . . . . . . . . . 21

EP
E

1.2.2: More Properties of Parabolas . . . . . . . . . . . . . . . . . . . . . . . . . . . . . 24


1.2.3: Situational Problems Involving Parabolas . . . . . . . . . . . . . . . . 29

Lesson 1.3: Ellipses . . . . . . . . . . . . . . . . . . . . . . . . . . . . . . . . . . . . . . . . . . . 33


1.3.1: Definition and Equation of an Ellipse . . . . . . . . . . . . . . . . . . . . . 33
1.3.2: More Properties of Ellipses . . . . . . . . . . . . . . . . . . . . . . . . . . . . . . . 37

1.3.3: Situational Problems Involving Ellipses . . . . . . . . . . . . . . . . . . . 41

Lesson 1.4: Hyperbolas . . . . . . . . . . . . . . . . . . . . . . . . . . . . . . . . . . . . . . . 46


1.4.1: Definition and Equation of a Hyperbola . . . . . . . . . . . . . . . . . . 47
1.4.2: More Properties of Hyperbolas . . . . . . . . . . . . . . . . . . . . . . . . . . . 51
1.4.3: Situational Problems Involving Hyperbolas . . . . . . . . . . . . . . . 55
Lesson 1.5: More Problems on Conic Sections . . . . . . . . . . . . . . . . 60
1.5.1: Identifying the Conic Section by Inspection . . . . . . . . . . . . . . . 60
1.5.2: Problems Involving Different Conic Sections . . . . . . . . . . . . . . 62

iii
All rights reserved. No part of this material may be reproduced or transmitted in any form or by any means electronic or mechanical including photocopying without written permission from the DepEd Central Office. First Edition, 2016.

Lesson 1.6: Systems of Nonlinear Equations . . . . . . . . . . . . . . . . . . 67


1.6.1: Review of Techniques in Solving Systems of Linear
Equations . . . . . . . . . . . . . . . . . . . . . . . . . . . . . . . . . . . . . . . . . . . . . . . . 67
1.6.2: Solving Systems of Equations Using Substitution . . . . . . . . . 69
1.6.3: Solving Systems of Equations Using Elimination . . . . . . . . . . 72
1.6.4: Applications of Systems of Nonlinear Equations . . . . . . . . . . 76
Unit 2:

Mathematical Induction (10 one-hour sessions)

81

PY

Lesson 2.1: Review of Sequences and Series . . . . . . . . . . . . . . . . . . . 82


Lesson 2.2: Sigma Notation . . . . . . . . . . . . . . . . . . . . . . . . . . . . . . . . . . . 87
2.2.1: Writing and Evaluating Sums in Sigma Notation . . . . . . . . . 87

C
O

2.2.2: Properties of Sigma Notation . . . . . . . . . . . . . . . . . . . . . . . . . . . . . 90


Lesson 2.3: Principle of Mathematical Induction . . . . . . . . . . . . . . 96
2.3.1: Proving Summation Identities . . . . . . . . . . . . . . . . . . . . . . . . . . . . 97
2.3.2: Proving Divisibility Statements. . . . . . . . . . . . . . . . . . . . . . . . . . . 102

? 2.3.3: Proving Inequalities . . . . . . . . . . . . . . . . . . . . . . . . . . . . . . . . . . . . . . 104


Lesson 2.4: The Binomial Theorem . . . . . . . . . . . . . . . . . . . . . . . . . . . 108

EP
E

2.4.1: Pascals Triangle and the Concept of Combination . . . . . . . . 109


2.4.2: The Binomial Theorem . . . . . . . . . . . . . . . . . . . . . . . . . . . . . . . . . . . 112
2.4.3: Terms of a Binomial Expansion . . . . . . . . . . . . . . . . . . . . . . . . . . 115

? 2.4.4: Approximation and Combination Identities . . . . . . . . . . . . . . . 117


Trigonometry (29 one-hour sessions)

122

Unit 3:

Lesson 3.1: Angles in a Unit Circle . . . . . . . . . . . . . . . . . . . . . . . . . . . 123


3.1.1: Angle Measure . . . . . . . . . . . . . . . . . . . . . . . . . . . . . . . . . . . . . . . . . . . 123
3.1.2: Coterminal Angles . . . . . . . . . . . . . . . . . . . . . . . . . . . . . . . . . . . . . . . 127
3.1.3: Arc Length and Area of a Sector . . . . . . . . . . . . . . . . . . . . . . . . . 129
Lesson 3.2: Circular Functions . . . . . . . . . . . . . . . . . . . . . . . . . . . . . . . . 135
3.2.1: Circular Functions on Real Numbers . . . . . . . . . . . . . . . . . . . . . 136
3.2.2: Reference Angle . . . . . . . . . . . . . . . . . . . . . . . . . . . . . . . . . . . . . . . . . . 140

All rights reserved. No part of this material may be reproduced or transmitted in any form or by any means electronic or mechanical including photocopying without written permission from the DepEd Central Office. First Edition, 2016.

Lesson 3.3: Graphs of Circular Functions and Situational


Problems . . . . . . . . . . . . . . . . . . . . . . . . . . . . . . . . . . . . . . . . . 144
3.3.1: Graphs of y = sin x and y = cos x . . . . . . . . . . . . . . . . . . . . . . . . 145
3.3.2: Graphs of y = a sin bx and y = a cos bx . . . . . . . . . . . . . . . . . . . 147
3.3.3: Graphs of y = a sin b(x c) + d and
y = a cos b(x c) + d . . . . . . . . . . . . . . . . . . . . . . . . . . . . . . . . . . . . . 151
3.3.4: Graphs of Cosecant and Secant Functions . . . . . . . . . . . . . . . . 156
3.3.5: Graphs of Tangent and Cotangent Functions . . . . . . . . . . . . . 160

PY

3.3.6: Simple Harmonic Motion . . . . . . . . . . . . . . . . . . . . . . . . . . . . . . . . . 163


Lesson 3.4: Fundamental Trigonometric Identities . . . . . . . . . . . . . 177
3.4.1: Domain of an Expression or Equation . . . . . . . . . . . . . . . . . . . . 177

C
O

3.4.2: Identity and Conditional Equation . . . . . . . . . . . . . . . . . . . . . . . 179


3.4.3: The Fundamental Trigonometric Identities . . . . . . . . . . . . . . . 181
3.4.4: Proving Trigonometric Identities . . . . . . . . . . . . . . . . . . . . . . . . . 184
Lesson 3.5: Sum and Difference Identities . . . . . . . . . . . . . . . . . . . . . 190
3.5.1: The Cosine Difference and Sum Identities . . . . . . . . . . . . . . . . 190

3.5.2: The Cofunction Identities and the Sine Sum and


Difference Identities . . . . . . . . . . . . . . . . . . . . . . . . . . . . . . . . . . . . . . 193

EP
E

3.5.3: The Tangent Sum and Difference Identities . . . . . . . . . . . . . . . 197

Lesson 3.6: Double-Angle and Half-Angle Identities . . . . . . . . . . . 205


3.6.1: Double-Angle Identities . . . . . . . . . . . . . . . . . . . . . . . . . . . . . . . . . . 206
3.6.2: Half-Angle Identities . . . . . . . . . . . . . . . . . . . . . . . . . . . . . . . . . . . . . 209

Lesson 3.7: Inverse Trigonometric Functions . . . . . . . . . . . . . . . . . . 216


3.7.1: Inverse Sine Function . . . . . . . . . . . . . . . . . . . . . . . . . . . . . . . . . . . . 217
3.7.2: Inverse Cosine Function . . . . . . . . . . . . . . . . . . . . . . . . . . . . . . . . . . 222
3.7.3: Inverse Tangent Function and the Remaining Inverse
Trigonometric Functions . . . . . . . . . . . . . . . . . . . . . . . . . . . . . . . . . 226
Lesson 3.8: Trigonometric Equations . . . . . . . . . . . . . . . . . . . . . . . . . . 239
3.8.1: Solutions of a Trigonometric Equation . . . . . . . . . . . . . . . . . . . . 240
3.8.2: Equations with One Term . . . . . . . . . . . . . . . . . . . . . . . . . . . . . . . . 244
3.8.3: Equations with Two or More Terms . . . . . . . . . . . . . . . . . . . . . . 247

All rights reserved. No part of this material may be reproduced or transmitted in any form or by any means electronic or mechanical including photocopying without written permission from the DepEd Central Office. First Edition, 2016.

Lesson 3.9: Polar Coordinate System . . . . . . . . . . . . . . . . . . . . . . . . . 257


3.9.1: Polar Coordinates of a Point . . . . . . . . . . . . . . . . . . . . . . . . . . . . . 258
3.9.2: From Polar to Rectangular, and Vice Versa . . . . . . . . . . . . . . . 263
3.9.3: Basic Polar Graphs and Applications . . . . . . . . . . . . . . . . . . . . . 266
278

References

305

EP
E

C
O

PY

Answers to All LM Exercises

All rights reserved. No part of this material may be reproduced or transmitted in any form or by any means electronic or mechanical including photocopying without written permission from the DepEd Central Office. First Edition, 2016.

About This Teaching Guide

The Precalculus course bridges basic mathematics and calculus. This course
completes the foundational knowledge on algebra, geometry, and trigonometry of
students who are planning to take courses in the STEM track. It provides them
with conceptual understanding and computational skills that are prerequisites
for Basic Calculus and future STEM courses.

PY

Based on the Curriculum Guide for Precalculus of the Department of Education (see pages 2-5), the primary aim of this Teaching Guide is to give math
teachers an adequate stand-alone material that can be used for each session of
the Grade 11 Precalculus course.

C
O

The Guide is divided into three units: analytic geometry, summation notation
and mathematical induction, and trigonometry. Each unit is composed of lessons
that bring together related learning competencies in the unit. Each lesson is
further divided into sub-lessons that focus on one or two competencies for effective
teaching and learning. Each sub-lesson is designed for a one-hour session, but the
teachers have the option to extend the time allotment to one-and-a-half hours for
some sub-lessons.

EP
E

Each sub-lesson ends with a Seatwork/Homework, which consists of exercises


related to the topic being discussed in the sub-lesson. As the title suggests, these
exercises can be done in school (if time permits) or at home. Moreover, at the
end of each lesson is a set of exercises (simply tagged as Exercises) that can be
used for short quizzes and long exams. Answers, solutions, or hints to most items
in Seatwork/Homework and Exercises are provided to guide the teachers as they
solve them.

Some items are marked with a star. A starred sub-lesson means the discussion
and accomplishment of the sub-lesson are optional. This will be decided by your
teacher. On the other hand, a starred example or exercise means the use of
calculator is required.
To further guide the teachers, Teaching Notes are provided on the margins.
These notes include simple recall of basic definitions and theorems, suggested
teaching methods, alternative answers to some exercises, quick approaches and
techniques in solving particular problems, and common errors committed by students.
We hope that Precalculus teachers will find this Teaching Guide helpful and
convenient to use. We encourage the teachers to study this Guide carefully and
solve the exercises themselves. Although great effort has been put into this Guide
for technical correctness and precision, any mistake found and reported to the
Team is a gain for other teachers. Thank you for your cooperation.
The Precalculus Team

All rights reserved. No part of this material may be reproduced or transmitted in any form or by any means electronic or mechanical including photocopying without written permission from the DepEd Central Office. First Edition, 2016.

All rights reserved. No part of this material may be reproduced or transmitted in any form or by any means electronic or mechanical including photocopying without written permission from the DepEd Central Office. First Edition, 2016.
Semester: First Semester
No. of Hours/ Semester: 80 hours/ semester
Pre-requisite (if needed):

key concepts of
conic sections and
systems of
nonlinear
equations

The learners
demonstrate an
understanding
of...

CONTENT
STANDARDS

model situations
appropriately and solve
problems accurately using
conic sections and systems
of nonlinear equations

The learners shall be able


to...

PERFORMANCE
STANDARDS

D
LEARNING COMPETENCIES

5.
6.
7.
8.
9.
10.
11.
12.

PY

graph a circle in a rectangular coordinate system

4.

define a parabola
determine the standard form of equation of a parabola
graph a parabola in a rectangular coordinate system
define an ellipse
determine the standard form of equation of an ellipse
graph an ellipse in a rectangular coordinate system
define a hyperbola
determine the standard form of equation of a hyperbola

determine the standard form of equation of a circle

3.

2.

illustrate the different types of conic sections: parabola, ellipse,


circle, hyperbola, and degenerate cases.***
define a circle.

1.

C
O

The learners...

K to 12 Senior High School STEM Specialized Subject Pre-Calculus December 2013

Analytic
Geometry

CONTENT

Page 1 of 4

STEM_PC11AG-Ia-5
STEM_PC11AG-Ib-1
STEM_PC11AG-Ib-2
STEM_PC11AG-Ic-1
STEM_PC11AG-Ic-2
STEM_PC11AG-Ic-3
STEM_PC11AG-Id-1
STEM_PC11AG-Id-2

STEM_PC11AG-Ia-4

STEM_PC11AG-Ia-3

STEM_PC11AG-Ia-2

STEM_PC11AG-Ia-1

CODE

Subject Description: At the end of the course, the students must be able to apply concepts and solve problems involving conic sections, systems of nonlinear equations,
series and mathematical induction, circular and trigonometric functions, trigonometric identities, and polar coordinate system.

EP
E

Grade: 11
Core Subject Title: Pre-Calculus

K to 12 BASIC EDUCATION CURRICULUM


SENIOR HIGH SCHOOL SCIENCE, TECHNOLOGY, ENGINEERING AND MATHEMATICS (STEM) SPECIALIZED SUBJECT

All rights reserved. No part of this material may be reproduced or transmitted in any form or by any means electronic or mechanical including photocopying without written permission from the DepEd Central Office. First Edition, 2016.

key concepts of
series and
mathematical
induction and the
Binomial
Theorem.

keenly observe and


investigate patterns, and
formulate appropriate
mathematical statements
and prove them using
mathematical induction
and/or Binomial Theorem.

PERFORMANCE
STANDARDS

EP
E

CONTENT
STANDARDS

recognize the equation and important characteristics of the


different types of conic sections
solves situational problems involving conic sections

14.
15.

PY

differentiate a series from a sequence


use the sigma notation to represent a series
illustrate the Principle of Mathematical Induction
apply mathematical induction in proving identities
illustrate Pascals Triangle in the expansion of + for small
positive integral values of
prove the Binomial Theorem
determine any term of + , where is a positive integer,
without expanding
solve problems using mathematical induction and the Binomial
Theorem

2.
3.
4.
5.
6.

9.

7.
8.

illustrate a series

1.

C
O

determine the solutions of systems of nonlinear equations using


techniques such as substitution, elimination, and graphing***
solve situational problems involving systems
of nonlinear equations

17.

18.

illustrate systems of nonlinear equations

16.

graph a hyperbola in a rectangular coordinate system

LEARNING COMPETENCIES
13.

K to 12 Senior High School STEM Specialized Subject Pre-Calculus December 2013

Series and
Mathematical
Induction

CONTENT

STEM_PC11SMI-Ih-1

STEM_PC11AG-Ig-2

STEM_PC11AG-If-g-1

STEM_PC11AG-If-1

STEM_PC11AG-Ie-2

STEM_PC11AG-Ie-1

STEM_PC11AG-Id-3

CODE

Page 2 of 4

STEM_PC11SMI-Ij-2

STEM_PC11SMI-Ij-1

STEM_PC11SMI-Ii-3

STEM_PC11SMI-Ii-2

STEM_PC11SMI-Ih-2
STEM_PC11SMI-Ih-3
STEM_PC11SMI-Ih-4
STEM_PC11SMI-Ih-i-1

K to 12 BASIC EDUCATION CURRICULUM


SENIOR HIGH SCHOOL SCIENCE, TECHNOLOGY, ENGINEERING AND MATHEMATICS (STEM) SPECIALIZED SUBJECT

All rights reserved. No part of this material may be reproduced or transmitted in any form or by any means electronic or mechanical including photocopying without written permission from the DepEd Central Office. First Edition, 2016.
3. formulate and solve
accurately situational
problems involving
appropriate trigonometric
functions
4. formulate and solve
accurately situational
problems involving the
polar coordinate system

uses reference angles to find exact values of circular functions


determine the domain and range of the different circular functions
graph the six circular functions (a) amplitude, (b) period, and (c)
phase shift
solve problems involving circular functions
determine whether an equation is an identity or a conditional
equation
derive the fundamental trigonometric identities
derive trigonometric identities involving sum and difference of
angles
derive the double and half-angle formulas
simplify trigonometric expressions
prove other trigonometric identities
solve situational problems involving trigonometric identities
illustrate the domain and range of the inverse trigonometric
functions.
evaluate an inverse trigonometric expression.
solve trigonometric equations.
solve situational problems involving inverse trigonometric
functions and trigonometric equations
locate points in polar coordinate system
convert the coordinates of a point from rectangular to polar
systems and vice versa
solve situational problems involving polar coordinate system

4.
5.
6.
7.

22.

20.
21.

17.
18.
19.

12.
13.
14.
15.
16.

10.
11.

8.
9.

K to 12 Senior High School STEM Specialized Subject Pre-Calculus December 2013

PY

illustrate the different circular functions

3.

C
O

illustrate angles in standard position and coterminal angles

2.

LEARNING COMPETENCIES
illustrate the unit circle and the relationship between the linear
and angular measures of a central angle in a unit circle
convert degree measure to radian measure and vice versa

1.

2. apply appropriate
trigonometric identities in
solving situational
problems

PERFORMANCE
STANDARDS
1. formulate and solve
accurately situational
problems involving
circular functions

EP
E

CONTENT
STANDARDS
key concepts of
circular functions,
trigonometric
identities, inverse
trigonometric
functions, and
the polar
coordinate
system

***Suggestion for ICT-enhanced lesson when available and where appropriate

Trigonometry

CONTENT

K to 12 BASIC EDUCATION CURRICULUM


SENIOR HIGH SCHOOL SCIENCE, TECHNOLOGY, ENGINEERING AND MATHEMATICS (STEM) SPECIALIZED SUBJECT

Page 3 of 4

STEM_PC11T-IIj-3

STEM_PC11T-IIj-2

STEM_PC11T-IIj-1

STEM_PC11T-IIi-2

STEM_PC11T-IIh-2
STEM_PC11T-IIh-i-1

STEM_PC11T-IIh-1

STEM_PC11T-IIf-1
STEM_PC11T-IIf-2
STEM_PC11T-IIf-g-1
STEM_PC11T-IIg-2

STEM_PC11T-IIe-3

STEM_PC11T-IIe-2

STEM_PC11T-IIe-1

STEM_PC11T-IId-2

STEM_PC11T-IIc-d-1

STEM_PC11T-IIb-2
STEM_PC11T-IIc-1

STEM_PC11T-IIb-1

STEM_PC11T-IIa-3

STEM_PC11T-IIa-1
STEM_PC11T-IIa-2

CODE

All rights reserved. No part of this material may be reproduced or transmitted in any form or by any means electronic or mechanical including photocopying without written permission from the DepEd Central Office. First Edition, 2016.
Competency

Week

Quarter

Domain/Content/
Component/ Topic

Grade Level

SAMPLE

Science, Technology,
Engineering and Mathematics
Pre-Calculus

illustrate the different types


of conic sections: parabola,
ellipse, circle, hyperbola,
and degenerate cases

Week one

First Quarter

Analytic Geometry

Grade 11

Code Book Legend

C
O

STEM_PC11AG

DOMAIN/ COMPONENT

PY

Trigonometry

Series and Mathematical Induction

Analytic Geometry

Sample: STEM_PC11AG-Ia-1

K to 12 Senior High School STEM Specialized Subject Pre-Calculus December 2013

Arabic Number

*Put a hyphen (-) in


between letters to
indicate more than a
specific week

Lowercase
Letter/s

*Zero if no specific
quarter

Roman Numeral

Uppercase
Letter/s

First Entry

Learning Area and


Strand/ Subject or
Specialization

LEGEND

EP
E

D
K to 12 BASIC EDUCATION CURRICULUM
SENIOR HIGH SCHOOL SCIENCE, TECHNOLOGY, ENGINEERING AND MATHEMATICS (STEM) SPECIALIZED SUBJECT

Page 4 of 4

SMI

AG

CODE

Unit 1

EP
E

C
O

PY

Analytic Geometry

San Juanico Bridge, by Morten Nrbe, 21 June 2009,

https://commons.wikimedia.org/wiki/File%3ASan Juanico Bridge 2.JPG. Public Domain.

Stretching from Samar to Leyte with a total length of more than two kilometers, the San Juanico Bridge has been serving as one of the main thoroughfares
of economic and social development in the country since its completion in 1973.
Adding picturesque effect on the whole architecture, geometric structures are
subtly built to serve other purposes. The arch-shaped support on the main span
of the bridge helps maximize its strength to withstand mechanical resonance and
aeroelastic flutter brought about by heavy vehicles and passing winds.

6
All rights reserved. No part of this material may be reproduced or transmitted in any form or by any means electronic or mechanical including photocopying without written permission from the DepEd Central Office. First Edition, 2016.

Lesson 1.1. Introduction to Conic Sections and Circles


Time Frame: 4 one-hour sessions
Learning Outcomes of the Lesson
At the end of the lesson, the student is able to:
(1) illustrate the different types of conic sections: parabola, ellipse, circle, hyperbola, and degenerate cases;

PY

(2) define a circle;


(3) determine the standard form of equation of a circle;

(4) graph a circle in a rectangular coordinate system; and

Lesson Outline

C
O

(5) solve situational problems involving conic sections (circles).

(1) Introduction of the four conic sections, along with the degenerate conics
(2) Definition of a circle
(4) Graphing circles

(3) Derivation of the standard equation of a circle


(5) Solving situational problems involving circles

EP
E

Introduction

We present the conic sections, a particular class of curves which sometimes


appear in nature and which have applications in other fields. In this lesson, we
first illustrate how each of these curves is obtained from the intersection of a
plane and a cone, and then discuss the first of their kind, circles. The other conic
sections will be covered in the next lessons.
1.1.1. An Overview of Conic Sections
We introduce the conic sections (or conics), a particular class of curves which
oftentimes appear in nature and which have applications in other fields. One
of the first shapes we learned, a circle, is a conic. When you throw a ball, the
trajectory it takes is a parabola. The orbit taken by each planet around the sun
is an ellipse. Properties of hyperbolas have been used in the design of certain
telescopes and navigation systems. We will discuss circles in this lesson, leaving
parabolas, ellipses, and hyperbolas for subsequent lessons.
Circle (Figure 1.1) - when the plane is horizontal
7
All rights reserved. No part of this material may be reproduced or transmitted in any form or by any means electronic or mechanical including photocopying without written permission from the DepEd Central Office. First Edition, 2016.

Ellipse (Figure 1.1) - when the (tilted) plane intersects only one cone to form
a bounded curve
Parabola (Figure 1.2) - when the plane intersects only one cone to form an
unbounded curve

Figure 1.1

C
O

PY

Hyperbola (Figure 1.3) - when the plane (not necessarily vertical) intersects
both cones to form two unbounded curves (each called a branch of the hyperbola)

Figure 1.2

Figure 1.3

We can draw these conic sections (also called conics) on a rectangular coordinate plane and find their equations. To be able to do this, we will present
equivalent definitions of these conic sections in subsequent sections, and use these
to find the equations.

EP
E

There are other ways for a plane and the cones to intersect, to form what are
referred to as degenerate conics: a point, one line, and two lines. See Figures 1.4,
1.5 and 1.6.

Figure 1.4

Figure 1.5

Figure 1.6

8
All rights reserved. No part of this material may be reproduced or transmitted in any form or by any means electronic or mechanical including photocopying without written permission from the DepEd Central Office. First Edition, 2016.

1.1.2. Definition and Equation of a Circle


A circle may also be considered a special kind of ellipse (for the special case when
the tilted plane is horizontal). As we get to know more about a circle, we will
also be able to distinguish more between these two conics.

C
O

PY

See Figure 1.7, with the point C(3, 1) shown. From the figure, the distance
of A(2, 1) from p
C is AC = 5. By the distance formula, the distance of B(6, 5)
from C is BC = (6 3)2 + (5 1)2 = 5. There are other points P such that
P C = 5. The collection of all such points which are 5 units away from C, forms
a circle.

Figure 1.8

EP
E

Figure 1.7

Let C be a given point. The set of all points P having the same
distance from C is called a circle. The point C is called the center of
the circle, and the common distance its radius.

The term radius is both used to refer to a segment from the center C to a
point P on the circle, and the length of this segment.

See Figure 1.8, where a circle is drawn. It has center C(h, k) and radius r > 0.
A point P (x, y) is on the circle if and only if P C = r. For any such point then,
its coordinates should satisfy the following.
p

PC = r
(x h)2 + (y k)2 = r
(x h)2 + (y k)2 = r2

This is the standard equation of the circle with center C(h, k) and radius r. If
the center is the origin, then h = 0 and k = 0. The standard equation is then
x2 + y 2 = r 2 .
9
All rights reserved. No part of this material may be reproduced or transmitted in any form or by any means electronic or mechanical including photocopying without written permission from the DepEd Central Office. First Edition, 2016.

Example 1.1.1. In each item, give the standard equation of the circle satisfying
the given conditions.
(1) center at the origin, radius 4

(2) center (4, 3), radius 7


(3) circle in Figure 1.7
(4) circle A in Figure 1.9
(5) circle B in Figure 1.9
(7) center (5, 6), tangent to the x-axis

PY

(6) center (5, 6), tangent to the y-axis

EP
E

C
O

(8) has a diameter with endpoints A(1, 4) and B(4, 2)

Figure 1.9

Solution. (1) x2 + y 2 = 16
(2) (x + 4)2 + (y 3)2 = 7

(3) The center is (3, 1) and the radius is 5, so the equation is (x 3)2 + (y 1)2 =
25.
(4) By inspection, the center is (2, 1) and the radius is 4. The equation is
(x + 2)2 + (y + 1)2 = 16.
(5) Similarly by inspection, we have (x 3)2 + (y 2)2 = 9.

(6) The center is 5 units away from the y-axis, so the radius is r = 5 (you can
make a sketch to see why). The equation is (x 5)2 + (y + 6)2 = 25.
10
All rights reserved. No part of this material may be reproduced or transmitted in any form or by any means electronic or mechanical including photocopying without written permission from the DepEd Central Office. First Edition, 2016.

(7) Similarly, since the center is 6 units away from the x-axis, the equation is
(x 5)2 + (y + 6)2 = 36.


4+2
3
,
=
,
3
. The
(8) The center C is the midpoint of A and B: C = 1+4
2 q 2
2
q

2
29
radius is then r = AC =
1 32 + (4 3)2 =
. The circle has
4

2
equation x 32 + (y 3)2 = 29
.
2
4
Seatwork/Homework 1.1.2

PY

Find the standard equation of the circle being described in each item.

Answer: x2 + y 2 = 11
(1) With center at the origin, radius 11
(2) With center (6, 7), tangent to the y-axis Answer: (x + 6)2 + (y 7)2 = 36
(3) Has a diameter with endpoints A(3, 2) and B(7, 4)

C
O

Answer: (x 2)2 + (y 3)2 = 26


1.1.3. More Properties of Circles

can be rewritten as

After expanding, the standard equation



2
3
29
x
+ (y 3)2 =
2
4

EP
E

x2 + y 2 3x 6y + 4 = 0,

an equation of the circle in general form.

If the equation of a circle is given in the general form


Ax2 + Ay 2 + Cx + Dy + E = 0,

or

A 6= 0,

x2 + y 2 + Cx + Dy + E = 0,

we can determine the standard form by completing the square in both variables.

Completing the square in an expression like x2 + 14x means determining


the term to be added that will produce a perfect polynomial square. Since the
coefficient of x2 is already 1, we take half the coefficient of x and square it, and
we get 49. Indeed, x2 + 14x + 49 = (x + 7)2 is a perfect square. To complete
the square in, say, 3x2 + 18x, we factor the coefficient of x2 from the expression:
3(x2 + 6x), then add 9 inside. When completing a square in an equation, any
extra term introduced on one side should also be added to the other side.

Teaching Notes
Recall the
technique of
completing the
square. This was
introduced in
Grade 9.

Example 1.1.2. Identify the center and radius of the circle with the given equation in each item. Sketch its graph, and indicate the center.
11
All rights reserved. No part of this material may be reproduced or transmitted in any form or by any means electronic or mechanical including photocopying without written permission from the DepEd Central Office. First Edition, 2016.

(1) x2 + y 2 6x = 7

(2) x2 + y 2 14x + 2y = 14

(3) 16x2 + 16y 2 + 96x 40y = 315


Solution. The first step is to rewrite each equation in standard form by completing the square in x and in y. From the standard equation, we can determine the
center and radius.
(1)

PY

x2 6x + y 2 = 7
x2 6x + 9 + y 2 = 7 + 9
(x 3)2 + y 2 = 16

Center (3, 0), r = 4, Figure 1.10


(2)

Center (7, 1), r = 6, Figure 1.11

16x2 + 96x + 16y 2 40y = 315




5
2
2
16(x + 6x) + 16 y y = 315
2


 
5
25
25
2
2
16(x + 6x + 9) + 16 y y +
= 315 + 16(9) + 16
2
16
16
2

5
= 484
16(x + 3)2 + 16 y
4

2
 2
5
484
121
11
2
(x + 3) + y
=
=
=
4
16
4
2

Center 3, 54 , r = 5.5, Figure 1.12.

Teaching Notes
A common mistake
committed by
students is to add 9
and 25
only. They
16

EP
E

(3)

C
O

x2 14x + y 2 + 2y = 14
x2 14x + 49 + y 2 + 2y + 1 = 14 + 49 + 1
(x 7)2 + (y + 1)2 = 36

often forget the


multiplier outside
the parenthesis.

Figure 1.10

Figure 1.11

Figure 1.12

12
All rights reserved. No part of this material may be reproduced or transmitted in any form or by any means electronic or mechanical including photocopying without written permission from the DepEd Central Office. First Edition, 2016.

In the standard equation (x h)2 + (y k)2 = r2 , both the two squared


terms on the left side have coefficient 1. This is the reason why in the preceding
example, we divided by 16 at the last equation.
Seatwork/Homework 1.1.3
Identify the center and radius of the circle with the given equation in each item.
Sketch its graph, and indicate the center.

(2) 4x2 + 4y 2 + 40x 32y = 5

13
2

= 7.5, Figure 1.13


= 6.5, Figure 1.14

C
O

Answer: center (5, 4), radius

15
2

PY

(1) x2 + y 2 5x + 4y = 46

Answer: center 25 , 2 , radius

Figure 1.14

EP
E

Figure 1.13

1.1.4. Situational Problems Involving Circles


Let us now take a look at some situational problems involving circles.

? Example 1.1.3. A street with two lanes, each 10 ft wide, goes through a
semicircular tunnel with radius 12 ft. How high is the tunnel at the edge of each
lane? Round off to 2 decimal places.

13
All rights reserved. No part of this material may be reproduced or transmitted in any form or by any means electronic or mechanical including photocopying without written permission from the DepEd Central Office. First Edition, 2016.

Solution. We draw a coordinate system with origin at the middle of the highway,
as shown. Because of the given radius, the tunnels boundary is on the circle
x2 + y 2 = 122 . Point P is the point on the arc just above the edge of a lane, so
its x-coordinate is 10. Weneed its y-coordinate. We then solve 102 + y 2 = 122
for y > 0, giving us y = 2 11 6.63 ft.
2

PY

C
O
EP
E

Teaching Notes
A perpendicular
bisector of a
segment is the line
that passes
through the
midpoint of the
segment and is
perpendicular to
the segment.

Example 1.1.4. A piece of a broken plate was dug up in an archaeological site.


It was put on top of a grid, as shown in Figure 1.15, with the arc of the plate
passing through A(7, 0), B(1, 4) and C(7, 2). Find its center, and the standard
equation of the circle describing the boundary of the plate.

Figure 1.15

Figure 1.16
Solution. We first determine the center. It is the intersection of the perpendicular
14
All rights reserved. No part of this material may be reproduced or transmitted in any form or by any means electronic or mechanical including photocopying without written permission from the DepEd Central Office. First Edition, 2016.

bisectors of AB and BC (see Figure 1.16). Recall that, in a circle, the perpendicular bisector of any
the center. Since the midpoint M
 chord passes through40
7+1 0+4
of AB is
, 2 = (3, 2), and mAB = 1+7 = 12 , the perpendicular bisector
2
of AB has equation y 2 = 2(x + 3), or equivalently, y = 2x 4.

= (4, 3), and mBC = 24
, 4+2
= 31 ,
Since the midpoint N of BC is 1+7
2
2
71
the perpendicular bisector of BC has equation y 3 = 3(x 4), or equivalently,
y = 3x 9.

PY

The intersection of the two lines y = 2x 4 and y = 3x 9 is (1, 6) (by


solving a system of linear equations). We can take the radius as the distance of
this point from any of A, B or C (its most convenient to use B in this case). We
then get r = 10. The standard equation is thus (x 1)2 + (y + 6)2 = 100.
2
Seatwork/Homework 1.1.4

C
O

? 1. A single-lane street 10 ft wide goes through a semicircular tunnel with radius


9 ft. How high is the tunnel at the edge of each lane? Round off to 2 decimal
places.
Answer: 7.48 ft

EP
E

Exercises 1.1

2. An archeologist found the remains of an ancient wheel, which she then placed
on a grid. If an arc of the wheel passes through A(7, 0), B(3, 4) and C(7, 0),
locate the center of the wheel, and the standard equation of the circle defining
its boundary.
Answer: (0, 3), x2 + (y + 3)2 = 58

1. Identify the center and radius of the circle with the given equation in each
item. Sketch its graph, and indicate the center.
(a) x2 + y 2 = 49

Answer: center (0, 0), r = 52



Answer: center 47 , 34 , r = 13
4

169
16

(b) 4x2 + 4y 2 = 25
2
2
(c) x 47 + y + 43 =

Answer: center (0, 0), r = 7

(d) x2 + y 2 12x 10y = 12

Answer: center (6, 5), r = 7

(e) x2 + y 2 + 8x 9y = 6

Answer: center (4, 4.5), r =

(f) x2 + y 2 + 10x + 12y = 12

13
2

Answer: center (5, 6), r = 7

(g) 2x2 + 2y 2 14x + 18y = 7

(h) 4x2 + 4y 2 20x + 40y = 5

(i) 9x2 + 9y 2 + 42x + 84y + 65 = 0

(j) 2x2 + 2y 2 + 10x = 2y + 7

Answer: center (3.5, 4.5), r = 6

Answer: center (2.5, 5), r = 30


Answer: center 73 , 14
,
r
=
2
5
3


Answer: center 25 , 21 , r = 10

15
All rights reserved. No part of this material may be reproduced or transmitted in any form or by any means electronic or mechanical including photocopying without written permission from the DepEd Central Office. First Edition, 2016.

PY
(b)

EP
E

C
O

(a)

(d)

(e)

(f)

(c)

16
All rights reserved. No part of this material may be reproduced or transmitted in any form or by any means electronic or mechanical including photocopying without written permission from the DepEd Central Office. First Edition, 2016.

PY
(h)

EP
E

C
O

(g)

(i)

(j)

2. Find the standard equation of the circle which satisfies the given conditions.

(a) center at the origin, radius 2 2


Answer: x2 + y 2 = 8

(b) center at (15, 20), radius 9

(c) center at (5, 6), through (9, 4)

Answer: (x 15)2 + (y + 20)2 = 81

Answer: (x 5)2 + (y 6)2 = 20

Teaching Notes
To determine the
equation of a
circle, we just need
to determine the
center and the
radius.

Solution. The radius is the distance from the center to (9, 4):
p

(5 9)2 + (6 4)2 = 20.

(d) center at (2, 3), tangent to the x-axis


Answer: (x + 2)2 + (y 3)2 = 9

(e) center at (2, 3), tangent to the y-axis


Answer: (x + 2)2 + (y 3)2 = 4
17

All rights reserved. No part of this material may be reproduced or transmitted in any form or by any means electronic or mechanical including photocopying without written permission from the DepEd Central Office. First Edition, 2016.

(f) center at (2, 3), tangent to the line y = 8


Answer: (x + 2)2 + (y 3)2 = 25
Solution. We need to determine the radius. This is best done by
sketching the center and line, to see that the center (2, 3) is 5 units
away from the nearest point on the line, (2, 8) (which is the point of
tangency).
(g) center at (2, 3), tangent to the line x = 10
Answer: (x + 2)2 + (y 3)2 = 64

PY

(h) center in the third quadrant, tangent to both the x-axis and y-axis,
radius 7
Answer: (x + 7)2 + (y + 7)2 = 49
(i) a diameter with endpoints (9, 2) and (15, 12)
Answer: (x 3)2 + (y 7)2 = 169

C
O

(j) concentric with x2 + y 2 + 2x 4y = 5, radius is 7


Answer: (x + 1)2 + (y 2)2 = 49

Solution. Two circles are said to be concentric if they have the same
center.
The standard equation of the given circle is
2
(x + 1) + (y 2)2 = 10. Thus, the circle were looking for has center
(1, 2) and radius 7.

EP
E

(k) concentric with x2 + y 2 8x 10y = 16 and 4 times the area


Answer: (x 4)2 + (y 5)2 = 100
Solution. The given circle has standard equation
(x 4)2 + (y 5)2 = 52 .

Its radius is 5, so its area is 25 sq. units. The circle we are looking
for should have area 100 sq. units, so its radius is 10.

(l) concentric with x2 + y 2 10x 6y = 2, same radius as


x2 + y 2 14x + 6y = 33
Answer: (x 5)2 + (y 3)2 = 25

(m) center at C(3, 4), tangent to the line y = 13 x 13


Answer: (x 3)2 + (y 4)2 = 10

Teaching Notes
The radius drawn
to a point on the
circle is
perpendicular to
the line tangent to
the circle at that
point.

Solution. (A sketch will greatly help in understanding the argument.)


If P is the point of tangency, then line CP is perpendicular to the given
tangent line. Since the tangent line has slope 31 , line CP has slope 3.
Because it passes through C, line CP has equation y 4 = 3(x 3),
or y = 3x + 13. Solving the system {y = 13 x 13 , y = 3x + 13}
yields p
x = 4 and y = 1, the
coordinates of P . The radius is then
2
2
CP = (3 4) + (4 1) = 10.
18

All rights reserved. No part of this material may be reproduced or transmitted in any form or by any means electronic or mechanical including photocopying without written permission from the DepEd Central Office. First Edition, 2016.

(n) center at (4, 3), tangent to the line y = 4x 30


Answer: (x + 4)2 + (y 3)2 = 17
Solution. (Similar to the previous problem) Let P be the point of
tangency, so line CP is perpendicular to the tangent line. The tangent
line has slope 4, so line CP has slope 14 . Line CP passes through C,
so it has equation y 3 = 14 (x + 4), or y = 14 x + 4. Solving the system
{y = 4x 30, y = 41 x + 4} yields x = 8 and y = 2, the coordinates
p

of P . The radius is then CP = (4 + 8)2 + (3 2)2 = 17.

PY

? 3. A seismological station is located at (0, 3), 3 km away from a straight


shoreline where the x-axis runs through. The epicenter of an earthquake
was determined to be 6 km away from the station.
(a) Find the equation of the curve that contains the possible location of
the epicenter.
Answer: x2 + (y + 3)2 = 62

C
O

(b) If furthermore, the epicenter was determined to be 2 km away from


the shore, find its possible coordinates (rounded off to two decimal
places).
Answer: (3.32, 2)

EP
E

Solution. Since the epicenter is 6 units away from (0, 3), it could be any
of the points of a circle with center (0, 3) and radius 6. The equation is
then x2 + (y + 3)2 = 62 . Next, we solve thisequation for x if y = 2, and we
get x2 = 62 (2 + 3)2 = 11, and so x = 11 3.32.
4. A ferris wheel is elevated 1 m above ground. When a car reaches the highest
point on the ferris wheel, its altitude from ground level is 31 m. How far
away from the center, horizontally, is the car when it is at an altitude of
25 m?
Answer: 12 m

Solution. The ferris wheel, as shown


below, is drawn 1 unit above the xaxis (ground level), center on the yaxis, and highest point at y = 31.
The diameter is thus 30, and the radius 15. We locate the center at
(0, 16), and write the equation of the
circle as x2 + (y 16)2 = 152 .

If y = 25, we have x2 + (25 16)2 =


152 , so x2 = 152 92 = 144, and
x = 12. (Clearly, there are two
points on the ferris wheel at an altitude of 25 m.) Thus, the car is 12 m
away horizontally from the center.
19

All rights reserved. No part of this material may be reproduced or transmitted in any form or by any means electronic or mechanical including photocopying without written permission from the DepEd Central Office. First Edition, 2016.

? 5. A window is to be constructed as shown, with its upper boundary the arc


of a circle having radius 4 ft and center at the midpoint of base AD. If the
vertical side is to be 34 as long as the base, find the dimensions (vertical side
and base) of this window. Round off your final answer to 1 decimal place.

C
O

PY

Answer: base 4.44 ft, side 3.33 ft

EP
E

Solution. We put two lines corresponding to the x-axis and y-axis, as shown,
with the origin coinciding with the midpoint of the windows base. This
origin is the center of the circle containing the arc. The equation of the circle
is then x2 + y 2 = 16. Let n be length of the base
 AD, so thenside
2 AD
has
n 3n
3n 2
3
length 4 n. Point B then has coordinates 2 , 4 . Therefore, 2 + 4 =
16. Solving this for n > 0 yields n = 1613 . The base is then n 4.44 ft and
the side 34 n 3.33 ft.

Lesson 1.2. Parabolas

Time Frame: 3 one-hour sessions


Learning Outcomes of the Lesson
At the end of the lesson, the student is able to:
(1) define a parabola;
(2) determine the standard form of equation of a parabola;
(3) graph a parabola in a rectangular coordinate system; and
(4) solve situational problems involving conic sections (parabolas).
20

All rights reserved. No part of this material may be reproduced or transmitted in any form or by any means electronic or mechanical including photocopying without written permission from the DepEd Central Office. First Edition, 2016.

Lesson Outline
(1) Definition of a parabola
(2) Derivation of the standard equation of a parabola
(3) Graphing parabolas
(4) Solving situational problems involving parabolas
Introduction

PY

A parabola is one of the conic sections. We have already seen parabolas which
open upward or downward, as graphs of quadratic functions. Here, we will see
parabolas opening to the left or right. Applications of parabolas are presented
at the end.

C
O

1.2.1. Definition and Equation of a Parabola

Consider the point F (0, 2) and the line ` having equation y = 2, as shown in
Figure 1.17. What are the distances of A(4, 2) from F and from `? (The latter
is taken as the distance of A from A` , the point on ` closest to A). How about
the distances of B(8, 8) from F and from ` (from B` )?
AF = 4

AA` = 4

p
(8 0)2 + (8 2)2 = 10

BF =

and

and

BB` = 10

EP
E

There are other points P such that P F = P P` (where P` is the closest point on
line `). The collection of all such points forms a shape called a parabola.

Figure 1.17

Figure 1.18

Let F be a given point, and ` a given line not containing F . The set of
all points P such that its distances from F and from ` are the same, is
called a parabola. The point F is its focus and the line ` its directrix.

21
All rights reserved. No part of this material may be reproduced or transmitted in any form or by any means electronic or mechanical including photocopying without written permission from the DepEd Central Office. First Edition, 2016.

Consider a parabola with focus F (0, c) and directrix ` having equation y = c.


See Figure 1.18. The focus and directrix are c units above and below, respectively,
the origin. Let P (x, y) be a point on the parabola so P F = P P` , where P` is the
point on ` closest to P . The point P has to be on the same side of the directrix
as the focus (if P was below, it would be closer to ` than it is from F ).
P F = P P`

PY

x2 + (y c)2 = y (c) = y + c
x2 + y 2 2cy + c2 = y 2 + 2cy + c2
x2 = 4cy
The vertex V is the point midway between the focus and the directrix. This
equation, x2 = 4cy, is then the standard equation of a parabola opening upward
with vertex V (0, 0).

C
O

Suppose the focus is F (0, c) and the directrix is y = c. In this case, a


point P on the resulting parabola would be below the directrix (just like the
focus).
Instead of opening upward, it will open downward. Consequently, P F =
p
x2 + (y + c)2 and P P` = c y (you may draw a version of Figure 1.18 for
this case). Computations similar to the one done above will lead to the equation
x2 = 4cy.

EP
E

We collect here the features of the graph of a parabola with standard equation
x2 = 4cy or x2 = 4cy, where c > 0.

(1) vertex : origin V (0, 0)


If the parabola opens upward, the vertex is the lowest point. If the
parabola opens downward, the vertex is the highest point.
(2) directrix : the line y = c or y = c
The directrix is c units below or above the vertex.

(3) focus: F (0, c) or F (0, c)


The focus is c units above or below the vertex.
22
All rights reserved. No part of this material may be reproduced or transmitted in any form or by any means electronic or mechanical including photocopying without written permission from the DepEd Central Office. First Edition, 2016.

Any point on the parabola has the same distance from the focus as it
has from the directrix.
(4) axis of symmetry: x = 0 (the y-axis)
This line divides the parabola into two parts which are mirror images
of each other.

PY

Example 1.2.1. Determine the focus and directrix of the parabola with the
given equation. Sketch the graph, and indicate the focus, directrix, vertex, and
axis of symmetry.
(1) x2 = 12y
(2) x2 = 6y

EP
E

C
O

Solution. (1) The vertex is V (0, 0) and the parabola opens upward. From 4c =
12, c = 3. The focus, c = 3 units above the vertex, is F (0, 3). The directrix,
3 units below the vertex, is y = 3. The axis of symmetry is x = 0.

(2) The vertex is V (0, 0) and the parabola opens downward.


From 4c = 6, c = 32 .

The focus, c = 23 units below the vertex, is F 0, 32 . The directrix, 32 units
above the vertex, is y = 32 . The axis of symmetry is x = 0.

23
All rights reserved. No part of this material may be reproduced or transmitted in any form or by any means electronic or mechanical including photocopying without written permission from the DepEd Central Office. First Edition, 2016.

Example 1.2.2. What is the standard equation of the parabola in Figure 1.17?
Solution. From the figure, we deduce that c = 2. The equation is thus x2 =
8y.
2
Seatwork/Homework 1.2.1

EP
E

C
O

PY

1. Give the focus and directrix of the parabola with equation x2 = 10y. Sketch
the graph, and indicate
the focus, directrix, vertex, and axis of symmetry.

Answer: focus 0, 25 , directrix y = 25

2. Find the standard equation of the parabola with focus F (0, 3.5) and directrix
y = 3.5.
Answer: x2 = 14y
1.2.2. More Properties of Parabolas

The parabolas we considered so far are vertical and have their vertices at the
origin. Some parabolas open instead horizontally (to the left or right), and some
have vertices not at the origin. Their standard equations and properties are given
in the box. The corresponding computations are more involved, but are similar
to the one above, and so are not shown anymore.

In all four cases below, we assume that c > 0. The vertex is V (h, k), and it
lies between the focus F and the directrix `. The focus F is c units away from
the vertex V , and the directrix is c units away from the vertex. Recall that, for
any point on the parabola, its distance from the focus is the same as its distance
from the directrix.

24
All rights reserved. No part of this material may be reproduced or transmitted in any form or by any means electronic or mechanical including photocopying without written permission from the DepEd Central Office. First Edition, 2016.

PY

(y k)2 = 4c(x h)

C
O

(x h)2 = 4c(y k)

(y k)2 = 4c(x h)

directrix `: horizontal

directrix `: vertical

axis of symmetry: x=h, vertical

axis of symmetry: y=k, horizontal

EP
E

(x h)2 = 4c(y k)

Note the following observations:

The equations are in terms of x h and y k: the vertex coordinates are


subtracted from the corresponding variable. Thus, replacing both h and k
with 0 would yield the case where the vertex is the origin. For instance, this
replacement applied to (x h)2 = 4c(y k) (parabola opening upward) would
yield x2 = 4cy, the first standard equation we encountered (parabola opening
upward, vertex at the origin).

If the x-part is squared, the parabola is vertical; if the y-part is squared,


the parabola is horizontal. In a horizontal parabola, the focus is on the left
or right of the vertex, and the directrix is vertical.
If the coefficient of the linear (non-squared) part is positive, the parabola
opens upward or to the right; if negative, downward or to the left.
25
All rights reserved. No part of this material may be reproduced or transmitted in any form or by any means electronic or mechanical including photocopying without written permission from the DepEd Central Office. First Edition, 2016.

C
O

PY

Teaching Notes
In finding the
equation of a
parabola, we just
need to determine
the vertex and the
value of c.

Example 1.2.3. The figure shows the graph of parabola, with only its focus and
vertex indicated. Find its standard equation. What is its directrix and its axis
of symmetry?

EP
E

Solution. The vertex is V (5, 4) and the focus is F (3, 4). From these, we
deduce the following: h = 5, k = 4, c = 2 (the distance of the focus from the
vertex). Since the parabola opens to the left, we use the template (y k)2 =
4c(x h). Our equation is
(y + 4)2 = 8(x 5).

Its directrix is c = 2 units to the right of V , which is x = 7. Its axis is the


horizontal line through V : y = 4.
The standard equation (y + 4)2 = 8(x 5) from the preceding example can
be rewritten as y 2 + 8x + 8y 24 = 0, an equation of the parabola in general
form.
If the equation is given in the general form Ax2 + Cx + Dy + E = 0 (A and C
are nonzero) or By 2 + Cx + Dy + E = 0 (B and C are nonzero), we can determine
the standard form by completing the square in both variables.
Example 1.2.4. Determine the vertex, focus, directrix, and axis of symmetry
of the parabola with the given equation. Sketch the parabola, and include these
points and lines.
(a) y 2 5x + 12y = 16

(b) 5x2 + 30x + 24y = 51


26

All rights reserved. No part of this material may be reproduced or transmitted in any form or by any means electronic or mechanical including photocopying without written permission from the DepEd Central Office. First Edition, 2016.

Solution. (1) We complete the square on y, and move x to the other side.
y 2 + 12y = 5x 16
y 2 + 12y + 36 = 5x 16 + 36 = 5x + 20
(y + 6)2 = 5(x + 4)

EP
E

C
O

PY

The parabola opens to the right. It has vertex V (4, 6). From 4c = 5, we
get c = 45 = 1.25. The focus is c = 1.25 units to the right of V : F (2.75, 6).
The (vertical) directrix is c = 1.25 units to the left of V : x = 5.25. The
(horizontal) axis is through V : y = 6.

(2) We complete the square on x, and move y to the other side.


5x2 + 30x = 24y + 51
5(x2 + 6x + 9) = 24y + 51 + 5(9)
5(x + 3)2 = 24y + 96 = 24(y 4)
24
(x + 3)2 = (y 4)
5

In the last line, we divided by 5 for the squared part not to have any coefficient. The parabola opens downward. It has vertex V (3, 4).
From 4c = 24
, we get c = 65 = 1.2. The focus is c = 1.2 units below V :
5
F (3, 2.8). The (horizontal) directrix is c = 1.2 units above V : y = 5.2. The
(vertical) axis is through V : x = 3.
27
All rights reserved. No part of this material may be reproduced or transmitted in any form or by any means electronic or mechanical including photocopying without written permission from the DepEd Central Office. First Edition, 2016.

PY

Example 1.2.5. A parabola has focus F (7, 9) and directrix y = 3. Find its
standard equation.

C
O

Solution. The directrix is horizontal, and the focus is above it. The parabola
then opens upward and its standard equation has the form (x h)2 = 4c(y k).
Since the distance from the focus to the directrix is 2c = 9 3 = 6, then c = 3.
Thus, the vertex is V (7, 6), the point 3 units below F . The standard equation is
then (x 7)2 = 12(y 6).
2

Seatwork/Homework 1.2.2

EP
E

1. Determine the vertex, focus, directrix, and axis of symmetry of the parabola
with equation x2 6x + 5y = 34. Sketch the graph, and include these points
and lines.

Answer: vertex (3, 5), focus (3, 6.25), directrix y = 3.75, axis x = 3

28
All rights reserved. No part of this material may be reproduced or transmitted in any form or by any means electronic or mechanical including photocopying without written permission from the DepEd Central Office. First Edition, 2016.

2. A parabola has focus F (2, 5) and directrix x = 6. Find the standard


equation of the parabola.
Answer: (y + 5)2 = 16(x 2)
1.2.3. Situational Problems Involving Parabolas
Let us now solve some situational problems involving parabolas.

EP
E

C
O

PY

Example 1.2.6. A satellite dish has a shape called a paraboloid, where each
cross-section is a parabola. Since radio signals (parallel to the axis) will bounce
off the surface of the dish to the focus, the receiver should be placed at the focus.
How far should the receiver be from the vertex, if the dish is 12 ft across, and 4.5
ft deep at the vertex?

Solution. The second figure above shows a cross-section of the satellite dish drawn
on a rectangular coordinate system, with the vertex at the origin. From the
problem, we deduce that (6, 4.5) is a point on the parabola. We need the distance
of the focus from the vertex, i.e., the value of c in x2 = 4cy.
x2 = 4cy
62 = 4c(4.5)
62
c=
=2
4 4.5

Thus, the receiver should be 2 ft away from the vertex.

Example 1.2.7. The cable of a suspension bridge hangs in the shape of a


parabola. The towers supporting the cable are 400 ft apart and 150 ft high.
If the cable, at its lowest, is 30 ft above the bridge at its midpoint, how high is
the cable 50 ft away (horizontally) from either tower?

29
All rights reserved. No part of this material may be reproduced or transmitted in any form or by any means electronic or mechanical including photocopying without written permission from the DepEd Central Office. First Edition, 2016.

PY

C
O

Solution. Refer to the figure above, where the parabolic cable is drawn with
its vertex on the y-axis 30 ft above the origin. We may write its equation as
(x 0)2 = a(y 30); since we dont need the focal distance, we use the simpler
variable a in place of 4c. Since the towers are 150 ft high and 400 ft apart, we
deduce from the figure that (200, 150) is a point on the parabola.

x2 = a(y 30)
2002 = a(150 30)
1000
2002
=
a=
120
3

EP
E

The parabola has equation x2 = 1000


(y 30), or equivalently,
3
2
y = 0.003x + 30. For the two points on the parabola 50 ft away from the
towers, x = 150 or x = 150. If x = 150, then
y = 0.003(1502 ) + 30 = 97.5.

Thus, the cable is 97.5 ft high 50 ft away from either tower. (As expected, we
get the same answer from x = 150.)
2
Seatwork/Homework 1.2.3

? 1. A satellite dish in the shape of a paraboloid is 10 ft across, and 4 ft deep at


its vertex. How far is the receiver from the vertex, if it is placed at the focus?
Round off your answer to 2 decimal places. (Refer to Example 1.2.6.)
Answer: 1.56 ft
Exercises 1.2
1. Determine the vertex, focus, directrix, and axis of symmetry of the parabola
with the given equation. Sketch the graph, and include these points and lines.
30

All rights reserved. No part of this material may be reproduced or transmitted in any form or by any means electronic or mechanical including photocopying without written permission from the DepEd Central Office. First Edition, 2016.

(a) x2 = 4y

(d) x2 + 6x + 8y = 7

(b) 3y 2 = 24x
2

(c) y + 52 = 5 x 92

(e) y 2 12x + 8y = 40

(f) 16x2 + 72x 112y = 221

Answer:

Vertex

Focus

Directrix

Axis of Symmetry

(a)

(0, 0)

(0, 1)

y=1

x=0

(b)

(0, 0)

(2, 0)

x = 2

y=0

(c)

(4.5, 2.5)

(3.25, 2.5)

x = 5.75

y = 2.5

(d)

(3, 2)

(3, 0)

y=4

x = 3

(e)

(2, 4)

(5, 4)

x = 1

y = 4

(f)

(2.25, 1.25)

(2.25, 3)

y = 0.5

x = 2.25

EP
E

C
O

PY

Item

(b)

(c)

(e)

(f)

(a)

(d)

2. Find the standard equation of the parabola which satisfies the given conditions.
Answer: (y + 9)2 = 16(x 1)

(a) vertex (1, 9), focus (3, 9)

Teaching Notes
It is helpful to
draw a diagram for
each item.

31
All rights reserved. No part of this material may be reproduced or transmitted in any form or by any means electronic or mechanical including photocopying without written permission from the DepEd Central Office. First Edition, 2016.

(b) vertex (8, 3), directrix x = 10.5

Answer: (y 3)2 = 10(x + 8)

(c) vertex (4, 2), focus (4, 1)

Answer: (x + 4)2 = 12(y 2)

(e) focus (7, 11), directrix y = 4

Answer: (x 7)2 = 14(y 7.5)

Answer: (y 11)2 = 12(x 4)

(d) focus (7, 11), directrix x = 1

(f) vertex (5, 7), vertical axis of symmetry, through the point P (7, 11)
Answer: (x + 5)2 = 8(y + 7)

PY

Solution. Since the axis is vertical and P is above the vertex, then the
parabola opens upward and has equation of the form (x + 5)2 = 4c(y + 7).
We plug the coordinates of P : (7 + 5)2 = 4c(11 + 7). We then get c = 2.
Thus, we have (x + 5)2 = 8(y + 7).
(g) vertex (5, 7), horizontal axis of symmetry, through the point P (7, 11)
Answer: (y + 7)2 = 27(x + 5)

C
O

Solution. Since the axis is horizontal and P is to the right of the vertex,
then the parabola opens to the right and has equation of the form (y +
7)2 = 4c(x + 5). We plug the coordinates of P : (11 + 7)2 = 4c(7 + 5).
We then get c = 6.75. Thus, we have (y + 7)2 = 27(x + 5).

3. A satellite dish shaped like a paraboloid, has diameter 2.4 ft and depth 0.9 ft.
If the receiver is placed at the focus, how far should the receiver be from the
vertex?
Answer: 0.4 ft

EP
E

4. If the diameter of the satellite dish from the previous problem is doubled, with
the depth kept the same, how far should the receiver be from the vertex?

Answer: 1.6 ft
? 5. A satellite dish is shaped like a paraboloid, with the receiver placed at the
focus. It is to have a depth of 0.44 m at the vertex, with the receiver placed
0.11 m away from the vertex. What should the diameter of the satellite dish
be?
Answer: 0.88 m
? 6. A flashlight is shaped like a paraboloid, so that if its light bulb is placed at
the focus, the light rays from the bulb will then bounce off the surface in a
focused direction that is parallel to the axis. If the paraboloid has a depth of
1.8 in and the diameter on its surface is 6 in, how far should the light source
be placed from the vertex?
Answer: 1.25 in

7. The towers supporting the cable of a suspension bridge are 1200 m apart and
170 m above the bridge it supports. Suppose the cable hangs, following the
shape of a parabola, with its lowest point 20 m above the bridge. How high is
the cable 120 m away from a tower?
Answer: 116 m

4
32
All rights reserved. No part of this material may be reproduced or transmitted in any form or by any means electronic or mechanical including photocopying without written permission from the DepEd Central Office. First Edition, 2016.

Lesson 1.3. Ellipses


Time Frame: 3 one-hour sessions
Learning Outcomes of the Lesson
At the end of the lesson, the student is able to:
(1) define an ellipse;
(2) determine the standard form of equation of an ellipse;

PY

(3) graph an ellipse in a rectangular coordinate system; and

(4) solve situational problems involving conic sections (ellipses).

(1) Definition of an ellipse

C
O

Lesson Outline

(2) Derivation of the standard equation of an ellipse


(3) Graphing ellipses

Introduction

(4) Solving situational problems involving ellipses

EP
E

Unlike circle and parabola, an ellipse is one of the conic sections that most students have not encountered formally before. Its shape is a bounded curve which
looks like a flattened circle. The orbits of the planets in our solar system around
the sun happen to be elliptical in shape. Also, just like parabolas, ellipses have
reflective properties that have been used in the construction of certain structures.
These applications and more will be encountered in this lesson.

1.3.1. Definition and Equation of an Ellipse


Consider the points F1 (3, 0) and F2 (3, 0), as shown in Figure 1.19. What is the
sum of the distances of A(4, 2.4) from F1 and from F2 ? How about the sum of
the distances of B (and C(0, 4)) from F1 and from F2 ?
AF1 + AF2 = 7.4 + 2.6 = 10
BF1 + BF2 = 3.8 + 6.2 = 10
CF1 + CF2 = 5 + 5 = 10

Teaching Notes
You may review
the distance
formula.

There are other points P such that P F1 + P F2 = 10. The collection of all such
points forms a shape called an ellipse.

33
All rights reserved. No part of this material may be reproduced or transmitted in any form or by any means electronic or mechanical including photocopying without written permission from the DepEd Central Office. First Edition, 2016.

Figure 1.19

PY

Figure 1.20

C
O

Let F1 and F2 be two distinct points. The set of all points P , whose
distances from F1 and from F2 add up to a certain constant, is called
an ellipse. The points F1 and F2 are called the foci of the ellipse.

Given are two points on the x-axis, F1 (c, 0) and F2 (c, 0), the foci, both c
units away from their center (0, 0). See Figure 1.20. Let P (x, y) be a point on
the ellipse. Let the common sum of the distances be 2a (the coefficient 2 will
make computations simpler). Thus, we have P F1 + P F2 = 2a.

EP
E

P F1 = 2a P F2
p
p
(x + c)2 + y 2 = 2a (x c)2 + y 2
p
x2 + 2cx + c2 + y 2 = 4a2 4a (x c)2 + y 2 + x2 2cx + c2 + y 2
p
a (x c)2 + y 2 = a2 cx


a2 x2 2cx + c2 + y 2 = a4 2a2 cx + c2 x2
(a2 c2 )x2 + a2 y 2 = a4 a2 c2 = a2 (a2 c2 )

b2 x 2 + a2 y 2 = a2 b2
by letting b = a2 c2 , so a > b
x2 y 2
+ 2 =1
a2
b

When we let b = a2 c2 , we assumed a > c. To see why this is true, look at


4P F1 F2 in Figure 1.20. By the Triangle Inequality, P F1 + P F2 > F1 F2 , which
implies 2a > 2c, so a > c.
We collect here the features of the graph of an ellipse with standard equation

x2 y 2
+ 2 = 1, where a > b. Let c = a2 b2 .
2
a
b
34
All rights reserved. No part of this material may be reproduced or transmitted in any form or by any means electronic or mechanical including photocopying without written permission from the DepEd Central Office. First Edition, 2016.

PY

(1) center : origin (0, 0)


(2) foci : F1 (c, 0) and F2 (c, 0)

Each focus is c units away from the center.

C
O

For any point on the ellipse, the sum of its distances from the foci is 2a.
(3) vertices: V1 (a, 0) and V2 (a, 0)

The vertices are points on the ellipse, collinear with the center and foci.
If y = 0, then x = a. Each vertex is a units away from the center.

The segment V1 V2 is called the major axis. Its length is 2a. It divides
the ellipse into two congruent parts.

EP
E

(4) covertices: W1 (0, b) and W2 (0, b)

The segment through the center, perpendicular to the major axis, is the
minor axis. It meets the ellipse at the covertices. It divides the ellipse
into two congruent parts.
If x = 0, then y = b. Each covertex is b units away from the center.

The minor axis W1 W2 is 2b units long. Since a > b, the major axis is
longer than the minor axis.

Example 1.3.1. Give the coordinates of the foci, vertices, and covertices of the
ellipse with equation
x2 y 2
+
= 1.
25
9
Sketch the graph, and include these points.
Solution. With a2 = 25 and b2 = 9, we have a = 5, b = 3, and c =
foci: F1 (4, 0), F2 (4, 0)

a2 b2 = 4.

vertices: V1 (5, 0), V2 (5, 0)

covertices: W1 (0, 3), W2 (0, 3)


35
All rights reserved. No part of this material may be reproduced or transmitted in any form or by any means electronic or mechanical including photocopying without written permission from the DepEd Central Office. First Edition, 2016.

PY

Example 1.3.2. Find the (standard) equation of the ellipse whose foci are
F1 (3, 0) and F2 (3, 0), such that for any point on it, the sum of its distances
from the foci is 10. See Figure 1.19.

a2 c2 = 4. The
2

Seatwork/Homework 1.3.1

C
O

Solution. We have 2a = 10 and c = 3, so a = 5 and b =


equation is
x2 y 2
+
= 1.
25 16

EP
E

1. Give the coordinates of the foci, vertices, and covertices of the ellipse with
y2
x2
+
= 1. Sketch the graph, and include these points.
equation
169 25
Answer: foci: F1 (12, 0) and F2 (12, 0), vertices: V1 (13, 0) and V2 (13, 0),
covertices: W1 (0, 5) and W2 (0, 5)

2. Find the equation in standard form of the ellipse whose foci are F1 (8, 0) and
F2 (8, 0), such that for any point on it, the sum of its distances from the foci
x2
y2
is 20.
Answer:
+
=1
100 36
36
All rights reserved. No part of this material may be reproduced or transmitted in any form or by any means electronic or mechanical including photocopying without written permission from the DepEd Central Office. First Edition, 2016.

1.3.2. More Properties of Ellipses


Some ellipses have their foci aligned vertically, and some have centers not at
the origin. Their standard equations and properties are given in the box. The
derivations are more involved, but are similar to the one above, and so are not
shown anymore.

PY

(0, 0)

Corresponding Graphs

C
O

Center

x2 y 2
+ 2 = 1, b > a
b2
a

EP
E

x2 y 2
+ 2 = 1, a > b
a2
b

(h, k)

(x h)2 (y k)2
+
=1
a2
b2
a>b

(x h)2 (y k)2
+
=1
b2
a2
b>a

major axis: horizontal

major axis: vertical

minor axis: vertical

minor axis: horizontal

37
All rights reserved. No part of this material may be reproduced or transmitted in any form or by any means electronic or mechanical including photocopying without written permission from the DepEd Central Office. First Edition, 2016.


In all four cases above, a > b and c = a2 b2 . The foci F1 and F2 are c
units away from the center. The vertices V1 and V2 are a units away from the
center, the major axis has length 2a, the covertices W1 and W2 are b units away
from the center, and the minor axis has length 2b. Recall that, for any point on
the ellipse, the sum of its distances from the foci is 2a.
In the standard equation, if the x-part has the bigger denominator, the ellipse
is horizontal. If the y-part has the bigger denominator, the ellipse is vertical.
Example 1.3.3. Give the coordinates of the center, foci, vertices, and covertices
of the ellipse with the given equation. Sketch the graph, and include these points.
(x + 3)2 (y 5)2
+
=1
24
49
(2) 9x2 + 16y 2 126x + 64y = 71

PY

(1)

center:
foci:

C
O

Solution. (1) From a2 = 49 and b2 = 24, we have a = 7, b = 2 6 4.9, and


c = a2 b2 = 5. The ellipse is vertical.
(3, 5)

F1 (3, 0), F2 (3, 10)

EP
E

vertices: V1 (3, 2), V2 (3, 12)

covertices: W1 (3 2 6, 5) (7.9, 5)

W2 (3 + 2 6, 5) (1.9, 5)

38
All rights reserved. No part of this material may be reproduced or transmitted in any form or by any means electronic or mechanical including photocopying without written permission from the DepEd Central Office. First Edition, 2016.

(2) We first change the given equation to standard form.

foci:

(7, 2)

F1 (7 2 7, 2) (1.7, 2)

F2 (7 + 2 7, 2) (12.3, 2)

C
O

center:

PY

9(x2 14x) + 16(y 2 + 4y) = 71


9(x2 14x + 49) + 16(y 2 + 4y + 4) = 71 + 9(49) + 16(4)
9(x 7)2 + 16(y + 2)2 = 576
(x 7)2 (y + 2)2
+
=1
64
36

We have a = 8 and b = 6. Thus, c = a2 b2 = 2 7 5.3. The ellipse is


horizontal.

vertices: V1 (1, 2), V2 (15, 2)


W1 (7, 8), W2 (7, 4)

EP
E

covertices:

Example 1.3.4. The foci of an ellipse are (3, 6) and (3, 2). For any point
on the ellipse, the sum of its distances from the foci is 14. Find the standard
equation of the ellipse.

39
All rights reserved. No part of this material may be reproduced or transmitted in any form or by any means electronic or mechanical including photocopying without written permission from the DepEd Central Office. First Edition, 2016.

Solution. The midpoint (3, 2) of the foci is the center of the ellipse. The
ellipse is vertical (because the foci are vertically
aligned)and c = 4. From the
given sum, 2a = 14 so a = 7. Also, b = a2 c2 = 33. The equation is
(x + 3)2 (y + 2)2
+
= 1.
2
33
49

Example 1.3.5. An ellipse has vertices (2 61, 5) and (2 + 61, 5), and
its minor axis is 12 units long. Find its standard equation and its foci.

C
O

Seatwork/Homework 1.3.2

PY

Solution. The midpoint (2, 5)of the vertices is the center of the ellipse, which is
horizontal. Each vertex is a = 61 units away from the center. From the length of
(x 2)2 (y + 5)2
the minor axis, 2b = 12 so b = 6. The standard equation is
+
=
61
36

1. Each focus is c = a2 b2 = 5 units away from (2, 5), so their coordinates


are (3, 5) and (7, 5).
2

1. Give the coordinates of the center, foci, vertices, and covertices of the ellipse
with equation 41x2 + 16y 2 + 246x 192y + 289 = 0. Sketch the graph, and
include these points.

EP
E

Answer: center C(3,


6), foci F1 (3, 1) and F2 (3, 11), vertices V1 (3, 6

41) and V2 (3, 6 + 41), covertices W1 (7, 6) and W2 (1, 6)

40
All rights reserved. No part of this material may be reproduced or transmitted in any form or by any means electronic or mechanical including photocopying without written permission from the DepEd Central Office. First Edition, 2016.

2. An ellipse has vertices (10, 4) and (6, 4), and covertices (2, 9) and
(2, 1). Find its standard equation and its foci.

(x + 2)2 (y + 4)2
+
= 1, foci (2 39, 4) and (2 + 39, 4)
Answer:
64
25
1.3.3. Situational Problems Involving Ellipses
Let us now apply the concept of ellipse to some situational problems.

EP
E

C
O

PY

? Example 1.3.6. A tunnel has the shape of a semiellipse that is 15 ft high at


the center, and 36 ft across at the base. At most how high should a passing truck
be, if it is 12 ft wide, for it to be able to fit through the tunnel? Round off your
answer to two decimal places.

Solution. Refer to the figure above. If we draw the semiellipse on a rectangular


coordinate system, with its center at the origin, an equation of the ellipse which
contains it, is
y2
x2
+
= 1.
182 152
To maximize its height, the corners of the truck, as shown in the figure, would
have to just touch the ellipse. Since the truck is 12 ft wide, let the point (6, n)
be the corner of the truck in the first quadrant, where n > 0, is the (maximum)
height of the truck. Since this point is on the ellipse, it should fit the equation.
Thus, we have
62
n2
+
=1
182 152



62
n = 15 1 2
18

n = 10 2 14.14 ft
2

41
All rights reserved. No part of this material may be reproduced or transmitted in any form or by any means electronic or mechanical including photocopying without written permission from the DepEd Central Office. First Edition, 2016.

C
O

PY

Example 1.3.7. The orbit of a planet has the shape of an ellipse, and on one
of the foci is the star around which it revolves. The planet is closest to the star
when it is at one vertex. It is farthest from the star when it is at the other vertex.
Suppose the closest and farthest distances of the planet from this star, are 420
million kilometers and 580 million kilometers, respectively. Find the equation of
the ellipse, in standard form, with center at the origin and the star at the x-axis.
Assume all units are in millions of kilometers.

EP
E

Solution. In the figure above, the orbit is drawn as a horizontal ellipse with
center at the origin. From the planets distances from the star, at its closest
and farthest points, it follows that the major axis is 2a = 420 + 580 = 1000
(million kilometers), so a = 500. If we place the star at the positive x-axis,
then it is c = 500 420 = 80 units away from the center. Therefore, we get
b2 = a2 c2 = 5002 802 = 243600. The equation then is
y2
x2
+
= 1.
250000 243600

The star could have been placed on the negative x-axis, and the answer would
still be the same.
2
Seatwork/Homework 1.3.3

? 1. The arch of a bridge is in the shape of a semiellipse, with its major axis at the
water level. Suppose the arch is 20 ft high in the middle, and 120 ft across its
major axis. How high above the water level is the arch, at a point 20 ft from
the center (horizontally)? Round off to 2 decimal places. Refer to Example
1.3.6.
Answer: 18.86 ft
42
All rights reserved. No part of this material may be reproduced or transmitted in any form or by any means electronic or mechanical including photocopying without written permission from the DepEd Central Office. First Edition, 2016.

Exercises 1.3
1. Give the coordinates of the center, vertices, covertices, and foci of the ellipse
with the given equation. Sketch the graph, and include these points.
y2
x2
+
=1
(a)
169 25
y2
x2
+
=1
(b)
144 169
(c) 4x2 + 13y 2 = 52
(x + 7)2 (y 4)2
+
=1
16
25
(e) 9x2 + 16y 2 + 72x 96y + 144 = 0

(f) 36x2 + 20y 2 144x + 120y 396 = 0


Answer:

PY

(d)

Center

Vertices

Covertices

Foci

(a)

(0, 0)

(13, 0)

(0, 5)

(12, 0)

(b)

(0, 0)

(12, 0)

(0, 5)

(c)

(0, 0)

(0, 13)

( 13, 0)

(0, 2)

(3, 0)

(d)

(7, 4)

(7, 1)

(11, 4)

(7, 1)

(7, 9)

(3, 4)

(7, 7)

(4 7, 3)

EP
E

C
O

Item

(e)

(8, 3)

(4, 0)

(0, 3)

(4, 6)

(2 2 5, 3)

(2, 3)

(2, 9)

(2, 3)

(2, 3)

(2, 7)
(2, 1)

(f)

(4, 3)

(a)

(b)

(c)

43
All rights reserved. No part of this material may be reproduced or transmitted in any form or by any means electronic or mechanical including photocopying without written permission from the DepEd Central Office. First Edition, 2016.

(d)

(e)

(f)

PY

2. Find the standard equation of the ellipse which satisfies the given conditions.

EP
E

C
O

(a) foci (7, 6) and (1, 6), the sum of the distances of any point from the
(x + 4)2 (y 6)2
foci is 14
Answer:
+
=1
49
40
(b) center (5, 3), horizontal major axis of length 20, minor axis of length 16
(x 5)2 (y 3)2
+
=1
Answer:
100
64
(c) major axis of length 22, foci 9 units above and below the center (2, 4)
(x 2)2 (y 4)2
+
=1
Answer:
40
121
(d) covertices (4, 8) and (10, 8), a focus at (3, 12)
(x 3)2 (y 8)2
Answer:
+
=1
49
65
Solution. The midpoint of the covertices is the center, (3, 8). From this
point, the given focus is c = 4 units away. Since b = 7 (the distance from
the center to a covertex), then a2 = b2 + c2 = 65. The ellipse then has
(x 3)2 (y 8)2
+
= 1.
equation
49
65

(e) focus (6, 2), covertex (1, 5), horizontal major axis
(x + 1)2 (y + 2)2
Answer:
+
=1
74
49
Solution. Make a rough sketch of the points to see that the center is to
the right of the given focus, and below the given covertex. The center is
thus (1, 2). It follows that c = 5, b = 7, so a2 = b2 + c2 = 74. The
(x + 1)2 (y + 2)2
ellipse then has equation
+
= 1.
74
49

3. A semielliptical tunnel has height 9 ft and a width of 30 ft. A truck that is


about to pass through is 12 ft wide and 8.3 ft high. Will this truck be able to
pass through the tunnel?
Answer: No

44
All rights reserved. No part of this material may be reproduced or transmitted in any form or by any means electronic or mechanical including photocopying without written permission from the DepEd Central Office. First Edition, 2016.

4. A truck that is about to pass through the tunnel from the previous item is 10
ft wide and 8.3 ft high. Will this truck be able to pass through the tunnel?
Answer: Yes
5. An orbit of a satellite around a planet is an ellipse, with the planet at one
focus of this ellipse. The distance of the satellite from this star varies from
300, 000 km to 500, 000 km, attained when the satellite is at each of the two
vertices. Find the equation of this ellipse, if its center is at the origin, and the
vertices are on the x-axis. Assume all units are in 100, 000 km.
Answer:

x2
16

y2
15

=1

x2

640,000

PY

6. The orbit of a planet around a star is described by the equation

y2

C
O

= 1, where the star is at one focus, and all units are in millions of
630,000
kilometers. The planet is closest and farthest from the star, when it is at the
vertices. How far is the planet when it is closest to the sun? How far is the
planet when it is farthest from the sun?
Answer: 700 million km, 900 million km

EP
E

Solution. The ellipse has center at the origin, and major axis on the x-axis.
Since a2 = 640, 000, then a = 800, so
the verticesare V1 (800, 0) and
2
V2 (800, 00). Since b = 630, 000, then c = a2 b2 = 10, 000 = 100. Suppose the star is at the focus at the right of the origin (this choice is arbitrary,
since we could have chosen instead the focus on the left). Its location is then
F (100, 0). The closest distance is then V2 F = 700 (million kilometers) and
the farthest distance is V1 F = 900 (million kilometers).

7. A big room is constructed so that the ceiling is a dome that is semielliptical


in shape. If a person stands at one focus and speaks, the sound that is made
bounces off the ceiling and gets reflected to the other focus. Thus, if two
people stand at the foci (ignoring their heights), they will be able to hear each
other. If the room is 34 m long and 8 m high, how far from the center should
each of two people stand if they would like to whisper back and forth and hear
each other?
Answer: 15 m

45
All rights reserved. No part of this material may be reproduced or transmitted in any form or by any means electronic or mechanical including photocopying without written permission from the DepEd Central Office. First Edition, 2016.

Solution. We could put a coordinate system with the floor of the room on
the x-axis, and the center of the room at the origin, as shown in the figures.
The major axis has length 34, and the height of the room is half of the minor
y2
x2
axis. The ellipse that contains the ceiling then has equation 17
2 + 82 = 1. The

distance of a focus from the center is c = a2 b2 = 172 82 = 15. Thus,


the two people should stand 15 m away from the center.
8. A whispering gallery has a semielliptical ceiling that is 9 m high and 30 m
long. How high is the ceiling above the two foci?
Answer: 5.4 m

C
O

PY

Solution. As in the previous problem, put a coordinate system with the floor
of the room on the x-axis, and the center of the room at the origin. The major
axis has length 30, and half the minor axis is 9. The ellipse that contains the
y2
x2
ceiling then has equation 15
2 + 92 = 1. The distance of a focus from the center

is c = a2 b2 = 152 92 = 12. If we put x = 12 in the equation of the


y2
27
122
ellipse, we get 15
2 + 92 = 1. Solving for y > 0 yields y = 5 = 5.4. The height
of the ceiling above each focus is 5.4 m.

EP
E

Lesson 1.4. Hyperbolas

Time Frame: 3 one-hour sessions

Learning Outcomes of the Lesson


At the end of the lesson, the student is able to:

(1) define a hyperbola;

(2) determine the standard form of equation of a hyperbola;


(3) graph a hyperbola in a rectangular coordinate system; and
(4) solve situational problems involving conic sections (hyperbolas).
Lesson Outline
(1) Definition of a hyperbola
(2) Derivation of the standard equation of a hyperbola
(3) Graphing hyperbolas
(4) Solving situational problems involving hyperbolas

46
All rights reserved. No part of this material may be reproduced or transmitted in any form or by any means electronic or mechanical including photocopying without written permission from the DepEd Central Office. First Edition, 2016.

Introduction
Just like ellipse, a hyperbola is one of the conic sections that most students
have not encountered formally before. Its graph consists of two unbounded
branches which extend in opposite directions. It is a misconception that each
branch is a parabola. This is not true, as parabolas and hyperbolas have very
different features. An application of hyperbolas in basic location and navigation
schemes are presented in an example and some exercises.
1.4.1. Definition and Equation of a Hyperbola

PY

Consider the points F1 (5, 0) and F2 (5, 0) as shown in Figure 1.21. What is the
absolute value of the difference of the distances of A(3.75, 3) from F1 and from
F2 ? How about the absolute value of the difference of the distances of B 5, 16
3
from F1 and from F2 ?

C
O

|AF1 AF2 | = |9.25 3.25| = 6




16 34

|BF1 BF2 | = = 6
3
3

EP
E

There are other points P such that |P F1 P F2 | = 6. The collection of all such
points forms a shape called a hyperbola, which consists of two disjoint branches.
For points P on the left branch, P F2 P F1 = 6; for those on the right branch,
P F1 P F2 = 6.

Figure 1.21

Figure 1.22

Let F1 and F2 be two distinct points. The set of all points P , whose
distances from F1 and from F2 differ by a certain constant, is called a
hyperbola. The points F1 and F2 are called the foci of the hyperbola.
In Figure 1.22, given are two points on the x-axis, F1 (c, 0) and F2 (c, 0), the
foci, both c units away from their midpoint (0, 0). This midpoint is the center
47
All rights reserved. No part of this material may be reproduced or transmitted in any form or by any means electronic or mechanical including photocopying without written permission from the DepEd Central Office. First Edition, 2016.

Figure 1.24

PY

Figure 1.23

C
O

of the hyperbola. Let P (x, y) be a point on the hyperbola, and let the absolute
value of the difference of the distances of P from F1 and F2 , be 2a (the coefficient
2 will make computations simpler). Thus, |P F1 P F2 | = 2a, and so
p

p


2
2
2
2
(x + c) + y (x c) + y = 2a.
Algebraic manipulations allow us to rewrite this into the much simpler
where b =

c2 a2 .

x2 y 2
2 = 1,
a2
b

Now we present a derivation. For now, assume P is closer to F2 so P F1 > P F2 ,


and P F1 P F2 = 2a.
P F1 = 2a + P F2
p
(x + c)2 + y 2 = 2a + (x c)2 + y 2
p
2 
2
p
2
2
2
2
(x + c) + y
= 2a + (x c) + y
p
cx a2 = a (x c)2 + y 2
 p
2
(cx a2 )2 = a (x c)2 + y 2
p

Teaching Notes
If it is assumed
that P is closer to
F1 , then the same
equation will be
obtained because
of symmetry.

EP
E

When we let b = c2 a2 , we assumed c > a. To see why this is true, suppose


that P is closer to F2 , so P F1 P F2 = 2a. Refer to Figure 1.22. Suppose also
that P is not on the x-axis, so 4P F1 F2 is formed. From the triangle inequality,
F1 F2 + P F2 > P F1 . Thus, 2c > P F1 P F2 = 2a, so c > a.

(c2 a2 )x2 a2 y 2 = a2 (c2 a2 )


b 2 x 2 a2 y 2 = a2 b2
x2 y 2
2 =1
a2
b

by letting b =

c2 a2 > 0

48
All rights reserved. No part of this material may be reproduced or transmitted in any form or by any means electronic or mechanical including photocopying without written permission from the DepEd Central Office. First Edition, 2016.

We collect here the features of the graph of a hyperbola with standard equation
x2 y 2
2 = 1.
a2
b

2
2
Let c = a + b .
(1) center : origin (0, 0)
(2) foci : F1 (c, 0) and F2 (c, 0)
Each focus is c units away from the center.

PY

For any point on the hyperbola, the absolute value of the difference of
its distances from the foci is 2a.
(3) vertices: V1 (a, 0) and V2 (a, 0)

C
O

The vertices are points on the hyperbola, collinear with the center and
foci.
If y = 0, then x = a. Each vertex is a units away from the center.
The segment V1 V2 is called the transverse axis. Its length is 2a.

(4) asymptotes: y = ab x and y = ab x, the lines `1 and `2 in Figure 1.24

EP
E

The asymptotes of the hyperbola are two lines passing through the center which serve as a guide in graphing the hyperbola: each branch of
the hyperbola gets closer and closer to the asymptotes, in the direction
towards which the branch extends. (We need the concept of limits from
calculus to explain this.)
An aid in determining the equations of the asymptotes: in the standard
2
2
equation, replace 1 by 0, and in the resulting equation xa2 yb2 = 0, solve
for y.

To help us sketch the asymptotes, we point out that the asymptotes


`1 and `2 are the extended diagonals of the auxiliary rectangle drawn
in Figure 1.24. This rectangle has sides 2a and 2b with its diagonals
intersecting at the center C. Two sides are congruent and parallel to
the transverse axis V1 V2 . The other two sides are congruent and parallel
to the conjugate axis, the segment shown which is perpendicular to the
transverse axis at the center, and has length 2b.

Example 1.4.1. Determine the foci, vertices, and asymptotes of the hyperbola
with equation
x2 y 2

= 1.
9
7
Sketch the graph, and include these points and lines, the transverse and conjugate
axes, and the auxiliary rectangle.
49
All rights reserved. No part of this material may be reproduced or transmitted in any form or by any means electronic or mechanical including photocopying without written permission from the DepEd Central Office. First Edition, 2016.

2
2
Solution. With
a = 9 and
b = 7, we have
a = 3, b = 7, and c = a2 + b2 = 4.
foci: F1 (4, 0) and F2 (4, 0)

vertices: V1 (3, 0) and V2 (3, 0)

asymptotes: y = 37 x and y = 37 x
The graph is shown at the right. The conju
gate axis drawn has its endpoints b = 7
2.7 units above and below the center.
2

PY

Example 1.4.2. Find the (standard) equation of the hyperbola whose foci are
F1 (5, 0) and F2 (5, 0), such that for any point on it, the absolute value of the
difference of its distances from the foci is 6. See Figure 1.21.

Seatwork/Homework 1.4.1

c2 a2 = 4. The
2

C
O

Solution. We have 2a = 6 and c = 5, so a = 3 and b =


x2 y 2
hyperbola then has equation

= 1.
9
16

1. Determine foci, vertices, and asymptotes of the hyperbola with equation

EP
E

y2
x2

= 1.
16 20
Sketch the graph, and include these points and lines, along with the auxiliary
rectangle.

Answer: foci
F1 (6, 0) and
F2 (6, 0), vertices V1 (4, 0) and V2 (4, 0), asymp

5
5
totes y = 2 x and y = 2 x

2. Find the equation


in
standard
form
of
the
hyperbola
whose
foci
are
F
(4
2, 0)
1

and F2 (4 2, 0), such that for any point on it, the absolute value of the
x2
y2
difference of its distances from the foci is 8.
Answer:

=1
16 16
50
All rights reserved. No part of this material may be reproduced or transmitted in any form or by any means electronic or mechanical including photocopying without written permission from the DepEd Central Office. First Edition, 2016.

1.4.2. More Properties of Hyperbolas


The hyperbolas we considered so far are horizontal and have the origin as their
centers. Some hyperbolas have their foci aligned vertically, and some have centers
not at the origin. Their standard equations and properties are given in the box.
The derivations are more involved, but are similar to the one above, and so are
not shown anymore.
Corresponding Hyperbola

(0, 0)

C
O

PY

Center

y 2 x2
2 =1
a2
b

EP
E

x2 y 2
2 =1
a2
b

(h, k)

(x h)2 (y k)2

=1
a2
b2

(y k)2 (x h)2

=1
a2
b2

transverse axis: horizontal

transverse axis: vertical

conjugate axis: vertical

conjugate axis: horizontal

51
All rights reserved. No part of this material may be reproduced or transmitted in any form or by any means electronic or mechanical including photocopying without written permission from the DepEd Central Office. First Edition, 2016.

PY

In all four cases above, we let c = a2 + b2 . The foci F1 and F2 are c units
away from the center C. The vertices V1 and V2 are a units away from the center.
The transverse axis V1 V2 has length 2a. The conjugate axis has length 2b and is
perpendicular to the transverse axis. The transverse and conjugate axes bisect
each other at their intersection point, C. Each branch of a hyperbola gets closer
and closer to the asymptotes, in the direction towards which the branch extends.
The equations of the asymptotes can be determined by replacing 1 in the standard
equation by 0. The asymptotes can be drawn as the extended diagonals of the
auxiliary rectangle determined by the transverse and conjugate axes. Recall that,
for any point on the hyperbola, the absolute value of the difference of its distances
from the foci is 2a.

C
O

In the standard equation, aside from being positive, there are no other restrictions on a and b. In fact, a and b can even be equal. The orientation of the
hyperbola is determined by the variable appearing in the first term (the positive
term): the corresponding axis is where the two branches will open. For example,
if the variable in the first term is x, the hyperbola is horizontal: the transverse
axis is horizontal, and the branches open to the left and right in the direction of
the x-axis.

Example 1.4.3. Give the coordinates of the center, foci, vertices, and asymptotes of the hyperbola with the given equation. Sketch the graph, and include
these points and lines, the transverse and conjugate axes, and the auxiliary rectangle.
(y + 2)2 (x 7)2

=1
25
9
(2) 4x2 5y 2 + 32x + 30y = 1

EP
E

(1)

2
2
Solution.
(1) From
a = 25 and b = 9, we have a = 5, b = 3, and c =

a2 + b2 = 34 5.8. The hyperbola is vertical. To determine the asymp2


(x7)2
totes, we write (y+2)

= 0, which is equivalent to y + 2 = 53 (x 7).


25
9
We can then solve this for y.

center: C(7, 2)
foci: F1 (7, 2

34) (7, 7.8) and F2 (7, 2 +

vertices: V1 (7, 7) and V2 (7, 3)


asymptotes: y = 35 x

41
3

and y = 35 x +

34) (7, 3.8)

29
3

The conjugate axis drawn has its endpoints b = 3 units to the left and right
of the center.

52
All rights reserved. No part of this material may be reproduced or transmitted in any form or by any means electronic or mechanical including photocopying without written permission from the DepEd Central Office. First Edition, 2016.

PY
C
O

(2) We first change the given equation to standard form.

EP
E

4(x2 + 8x) 5(y 2 6y) = 1


4(x2 + 8x + 16) 5(y 2 6y + 9) = 1 + 4(16) 5(9)
4(x + 4)2 5(y 3)2 = 20
(x + 4)2 (y 3)2

=1
5
4

We have a = 5 2.2 and b = 2. Thus, c = a2 + b2 = 3. The hyperbola


2
2
is horizontal. To determine the asymptotes, we write (x+4)
(y3)
= 0
5
4
2

which is equivalent to y 3 = 5 (x + 4), and solve for y.


center: C(4, 3)
foci: F1 (7, 3) and F2 (1, 3)

vertices: V1 (4 5, 3) (6.2, 3) and V2 (4 + 5, 3) (1.8, 3)

asymptotes: y =

2 x
5

8
5

+ 3 and y = 25 x

8
5

+3

The conjugate axis drawn has its endpoints b = 2 units above and below
the center.

53
All rights reserved. No part of this material may be reproduced or transmitted in any form or by any means electronic or mechanical including photocopying without written permission from the DepEd Central Office. First Edition, 2016.

PY
C
O

EP
E

Example 1.4.4. The foci of a hyperbola are (5, 3) and (9, 3). For any point
on the hyperbola, the absolute value of the difference of its of its distances from
the foci is 10. Find the standard equation of the hyperbola.
Solution. The midpoint (2, 3) of the foci is the center of the hyperbola. Each
focus is c = 7 units away from the center. From the given difference, 2a = 10 so
a = 5. Also, b2 = c2 a2 = 24. The hyperbola is horizontal (because the foci are
horizontally aligned), so the equation is
2

(x 2)2 (y + 3)2

= 1.
25
24

Example 1.4.5.
A hyperbola has vertices (4, 5) and (4, 9), and one of its
foci is (4, 2 65). Find its standard equation.

Solution. The midpoint (4, 2) of the vertices is the center of the hyperbola,
which is vertical (because the vertices are vertically aligned).
Each vertex is

a = 7 units away from the center. The given focus is c = 65 units away from
the center. Thus, b2 = c2 a2 = 16, and the standard equation is
(y 2)2 (x + 4)2

= 1.
49
16

54
All rights reserved. No part of this material may be reproduced or transmitted in any form or by any means electronic or mechanical including photocopying without written permission from the DepEd Central Office. First Edition, 2016.

Seatwork/Homework 1.4.2

EP
E

C
O

PY

1. Give the coordinates of the center, foci, vertices, and asymptotes of the hyperbola with equation 9x2 4y 2 90x 32y = 305. Sketch the graph, and
include these points and lines, along with the auxiliary rectangle.

Answer: center C(5, 4), foci F1 (5, 42 13) and F2 (5, 4+2 13), vertices
V1 (5, 10) and V2 (5, 2), asymptotes y = 32 x + 72 and y = 23 x 23
2

2. A hyperbola has vertices (1, 9) and (13, 9), and one of its foci is (2, 9). Find
(x 7)2 (y 9)2
its standard equation.
Answer:

=1
36
45

1.4.3. Situational Problems Involving Hyperbolas


Let us now give an example on an application of hyperbolas.
Example 1.4.6. An explosion was heard by two stations 1200 m apart, located
at F1 (600, 0) and F2 (600, 0). If the explosion was heard in F1 two seconds before
it was heard in F2 , identify the possible locations of the explosion. Use 340 m/s
as the speed of sound.
Solution. Using the given speed of sound, we can deduce that the sound traveled
340(2) = 680 m farther in reaching F2 than in reaching F1 . This is then the
difference of the distances of the explosion from the two stations. Thus, the
explosion is on a hyperbola with foci are F1 and F2 , on the branch closer to F1 .
55
All rights reserved. No part of this material may be reproduced or transmitted in any form or by any means electronic or mechanical including photocopying without written permission from the DepEd Central Office. First Edition, 2016.

PY

Seatwork/Homework 1.4.3

C
O

We have c = 600 and 2a = 680, so a = 340 and b2 = c2 a2 = 244400.


The explosion could therefore be anywhere on the left branch of the hyperbola
y2
x2
244400
= 1.
2
115600

EP
E

? 1. Two stations, located at M (1.5, 0) and N (1.5, 0) (units are in km), simultaneously send sound signals to a ship, with the signal traveling at the speed of
0.33 km/s. If the signal from N was received by the ship four seconds before
the signal it received from M , find the equation of the curve containing the
y2
x2
possible location of the ship.
Answer: 0.4356
1.8144
= 1 (right branch)
Exercises 1.4

1. Give the coordinates of the center, foci, vertices, and the asymptotes of the
hyperbola with the given equation. Sketch the graph, and include these points
and lines.
x2
y2

=1
36 64
y2
x2
(b)

=1
25 16
(c) (x 1)2 y 2 = 4
(a)

(y + 2)2 (x + 3)2

=1
15
10
(e) 3x2 2y 2 42x 16y = 67

(d)

(f) 25x2 39y 2 + 150x + 390y = 225


56

All rights reserved. No part of this material may be reproduced or transmitted in any form or by any means electronic or mechanical including photocopying without written permission from the DepEd Central Office. First Edition, 2016.

Answer:
Item

Center

Vertices

Foci

(a)

(0, 0)

(6, 0)

(b)

(0, 0)

(0, 5)

(c)

(1, 0)

(d)

(3, 2)

(1, 0), (3, 0)

(3, 2 15)

(10, 0)

(0, 41)

(1 2 2, 0)

(e)

(7, 4)

(3, 4), (11, 4)

(f)

(3, 5)

(3, 0), (3, 10)

(a)
(b)
(c)

(e)

y = 34 x

y = 54 x

y = x 1, y = x + 1
q
q
y = 32 x 3 32 2
q
q
3
y = 2 x 7 32 4
y = 539 x

15
39

+5

EP
E

(f)

PY

Asymptotes

(d)

(3, 3), (3, 13)

C
O

Item

(3, 7), (3, 3)

(7 2 10, 4)

(a)

(b)

57
All rights reserved. No part of this material may be reproduced or transmitted in any form or by any means electronic or mechanical including photocopying without written permission from the DepEd Central Office. First Edition, 2016.

PY
(d)

C
O

(c)

EP
E

(e)
(f)
2. Find the standard equation of the hyperbola which satisfies the given conditions.

(a) foci (4, 3) and (4, 13), the absolute value of the difference of the
distances of any point from the foci is 14
(y 5)2 (x + 4)2

=1
Answer:
49
15
(b) vertices (2, 8) and (8, 8), a focus (12, 8)
(x 3)2 (y 8)2
Answer:

=1
25
56
(c) center (6, 9), a vertex (6, 15), conjugate axis of length 12
(y 9)2 (x + 6)2
Answer:

=1
25
36
(d) asymptotes y = 34 x + 13 and y = 43 x + 41
, a vertex (1, 7)
3
(x 5)2 (y 7)2
Answer:

=1
36
64
Solution. The asymptotes intersect at (5, 7). This is the center. The
distance of the given vertex from the center is a = 6. This vertex and
center are aligned horizontally, so the hyperbola has equation of the form
2
(xh)2
(yk)
= 1. The asymptotes consequently have the form y k =
a2
b2
58

All rights reserved. No part of this material may be reproduced or transmitted in any form or by any means electronic or mechanical including photocopying without written permission from the DepEd Central Office. First Edition, 2016.

ab (x h), and thus, have slopes ab . From the given asymptotes,


Since a = 6, then b = 8. The standard equation is then

b
a

= 43 .

(x 5)2 (y 7)2

= 1.
36
64
and y = 31 x + 37 , a focus (1, 12)
(y 2)2 (x 1)2
Answer:

=1
10
90
Solution. The asymptotes intersect at (1, 2). This is the center. The
distance of the given focus from the center is c = 10. This focus and
center are aligned vertically, so the hyperbola has equation of the form
2
(yk)2
(xh)
= 1. The asymptotes consequently have the form y k =
a2
b2
a
b (x h), and thus, have slopes ab . From the given asymptotes, ab = 13 ,
so b = 3a.
c2 = 100 = a2 + b2 = a2 + (3a)2 = 10a2

5
3

PY

(e) asymptotes y = 31 x +

C
O

Thus, a2 = 10, and b2 = 9a2 = 90. The standard equation is


(y 2)2 (x 1)2

= 1.
10
90

EP
E

3. Two control towers are located at points Q(500, 0) and R(500, 0), on a
straight shore where the x-axis runs through (all distances are in meters).
At the same moment, both towers sent a radio signal to a ship out at sea, each
traveling at 300 m/s. The ship received the signal from Q 3 s (microseconds)
before the message from R.
(a) Find the equation of the curve containing the possible location of the
y2
x2

= 1 (left branch)
ship.
Answer:
202500 47500
(b) Find the coordinates (rounded off to two decimal places) of the ship if it
is 200 m from the shore (y = 200).
Answer: (610.76, 200)

Solution. Since the time delay between the two signals is 3 s, then the difference between the distances traveled by the two signals is 300 3 = 900 m. The
ship is then on a hyperbola, consisting of points whose distances from Q and R
(the foci) differ by 2a = 900. With a = 450 and c = 500 (the distance of each
focus from the center, the origin), we have b2 = c2 a2 = 5002 4502 = 47500.
y2
x2
Since a2 = 202500, the hyperbola then has equation 202500
47500
= 1. Since
the signal from Q was received first, the ship is closer to Q than R, so the
ship is on the left branch of this hyperbola. Using y = 200, we then solve
x2
2002
47500
= 1 for x < 0 (left branch), and we get x 610.76.
202500

59
All rights reserved. No part of this material may be reproduced or transmitted in any form or by any means electronic or mechanical including photocopying without written permission from the DepEd Central Office. First Edition, 2016.

Lesson 1.5. More Problems on Conic Sections


Time Frame: 2 one-hour sessions
Learning Outcomes of the Lesson
At the end of the lesson, the student is able to:
(1) recognize the equation and important characteristics of the different types of
conic sections; and
(2) solve situational problems involving conic sections.

PY

Lesson Outline

(1) Conic sections with associated equations in general form

(2) Problems involving characteristics of various conic sections

Introduction

C
O

(3) Solving situational problems involving conic sections

In this lesson, we will identify the conic section from a given equation. We
will analyze the properties of the identified conic section. We will also look at
problems that use the properties of the different conic sections. This will allow
us to synthesize what has been covered so far.

EP
E

1.5.1. Identifying the Conic Section by Inspection


The equation of a circle may be written in standard form
Ax2 + Ay 2 + Cx + Dy + E = 0,

that is, the coefficients of x2 and y 2 are the same. However, it does not follow
that if the coefficients of x2 and y 2 are the same, the graph is a circle.

General Equation
Standard Equation
graph


1 2
3 2
2
2
2x + 2y 2x + 6y + 5 = 0 x 2 + y + 2 = 0
point
2
2
2
2
x + y 6x 8y + 50 = 0 (x 3) + (y 4) = 25 empty set

(A)
(B)

For a circle with equation (x h)2 + (y k)2 = r2 , we have r2 > 0. This is


not the case for the standard equations of (A) and (B).
In (A), because the sum of two squares can only be 0 if and only if each square
is 0, it follows that x 21 = 0 and y + 32 = 0. The graph is thus the single point
1
, 32 .
2

In (B), no real values of x and y can make the nonnegative left side equal to
the negative right side. The graph is then the empty set.
60

All rights reserved. No part of this material may be reproduced or transmitted in any form or by any means electronic or mechanical including photocopying without written permission from the DepEd Central Office. First Edition, 2016.

Let us recall the general form of the equations of the other conic sections. We
may write the equations of conic sections we discussed in the general form
Ax2 + By 2 + Cx + Dy + E = 0.
Some terms may vanish, depending on the kind of conic section.
(1) Circle: both x2 and y 2 appear, and their coefficients are the same
Ax2 + Ay 2 + Cx + Dy + E = 0
Example: 18x2 + 18y 2 24x + 48y 5 = 0
(2) Parabola: exactly one of x2 or y 2 appears

PY

Degenerate cases: a point, and the empty set

C
O

Ax2 + Cx + Dy + E = 0 (D 6= 0, opens upward or downward)


By 2 + Cx + Dy + E = 0 (C 6= 0, opens to the right or left)
Examples: 3x2 12x + 2y + 26 = 0 (opens downward)
2y 2 + 3x + 12y 15 = 0 (opens to the right)

(3) Ellipse: both x2 and y 2 appear, and their coefficients A and B have the same
sign and are unequal

Examples: 2x2 + 5y 2 + 8x 10y 7 = 0 (horizontal major axis)


4x2 + y 2 16x 6y + 21 = 0 (vertical major axis)

EP
E

If A = B, we will classify the conic as a circle, instead of an ellipse.


Degenerate cases: a point, and the empty set

(4) Hyperbola: both x2 and y 2 appear, and their coefficients A and B have different signs

Examples: 5x2 3y 2 20x 18y 22 = 0 (horizontal transverse axis)


4x2 + y 2 + 24x + 4y 36 = 0 (vertical transverse axis)
Degenerate case: two intersecting lines

The following examples will show the possible degenerate conic (a point, two
intersecting lines, or the empty set) as the graph of an equation following a similar
pattern as the non-degenerate cases.
(1) 4x2 + 9y 2 16x + 18y + 25 = 0

=
=

(2) 4x2 + 9y 2 16x + 18y + 61 = 0

=
=

(x 2)2 (y + 1)2
+
=0
32
22
one point: (2, 1)
(x 2)2 (y + 1)2
+
= 1
32
22
empty set

61
All rights reserved. No part of this material may be reproduced or transmitted in any form or by any means electronic or mechanical including photocopying without written permission from the DepEd Central Office. First Edition, 2016.

(3) 4x2 9y 2 16x 18y + 7 = 0

=
=

(x 2)2 (y + 1)2

=0
32
22
2
two lines: y + 1 = (x 2)
3

A Note on Identifying a Conic Section


by Its General Equation

PY

It is only after transforming a given general equation to standard


form that we can identify its graph either as one of the degenerate
conic sections (a point, two intersecting lines, or the empty set) or as
one of the non-degenerate conic sections (circle, parabola, ellipse, or
hyperbola).

Seatwork/Homework 1.5.1

(1) 5x2 3y 2 + 10x 12y = 22


(2) 2y 2 5x 12y = 17

C
O

The graphs of the following equations are (nondegenerate) conic sections. Identify
the conic section.

(3) 3x2 + 3y 2 + 42x 12y = 154


2
2

(4) 3x + 6x + 4y = 18

(5) 7x + 3y 14x + 12y = 14

EP
E

(6) 4x2 + 3y 2 + 24x 12y = 36

Answer: hyperbola
Answer: parabola
Answer: circle
Answer: parabola
Answer: ellipse
Answer: hyperbola

1.5.2. Problems Involving Different Conic Sections


The following examples require us to use the properties of different conic sections
at the same time.

Example 1.5.1. A circle has center at the focus of the parabola y 2 + 16x + 4y =
44, and is tangent to the directrix of this parabola. Find its standard equation.
Solution. The standard equation of the parabola is (y + 2)2 = 16(x 3). Its
vertex is V (3, 2). Since 4c = 16 or c = 4, its focus is F (1, 2) and its directrix
is x = 7. The circle has center at (1, 2) and radius 8, which is the distance
from F to the directrix. Thus, the equation of the circle is
(x + 1)2 + (y + 2)2 = 64.

Example 1.5.2. The vertices and foci of 5x2 4y 2 + 50x + 16y + 29 = 0 are,
respectively, the foci and vertices of an ellipse. Find the standard equation of
this ellipse.
62
All rights reserved. No part of this material may be reproduced or transmitted in any form or by any means electronic or mechanical including photocopying without written permission from the DepEd Central Office. First Edition, 2016.

Solution. We first write the equation of the hyperbola in standard form:


(x + 5)2 (y 2)2

= 1.
16
20
For this hyperbola, using the notations ah , bh , and ch to refer to a, b, and
c of
the standard
equation of the hyperbola, respectively, we have ah = 4, bh = 2 5,
p
2
2
ch = ah + bh = 6, so we have the following points:
center: (5, 2)

vertices: (9, 2) and (1, 2)

PY

foci: (11, 2) and (1, 2).


It means that, for the ellipse, we have these points:
center: (5, 2)
vertices: (11, 2) and (1, 2)

C
O

foci: (9, 2) and (1, 2).

In this case, ce = 4 and ae = 6, so that be =


equation of the ellipse is

a2e c2e = 20. The standard


2

(x + 5)2 (y 2)2
+
= 1.
36
20
Seatwork/Homework 1.5.2

EP
E

1. Find the standard equation of all circles having center at a focus of 21x2
4y 2 + 84x 24y = 36 and passing through the farther vertex.

Answer: (x + 7)2 + (y + 3)2 = 49, (x 3)2 + (y + 3)2 = 49

2. Find the standard equation of the hyperbola one branch of which has focus and
vertex that are the same as those of x2 6x + 8y = 23, and whose conjugate
axis is on the directrix of the same parabola.
(y 6)2 (x 3)2
Answer:

=1
4
12

Exercises 1.5
1. The graphs of the following equations are non-degenerate conic sections. Identify the conic section.
(a) 5x2 + 7y 2 40x 28y = 73

Answer: ellipse

(b) 5y + 2x 30y = 49

Answer: parabola

(c) 3x2 3y 2 + 12x 12y = 5

Answer: hyperbola

(d) 3x2 + 3y 2 + 12x + 12y = 4

Answer: circle
63

All rights reserved. No part of this material may be reproduced or transmitted in any form or by any means electronic or mechanical including photocopying without written permission from the DepEd Central Office. First Edition, 2016.

(e) 2x2 + 24x 5y = 57

Answer: parabola

2. The graphs of the following equations are degenerate conic sections. What are
the specific graphs?
(a) x2 + 3y 2 4x + 24y = 52

Answer: point: (2, 4)

Answer: lines: y + 2 = 32 (x + 1)

(b) 9x2 4y 2 + 18x 16y = 7

(c) 3x2 + 5y 2 6x 20y = 25

Answer: empty set

PY

3. An ellipse has equation 25x2 + 16y 2 + 150x 32y = 159. Find the standard
equations of all parabolas whose vertex is a focus of this ellipse and whose
focus is a vertex of this ellipse.
Answer: (x + 3)2 = 8(y + 2), (x + 3)2 = 32(y + 2), (x + 3)2 = 32(y 4),
and (x + 3)2 = 8(y 4)

C
O

Solution. The standard equation of the ellipse is

(x + 3)2 (y 1)2
+
= 1.
16
25

focus

focal distance

standard equation

Q(3, 2) P (3, 4)

(x + 3)2 = 8(y + 2)

Q(3, 2)

S(3, 6)

(x + 3)2 = 32(y + 2)

R(3, 4)

P (3, 4)

(x + 3)2 = 32(y 4)

R(3, 4)

S(3, 6)

EP
E

vertex

Its center is (3, 1). Since a = 5 and b = 4, we get c = 3, so the vertices are
P (3, 4) and S(3, 6), while its foci are Q(3, 2) and R(3, 4). We then
get four parabolas satisfying the conditions of the problem. The focal distance
indicated below is the distance from the vertex to the focus.

(x + 3)2 = 8(y 4)

4. Find the standard equation of the hyperbola whose conjugate axis is on the
directrix of the parabola y 2 + 12x + 6y = 39, having the focus of the parabola
as one of its foci, and the vertex of the parabola as one of its vertices.
(x 7)2 (y + 3)2
Answer:

=1
9
27
Solution. The standard equation of the parabola is (y + 3)2 = 12(x 4), so
its vertex is V (4, 3), and it opens to the left. With 4c = 12, or c = 3, its
focus is F (1, 3), and its directrix is x = 7. The hyperbola has its center on
64
All rights reserved. No part of this material may be reproduced or transmitted in any form or by any means electronic or mechanical including photocopying without written permission from the DepEd Central Office. First Edition, 2016.

x = 7, its conjugate axis, and a vertex at (4, 3). Its center is then C(7, 3).
The conjugate axis is vertical so the hyperbola is horizontal, with constants
ah = CV = 3 and ch = CF = 6, so b2h = c2h a2h = 27. The standard equation
of the required hyperbola is
(x 7)2 (y + 3)2

= 1.
9
27

Solution. The standard form of the ellipse is

PY

5. Find the standard equation of the parabola opening to the left whose axis
contains the major axis of the ellipse x2 + 4y 2 10x 24y + 45 = 0, whose
focus is the center of the ellipse, and which passes through the covertices of
this ellipse.
Answer: (y 3)2 = 4(x 6)
(x 5)2 (y 3)2
+
= 1.
16
4

C
O

Its center (5, 3) is the focus of the parabola. Since b = 2, its covertices are
W1 (5, 1) and W2 (5, 5). The vertex of the parabola, c units to the right of (5, 3),
is (5 + c, 3). Its equation can be written as (y 3)2 = 4c(x (5 + c)). Since
(5, 5) is a point on this parabola, we have (5 3)2 = 4c(5 (5 + c)). Solving
this equation for c > 0 yields c = 1. Therefore, the standard equation of the
required parabola is (y 3)2 = 4(x 6).

EP
E

6. Find the standard equation of the ellipse whose major and minor axes are the
transverse and conjugate axes (not necessarily in that order) of the hyperbola
(x 2)2 (y + 3)2
+
=1
4x2 9y 2 16x 54y = 29.
Answer:
9
4
Solution. The standard equation of the hyperbola is
(y + 3)2 (x 2)2

= 1,
4
9

with center (2, 3), and constants ah = 2 and bh = 3. Since its conjugate axis
(which is horizontal and has length 2bh = 6) is longer than its transverse axis
(length 2ah = 4), the ellipse is horizontal. Its major axis has length 2ae = 6
and its minor axis has length 2be = 4, so ae = 3 and be = 2. The ellipse shares
the same center as the hyperbola. Thus, the standard equation of the required
ellipse is
(x 2)2 (y + 3)2
+
= 1.
9
4

7. If m 6= 3, 2, find the value(s) of m so that the graph of


(2m 4)x2 + (m + 3)y 2 = (m + 3)(2m 4)
is
65
All rights reserved. No part of this material may be reproduced or transmitted in any form or by any means electronic or mechanical including photocopying without written permission from the DepEd Central Office. First Edition, 2016.

(a) a circle,
(b) a horizontal ellipse,
(c) a vertical ellipse,
(d) a hyperbola (is it horizontal or vertical?), or
(e) the empty set.
Answer: (a) m = 7, (b) 2 < m < 7, (c) m > 7, (d) 3 < m < 2 (horizontal),
(e) m < 3

PY

Solution. It might be helpful to observe that the equation is equivalent to


y2
x2
+
= 1.
m + 3 2m 4

C
O

(a) The graph is a circle if m + 3 = 2m 4 > 0 (positive, so the graph is not


a point or the empty set). This happens if m = 7.
(b) We require 0 < 2m 4 < m + 3. Thus, 2 < m < 7.
(c) We require 0 < m + 3 < 2m 4. Thus, m > 7.

(d) We need m + 3 and 2m 4 to have different signs. We consider two cases.

EP
E

i. If m + 3 < 0 < 2m 4, then m < 3 AND m > 2, which cannot


happen.
ii. If 2m 4 < 0 < m + 3, then 3 < m < 2. In this case, the equation
can be written, with positive denominators, as
y2
x2

= 1.
m + 3 4 2m

The hyperbola is horizontal.

(e) The remaining case is when m < 3. In this case, m + 3 < 0 and
2m 4 < 0. This makes the expression
x2
y2
+
m + 3 2m 4

negative, and never equal to 1. The graph is then the empty set.

66
All rights reserved. No part of this material may be reproduced or transmitted in any form or by any means electronic or mechanical including photocopying without written permission from the DepEd Central Office. First Edition, 2016.

Lesson 1.6. Systems of Nonlinear Equations


Time Frame: 4 one-hour sessions
Learning Outcomes of the Lesson
At the end of the lesson, the student is able to:
(1) illustrate systems of nonlinear equations;
(2) determine the solutions of systems of nonlinear equations using techniques
such as substitution, elimination, and graphing; and

PY

(3) solve situational problems involving systems of nonlinear equations.


Lesson Outline
(1) Review systems of linear equations

C
O

(2) Solving a system involving one linear and one quadratic equation
(3) Solving a system involving two quadratic equations
(4) Applications of systems of nonlinear equations
Introduction

EP
E

After recalling the techniques used in solving systems of linear equations in


Grade 8, we extend these methods to solving a system of equations to systems
in which the equations are not necessarily linear. In this lesson, the equations
are restricted to linear and quadratic types, although it is possible to adapt the
methodology to systems with other types of equations. We focus on quadratic
equations for two reasons: to include a graphical representation of the solution
and to ensure that either a solution is obtained or it is determined that there is
no solution. The latter is possible because of the quadratic formula.

1.6.1. Review of Techniques in Solving Systems of Linear


Equations
Recall the methods we used to solve systems of linear equations. There were
three methods used: substitution, elimination, and graphical.
Example 1.6.1. Use the substitution method to solve the system, and sketch
the graphs in one Cartesian plane showing the point of intersection.

4x + y = 6
5x + 3y = 4

Teaching Notes
Recall that the
task of solving a
system of
equations is
equivalent to
finding points of
intersection.
Teaching Notes
Systems of linear
equations and
solving them were
introduced and
studied in Grade 8
at the last part of
Quarter I.

Solution. Isolate the variable y in the first equation, and then substitute into the
second equation.
67
All rights reserved. No part of this material may be reproduced or transmitted in any form or by any means electronic or mechanical including photocopying without written permission from the DepEd Central Office. First Edition, 2016.

4x + y = 6
= y = 6 4x

PY

5x + 3y = 4
5x + 3(6 4x) = 4
7x + 18 = 4
x=2
y = 6 4(2) = 2

C
O

Example 1.6.2. Use the elimination method to solve the system, and sketch the
graphs in one Cartesian plane showing the point of intersection.

2x + 7 = 3y
4x + 7y = 12

Solution. We eliminate first the variable x. Rewrite the first equation wherein
only the constant term is on the right-hand side of the equation, then multiply
it by 2, and then add the resulting equation to the second equation.

EP
E

2x 3y = 7
(2)(2x 3y) = (2)(7)
4x + 6y = 14
4x + 6y = 14
4x + 7y = 12

13y = 26
y=2
1
x=
2

Seatwork/Homework 1.6.1
Use either substitution or elimination method to solve the system, and sketch the
graphs in one Cartesian plane showing the point of intersection.

x 3y = 5
1.
2x + 5y = 1
68

All rights reserved. No part of this material may be reproduced or transmitted in any form or by any means electronic or mechanical including photocopying without written permission from the DepEd Central Office. First Edition, 2016.

C
O

EP
E

5x + 3y = 4
2.
3x + 5y = 9

Answer: 21 , 32

PY

Answer: (2, 1)

1.6.2. Solving Systems of Equations Using Substitution


We begin our extension with a system involving one linear equation and one
quadratic equation. In this case, it is always possible to use substitution by
solving the linear equation for one of the variables.
Example 1.6.3. Solve the following system, and sketch the graphs in one Cartesian plane.

xy+2=0
y 1 = x2
69

All rights reserved. No part of this material may be reproduced or transmitted in any form or by any means electronic or mechanical including photocopying without written permission from the DepEd Central Office. First Edition, 2016.

Solution. We solve for y in terms of x in the first equation, and substitute this
expression to the second equation.
xy+2=0

Solutions:

y =x+2

1+ 5
1+ 5
5+ 5
x=
= y =
+2=
2
2
2
1 5
1 5
5 5
x=
= y =
+2=
2
2
2

!
1+ 5 5+ 5
,
and
2
2

PY

y 1 = x2
(x + 2) 1 = x2
x2 x 1 = 0

1 5
x=
2

1 5 5 5
,
2
2

EP
E

C
O

The first equation represents a line with x-intercept 2 and y-intercept 2,


while the second equation represents a parabola with vertex at (0, 1) and which
opens upward.

Seatwork/Homework 1.6.2

Solve each system, and sketch the graphs in one Cartesian plane showing the
point(s) of intersection.

x2 + y 2 = 16
1.
xy =4

Answer: (4, 0) and (0, 4)


Solution. Solving for x in the second equation, we get x = y + 4. Substitute

70
All rights reserved. No part of this material may be reproduced or transmitted in any form or by any means electronic or mechanical including photocopying without written permission from the DepEd Central Office. First Edition, 2016.

this expression into the first equation.


x2 + y 2 = 16 =

(y + 4)2 + y 2 = 16
y 2 + 8y + 16 + y 2 = 16
2y 2 + 8y = 0
y = 0 or y = 4

y = 0 = x = 4

and

y = 4 = x = 0

EP
E

C
O

PY

Solutions: (4, 0) and (0, 4)

Teaching Notes
We substitute each
value of y (0 and
4) to the second
equation x y = 4
(or x = y + 4).

y = x2
2.
x = y2

Answer: (0, 0) and (1, 1)


Solution. Since the equations represent parabolas, we can use either of them
to isolate one variable. This is in fact the form in which both equations are
given. Substituting y = x2 into x = y 2 , we get
x = y 2 =

x = (x2 )2
x4 x = 0

x(x3 1) = 0
x = 0 or x = 1
71
All rights reserved. No part of this material may be reproduced or transmitted in any form or by any means electronic or mechanical including photocopying without written permission from the DepEd Central Office. First Edition, 2016.

x = 0 = y = 0

and

x = 1 = y = 1

C
O

PY

Solutions: (0, 0) and (1, 1)

1.6.3. Solving Systems of Equations Using Elimination


Elimination method is also useful in systems of nonlinear equations. Sometimes,
some systems need both techniques (substitution and elimination) to solve them.

Solution 1. We expand the second equation, and eliminate the variable x by


adding the equations.
4(3 x) = (y 3)2 = 12 4x = y 2 6y + 9 = y 2 + 4x 6y = 3

y 2 4x 6y = 11
y 2 + 4x 6y = 3

Teaching Notes
The variable y
could also be
eliminated first by
subtracting the
second equation
from the first.

EP
E

Example 1.6.4. Solve the following system:

y 2 4x 6y = 11
4(3 x) = (y 3)2 .

Adding these equations, we get

2y 2 12y = 14 = y 2 6y7 = 0 = (y7)(y+1) = 0 = y = 7 or y = 1.


Teaching Notes
We may actually
substitute y = 7
and y = 1 (one at
a time) into any of
the two given
equations, and
then solve for x.

Solving for x in the second equation, we have


(y 3)2
.
x=3
4
y = 7 = x = 1

and

y = 1 = x = 1

Solutions: (1, 7) and (1, 1)

72
All rights reserved. No part of this material may be reproduced or transmitted in any form or by any means electronic or mechanical including photocopying without written permission from the DepEd Central Office. First Edition, 2016.

C
O

PY

The graphs of the equations in the preceding example with the points of
intersection are shown below.

Usually, the general form is more convenient to use in solving systems of


equations. However, sometimes the solution can be simplified by writing the
equations in standard form. Moreover, the standard form is best for graphing.

Let us again solve the previous example in a different way.

EP
E

Solution 2. By completing the square, we can change the first equation into standard form:
y 2 4x 6y = 11 = 4(x + 5) = (y 3)2 .

4(x + 5) = (y 3)2
4(3 x) = (y 3)2

Using substitution or the transitive property of equality, we get


4(x + 5) = 4(3 x) = x = 1.

Substituting this value of x into the second equation, we have


4[3 (1)] = (y 3)2 = 16 = (y 3)2 = y = 7 or y = 1.

The solutions are (1, 7) and (1, 1), same as Solution 1.

Example 1.6.5. Solve the system and graph the curves:

(x 3)2 + (y 5)2 = 10
x2 + (y + 1)2 = 25.
73

All rights reserved. No part of this material may be reproduced or transmitted in any form or by any means electronic or mechanical including photocopying without written permission from the DepEd Central Office. First Edition, 2016.

Subtracting these two equations, we get

6x 12y + 48 = 0 = x + 2y 8 = 0
x = 8 2y.

PY

We can substitute x = 8 2y to either the first equation or the second equation.


For convenience, we choose the second equation.
x2 + y 2 + 2y 24 = 0
(8 2y)2 + y 2 + 2y 24 = 0
y 2 6y + 8 = 0
y = 2 or y = 4

C
O

Teaching Notes
Because the
equation
x + 2y 8 = 0 is
obtained by
combining the two
equations (through
substraction), this
equation also
contains the
solutions of the
original system. In
fact, this is the line
passing through
the common points
of the two circles.

Solution. Expanding both equations, we obtain

x2 + y 2 6x 10y + 24 = 0
x2 + y 2 + 2y 24 = 0.

y = 2 = x = 8 2(2) = 4

and

The solutions are (4, 2) and (0, 4).

y = 4 = x = 8 2(4) = 0

EP
E

The graphs of both equations are circles. One has center (3, 5) and radius
10, while the other has center (0, 1) and radius 5. The graphs with the points
of intersection are show below.

74
All rights reserved. No part of this material may be reproduced or transmitted in any form or by any means electronic or mechanical including photocopying without written permission from the DepEd Central Office. First Edition, 2016.

Seatwork/Homework 1.6.3
Solve the system, and graph the curves in one Cartesian plane showing the
point(s) of intersection.

C
O

PY

x2 + y 2 = 25
1.
2
2
x +y =1
18 32
Answer: (3, 4), (3, 4), (3, 4), and (3, 4)

EP
E

x2 + 2y 12 = 0
2.
x2 + y 2 = 36



Answer: (0, 6), 2 5, 4 , and 2 5, 4

75
All rights reserved. No part of this material may be reproduced or transmitted in any form or by any means electronic or mechanical including photocopying without written permission from the DepEd Central Office. First Edition, 2016.


(x 1)2 + (y 3)2 = 10
3.
x2 + (y 1)2 = 5

EP
E

C
O

PY

Answer: (2, 2) and (2, 0)

1.6.4. Applications of Systems of Nonlinear Equations


Let us apply systems of equations to a problem involving modern-day television
sets.

? Example 1.6.6. The screen size of television sets is given in inches. This
indicates the length of the diagonal. Screens of the same size can come in different
shapes. Wide-screen TVs usually have screens with aspect ratio 16 : 9, indicating
the ratio of the width to the height. Older TV models often have aspect ratio
4 : 3. A 40-inch LED TV has screen aspect ratio 16 : 9. Find the length and the
width of the screen.
Solution. Let w represent the width and h the height of the screen. Then, by
Pythagorean Theorem, we have the system

w2 + h2 = 402 = w2 + h2 = 1600
w = 16 = h = 9w
h
9
16
76

All rights reserved. No part of this material may be reproduced or transmitted in any form or by any means electronic or mechanical including photocopying without written permission from the DepEd Central Office. First Edition, 2016.

w + h = 1600 =

w +

9w
16

2

= 1600

337w2
= 1600
256
r
409 600
w=
34.86
337
h=

19(34.86)
19x

= 19.61
16
16

PY

Therefore, a 40-inch TV with aspect ratio 16 : 9 is about 35.86 inches wide and
19.61 inches high.
2
Seatwork/Homework 1.6.4

EP
E

C
O

1. From a circular piece of metal sheet with diameter 20 cm, a rectangular


piece with perimeter 28 cm is to be cut as shown. Find the dimensions of
the rectangular piece.
Answer: 6 cm 8 cm

Exercises 1.6
1. Solve the system, and graph the curves.

y = 2x + 4
(a)
y = 2x2
Answer: (1, 2) and (2, 8)

77
All rights reserved. No part of this material may be reproduced or transmitted in any form or by any means electronic or mechanical including photocopying without written permission from the DepEd Central Office. First Edition, 2016.

PY
C
O

x2 + y 2 = 25
(b)
2x 3y = 6

EP
E

16
, 13
Answer: (3, 4) and 63
13

x2 + y 2 = 12
(c)
x2 y 2 = 4





Answer: 2 2, 2 , 2 2, 2 , 2 2, 2 , and 2 2, 2
78
All rights reserved. No part of this material may be reproduced or transmitted in any form or by any means electronic or mechanical including photocopying without written permission from the DepEd Central Office. First Edition, 2016.

PY
C
O

EP
E

x2 4y 2 = 200
(d)
x + 2y = 100

Answer: 51, 49
2

1 (x + 1)2 (y + 2)2 = 1
4
(e)
(y + 2)2 = 1 (x 1)
4

Answer: (1, 2), 4, 2 +

5
2



, and 4, 2

5
2

79
All rights reserved. No part of this material may be reproduced or transmitted in any form or by any means electronic or mechanical including photocopying without written permission from the DepEd Central Office. First Edition, 2016.

? 2. A laptop has screen size 13 inches with aspect ratio 5 : 4. Find the length and
the width of the screen.
Answer: 10.15 in 8.12 in
? 3. What are the dimensions of a rectangle whose perimeter is 50 cm and diagonal
Answer: 14.9 cm 10.1 cm

18 cm?
2

4. The graph of 2xyy +5x+20 = 0 is a rotated hyperbola. Find the intersection


of this hyperbola with the graph of 3x + 2y = 3. (The graph is not required.)

71
, 25
Answer: (1, 3), 21
7

Answer: a = 1 or a = 3

C
O

have only one solution?

xy+a=0

PY

5. For what values of a will the system

x2 + y 2 + 2x 1 = 0

EP
E

80
All rights reserved. No part of this material may be reproduced or transmitted in any form or by any means electronic or mechanical including photocopying without written permission from the DepEd Central Office. First Edition, 2016.

Unit 2

EP
E

C
O

PY

Mathematical Induction

Batad Rice Terraces in Ifugao, by Ericmontalban, 30 September 2012,

https://commons.wikimedia.org/wiki/File%3ABatad rice terraces in Ifugao.jpg. Public Domain.

Listed as one of the United Nations Educational, Scientific and Cultural


Organization (UNESCO) World Heritage sites since 1995, the two-millenniumold Rice Terraces of the Philippine Cordilleras by the Ifugaos is a living testimony
of mankinds creative engineering to adapt to physically-challenging environment
in nature. One of the five clusters of terraces inscribed in the UNESCO list is
the majestic Batad terrace cluster (shown above), which is characterized by its
amphitheater-like, semicircular terraces with a village at its base.

81
All rights reserved. No part of this material may be reproduced or transmitted in any form or by any means electronic or mechanical including photocopying without written permission from the DepEd Central Office. First Edition, 2016.

Lesson 2.1. Review of Sequences and Series


Time Frame: 1 one-hour session
Learning Outcomes of the Lesson
At the end of the lesson, the student is able to:
(1) illustrate a series; and

Lesson Outline
(1) Sequences and series

PY

(2) differentiate a series from a sequence.

(2) Different types of sequences and series (Fibonacci sequence, arithmetic and
geometric sequence and series, and harmonic series)

Introduction

C
O

(3) Difference between sequence and series

Pose the following problem to the class:

EP
E

Jasons classroom is on the second floor of the school. He can take


one or two steps of the stairs in one leap. In how many ways can
Jason climb the stairs if it has 16 steps?

Get students to suggest strategies they can use to solve this problem. Lead
or encourage them to try out smaller number of steps and find a pattern. Work
with the class to complete the following table (on the board):

82
All rights reserved. No part of this material may be reproduced or transmitted in any form or by any means electronic or mechanical including photocopying without written permission from the DepEd Central Office. First Edition, 2016.

Number of Ways
to Climb the Stairs

5
..
.

8
..
.

PY

Number of Steps
in the Stairs

C
O

The students should be able to recognize the Fibonacci sequence. Ask the
students to recall what Fibonacci sequences are and where they had encountered
this sequence before.

Teaching Notes
This is equivalent
to the number of
ways to express a
number (number of
steps in the stairs)
as a sum of 1s and
2s. For example,
we can write 3 as a
sum of 1s and 2s
in three ways:
2 + 1, 1 + 2, and
1 + 1 + 1. In 2 + 1,
it means Jason
leaps 2 steps first,
then 1 step to
finish the
three-step stairs.

In this lesson, we will review the definitions and different types of sequences
and series.
Lesson Proper

Recall the following definitions:

EP
E

A sequence is a function whose domain is the set of positive integers


or the set {1, 2, 3, . . . , n}.
A series represents the sum of the terms of a sequence.

If a sequence is finite, we will refer to the sum of the terms of the


sequence as the series associated with the sequence. If the sequence has
infinitely many terms, the sum is defined more precisely in calculus.

A sequence is a list of numbers (separated by commas), while a series is a


sum of numbers (separated by + or sign). As an illustration, 1, 21 , 13 , 41
7
is a sequence, and 1 21 + 13 41 = 12
is its associated series.

The sequence with nth term an is usually denoted by {an }, and the associated
series is given by
S = a1 + a2 + a3 + + an .

83
All rights reserved. No part of this material may be reproduced or transmitted in any form or by any means electronic or mechanical including photocopying without written permission from the DepEd Central Office. First Edition, 2016.

Example 2.1.1. Determine the first five terms of each defined sequence, and
give their associated series.
(1) {2 n}
(3) {(1)n }
(2) {1 + 2n + 3n2 }

(4) {1 + 2 + 3 + + n}

Solution. We denote the nth term of a sequence by an , and S = a1 + a2 + a3 +


a4 + a5 .
(1) an = 2 n

First five terms: a1 = 2 1 = 1, a2 = 2 2 = 0, a3 = 1, a4 = 2, a5 = 3

PY

Associated series: S = a1 + a2 + a3 + a4 + a5 = 1 + 0 1 2 3 = 5

(2) an = 1 + 2n + 3n2

First five terms: a1 = 1 + 2 1 + 3 12 = 6, a2 = 17, a3 = 34, a4 = 57, a5 = 86

C
O

Associated series: S = 6 + 17 + 34 + 57 + 86 = 200


(3) an = (1)n

First five terms: a1 = (1)1 = 1, a2 = (1)2 = 1, a3 = 1, a4 = 1,


a5 = 1
Associated series: S = 1 + 1 1 + 1 1 = 1

(4) an = 1 + 2 + 3 + + n

EP
E

First five terms: a1 = 1, a2 = 1+2 = 3, a3 = 1+2+3 = 6, a4 = 1+2+3+4 =


10, a5 = 1 + 2 + 3 + 4 + 5 = 15
Associated series: S = 1 + 3 + 6 + 10 + 15 = 35

The sequence {an } defined by an = an1 + an2 for n 3, where a1 =


a2 = 1, is called a Fibonacci sequence. It terms are 1, 1, 2, 3, 5, 8, 13, . . ..

An arithmetic sequence is a sequence in which each term after the first


is obtained by adding a constant (called the common difference) to the
preceding term.

If the nth term of an arithmetic sequence is an and the common difference is


d, then
an = a1 + (n 1)d.

The associated arithmetic series with n terms is given by


Sn =

n(a1 + an )
n[2a1 + (n 1)d]
=
.
2
2
84

All rights reserved. No part of this material may be reproduced or transmitted in any form or by any means electronic or mechanical including photocopying without written permission from the DepEd Central Office. First Edition, 2016.

A geometric sequence is a sequence in which each term after the first


is obtained by multiplying the preceding term by a constant (called
the common ratio).
If the nth term of a geometric sequence is an and the common ratio is r, then
an = a1 rn1 .

PY

The associated geometric series with n terms is given by

na1
if r = 1
n
Sn =
a1 (1 r )

if r =
6 1.

(1 r)

C
O

The proof of this sum formula is an example in Lesson 2.3.


When 1 < r < 1, the infinite geometric series

a1 + a1 r + a1 r2 + + a1 rn1 +
has a sum, and is given by

a1
.
1r

S=

EP
E

If {an } is an arithmetic sequence, then the sequence with nth term


bn = a1n is a harmonic sequence.

Teaching Notes
The proof of the
fact that the
infinite geometric
series
a1 + a1 r + has
a sum when |r| < 1
is beyond the scope
of Precalculus, and
can be found in
university
Calculus.

Seatwork/Homework

1. Write SEQ if the given item is a sequence, and write SER if it is a series.
Answer: SEQ

(b) 2, 8, 10, 18, . . .

Answer: SEQ

(c) 1 + 1 1 + 1 1

Answer: SER

(a) 1, 2, 4, 8, . . .

(d)

1 2 3 4
, , , ,...
2 3 4 5
2

Answer: SEQ

(e) 1 + 2 + 2 + 23 + 24

Answer: SER

(f) 1 + 0.1 + 0.001 + 0.0001

Answer: SER

2. Write A if the sequence is arithmetic, G if it is geometric, F if Fibonacci, and


O if it is not one of the mentioned types.
(a) 3, 5, 7, 9, 11, . . .

Answer: A
85

All rights reserved. No part of this material may be reproduced or transmitted in any form or by any means electronic or mechanical including photocopying without written permission from the DepEd Central Office. First Edition, 2016.

(b) 2, 4, 9, 16, 25, . . .


(c)
(d)
(e)

1
,
4
1
,
3
1
,
5

Answer: O

1 1
, , 1 ,...
16 64 256
2 3 4
, , ,...
9 27 81
1 1 1 1
, , , ,...
9 13 17 21

Answer: G
Answer: O
Answer: A

(f) 4, 6, 10, 16, 26, . . .



(g) 3, 4, 5, 6, . . .

Answer: F
Answer: O

(h) 0.1, 0.01, 0.001, 0.0001, . . .

Answer: G

PY

3. Determine the first five terms of each defined sequence, and give their associated series.

C
O

(a) {1 + n n2 }
Answer: a1 = 1, a2 = 1, a3 = 5, a4 = 11, a5 = 19
Associated series: 1 1 5 11 19 = 35

(b) {1 (1)n+1 }
Answer: a1 = 0, a2 = 2, a3 = 0, a4 = 2, a5 = 0
Associated series: 0 + 2 + 0 + 2 + 0 = 4

(c) a1 = 3 and an = 2an1 + 3 for n 2


Answer: a1 = 3, a2 = 9, a3 = 21, a4 = 45, a5 = 93
Associated series: 1 1 5 11 19 = 35

EP
E

(d) {1 2 3 n}
Answer: a1 = 1, a2 = 1 2 = 2, a3 = 1 2 3 = 6, a4 = 24, a5 = 120
Associated series: 1 + 2 + 6 + 24 + 120 = 153

4. Identify the series (and write NAGIG if it is not arithmetic, geometric, and
infinite geometric series), and determine the sum (and write NO SUM if it
cannot be summed up).

(a) 4 + 9 + 14 + + 64

(b) 81 + 27 + 9 + +

Answer: Arithmetic, 442

1
81

Answer: Geometric,

(c) 1 + 3 + 6 + 10 + 15 + 21 + + 55

(d) 10 2 + 6 + + 46

(e) 10 + 2 + 0.4 + 0.08 +


(f)

1
2

1
3

1
5

9841
81

Answer: NAGIG, 220


Answer: Arithmetic, 144
Answer: Infinite geometric, 12.5

1
7

+ + + +

Answer: NAGIG, NO SUM

(g) 1 0.1 + 0.01 0.001 +

Answer: Infinite geometric,

10
11

86
All rights reserved. No part of this material may be reproduced or transmitted in any form or by any means electronic or mechanical including photocopying without written permission from the DepEd Central Office. First Edition, 2016.

Lesson 2.2. Sigma Notation


Time Frame: 2 one-hour sessions
Learning Outcomes of the Lesson
At the end of the lesson, the student is able to use the sigma notation to
represent a series.

(1) Definition of and writing in sigma notation


(2) Evaluate sums written in sigma notation
(3) Properties of sigma notation

PY

Lesson Outline

Introduction

C
O

(4) Calculating sums using the properties of sigma notation

The sigma notation is a shorthand for writing sums. In this lesson, we will
see the power of this notation in computing sums of numbers as well as algebraic
expressions.

EP
E

2.2.1. Writing and Evaluating Sums in Sigma Notation


Mathematicians use the sigma notation to denote a sum. The uppercase Greek
letter (sigma) is used to indicate a sum. The notation consists of several
components or parts.

Let f (i) be an expression involving an integer i. The expression


f (m) + f (m + 1) + f (m + 2) + + f (n)

can be compactly written in sigma notation, and we write it as


n
X

f (i),

Teaching Notes
Emphasize that
the value of i starts
at m, increases by
1, and ends at n.

i=m

which is read the summation of f (i) from i = m to n. Here, m


and n are integers with m n, f (i) is a term (or summand ) of the
summation, and the letter i is the index, m the lower bound, and n
the upper bound.

87
All rights reserved. No part of this material may be reproduced or transmitted in any form or by any means electronic or mechanical including photocopying without written permission from the DepEd Central Office. First Edition, 2016.

Example 2.2.1. Expand each summation, and simplify if possible.


n
4
X
X
(3)
ai
(1)
(2i + 3)
i=1

i=2

(2)

5
X

6
X
n
(4)
n+1
n=1

i=0

Solution. We apply the definition of sigma notation.


4
X

(2i + 3) = [2(2) + 3] + [2(3) + 3] + [2(4) + 3] = 27

i=2

(2)

5
X

2i = 20 + 21 + 22 + 23 + 24 + 25 = 63

(3)

n
X
i=1

C
O

i=0

PY

(1)

ai = a1 + a2 + a3 + + an

6
X
n
1
2
3 2
5
6
= +
+
+ +
+
(4)
n+1
2
3
4
5
6
7
n=1

Example 2.2.2. Write each expression in sigma notation.


1 1 1
1
+ + + +
2 3 4
100
(2) 1 + 2 3 + 4 5 + 6 7 + 8 9 + 25

EP
E

(1) 1 +

(3) a2 + a4 + a6 + a8 + + a20
(4) 1 +

1
1
1
1
1 1 1
+ + +
+
+
+
2 4 8 16 32 64 128
100

X1
1 1 1
1
Solution. (1) 1 + + + + +
=
2 3 4
100 n=1 n

(2) 1 + 2 3 + 4 5 + 25
= (1)1 1 + (1)2 2 + (1)3 3 + (1)4 4
+ (1)5 5 + + (1)25 25
=

25
X

(1)j j

j=1

88
All rights reserved. No part of this material may be reproduced or transmitted in any form or by any means electronic or mechanical including photocopying without written permission from the DepEd Central Office. First Edition, 2016.

(3) a2 + a4 + a6 + a8 + + a20
= a2(1) + a2(2) + a2(3) + a2(4) + + a2(10)
=

10
X

a2i

i=1

X 1
1
1
1
1
1 1 1
+
+
+
=
(4) 1 + + + +
2 4 8 16 32 64 128 k=0 2k

PY

The sigma notation of a sum expression is not necessarily unique. For example, the last item in the preceding example can also be expressed in sigma
notation as follows:
8

X 1
1
1
1
1
1 1 1
+
+
+
=
.
1+ + + +
2 4 8 16 32 64 128 k=1 2k1

Seatwork/Homework 2.2.1

C
O

However, this last sigma notation is equivalent to the one given in the example.

(a)

5
X

1. Expand each summation, and simplify if possible.


(2 3k)

k=1
n
X

EP
E
(b)

xj

Answer: x + x2 + x3 + + xn

j=1

(c)

6
X
j=3

4
X

(j 2 j)

Answer: 68

(1)k+1 k

Answer: 2

(d)

Answer: 28

k=1

(e)

3
X
n=1

(an+1 an )

Answer: a4 a1

2. Write each expression in sigma notation.


2

(a) x + 2x + 3x + 4x + 5x

Answer:

5
X

kxk

k=1

(b) 1 2 + 3 4 + 5 6 + 10

Answer:

10
X

(1)k+1 k

k=1

89
All rights reserved. No part of this material may be reproduced or transmitted in any form or by any means electronic or mechanical including photocopying without written permission from the DepEd Central Office. First Edition, 2016.

Teaching Notes
Equivalent answer:
1+3+5+ +101 =
51
X
(2k 1)
k=1

(c) 1 + 3 + 5 + 7 + + 101

Answer:

50
X

(2k + 1)

k=0

(d) a4 + a8 + a12 + a16

Answer:

4
X

a4k

k=1

(e) 1

1 1 1 1
+ +
3 5 7 9

Answer:

4
X
k=0

(1)k
2k + 1

We start with finding a formula for the sum of

i=1

in terms of n.

i = 1 + 2 + 3 + + n

C
O

n
X

PY

2.2.2. Properties of Sigma Notation

Teaching Notes
This illustration
can be done with
manipulatives, and
allow the students
to guess.

EP
E

The sum can be evaluated in different ways. One informal but simple approach
is pictorial.

n
X
i=1

i = 1 + 2 + 3 + + n =

n(n + 1)
2

Another way is to use the formula for an arithmetic series with a1 = 1 and
an = n:
n(a1 + an )
n(n + 1)
S=
=
.
2
2
We now derive some useful summation facts. They are based on the axioms
of arithmetic addition and multiplication.
90
All rights reserved. No part of this material may be reproduced or transmitted in any form or by any means electronic or mechanical including photocopying without written permission from the DepEd Central Office. First Edition, 2016.

n
X

cf (i) = c

i=m

n
X

f (i),

c any real number.

i=m

Proof.

i=m

cf (i) = cf (m) + cf (m + 1) + cf (m + 2) + + cf (n)


= c[f (m) + f (m + 1) + + f (n)]
n
X
=c
f (i)

PY

n
X

Teaching Notes
Some proofs can be
skipped. However,
it is helpful if they
are all discussed in
class.

i=m

[f (i) + g(i)] =

i=m

n
X

g(i)

i=m

i=m

f (i) +

i=m

Proof.
n
X

n
X

C
O

n
X

[f (i) + g(i)]

EP
E

= [f (m) + g(m)] + + [f (n) + g(n)]


= [f (m) + + f (n)] + [g(m) + + g(n)]
n
n
X
X
=
f (i) +
g(i)
i=m

i=m

n
X
i=m

c = c(n m + 1)

Proof.
n
X
i=m

c = c| + c + c{z+ + }c
nm+1 terms

= c(n m + 1)

91
All rights reserved. No part of this material may be reproduced or transmitted in any form or by any means electronic or mechanical including photocopying without written permission from the DepEd Central Office. First Edition, 2016.

A special case of the above result which you might encounter more often is
the following:
n
X
c = cn.
i=1

Telescoping Sum

i=m

[f (i + 1) f (i)] = f (n + 1) f (m)

PY

n
X

Proof.
n
X


f (i + 1) f (i)

C
O

i=m

= [f (m + 1) f (m)] + [f (m + 2) f (m + 1)]
+ [f (m + 3) f (m + 2)] + + [f (n + 1) f (n)]

Note that the terms, f (m + 1), f (m + 2), . . . , f (n), all cancel out. Hence, we have
[f (i + 1) f (i)] = f (n + 1) f (m).

n
X

30
X

EP
E

i=m

Example 2.2.3. Evaluate:

i=1

(4i 5).

Solution.

30
X
i=1

(4i 5) =

30
X

4i

i=1
30
X

=4

i=1

30
X

i=1
30
X

5
5

i=1

(30)(31)
5(30)
2
= 1710
=4

Example 2.2.4. Evaluate:


1
1
1
1
+
+
+ +
.
12 23 34
99 100
92
All rights reserved. No part of this material may be reproduced or transmitted in any form or by any means electronic or mechanical including photocopying without written permission from the DepEd Central Office. First Edition, 2016.

Solution.
1
1
1
1
+
+
+ +
12 23 34
99 100
99
X
1
=
i(i + 1)
i=1
=

99
X
i+1i

i(i + 1)

i=1
99 
X

C
O

1
and the telescoping-sum property, we get
i


99
X
1
1
1
99
=

.
=
i(i + 1)
100 1
100
i=1

Using f (i) =

PY

i
i+1

=
i(i + 1) i(i + 1)
i=1

99 
X
1
1
=

i
i+1
i=1

99 
X
1
1
=

i
+
1
i
i=1

EP
E

Example 2.2.5. Derive a formula for

n
X

i2 using a telescoping sum with terms

i=1

f (i) = i3 .

Solution. The telescoping sum property implies that

n
X
3

i (i 1)3 = n3 03 = n3 .
i=1

On the other hand, using expansion and the other properties of summation,
we have
n
n
X
3
 X
i (i 1)3 =
(i3 i3 + 3i2 3i + 1)
i=1

i=1
n
X

=3

i=1

n
n
X
X
i 3
i+
1
2

i=1

i=1

n
X
n(n + 1)
=3
i2 3
+ n.
2
i=1

93
All rights reserved. No part of this material may be reproduced or transmitted in any form or by any means electronic or mechanical including photocopying without written permission from the DepEd Central Office. First Edition, 2016.

Equating the two results above, we obtain


n
X
3n(n + 1)
3
+ n = n3
i2
2
i=1

n
X
6
i2 3n(n + 1) + 2n = 2n3
i=1

n
X
6
i2 = 2n3 2n + 3n(n + 1)

PY

i=1

C
O

= 2n(n2 1) + 3n(n + 1)
= 2n(n 1)(n + 1) + 3n(n + 1)
= n(n + 1)[2(n 1) + 3]
= n(n + 1)(2n + 1).

Finally, after dividing both sides of the equation by 6, we obtain the desired
formula
n
X
n(n + 1)(2n + 1)
i2 =
.
2
6
i=1
Seatwork/Homework 2.2.2

EP
E

1. Use the properties of sigma notation to evaluate the following summations.


(a)

50
X
k=1

(b)

n
X

(2 3k)

Answer: 3725

(1 + 2j)

Answer: 2n + n2

j=1

99
X

(c)

j=1

i+1+ i

Solution:
99
X
j=1

Answer: 9

99
X
i+1 i
1
1
=

i+1+ i
i
+
1
+
i
i
+
1

i
j=1
99 
X


=
i+1 i
j=1

= 99 + 1 1
=9
94

All rights reserved. No part of this material may be reproduced or transmitted in any form or by any means electronic or mechanical including photocopying without written permission from the DepEd Central Office. First Edition, 2016.

2. If

n
X

(i + 1)2 = an3 + bn2 + cn + d, what is a + b + c + d?

Answer: 4

i=1

Exercises 2.2
1. Expand each sum.
i
x+i

i=5

(b)

6
X

Answer:

2i

1
2
3
4
5
+
+
+
+
x+1 x+2 x+3 x+4 x+5

Answer: 0 +

i=0

3
X

(c)

3i

3
6 + 2 + 3 10 + 3 12

PY

(a)

9
X

2+

4+

Answer: 9 + 3 + 1 + 1/3 + 1/9 + 1/27

C
O

i=2

2. Write each expression in sigma notation.


2

(a) 1 + 2 + 3 + 4 + + 12

12

Answer:

12
X

ii

i=1

(b) (x 5) + (x 3) + (x 1) + (x + 1) + (x + 3) + + (x + 15)
7
X
Answer:
[x + (2i + 1)]

EP
E

i=3

(c) a1 + a4 + a9 + a16 + + a81

Answer:

9
X

Teaching Notes
Another possible
answer for (b) is
11
X
[x + (2i 7)].
i=1

ai 2

i=1

3. Evaluate each sum.


(4i 15)

Answer: 27240

(a)

120
X
i=1

(b)

50
X
i=1

(c)

n
X
i=1

4. If

30
X
i=1

[(5i 2)(i + 3)]

Answer: 230900

(3i 1)2

f (i) = 70 and

Answer:
30
X

g(i) = 50, what is the value of

i=1

6n3 + 3n2 3n + 2
2

30
X
3g(i) f (i) + 7
i=1

Answer: 145

95
All rights reserved. No part of this material may be reproduced or transmitted in any form or by any means electronic or mechanical including photocopying without written permission from the DepEd Central Office. First Edition, 2016.

5. If s =
6. If s =

100
X

i=1
n
X

200
X

i, express

i in terms of s.

Answer: 2s + 100000

i=1

ai , does it follow that

i=1

n
X

a2i = s2 ?

i=1

Answer: No. If s =

2
X

ai = a1 + a2 , then

i=1

7. Derive a formula for

n
X

a2i = a21 + a22 , while s2 =

i=1

i3 by using a telescoping sum with terms f (i) = i4 .

PY

a21 + 2a1 a2 + a22 .

n
X

i=1

Answer:

n2 (n + 1)2
4

C
O

Lesson 2.3. Principle of Mathematical Induction

Time Frame: 3 one-hour sessions

Learning Outcomes of the Lesson

EP
E

At the end of the lesson, the student is able to:

(1) illustrate the Principle of Mathematical Induction; and


(2) apply mathematical induction in proving identities.
Lesson Outline

(1) State the Principle of Mathematical Induction

(2) Prove summation identities using mathematical induction


(3) Prove divisibility statements using mathematical induction
(4) Prove inequalities using mathematical induction
Introduction
We have derived and used formulas for the terms of arithmetic and geometric
sequences and series. These formulas and many other theorems involving positive
integers can be proven with the use of a technique called mathematical induction.

96
All rights reserved. No part of this material may be reproduced or transmitted in any form or by any means electronic or mechanical including photocopying without written permission from the DepEd Central Office. First Edition, 2016.

2.3.1. Proving Summation Identities


There are many mathematical results that can be proven using mathematical
induction. In this lesson, we will focus on three main categories: summation
identities, divisibility statements, and inequalities.
We first state the Principle of Mathematical Induction, and see how the principle works in general sense.
The Principle of Mathematical Induction

PY

Let P (n) be a property or statement about an integer n. Suppose


that the following conditions can be proven:
(1) P (n0 ) is true (that is, the statement is true when n = n0 ).

C
O

(2) If P (k) is true for some integer k n0 , then P (k + 1) is true


(that is, if the statement is true for n = k, then it is also true for
n = k + 1).
Then the statement P (n) is true for all integers n n0 .

EP
E

The Principle of Mathematical Induction is often compared to climbing an


infinite staircase. First, you need to be able to climb up to the first step. Second,
if you are on any step (n = k), you must be able to climb up to the next step
(n = k + 1). If you can do these two things, then you will be able to climb up
the infinite staircase.

Part 1

Part 2

Another analogy of the Principle of Mathematical Induction that is used is


toppling an infinite line of standing dominoes. You need to give the first domino
a push so that it falls down. Also, the dominoes must be arranged so that if the
kth domino falls down, the next domino will also fall down. These two conditions
will ensure that the entire line of dominoes will fall down.

97
All rights reserved. No part of this material may be reproduced or transmitted in any form or by any means electronic or mechanical including photocopying without written permission from the DepEd Central Office. First Edition, 2016.

PY
C
O

Standing Domino Tiles, by Nara Cute, 16 October 2015,


https://commons.wikimedia.org/wiki/File:Wallpaper kartu domino.png. Public Domain.

Let us now take a look at some examples on the use of mathematical induction
in proving summation identities.

EP
E

Example 2.3.1. Using mathematical induction, prove that


1 + 2 + 3 + + n =

n(n + 1)
2

for all positive integers n.

Solution. We need to establish the two conditions stated in the Principle of Mathematical Induction.
Part 1. Prove that the identity is true for n = 1.
The left-hand side of the equation consists of one term equal to 1. The righthand side becomes
1(1 + 1)
2
= = 1.
2
2
Hence, the formula is true for n = 1.
Part 2. Assume that the formula is true for n = k 1:
1 + 2 + 3 + + k =

k(k + 1)
.
2

98
All rights reserved. No part of this material may be reproduced or transmitted in any form or by any means electronic or mechanical including photocopying without written permission from the DepEd Central Office. First Edition, 2016.

We want to show that the formula is true for n = k + 1; that is,


1 + 2 + 3 + + k + (k + 1) =

(k + 1)(k + 1 + 1)
.
2

Using the formula for n = k and adding k + 1 to both sides of the equation,
we get
k(k + 1)
+ (k + 1)
2
k(k + 1) + 2(k + 1)
=
2
(k + 1)(k + 2)
=
2
(k + 1) [(k + 1) + 1]
=
2

C
O

PY

1 + 2 + 3 + + k + (k + 1) =

We have proven the two conditions required by the Principle of Mathematical


Induction. Therefore, the formula is true for all positive integers n.
2
Example 2.3.2. Use mathematical induction to prove the formula for the sum
of a geometric series with n terms:
a1 (1 rn )
,
1r

Sn =

r 6= 1.

EP
E

Solution. Let an be the nth term of a geometric series. From Lesson 2.1, we know
that an = a1 rn1 .
Part 1. Prove that the formula is true for n = 1.
a1 (1 r1 )
= a1 = S 1
1r

The formula is true for n = 1.

Part 2. Assume that the formula is true for n = k 1: Sk =

want to prove that it is also true for n = k + 1; that is,


Sk+1 =

a1 (1 rk )
. We
1r

Teaching Notes
The fact that
an = a1 rn1 can
also be proven by
mathematical
induction. Here,
however, we simply
recall a formula in
Lesson 2.1 because
our focus in this
example is the
proof of the sum.

a1 (1 rk+1 )
.
1r

We know that
Sk+1 = a1 + a2 + + ak +ak+1
|
{z
}
Sk

= Sk + ak+1

99
All rights reserved. No part of this material may be reproduced or transmitted in any form or by any means electronic or mechanical including photocopying without written permission from the DepEd Central Office. First Edition, 2016.

=
=
=
=


a1 1 r k
+ a1 rk
1r 
a1 1 rk + a1 rk (1 r)
1r

k
a1 1 r + rk rk+1
1 r
k+1
a1 1 r
1r

By the Principle of Mathematical Induction, we have proven that


a1 (1 rn )
1r

PY

Sn =
for all positive integers n.

C
O

Example 2.3.3. Using mathematical induction, prove that


12 + 22 + 32 + + n2 =
for all positive integers n.

n(n + 1)(2n + 1)
6

EP
E

Part 1

Solution. We again establish the two conditions stated in the Principle of Mathematical Induction.
123
1(1 + 1)(2 1 + 1)
=
= 1 = 12
6
6
The formula is true for n = 1.

Part 2

k(k + 1)(2k + 1)
.
6
Prove: 12 + 22 + 32 + + k 2 + (k + 1)2
(k + 1)(k + 2) [2(k + 1) + 1]
=
6
(k + 1)(k + 2)(2k + 3)
=
.
6

Assume: 12 + 22 + 32 + + k 2 =

12 + 22 + 32 + + k 2 + (k + 1)2
k(k + 1)(2k + 1)
=
+ (k + 1)2
6
k(k + 1)(2k + 1) + 6(k + 1)2
=
6
100
All rights reserved. No part of this material may be reproduced or transmitted in any form or by any means electronic or mechanical including photocopying without written permission from the DepEd Central Office. First Edition, 2016.

(k + 1) [k(2k + 1) + 6(k + 1)]


6
(k + 1) (2k 2 + 7k + 6)
=
6
(k + 1)(k + 2)(2k + 3)
=
6
=

Therefore, by the Principle of Mathematical Induction,


n(n + 1)(2n + 1)
6

for all positive integers n.


Seatwork/Homework 2.3.1

C
O

Using mathematical induction, prove that

PY

12 + 22 + 32 + + n2 =

1 3 + 2 4 + 3 5 + + n(n + 2) =
Answer:

Part 1

n(n + 1)(2n + 7)
.
6

EP
E

29
1(1 + 1)[2(1) + 7]
=
=3=13
6
6
The formula is true for n = 1.
Part 2

k(k + 1)(2k + 7)
6
To show: 1 3 + 2 4 + + k(k + 2) + (k + 1)(k + 3)
(k + 1)(k + 2) [2(k + 1) + 7]
=
6
(k + 1)(k + 2)(2k + 9)
=
6

Assume: 1 3 + 2 4 + 3 5 + + k(k + 2) =

1 3 + 2 4 + + k(k + 2) + (k + 1)(k + 3)
k(k + 1)(2k + 7)
=
+ (k + 1)(k + 3)
6
(k + 1)
=
[k(2k + 7) + 6(k + 3)]
6

(k + 1)  2
=
2k + 13k + 18
6
101

All rights reserved. No part of this material may be reproduced or transmitted in any form or by any means electronic or mechanical including photocopying without written permission from the DepEd Central Office. First Edition, 2016.

Teaching Notes
Recall the
definition of
divisibility: an
integer n is
divisible by an
integer k if n = kr
for some integer r.

(k + 1)(k + 2)(2k + 9)
6

Therefore, by the Principle of Math Induction, the formula is true for all positive
integers n.
2.3.2. Proving Divisibility Statements
We now prove some divisibility statements using mathematical induction.

PY

Example 2.3.4. Use mathematical induction to prove that, for every positive
integer n, 7n 1 is divisible by 6.

Part 1
71 1 = 6 = 6 1

71 1 is divisible by 6.
Part 2

C
O

Solution. Similar to what we did in the previous session, we establish the two
conditions stated in the Principle of Mathematical Induction.

Assume: 7k 1 is divisible by 6.

To show: 7k+1 1 is divisible by 6.

EP
E

7k+1 1 = 7 7k 1 = 6 7k + 7k 1 = 6 7k + (7k 1)

By definition of divisibility, 6 7k is divisible by 6. Also, by the hypothesis


(assumption), 7k 1 is divisible by 6. Hence, their sum (which is equal to
7k+1 1) is also divisible by 6.

Therefore, by the Principle of Math Induction, 7n 1 is divisible by 6 for all


positive integers n.
2
Note that 70 1 = 1 1 = 0 = 6 0 is also divisible by 6. Hence, a stronger
and more precise result in the preceding example is: 7n 1 is divisible by 6 for
every nonnegative integer n. It does not make sense to substitute negative values
of n since this will result in non-integer values for 7n 1.
Example 2.3.5. Use mathematical induction to prove that, for every nonnegative integer n, n3 n + 3 is divisible by 3.
Solution. We again establish the two conditions in the Principle of Mathematical
Induction.

102
All rights reserved. No part of this material may be reproduced or transmitted in any form or by any means electronic or mechanical including photocopying without written permission from the DepEd Central Office. First Edition, 2016.

Part 1. Note that claim of the statement is that it is true for every nonnegative
integer n. This means that Part 1 should prove that the statement is true for
n = 0.
03 0 + 3 = 3 = 3(1)

03 0 + 3 is divisible by 3.
Part 2. We assume that k 3 k + 3 is divisible by 3. By definition of divisibility,
we can write k 3 k + 3 = 3a for some integer a.

PY

To show: (k + 1)3 (k + 1) + 3 is divisible by 3.

C
O

(k + 1)3 (k + 1) + 3 = k 3 + 3k 2 + 2k + 3
= (k 3 k + 3) + 3k 2 + 3k
= 3a + 3k 2 + 3k
= 3(a + k 2 + k)

Since a + k 2 + k is also an integer, by definition of divisibility, (k + 1)3 (k + 1) + 3


is divisible by 3.

EP
E

Therefore, by the Principle of Math Induction, n3 n + 3 is divisible by 3 for


all positive integers n.
2
Seatwork/Homework 2.3.1

Use mathematical induction to prove each divisibility statement for all nonnegative integers n.
(1) 72n 3 5n + 2 is divisible by 12.

Answer:
Part 1

72(0) 3 50 + 2 = 1 3(1) + 2 = 0 = 12(0)


72(0) 3 50 + 2 is divisible by 12
Part 2
Assume: 72k 3 5k + 2 is divisible by 12

To show: 72(k+1) 3 5(k+1) + 2 is divisible by 12


72(k+1) 3 5(k+1) + 2

= 72 72k 3 5 5k + 2
103

All rights reserved. No part of this material may be reproduced or transmitted in any form or by any means electronic or mechanical including photocopying without written permission from the DepEd Central Office. First Edition, 2016.

= 49 72k 15 5k + 2

= 72k + 48 72k 3 5k 12 5k + 2

= 72k 3 5k + 2 + 48 72k 12 5k


= 72k 3 5k + 2 + 12 4 72k 5k

By the hypothesis,
72k 3 5k + 2 is divisible by 12. The second term,

12 4 72k 5k , is divisible by 12 because 4 72k 5k is an integer. Hence
their sum, which is equal to 72(k+1) 3 5(k+1) + 2, is divisible by 12.

PY

Therefore, by the Principle of Math Induction, 72n 3 5n + 2 is divisible by


12 for every nonnegative integer n.

(2) n3 + 3n2 + 2n is divisible by 3.


Answer:

03 + 3 02 + 2(0) = 0 = 3(0)

C
O

Part 1
Thus, 03 + 3 02 + 2(0) is divisible by 3.
Part 2

Assume: k 3 + 3k 2 + 2k is divisible by 3.

= k 3 + 3k 2 + 2k = 3a, a integer

EP
E

To show: (k + 1)3 + 3(k + 1)2 + 2(k + 1) is divisible by 3.

(k + 1)3 + 3(k + 1)2 + 2(k + 1)


= k 3 + 6k 2 + 11k + 6
= (k 3 + 3k 2 + 2k) + 3k 2 + 9k + 6
= 3a + 3k 2 + 9k + 6
= 3(a + k 2 + 3k + 2)

Since a + k 2 + 3k + 2 is also an integer, by definition of divisibility, (k + 1)3 +


3(k + 1)2 + 2(k + 1) is divisible by 3.
Therefore, by the Principle of Math Induction, n3 + 3n2 + 2n is divisible by
3 for all positive integers n.

? 2.3.3. Proving Inequalities


Finally, we now apply the Principle of Mathematical Induction in proving some
inequalities involving integers.
Example 2.3.6. Use mathematical induction to prove that 2n > 2n for every
integer n 3.
104
All rights reserved. No part of this material may be reproduced or transmitted in any form or by any means electronic or mechanical including photocopying without written permission from the DepEd Central Office. First Edition, 2016.

Solution. Just like the previous example, we establish the two conditions in the
Principle of Mathematical Induction.
Part 1
23 = 8 > 6 = 2(3)
This confirms that 23 > 2(3).
Part 2
Assume: 2k > 2k, where k is an integer with k 3

PY

To show: 2k+1 > 2(k + 1) = 2k + 2

C
O

We compare the components of the assumption and the inequality we need to


prove. On the left-hand side, the expression is doubled. On the right-hand side,
the expression is increased by 2. We choose which operation we want to apply to
both sides of the assumed inequality.
Alternative 1. We double both sides.

Since 2k > 2k, by the multiplication property of inequality, we have 2 2k >


2 2k.
2k+1 > 2(2k) = 2k + 2k > 2k + 2 if k 3.

Hence, 2k+1 > 2(k + 1).

Alternative 2. We increase both sides by 2.

EP
E

Since 2k > 2k, by the addition property of inequality, we have 2k + 2 > 2k + 2.


2(k + 1) = 2k + 2 < 2k + 2 < 2k + 2k if k 3.


The right-most expression above, 2k + 2k , is equal to 2 2k = 2k+1 .

Hence, 2(k + 1) < 2k+1 .

Therefore, by the Principle of Math Induction, 2n > 2n for every integer


n 3.
2
We test the above inequality for integers less than 3.
20 = 1 > 0 = 2(0)

True

21 = 2 = 2(1)

False

22 = 4 = 2(2)

False

The inequality is not always true for nonnegative integers less than 3. This
illustrates the necessity of Part 1 of the proof to establish the result. However,
the result above can be modified to: 2n 2n for all nonnegative integers n.
Before we discuss the next example, we review the factorial notation. Recall
105
All rights reserved. No part of this material may be reproduced or transmitted in any form or by any means electronic or mechanical including photocopying without written permission from the DepEd Central Office. First Edition, 2016.

that 0! = 1 and, for every positive integer n, n! = 1 2 3 n. The factorial also


satisfies the property that (n + 1)! = (n + 1) n!.
Example 2.3.7. Use mathematical induction to prove that 3n < (n + 2)! for
every positive integer n. Can you refine or improve the result?
Solution. We proceed with the usual two-part proof.
Part 1
31 = 3 < 6 = 3! = (1 + 2)! = 31 < (1 + 2)!

PY

Thus, the desired inequality is true for n = 1.


Part 2
Assume: 3k < (k + 2)!

C
O

To show: 3k+1 < (k + 3)!

Given that 3k < (k + 2)!, we multiply both sides of the inequality by 3 and
obtain

3 3k < 3 [(k + 2)!] .

This implies that



3 3k < 3 [(k + 2)!] < (k + 3) [(k + 2)!] ,

and so

since k > 0,

EP
E

3k+1 < (k + 3)!.

Therefore, by the Principle of Math Induction, we conclude that 3n < (n + 2)!


for every positive integer n.
The left-hand side of the inequality is defined for any integer n. The righthand side makes sense only if n + 2 0, or n 2.
1
< 1 = 0! = (2 + 2)!
9
1
When n = 1: 31 = < 1 = 1! = (1 + 2)!
3
0
When n = 0: 3 = 1 < 2 = 2! = (0 + 2)!

When n = 2: 32 =

Therefore, 3n < (n + 2)! for any integer n 2.

Seatwork/Homework 2.3.3
Use mathematical induction to prove that 2n + 3 < 2n for n 4.
106
All rights reserved. No part of this material may be reproduced or transmitted in any form or by any means electronic or mechanical including photocopying without written permission from the DepEd Central Office. First Edition, 2016.

Answer:
Part 1
2(4) + 3 = 11 < 16 = 24
Thus, 2(4) + 3 <= 24 .
Part 2
Assume: 2k + 3 < 2k , k 4

To show: 2(k + 1) + 3 < 2k+1

PY

2(k + 1) + 3 = 2k + 5 = (2k + 3) + 2

< 2k + 2 < 2k + 2k = 2k+1

Exercises 2.3

C
O

Therefore, by the Principle of Math Induction, 2n + 3 < 2n for n 4.

Prove the following statements by mathematical induction.


n
X
i=1

(3)

3n2 + n
2

1
1
1
n
1
+
+
+ +
=
12 23 34
n(n + 1)
n+1

EP
E

(2)

(3i 1) =

(1)

n
X
i=1

2 3i1 = 3n 1

Hint:

k+1
X

i1

23

i=1

n
X

i3 =

i=1

23i1 +23( k+1)1 = 3k 1+23k = 33k 1 = 3k+1 1

n2 (n + 1)2
4

(4)

k
X

i=1

(5) a1 + (a1 + d) + (a1 + 2d) + + [a1 + (n 1)d] =

n [2a1 + (n 1)d]
2

(6) 1 (1!) + 2 (2!) + + n (n!) = (n + 1)! 1


Hint:

k+1
X
i=1

i i! =

k
X
i=1

i i! + (k + 1)(k + 1)! = (k + 1)! 1 + (k + 1)(k + 1)! =

(k + 1)!(1 + k + 1) 1 = (k + 2)! 1
(7) 7n 4n is divisible by 3

Hint: 7k+1 4k+1 = 7 7k 4 4k = (3 + 4)7k 4 4k = 3 7k + (7k 4k )


107

All rights reserved. No part of this material may be reproduced or transmitted in any form or by any means electronic or mechanical including photocopying without written permission from the DepEd Central Office. First Edition, 2016.

(8) 10n + 3 4n+2 + 5 is divisible by 9

Hint: 10k+1 +34k+3 +5 = 1010k +344k+2 +5 = (9+1)10k +(9+3)4k+2 +5 =


9(10k + 4k+2 ) + 10k + 3 4k+2 + 5

(9) 11n+2 + 122n+1 is divisible by 133


Hint: 11k+3 +122k+3 = 1111k+2 +122 122k+1 = 1111k+2 +(133+11)122k+1 =
11(11k+2 + 122k+1 ) + 133 122k+1
(10) xn y n is divisible by x y for any positive integer n

Hint: xk+1 y k+1 = x xk y xk + y xk y y k = (x y)xk + y(xk y k )

PY

(11) xn + y n is divisible by x + y for any odd positive integer n

Hint: xk+2 + y k+2 = x2 xk + y 2 y k = x2 xk + x2 y k x2 y k + y 2 y k = x2 (xk + y k )


y k (x y)(x + y)
(12) If 0 < a < 1, then 0 < an < 1 for any positive integer n

C
O

Hint: 0 < ak < 1 = 0 a < ak a < 1 a = 0 < ak+1 < a < 1

(13) (1 + a)n > 1 + na for a > 1, a 6= 0 and n an integer greater than 1

Hint: (1 + a)k+1 > (1 + ka)(1 + a) = 1 + (k + 1)a + ka2 > 1 + (k + 1)a

(14) 2n > n2 for every integer n > 4

Hint: 2k+1 = 2 2k > 2k 2 = k 2 + k 2 > k 2 + 2k + 1 = (k + 1)2 . The last


inequality follows from (k 1)2 > 2 for k > 4, which implies that k 2 > 2k + 1.

EP
E

For k > 4, (k 1)2 > 2

(15) 2n < n! for every integer n > 3

Hint: 2k+1 = 2 2k = 2k! < (k + 1)k! = (k + 1)!

Lesson 2.4. The Binomial Theorem


Time Frame: 4 one-hour sessions
Learning Outcomes of the Lesson
At the end of the lesson, the student is able to:
(1) illustrate Pascals Triangle in the expansion of (x + y)n for small positive
integral values of n;
(2) prove the Binomial Theorem;

108
All rights reserved. No part of this material may be reproduced or transmitted in any form or by any means electronic or mechanical including photocopying without written permission from the DepEd Central Office. First Edition, 2016.

(3) determine any term in (x + y)n , where n is a positive integer, without expanding; and
(4) solve problems using mathematical induction and the Binomial Theorem.
Lesson Outline
(1) Expand (x + y)n for small values of n using Pascals Triangle
(2) Review the definition of and formula for combination
(3) State and prove the Binomial Theorem
(4) Compute all or specified terms of a binomial expansion

PY

(5) Prove some combination identities using the Binomial Theorem


Introduction

C
O

In this lesson, we study two ways to expand (a + b)n , where n is a positive


integer. The first, which uses Pascals Triangle, is applicable if n is not too big,
and if we want to determine all the terms in the expansion. The second method
gives a general formula for the expansion of (a + b)n for any positive integer n.
This formula is useful especially when n is large because it avoids the process of
going through all the coefficients for lower values of n obtained through Pascals
Triangle. Moreover, if only a specific term is required, it can be computed directly
using a simple formula. Lastly, the theorem can be used to derive and prove some
useful and interesting results about sums of combinations.

EP
E

2.4.1. Pascals Triangle and the Concept of Combination


Consider the following powers of a + b:

Teaching Notes
Calculations with
big numbers are
required in many
of the examples
and exercises in
this section. The
use of scientific
calculators is
desirable.
Teaching Notes
You may ask the
students to expand
these powers using
long multiplication.

(a + b)1 = a + b
(a + b)2 = a2 + 2ab + b2
(a + b)3 = a3 + 3a2 b + 3ab2 + b3
(a + b)4 = a4 + 4a3 b + 6a2 b2 + 4ab3 + b4
(a + b)5 = a5 + 5a4 b + 10a3 b2 + 10a2 b3 + 5ab4 + b5

Teaching Notes
The concept of
combination was
introduced in
Grade 10. In
particular, the
concept was
discussed with
competency codes
from M10SP-IIIc-1
to M10SP-IIId-e-1.

We list down the coefficients of each expansion in a triangular array as follows:


n=1:

n=2:

n=3:
n=4:
n=5: 1

1
1

2
3

4
5

1
1
3
6

10

1
4

10

1
5

109
All rights reserved. No part of this material may be reproduced or transmitted in any form or by any means electronic or mechanical including photocopying without written permission from the DepEd Central Office. First Edition, 2016.

The preceding triangular array of numbers is part of what is called the Pascals Triangle, named after the French mathematician, Blaise Pascal (1623-1662).
Some properties of the Triangle are the following:
(1) Each row begins and ends with 1.
(2) Each row has n + 1 numbers.
(3) The second and second to the last number of each row correspond to the
row number.

PY

(4) There is symmetry of the numbers in each row.


(5) The number of entries in a row is one more than the row number (or one
more than the number of entries in the preceding row).

C
O

(6) Every middle number after first row is the sum of the two numbers above
it.
It is the last statement which is useful in constructing the succeeding rows of the
triangle.
Example 2.4.1. Use Pascals Triangle to expand the expression (2x 3y)5 .

Solution. We use the coefficients in the fifth row of the Pascals Triangle.

EP
E

(2x 3y)5 = (2x)5 + 5(2x)4 (3y) + 10(2x)3 (3y)2


+ 10(2x)2 (3y)3 + 5(2x)(3y)4
+ (3y)5
= 32x5 240x4 y + 720x3 y 2 1080x2 y 3
+ 810xy 4 243y 5

Example 2.4.2. Use Pascals Triangle to expand (a + b)8 .

Solution. We start with the sixth row (or any row of the Pascals Triangle that
we remember).
n=6:
n=7:
n=8: 1

1
1

6
7

15
21

28

20
35

56

15
35

70

6
21

56

1
7

28

1
8

Therefore, we get
(a + b)8 = a8 + 8a7 b + 28a6 b2 + 56a5 b3
+ 70a4 b4 + 56a3 b5 + 28a2 b6
+ 8ab7 + b8

110
All rights reserved. No part of this material may be reproduced or transmitted in any form or by any means electronic or mechanical including photocopying without written permission from the DepEd Central Office. First Edition, 2016.

We observe that, for each n, the expansion of (a + b)n starts with an


and the exponent of a in the succeeding terms decreases by 1, while
the exponent of b increases by 1. This observation will be shown to
be true in general.

Let us review the concept of combination. Recall that C(n, k) or nk counts
the number of ways of choosing k objects from a set of n objects. It is also useful
to know some properties of C(n, k):

PY

(1) C(n, 0) = C(n, n) = 1,


(2) C(n, 1) = C(n, n 1) = n, and
(3) C(n, k) = C(n, n k).

C
O

These properties can explain some of the observations we made on the numbers in the Pascals Triangle. Also recall the general formula for the number of
combinations of n objects taken k at a time:
 
n
n!
,
C(n, k) =
=
k!(n k)!
k

EP
E

where 0! = 1 and, for every positive integer n, n! = 1 2 3 n.


 
 
5
8
Example 2.4.3. Compute
and
.
3
5

Solution.

 
5!
5
5!
=
= 10
=
(5 3)!3!
2!3!
3
 
8
8!
10!
=
=
= 56
5
(8 5)!5!
3!5!


You may observe that the value of 53 and the fourth coefficient
in the fifth

8
row of Pascals Triangle are the same. In the same manner, 5 is equal to the
sixth coefficient in the expansion of (a + b)8 (see Example 2.4.2). These observed
equalities are not coincidental, and they are, in fact, the essence embodied in the
Binomial Theorem, as you will see in the succeeding sessions.
Seatwork/Homework 2.4.1
1. Use Pascals Triangle to expand each expression.
(a) (x 2y)4

Answer: x4 8x3 y + 24x2 y 2 32xy 3 + 16y 4


111

All rights reserved. No part of this material may be reproduced or transmitted in any form or by any means electronic or mechanical including photocopying without written permission from the DepEd Central Office. First Edition, 2016.

(b) (2a b2 )3

Answer: 8a3 12a2 b2 + 6ab4 b6

(c) (a + b)9
Answer: a9 +9a8 b+36a7 b2 +84a6 b3 +126a5 b4 +126a4 b5 +84a3 b6 +36a2 b7 +
9ab8 + b9

C
O

PY

2. Compute.
 
5
(a)
Answer: 10
2
 
9
(b)
Answer: 36
7
 
12
(c)
Answer: 66
10
 
20
(d)
Answer: 15504
5
 
n
n(n 1)
3. Prove:
=
.
2
2
Answer:
 
n
n!
n(n 1)(n 2)!
n(n 1)
=
=
=
2
(n 2)!2!
(n 2)!2!
2

EP
E

2.4.2. The Binomial Theorem

As the power n gets larger, the more laborious it would be to use Pascals Triangle
(and impractical to use long multiplication) to expand (a + b)n . For example,
using Pascals Triangle, we need to compute row by row up to the thirtieth row
to know the coefficients of (a + b)30 . It is, therefore, delightful to know that it is
possible to compute the terms of a binomial expansion of degree n without going
through the expansion of all the powers less than n.

We now explain how the concept of combination is used in the expansion of


(a + b)n .
(a + b)n = (a + b)(a + b)(a + b) (a + b)
{z
}
|
n factors

When the distributive law is applied, the expansion of (a + b)n consists of


terms of the form am bi , where 0 m, i n. This term is obtained by choosing
a for m of the factors and b for the rest of the factors. Hence, m + i = n, or
m = n i. This means that the number of times the term ani bi will appear
in the expansion of (a + b)n equals the number of ways of choosing (n i) or i

112
All rights reserved. No part of this material may be reproduced or transmitted in any form or by any means electronic or mechanical including photocopying without written permission from the DepEd Central Office. First Edition, 2016.

factors from the n factors, which is exactly C(n, i). Therefore, we have
n  
X
n ni i
n
(a + b) =
a b.
i
i=0
To explain the reasoning above, consider the case n = 3.
(a + b)3 = (a + b)(a + b)(a + b)
= aaa + aab + aba + abb + baa + bab + bba + bbb
= a3 + 3a2 b + 3ab2 + b3

PY

That is, each term in the expansion is obtained by choosing either a or b in each
factor. The term a3 is obtained when a is chosen each time, while a2 b is obtained
when a is selected 2 times, or equivalently, b is selected exactly once.

C
O

We will give another proof of this result using mathematical induction. But
first, we need to prove a result about combinations.
Pascals Identity

If n and k are positive integers with k n, then



   

n+1
n
n
=
+
.
k
k
k1

EP
E

Proof. The result follows from the combination formula.


  

n
n
n!
n!
+
=
+
k
k1
k!(n k)! (k 1)!(n k + 1)!
n!(n k + 1) + n!(k)
=
k!(n k + 1)!
n!(n k + 1 + k)
=
k!(n + 1 k)!
n!(n + 1)
=
k!(n + 1 k)!
(n + 1)!
=
k!(n + 1 k)!


n+1
=
k

Teaching Notes
The formula can
also be proved
using
the fact that
n
is the number
k
of ways to choose k
from n distinct
objects. Suppose a
is one of the n
objects. Then, in
selecting k objects,
either a is selected
or not. If a is
included in the k
objects, then
there
n 
are k1
ways to
complete the
selection of the k
objects; if a is not
included, then

there are n
ways.
k

Pascals identity explains the method of constructing Pascals Triangle, in


which an entry is obtained by adding the two numbers above it. This identity
is also an essential part of the second proof of the Binomial Theorem, which we
now state.
113
All rights reserved. No part of this material may be reproduced or transmitted in any form or by any means electronic or mechanical including photocopying without written permission from the DepEd Central Office. First Edition, 2016.

The Binomial Theorem


For any positive integer n,
n  
X
n ni i
(a + b) =
a b.
i
i=0
n

Proof. We use mathematical induction.


 
 
1  
X
1 1i i
1 1 0
1 0 1
a b =
ab +
a b =a+b
i
0
1
i=0

Hence, the formula is true for n = 1.

k  
X
k

C
O

Part 2. Assume that


k

(a + b) =

i=0

We want to show that

k+1

aki bi .


k+1 
X
k + 1 k+1i i
=
a
b.
i
i=0

(a + b)

PY

Part 1

EP
E

(a + b)k+1 = (a + b)(a + b)k


k  
X
k ki i
= (a + b)
a b
i
i=0
k  
k  
X
X
k ki i
k ki i
=a
a b +b
a b
i
i
i=0
i=0
k  
k  
X
k ki+1 i X k ki i+1
=
a
b +
a b
i
i
i=0
i=0
 
 
 
k  
X
k k+1 0
k k+1i i
k k 1
k k1 2
=
a b +
a
b +
a b +
a b
0
i
0
1
i=1
 


 
k
k 0 k+1
k k2 3
1 k
+
a b + +
ab +
ab
2
k1
k

k  
k 
X
k k+1i i X
k
k+1
=a
+
a
b +
ak+1i bi + bk+1
i
i1
i=1
i=1






k  
k + 1 k+1 0 X k
k
k + 1 0 k+1
k+1i i
=
a b +
+
a
b +
ab
0
i
i1
k+1
i=1
114

All rights reserved. No part of this material may be reproduced or transmitted in any form or by any means electronic or mechanical including photocopying without written permission from the DepEd Central Office. First Edition, 2016.


k+1 
X
k + 1 k+1i i
=
a
b
i
i=0
The last expression above follows from Pascals Identity.
Therefore, by the Principle of Mathematical Induction,
n  
X
n ni i
n
(a + b) =
a b
i
i=1

2.4.3. Terms of a Binomial Expansion

PY

for any positive integer n.

We now apply the Binomial Theorem in different examples.

Solution.

C
O

Example 2.4.4. Use the Binomial Theorem to expand (x + y)6 .

6  
X
6 6k k
(x + y) =
x y
k
k=0
 
 


6 5 1
6 4 2
6 6 0
=
xy +
xy +
xy
1
2
0
 
 
 
6 3 3
6 2 4
6 1 5
+
xy +
xy +
xy
3
4
5
 
6 0 6
+
xy
6
= x6 + 6x5 y + 15x4 y 2 + 20x3 y 3
+ 15x2 y 2 + 6xy 5 + y 6

EP
E

Since the expansion of (a + b)n begins with k = 0 and ends with


the
 k = n,
n n
n
expansion has n + 1 terms. The first term in the expansion is 0 a =
, the
 an1
n
second term is n1 an1 b = nan=1 b, the second to the last term is n1
ab
=

n n
n1
n
nab , and the last term is n b = b .
 nk+1 k1
n
The kth term of the expansion is k1
a
b . If n is even, there is a

n
middle term, which
is the 2 +1 th term. If n is odd, there are two middle

n+1
terms, the n+1
th
and
+ 1 th terms.
2
2

The general term is often represented by nk ank bk . Notice that, in any term,
the sum of the exponents of a and b is n. The combination nk is the coefficient
of the term involving bk . This allows us to compute any particular term without
needing to expand (a + b)n and without listing all the other terms.
115

All rights reserved. No part of this material may be reproduced or transmitted in any form or by any means electronic or mechanical including photocopying without written permission from the DepEd Central Office. First Edition, 2016.

Teaching Notes
To find a specific
term in the
expansion of
(a + b)n , it is
important to find
the value of k.

Example 2.4.5. Find the fifth term in the expansion of 2x

20
y .

Solution. The fifth term in the expansion of a fifth power corresponds to k = 4.


 

20

(2x)204 ( y)4 = 4845 65536x16 y 2


4
= 317521920x16 y 2
2
Example 2.4.6. Find the middle term in the expansion of

x
2

+ 3y

6

C
O

PY

Solution. Since there are seven terms in the expansion, the middle term is the
fourth term (k = 3), which is
 3
  
 135x3 y 3
x
6
x 3
3
3
(3y) = 20
.
2
27y =
2
8
2
3
Example 2.4.7. Find the term involving x (with exponent 1) in the expansion

8
2y
2
of x
.
x
Solution. The general term in the expansion is

EP
E

k  
 


8 162k (2)k y k
8
2y
2 8k
=
x

k
x
xk
k
 
8
=
(2)k x162kk y k
k
 
8
=
(2)k x163k y k .
k

The term involves x if the exponent of x is 1, which means 16 3k = 1, or


k = 5. Hence, the term is
 
8
(2)5 xy 5 = 1792xy 5 .
2
5
Seatwork/Homework 2.4.3
5

1. Use the Binomial Theorem to expand (2a b2 ) .


Answer:

2a b


2 5

 
 
5
5
5
=
(2a) +
(2a)4 b2
0
1
116

All rights reserved. No part of this material may be reproduced or transmitted in any form or by any means electronic or mechanical including photocopying without written permission from the DepEd Central Office. First Edition, 2016.

 
 
3

5
5
2 2
3
(2a)2 b2
+
(2a) b +
3
2
 
 
5
4
5
5
b2
+
(2a) b2 +
5
4
5
4 2
3 4
= 32a 80a b + 80a b 40a2 b6
+ 10ab8 b10


11
2
2. Find the two middle terms in the expansion of x +
.
y
Answer: There are 12 terms in the expansion, so the two middle terms are the
6th (corresponding to k = 5) and the 7th (corresponding to k = 6) terms.

PY

1/3

C
O

 5
 
 

2
11
32
14784x2
1/3 115
2
x
= 462x
=
5
y
y5
y5

 
 
 6

29568x5/3
11
64
2
5/3
1/3 116
= 462x
=
x
y
y6
y6
6
3. Find the constant term in the expansion of

x3
3
+ 2
2
x

10

Answer: The general term is

EP
E

   3 10k  k    303k   k 
3
10
x
3
10
x
=
2
10k
2
x
k
2
x2k
k
  k
10 3
x305k
=
k 210k

The constant term contains x0 , which means 30 5k = 0, or k = 6.


  6
10 3 0 76545
x =
8
6 24

? 2.4.4. Approximation and Combination Identities


We continue applying the Binomial Theorem.
? Example 2.4.8. (1) Approximate (0.8)8 by using the first three terms in the
expansion of (1 0.2)8 . Compare your answer with the calculator value.
(2) Use 5 terms in the binomial expansion to approximate (0.8)8 . Is there an
improvement in the approximation?

117
All rights reserved. No part of this material may be reproduced or transmitted in any form or by any means electronic or mechanical including photocopying without written permission from the DepEd Central Office. First Edition, 2016.

Solution.
8

8  
X
8

(1)8k (0.2)k
k
k=0
8  
X
8
=
(0.2)k
k
k=0

(0.8) = (1 0.2) =

= 1 1.6 + 1.12 = 0.52

PY

   
 
2  
X
8
8
8
8
k
(1)
(0.2) =
+
(0.2) +
(0.2)2
k
0
1
2
k=0

The calculator value is 0.16777216, so the error is 0.35222784.

C
O

   
 
4  
X
8
8
8
8
k
(2)
(0.2) =
+
(0.2) +
(0.2)2
k
0
1
2
k=0
 
 
8
8
3
+
(0.2) +
(0.2)4
3
4
= 0.52 0.448 + 0.112 = 0.184

The error is 0.01622784, which is an improvement on the previous estimate.


2

EP
E

Example 2.4.9. Use the Binomial Theorem to prove that, for any positive integer n,
n  
X
n
= 2n .
k
k=0

Solution. Set a = b = 1 in the expansion of (a + b)n . Then


n  
n  
X
X
n
n
n
n
nk
k
2 = (1 + 1) =
(1) (1) =
.
k
k
k=0
k=0
Example 2.4.10. Use the Binomial Theorem to prove that
 
     
100
100
100
100
+
+
+ +
0
2
4
100
     
 
100
100
100
100
=
+
+
+ +
1
3
5
99
Solution. Let a = 1 and b = 1 in the expansion of (a + b)100 . Then



100 
100 X
100
1 + (1)
=
(1)100k (1)k .
k
k=0
118

All rights reserved. No part of this material may be reproduced or transmitted in any form or by any means electronic or mechanical including photocopying without written permission from the DepEd Central Office. First Edition, 2016.

   
 
 
100
100
100
100
2
0=
+
(1) +
(1) +
(1)3
0
1
2
3
 
 
100
100
99
+ +
(1) +
(1)100
99
100

PY

If k is even, then (1)k = 1. If k is odd, then (1)k = 1. Hence, we have


       
100
100
100
100
0=

0
1
2
3
   
100
100
+
+
99
100

C
O

Therefore, after transposing the negative terms to other side of the equation, we
obtain
     
 
100
100
100
100
+
+
+ +
0
2
4
100
     
 
100
100
100
100
=
+
+
+ +
2
1
3
5
99
Seatwork/Homework 2.4.4

EP
E

? 1. Approximate (1.9)10 using the first three terms in the expansion of


(2 0.1)10 , and find its error compared to the calculator value.
Answer:

10

(1.9)

10

= (2 0.1)

2  
X
10

k=0
10

210k (0.1)k

= 2 10 29 0.1 + 45 28 0.12
= 627.2

Calculator value = 613.1066258


Error from the calculator value = 14.09337422

2. Prove that, for any positive integer n,


n  
X
n k
3 = 4n .
k
k=0
n

Answer: 4 = (1 + 3) =

n  
X
n
k=0

nk k

3 =

n  
X
n
k=0

3k

119
All rights reserved. No part of this material may be reproduced or transmitted in any form or by any means electronic or mechanical including photocopying without written permission from the DepEd Central Office. First Edition, 2016.

Exercises 2.4
1. Use the Binomial Theorem to expand each expression.
(a) (x 2)5

7
1
(b) x +
y

Answer: x5 10x4 + 40x3 80x2 + 80x 32

Answer: x7 +
1
3
2

4

PY

(c)

7x6 21x5 35x4 35x3 21x2 7x


1
+ 2 + 3 + 4 + 5 + 6 + 7
y
y
y
y
y
y
y
 
 
1
1
377
1
+ 3(9)
=
Answer: 81 3(27)
2
4
8
8

C
O

2. Without expanding completely, compute the indicated term(s) in the expansion of the given expression.

15
1
15x41 105x37
3
+
(a) x +
, first 3 terms
Answer: x45 +
2x
2
4

EP
E

(b) (4 3x)6 , last 3 terms



12
3
(c) x +
, 9th term
2

25

1
(d)
x
, 6th term
y

18
1
1
(e)
, middle term
p+
2
q

11
2 a2
(f)
+
, two middle terms
a
3
9

(g)
y + x , term involving y 3

16
1
2x , constant term
(h)
x3

Answer: 19440x4 5832x5 + 729x6


Answer:

3247695 4
x
256

Answer:

53130x10
y5

Answer:

12155p9
128q 9

Answer:

9856 4 4928 7
a +
a
81
243
Answer: 84x3 y 3
Answer:

366080
729

21

(i) (xy 2y 2 ) , term that does not contain y


Answer: 14883840x14

18
x y

(j)
, term in which the exponents of x and y are equal
y2
x
43758
Answer: 6 6
xy

? 3. Approximate (1.1)10 by using the first 4 terms in the expansion of


(1 + 0.1)10 . Compare your answer with the calculator result.
Answer: 2.57, with an error of 0.0237424601 from the calculator value of
2.59374246
120
All rights reserved. No part of this material may be reproduced or transmitted in any form or by any means electronic or mechanical including photocopying without written permission from the DepEd Central Office. First Edition, 2016.

4. Use the Binomial Theorem to prove that


n  
X
n k
2 = 3n .
k
k=0

Hint: Expand (1 + 2)n .


5. Use the Binomial Theorem to prove that

PY

50  
X
50
(2)k = 1.
k
k=0

Hint: Expand (1 2)50 .

EP
E

C
O

121
All rights reserved. No part of this material may be reproduced or transmitted in any form or by any means electronic or mechanical including photocopying without written permission from the DepEd Central Office. First Edition, 2016.

Unit 3

EP
E

C
O

PY

Trigonometry

Puerto Princesa Subterranean River National Park, by Giovanni G. Navata, 12 November 2010,
https://commons.wikimedia.org/wiki/File%3AUnderground River.jpg. Public Domain

Named as one of the New Seven Wonders of Nature in 2012 by the New7Wonders
Foundation, the Puerto Princesa Subterranean River National Park is worldfamous for its limestone karst mountain landscape with an underground river.
The Park was also listed as UNESCO World Heritage Site in 1999. The underground river stretches about 8.2 km long, making it one of the worlds longest
rivers of its kind.

122
All rights reserved. No part of this material may be reproduced or transmitted in any form or by any means electronic or mechanical including photocopying without written permission from the DepEd Central Office. First Edition, 2016.

Lesson 3.1. Angles in a Unit Circle


Time Frame: 3 one-hour sessions
Learning Outcomes of the Lesson
At the end of the lesson, the student is able to:
(1) illustrate the unit circle and the relationship between the linear and angular
measures of arcs in a unit circle.

PY

(2) convert degree measure to radian measure, and vice versa.


(3) illustrate angles in standard position and coterminal angles.
Lesson Outline
(1) Linear and angular measure of arcs

C
O

(2) Conversion of degree to radian, and vice versa


(3) Arc length and area of the sector

(4) Angle in standard position and coterminal angles


Introduction

EP
E

Angles are being used in several fields like engineering, medical imaging, electronics, astronomy, geography and many more. Added to that, surveyors, pilots,
landscapers, designers, soldiers, and people in many other professions heavily use
angles and trigonometry to accomplish a variety of practical tasks. In this lesson, we will deal with the basics of angle measures together with arc length and
sectors.
3.1.1. Angle Measure

An angle is formed by rotating a ray about its endpoint. In the figure shown
below, the initial side of AOB is OA, while its terminal side is OB. An angle
is said to be positive if the ray rotates in a counterclockwise direction, and the
angle is negative if it rotates in a clockwise direction.

Teaching Notes
Angles in
trigonometry differ
from angles in
Euclidean
geometry in the
sense of motion.
An angle in
geometry is defined
as a union of rays
(that is, static)
and has measure
between 0 and
180 . An angle in
trigonometry is a
rotation of a ray,
and, therefore, has
no limit. It has
positive and
negative directions
and measures.

123
All rights reserved. No part of this material may be reproduced or transmitted in any form or by any means electronic or mechanical including photocopying without written permission from the DepEd Central Office. First Edition, 2016.

C
O

PY

An angle is in standard position if it is drawn in the xy-plane with its vertex


at the origin and its initial side on the positive x-axis. The angles , , and in
the following figure are angles in standard position.

To measure angles, we use degrees, minutes, seconds, and radians.

EP
E

A central angle of a circle measures one degree, written 1 , if it inter1


of the circumference of the circle. One minute, written 10 , is
cepts 360
1
1

of 1 , while one second, written 100 , is 60


of 10 .
60

For example, in degrees, minutes, and seconds,


0

18

0
00

10 30 18 = 10 30 +
60

0
= 10 30.3


30.3
= 10 +
60

= 10.505

and

79.251 = 79 (0.251 60)0


= 79 15.060
= 79 150 (0.06 60)00
= 79 150 3.600 .
Recall that the unit circle is the circle with center at the origin and radius 1
unit.
124
All rights reserved. No part of this material may be reproduced or transmitted in any form or by any means electronic or mechanical including photocopying without written permission from the DepEd Central Office. First Edition, 2016.

C
O

PY

A central angle of the unit circle that intercepts an arc of the circle
with length 1 unit is said to have a measure of one radian, written 1
rad. See Figure 3.1.

Figure 3.1

EP
E

In trigonometry, as it was studied in Grade 9, the degree measure is often used.


On the other hand, in some fields of mathematics like calculus, radian measure of
angles is preferred. Radian measure allows us to treat the trigonometric functions
as functions with the set of real numbers as domains, rather than angles.
Example 3.1.1. In the following figure, identify the terminal side of an angle in
standard position with given measure.
(1) degree measure: 135 , 135 , 90 , 405

rad, 3
rad,
4

3
2

rad, 2 rad

(2) radian measure:

125
All rights reserved. No part of this material may be reproduced or transmitted in any form or by any means electronic or mechanical including photocopying without written permission from the DepEd Central Office. First Edition, 2016.

Solution.

(1) 135 : OC; 135 : OD; 90 : OE; and 405 : OB

(2) radian measure:

OE

rad: OB; 3
rad: OD;
4

3
2

rad: OE; and 2 rad:


2

Since a unit circle has circumference 2, a central angle that measures 360
has measure equivalent to 2 radians. Thus, we obtain the following conversion
rules.

PY

Converting degree to radian, and vice versa


1. To convert a degree measure to radian, multiply it by

C
O

2. To convert a radian measure to degree, multiply it by

.
180
180
.

EP
E

Figure 3.2 shows some special angles in standard position with the indicated
terminal sides. The degree and radian measures are also given.

Figure 3.2

126
All rights reserved. No part of this material may be reproduced or transmitted in any form or by any means electronic or mechanical including photocopying without written permission from the DepEd Central Office. First Edition, 2016.

Example 3.1.2. Express 75 and 240 in radians.

75

  5
=
180
12

  4
240
=
180
3

Solution.

11
6

180

rad and

75 =

240 =

11
6

4
rad
3

= 22.5

rad = 22.5
8

= 330

11
rad = 330
6

180

Seatwork/Homework 3.1.1

rad in degrees.

C
O

Example 3.1.3. Express

5
rad
12

PY

Solution.

1. Convert the following degree measures to radian measure.


(a) 60

EP
E

(c) 150

(b) 90

Answer: 2
Answer: 5
6

Answer:

rad
rad
rad

2. Convert the following radian measures to degree measure.


(a)

(b)

rad
9
3
rad
4

Answer: 20
Answer: 135

3.1.2. Coterminal Angles

Two angles in standard position that have a common terminal side are called
coterminal angles. Observe that the degree measures of coterminal angles differ
by multiples of 360 .
Two angles are coterminal if and only if their degree measures differ
by 360k, where k Z.

Similarly, two angles are coterminal if and only if their radian measures differ by 2k, where k Z.

127
All rights reserved. No part of this material may be reproduced or transmitted in any form or by any means electronic or mechanical including photocopying without written permission from the DepEd Central Office. First Edition, 2016.

C
O

PY

As a quick illustration, to find one coterminal angle with an angle that measures 410 , just subtract 360 , resulting in 50 . See Figure 3.3.

Figure 3.3

Example 3.1.4. Find the angle coterminal with 380 that has measure

(1) between 0 and 360 , and


(2) between 360 and 0 .

EP
E

Solution. A negative angle moves in a clockwise direction, and the angle 380
lies in Quadrant IV.
(1) 380 + 2 360 = 340

(2) 380 + 360 = 20

Seatwork/Homework 3.1.2

1. Find the angle between 0 and 360 (if in degrees) or between 0 rad and 2 rad
(if in radians) that is coterminal with the given angle.
(a) 736

Answer: 16

(b) 28 480 6500


(c)

13
2

Answer: 331 100 5500

rad

Answer:

? (d) 10 rad

rad

Answer: 3.72 rad

2. Find the angle between 360 and 0 (if in degrees) or between 2 rad and
0 rad (if in radians) that is coterminal with the given angle.
128
All rights reserved. No part of this material may be reproduced or transmitted in any form or by any means electronic or mechanical including photocopying without written permission from the DepEd Central Office. First Edition, 2016.

(a) 142

Answer: 218

(b) 400 10 2300


(c)

Answer: 40 10 2300

Answer: 11
rad
6

rad

? (d) 20 rad

Answer: 1.15 rad

3.1.3. Arc Length and Area of a Sector

circumference of circle =
(2r) = r.
2
2

Teaching Notes
Review how arcs
were measured in
Grade 10. What
unit of measure
was used? For two
circles with
different radii, do
equal central
angles intercept
arcs of the same
measure?
Conclude that
previous notion of
arc measure is not
the same as length.
Arcs are now
measured in terms
of length and
measure changes
with the radius of
the circle.

EP
E

C
O

s=

PY

In a circle, a central angle whose radian measure is subtends an arc that is the

of the circumference of the circle. Thus, in a circle of radius r (see


fraction 2
Figure 3.4), the length s of an arc that subtends the angle is

Figure 3.4

In a circle of radius r, the length s of an arc intercepted by a central


angle with measure radians is given by
s = r.

Example 3.1.5. Find the length of an arc of a circle with radius 10 m that
subtends a central angle of 30 .
Solution. Since the given central angle is in degrees, we have to convert it into
radian measure. Then apply the formula for an arc length.
 
= rad
30
180
6
  5
s = 10
=
m
2
6
3
129

All rights reserved. No part of this material may be reproduced or transmitted in any form or by any means electronic or mechanical including photocopying without written permission from the DepEd Central Office. First Edition, 2016.

Example 3.1.6. A central angle in a circle of radius 4 m is subtended by an


arc of length 6 m. Find the measure of in radians.
Solution.
=

6
3
s
= = rad
r
4
2

EP
E

C
O

PY

A sector of a circle is the portion of the interior of a circle bounded by the


initial and terminal sides of a central angle and its intercepted arc. It is like a
slice of pizza. Note that an angle with measure 2 radians will define a sector
that corresponds to the whole pizza. Therefore, if a central angle of a sector

of a complete
has measure radians, then the sector makes up the fraction 2
circle. See Figure 3.5. Since the area of a complete circle with radius r is r2 , we
have

1
Area of a sector =
(r2 ) = r2 .
2
2

Figure 3.5

In a circle of radius r, the area A of a sector with a central angle


measuring radians is
1
A = r2 .
2

Example 3.1.7. Find the area of a sector of a circle with central angle 60 if
the radius of the circle is 3 m.
Solution. First, we have to convert 60 into radians. Then apply the formula for
computing the area of a sector.
 
60
= rad
180
3
1

3 2
A = (32 ) =
m
2
2
3
2
130
All rights reserved. No part of this material may be reproduced or transmitted in any form or by any means electronic or mechanical including photocopying without written permission from the DepEd Central Office. First Edition, 2016.

Example 3.1.8. A sprinkler on a golf course fairway is set to spray water over
a distance of 70 feet and rotates through an angle of 120 . Find the area of the
fairway watered by the sprinkler.
Solution.

  2
=
rad
180
3
1
2
4900
A = (702 )
=
5131 ft2
2
3
3
120

PY

Seatwork/Homework 3.1.3
1. In a circle of radius 7 feet, find the length of the arc that subtends a central
angle of 5 radians.
Answer: 35 ft

C
O

2. A central angle in a circle of radius 20 m is subtended by an arc of length


15 m. Find the measure of in degrees.
Answer: 135
3. Find the area of a sector of a circle with central angle that measures 75 if the
radius of the circle is 6 m.
Answer: 7.5 m2

Exercises 3.1

EP
E

1. Give the degree/radian measure of the following special angles.

131
All rights reserved. No part of this material may be reproduced or transmitted in any form or by any means electronic or mechanical including photocopying without written permission from the DepEd Central Office. First Edition, 2016.

2. Convert each degree measure to radians. Leave answers in terms of .


11
6
8
Answer: 3

Answer: 12
Answer: 7
12
53
Answer: 36
Answer: 2
3
7
Answer: 4

(a) 330

Answer:

(b) 480
(c) 15
(d) 105
(e) 265
(f) 120

PY

(g) 315

rad
rad
rad
rad
rad
rad
rad

(b)
(c)
(d)

5
rad
6
8
rad
3
15
rad
4
6 rad
rad
7
20

(e)
? (f) 20 rad
? (g) 35 rad

EP
E

? (h) 5 rad

(a)

C
O

3. Convert each radian measure to degree-minute-second measure (approximate


if necessary).
Answer: 150
Answer: 480
Answer: 675
Answer: 30

Answer: 63

Answer: 1145 540 56.1200


Answer: 2005 210 8.2200

Answer: 286 280 44.0300

4. Find the angle between 0 and 360 (if in degrees) or between 0 rad and 2 rad
(if in radians) that is coterminal with the given angle.
Answer: 325

(b) 451

Answer: 91

(a) 685

(c) 1400

Answer: 40

(d) 960 450 3400

Answer: 240 450 3400

(e) 728 150 4300

Answer: 352 150 4300

(f)

(g)

29
rad
6
3
2 rad

5
6
Answer: 2

Answer:

? (h) 16 rad
? (i) 20 rad

rad
rad

Answer: 3.43 rad


Answer: 5.13 rad

5. Find the angle between 360 and 0 (if in degrees) or between 2 rad and
0 rad (if in radians) that is coterminal with the given angle.
132
All rights reserved. No part of this material may be reproduced or transmitted in any form or by any means electronic or mechanical including photocopying without written permission from the DepEd Central Office. First Edition, 2016.

(a) 685

Answer: 35

(b) 451

Answer: 269

(c) 1400

Answer: 320

(d) 960 450 3400

Answer: 120 450 3400

(e) 728 150 4300


(f)

(g)

Answer: 8 150 4300

29
rad
6
3
2 rad

Answer: 7
rad
6

Answer: 3
rad
2

? (h) 16 rad
? (i) 20 rad

Answer: 2.850 rad

PY

Answer: 1.150 rad

6. Find the length of an arc of a circle with radius 21 m that subtends a central
m
angle of 15 .
Answer: 7
4

C
O

7. A central angle in a circle of radius 9 m is subtended by an arc of length 12


m. Find the measure of in radians.
Answer: 43 rad
8. Find the radius of a circle in which a central angle of
of area 64 m2 .

rad determines a sector


Answer: 16 m

9. If the radius of a circle is doubled, how is the length of the arc intercepted by
a fixed central angle changed?
Answer: The length is doubled.

EP
E

10. Radian measure simplifies many formulas, such as the formula for arc length,
s = r. Give the corresponding formula when is measured in degrees instead
of radians.
Answer: s = r
180

? 11. As shown below, find the radius of the pulley if a rotation of 51.6 raises the
weight by 11.4 cm.
Answer: 12.7 cm

? 12. How many inches will the weight rise if the pulley whose radius is 9.27 inches
is rotated through an angle of 71 500 ?
Answer: 11.6 in
133
All rights reserved. No part of this material may be reproduced or transmitted in any form or by any means electronic or mechanical including photocopying without written permission from the DepEd Central Office. First Edition, 2016.

? 13. Continuing with the previous item, through what angle (to the nearest minute)
must the pulley be rotated to raise the weight 6 in?
Answer: 37 50
? 14. Given a circle of radius 3 in, find the measure (in radians) of the central angle
of a sector of area 16 in2 .

EP
E

C
O

PY

Answer: 3.6 rad


? 15. An automatic lawn sprinkler sprays up to a distance of 20 feet while rotating
30 . What is the area of the sector the sprinkler covers? Answer: 104.72 ft2
? 16. A jeepney has a windshield wiper on the drivers side that has total arm and
blade 10 inches long and rotates back and forth through an angle of 95 . The
shaded region in the figure is the portion of the windshield cleaned by the
7-inch wiper blade. What is the area of the region cleaned? Answer: 75.4 in2

17. If the radius of a circle is doubled and the central angle of a sector is unchanged,
how is the area of the sector changed?
Answer: The area is quadrupled.
18. Give the corresponding formula for the area of a sector when the angle is
2
measured in degrees.
Answer: A = r
360

? 19. A frequent problem in surveying city lots and rural lands adjacent to curves
of highways and railways is that of finding the area when one or more of the
boundary lines is the arc of a circle. Approximate the total area of the lot
shown in the figure.
Answer: 1909.0 m2

134
All rights reserved. No part of this material may be reproduced or transmitted in any form or by any means electronic or mechanical including photocopying without written permission from the DepEd Central Office. First Edition, 2016.

20. Two gears of radii 2.5 cm and 4.8 cm are adjusted so that the smaller gear
drives the larger one, as shown. If the smaller gear rotates counterclockwise
through 225 , through how many degrees will the larger gear rotate?

C
O

PY

Answer: 117

Lesson 3.2. Circular Functions

EP
E

Time Frame: 2 one-hour sessions

Learning Outcomes of the Lesson


At the end of the lesson, the student is able to:
(1) illustrate the different circular functions; and

(2) use reference angles to find exact values of circular functions.


Lesson Outline
(1) Circular functions
(2) Reference angles
Introduction
We define the six trigonometric function in such a way that the domain of
each function is the set of angles in standard position. The angles are measured
either in degrees or radians. In this lesson, we will modify these trigonometric
functions so that the domain will be real numbers rather than set of angles.

Teaching Notes
The teacher can
give a review of
trigonometric
ratios as discussed
in Grade 9.

135
All rights reserved. No part of this material may be reproduced or transmitted in any form or by any means electronic or mechanical including photocopying without written permission from the DepEd Central Office. First Edition, 2016.

3.2.1. Circular Functions on Real Numbers


Recall that the sine and cosine functions (and four others: tangent, cosecant,
secant, and cotangent) of angles measuring between 0 and 90 were defined in
the last quarter of Grade 9 as ratios of sides of a right triangle. It can be verified
that these definitions are special cases of the following definition.
Let be an angle in standard position and P () = P (x, y) the point
on its terminal side on the unit circle. Define
1
csc = , y 6= 0
y

cos = x

sec =

1
, x 6= 0
x
x
cot = , y 6= 0
y

y
, x 6= 0
x

C
O

tan =

PY

sin = y

Example 3.2.1. Find the values of cos 135 , tan 135 , sin(60 ), and sec(60 ).

EP
E

Solution. Refer to Figure 3.6(a).

(a)

(b)
Figure 3.6

Teaching Notes
A 45 -45 right
triangle is isosceles.
Moreover, the
opposite side of the
30 -angle in a
30 -60 right
triangle is half the
length of its
hypotenuse.

From properties of 45 -45 and 30 -60 right triangles (with hypotenuse 1


unit), we obtain the lengths of the legs as in Figure 3.6(b). Thus, the coordinates
of A and B are
!
!
2 2
1
3
A=
,
and B =
,
.
2 2
2
2
136

All rights reserved. No part of this material may be reproduced or transmitted in any form or by any means electronic or mechanical including photocopying without written permission from the DepEd Central Office. First Edition, 2016.

Therefore, we get

cos 135 =

2
,
2

3
sin(60 ) =
,
2

tan 135 = 1,
and

sec(60 ) = 2.

PY

From the last example, we may then also say that

 2



3
rad =
, sin rad =
,
cos
4
2
3
2

and so on.

C
O

From the above definitions, we define the same six functions on real numbers.
These functions are called trigonometric functions.
Let s be any real number. Suppose is the angle in standard position
with measure s rad. Then we define
csc s = csc

cos s = cos

sec s = sec

tan s = tan

cot s = cot

EP
E

sin s = sin

From the last example, we then have


cos

and

 
4


2

= cos
rad = cos 45 =
4
2


 


3
sin
= sin rad = sin(60 ) =
.
3
3
2
In the same way, we have
tan 0 = tan(0 rad) = tan 0 = 0.

Example 3.2.2. Find the exact values of sin 3


, cos 3
, and tan 3
.
2
2
2

Solution. Let P 3
be the point on the unit circle and on the terminal
side of
2

3
3
the angle in the standard position with measure 2 rad. Then P 2 = (0, 1),
and so
3
3
sin
= 1, cos
= 0,
2
2
but tan 3
is undefined.
2
2
137
All rights reserved. No part of this material may be reproduced or transmitted in any form or by any means electronic or mechanical including photocopying without written permission from the DepEd Central Office. First Edition, 2016.

Example 3.2.3. Suppose s is a real number such that sin s = 43 and cos s > 0.
Find cos s.
Solution. We may consider s as the angle with measure s rad. Let P (s) = (x, y)
be the point on the unit circle and on the terminal side of angle s.
Since P (s) is on the unit circle, we know that x2 + y 2 = 1. Since sin s = y =
we get


3
x =1y =1
4
2

7
=
16

7
.
4

7
x=
.
4
2

EP
E

C
O

Since cos s = x > 0, we have cos s =

2

PY

34 ,

Let P (x1 , y1 ) and Q(x, y) be points on the terminal side of an angle in


standard position, where P is on the unit circle and Q on the circle of radius r
(not necessarily 1) with center also at the origin, as shown above. Observe that
we can use similar triangles to obtain
cos = x1 =

x1
x
=
1
r

and

sin = y1 =

y1
y
= .
1
r

We may then further generalize the definitions of the six circular functions.

138
All rights reserved. No part of this material may be reproduced or transmitted in any form or by any means electronic or mechanical including photocopying without written permission from the DepEd Central Office. First Edition, 2016.

Let be an angle in standard


position, Q(x, y) any point on the terp
minal side of , and r = x2 + y 2 > 0. Then
sin =

y
r

r
csc = , y 6= 0
y

cos =

x
r

sec =

tan =

r
, x 6= 0
x
x
cot = , y 6= 0
y

y
, x 6= 0
x

Seatwork/Homework 3.2.1

C
O

PY

We then have a second solution for Example 3.2.3 as follows. With sin s = 43
and sin s = yr , we may choose y = 3 and r = 4 (which is always positive). In
this case, we can solve for x, which is positive since cos s = x4 is given to be
positive.

7
4 = x2 + (3)2 = x = 7 = cos s =
4
1. Given , find the exact values of the six circular functions.

(a) = 30
Answer:
sin 30 = 21 , cos 30 =

2 3
, cot 30 = 3
3
(b) = 3
4

tan 30 =

3
,
3

csc 30 = 2, sec 30 =

EP
E

3
,
2

2
2
3
=
,
cos
=

, tan 3
= 1, csc 3
= 2, sec 3
=
Answer: sin 3
4
2
4
2
4
4
4

3
2, cot 4 = 1
(c) = 150

Answer: sin(150 ) = 12 , cos(150 ) = 23 , tan(150 ) = 33 , csc(150 ) =

2, sec(150 ) = 2 3 3 , cot(150 ) = 3
(d) = 4
3

3
4
4
1
4
Answer:
sin(
)
=
,
cos(
)
=

,
tan(
)
=

3, csc( 4
)=
3
2
3
2
3
3

2 3
3
4
4
, sec( 3 ) = 2, cot( 3 ) = 3
3

2. Given a value of one circular function and sign of another function (or the
quadrant where the angle lies), find the value of the indicated function.

Answer: 23
Answer: 54

(a) sin = 21 , in QI; cos


(b) cos = 53 , in QIV; csc
(c) sin = 37 , sec < 0; tan

Answer:

(d) cot = 29 , cos > 0; csc

Answer:

3 10
20

985

139
All rights reserved. No part of this material may be reproduced or transmitted in any form or by any means electronic or mechanical including photocopying without written permission from the DepEd Central Office. First Edition, 2016.

3.2.2. Reference Angle

C
O

PY

We observe that if 1 and 2 are coterminal angles, the values of the six circular
or trigonometric functions at 1 agree with the values at 2 . Therefore, in finding
the value of a circular function at a number , we can always
to a number
 reduce
14
2
=
sin

4
=
sin
.
Also, observe
between 0 and 2. For example, sin 14
3
3
3
2

from Figure 3.7 that sin 3 = sin 3 .

Figure 3.7

EP
E

In general, if 1 , 2 , 3 , and 4 are as shown in Figure 3.8 with P (1 ) =


(x1 , y1 ), then each of the x-coordinates of P (2 ), P (3 ), and P (4 ) is x1 , while
the y-coordinate is y1 . The correct sign is determined by the location of the
angle. Therefore, together with the correct sign, the value of a particular circular
function at an angle can be determined by its value at an angle 1 with radian
measure between 0 and 2 . The angle 1 is called the reference angle of .

Figure 3.8

140
All rights reserved. No part of this material may be reproduced or transmitted in any form or by any means electronic or mechanical including photocopying without written permission from the DepEd Central Office. First Edition, 2016.

C
O

Figure 3.9

PY

The signs of the coordinates of P () depends on the quadrant or axis where


it terminates. It is important to know the sign of each circular function in each
quadrant. See Figure 3.9. It is not necessary to memorize the table, since the
sign of each function for each quadrant is easily determined from its definition.
We note that the signs of cosecant, secant, and cotangent are the same as sine,
cosine, and tangent, respectively.

EP
E

Using the fact that the unit circle is symmetric with respect to the x-axis, the
y-axis, and the origin, we can identify the coordinates of all the points using the
coordinates of corresponding points in the Quadrant I, as shown in Figure 3.10
for the special angles.

Figure 3.10

141
All rights reserved. No part of this material may be reproduced or transmitted in any form or by any means electronic or mechanical including photocopying without written permission from the DepEd Central Office. First Edition, 2016.

Example 3.2.4. Use reference angle and appropriate sign to find the exact value
of each expression.
and cos 11
(3) sin 150
(1) sin 11
6
6

(2) cos 7
(4) tan 8
6
3

Solution. (1) The reference angle of 11


is 6 , and it lies in Quadrant IV wherein
6
sine and cosine are negative and positive, respectively.
11

1
= sin =
6
6
2

3
11
= cos =
cos
6
6
2

PY

sin

(3) sin 150 = sin 30 =


= tan

cos 3
3
sin

3
2
1
2

= 3

(4) tan

8
3

1
2

C
O

(2) The angle 7


lies in Quadrant II wherein cosine is negative, and its refer6
ence angle is 6 .



7

3
cos
= cos =
6
6
2

EP
E

Seatwork/Homework 3.2.2

Use reference angle and appropriate sign to find the exact value of each expression.
(1) sin 510

(2) tan(225 )

Answer: 1

(3) sec 13
3

(4) cot 10
3

1
2

Answer:

Answer: 2

Answer:

3
3

Exercises 3.2
1. Find the exact value.
(a) sin 600

Answer:

(b) tan(810 )

3
2

Answer: Undefined

Answer: 2

(c) sec 585


142

All rights reserved. No part of this material may be reproduced or transmitted in any form or by any means electronic or mechanical including photocopying without written permission from the DepEd Central Office. First Edition, 2016.

1
2
Answer: 12
Answer: 12

(d) cos(420 )

Answer:

(e) sin 7
6
(f) cos 5
3
(g) tan 3
4

Answer: 1

(h) sec 2
3

Answer: 2

(i) csc
(j) cot

(k) cos 4
3
(l) tan 17
3

Answer: 2

Answer: 3

Answer: 12

Answer:

PY

11
6
35
6

(m) cos 7
4

Answer:

(o) sin 4
3

(p) sec 23
6

3
3

2
2

Answer: 2

(q) csc 13
3
(r) tan 5
6

3
2

Answer: 2 3 3

Answer: 2 3 3

Answer: 33

Answer:

C
O

(n) sec 19
4

2. Find the exact value of each expression.


(a) sin2 150 + cos2 150

EP
E

(b) cos(30 ) + sin 420

Answer: 1

Answer: 3

(c) tan(225 ) + tan 405

Answer: 0

(d) sec 750 csc(300 )

Answer: 0

(e) cos2

Answer: 1

(f)

Answer: 0

(g)

2
+ sin2 2
3
3
11
5
sin 6 + cos 3
2 cos 5
sin 5
3
2
2
tan 4 + 2 cos 8
3

(h)
(i)

(j)

tan 2
tan 5
3
6
1+tan 2
tan 5
3
6

Answer: 0

sin 13
6

Answer: 12
Answer:

11
cos 6
6

6 +cos 5
6

sin
sin(

Teaching Notes
(sin x)2 is denoted
by sin2 x.
Similarly, this
notation is used
with the other
trigonometric
functions. In
general, for a
positive integer n,
sinn x = (sin x)n .

3
3

Answer: 1

3. Compute P (), and find the exact values of the six circular functions.
(a) =

19
6

Answer: P () =

csc 19
= 2, sec
6

23 , 21 , sin 19
= 21 ,
6

19
= 2 3 3 , cot 19
= 3
6
6

cos 19
=
6

3
,
2

tan 19
=
6

3
,
3

143
All rights reserved. No part of this material may be reproduced or transmitted in any form or by any means electronic or mechanical including photocopying without written permission from the DepEd Central Office. First Edition, 2016.

(b) =

32
3



=
Answer: P () = 21 , 23 , sin 32
3
=
csc 32
3

2 3
,
3

3
,
2

sec 32
= 2, cot 32
=
3
3

1
32
cos 32
=

,
tan
=

3,
3
2
3

3
3

4. Given the value of a particular circular function and an information about the
angle , find the values of the other circular functions.
1
2

and

3
2

< < 2

Answer: sec = 2, sin =

8
and 0 < < 2
(b) sin = 17
Answer: csc = 17
, cos =
8

2 13
13

and

Answer: sec =
23

3
,
2

15
,
17

3
< < 2
2

13
, sin =
2

tan = 3, csc = 2 3 3 , cot = 33

tan =

8
,
15

sec =

17
,
15

cot =

15
8

3 1313 , tan = 23 , csc =

13
,
3

cot =

C
O

(c) cos =

PY

(a) cos =

Lesson 3.3. Graphs of Circular Functions and Situational


Problems

EP
E

Time Frame: 6 one-hour sessions

Learning Outcomes of the Lesson


At the end of the lesson, the student is able to:

(1) determine the domain and range of the different circular functions;

(2) graph the six circular functions with its amplitude, period, and phase shift;
and
(3) solve situational problems involving circular functions.
Lesson Outline
(1) Domain and range of circular functions
(2) Graphs of circular functions
(3) Amplitude, period, and phase shift
Introduction
There are many things that occur periodically. Phenomena like rotation of
the planets and comets, high and low tides, and yearly change of the seasons
144

All rights reserved. No part of this material may be reproduced or transmitted in any form or by any means electronic or mechanical including photocopying without written permission from the DepEd Central Office. First Edition, 2016.

follow a periodic pattern. In this lesson, we will graph the six circular functions
and we will see that they are periodic in nature.
3.3.1. Graphs of y = sin x and y = cos x
Recall that, for a real number x, sin x = sin for an angle with measure x
radians, and that sin is the second coordinate of the point P () on the unit
circle. Since each x corresponds to an angle , we can conclude that

PY

(1) sin x is defined for any real number x or the domain of the sine function is
R, and
(2) the range of sine is the set of all real numbers between 1 and 1 (inclusive).

C
O

From the definition, it also follows that sin(x+2) = sin x for any real number
x. This means that the values of the sine function repeat every 2 units. In this
case, we say that the sine function is a periodic function with period 2.
Table 3.11 below shows the values of y = sin x, where x is the equivalent radian
measure of the special angles and their multiples from 0 to 2. As commented
above, these values determine the behavior of the function on R.

2
2

3
2

7
6

12

0.5

2
3

3
2

1
2

EP
E

0.5

5
4

2
2

0.71 0.87
4
3

3
2

0.87 0.71

3
2

0.71 0.87 1

3
4

2
2

5
3

3
2

7
4

2
2

5
6

1
2

0.5

11
6

12

0.87 0.71 0.5

Table 3.11

From the table, we can observe that as x increases from 0 to 2 , sin x also
increases from 0 to 1. Similarly, as x increases from 3
to 2, sin x also increases
2
from 1 to 0. On the other hand, notice that as x increases from 2 to , sin x
decreases from 1 to 0. Similarly, as x increases from to 3
, sin x decreases from
2
0 to 1.

To sketch the graph of y = sin x, we plot the points presented in Table 3.11,
and join them with a smooth curve. See Figure 3.12. Since the graph repeats
every 2 units, Figure 3.13 shows periodic graph over a longer interval.
145

All rights reserved. No part of this material may be reproduced or transmitted in any form or by any means electronic or mechanical including photocopying without written permission from the DepEd Central Office. First Edition, 2016.

Teaching Notes
It is a good
exercise to
construct the
graph of the sine
function using the
height of P ().
Put the unit circle
side-by-side with
the coordinate
plane for the
graph, and trace
the height for each
value of x onto the
graph of y = sin x.

C
O

PY

Figure 3.12

Figure 3.13

We can make observations about the cosine function that are similar to the
sine function.
y = cos x has domain R and range [1, 1].

EP
E

y = cos x is periodic with period 2. The graph of y = cos x is shown in


Figure 3.14.

Figure 3.14

From the graphs of y = sin x and y = cos x in Figures 3.13 and 3.14, respectively, we observe that sin(x) = sin x and cos(x) = cos x for any real
number x. In other words, the graphs of y = cos(x) and y = cos x are the same,
while the graph of y = sin(x) is the same as that of y = sin x.
In general, if a function f satisfies the property that f (x) = f (x) for all x
in its domain, we say that such function is even. On the other hand, we say that
a function f is odd if f (x) = f (x) for all x in its domain. For example, the
functions x2 and cos x are even, while the functions x3 3x and sin x are odd.
146
All rights reserved. No part of this material may be reproduced or transmitted in any form or by any means electronic or mechanical including photocopying without written permission from the DepEd Central Office. First Edition, 2016.

3.3.2. Graphs of y = a sin bx and y = a cos bx

C
O

Figure 3.15

PY

Using a table of values from 0 to 2, we can sketch the graph of y = 3 sin x, and
compare it to the graph of y = sin x. See Figure 3.15 wherein the solid curve
belongs to y = 3 sin x, while the dashed curve to y = sin x. For instance, if x = 2 ,
then y = 1 when y = sin x, and y = 3 when y = 3 sin x. The period, x-intercepts,
and domains are the same for both graphs, while they differ in the range. The
range of y = 3 sin x is [3, 3].

In general, the graphs of y = a sin x and y = a cos x with a > 0 have the same
shape as the graphs of y = sin x and y = cos x, respectively. If a < 0, there is
a reflection across the x-axis. The range of both y = a sin x and y = a cos x is
[|a|, |a|].

EP
E

In the graphs of y = a sin x and y = a cos x, the number |a| is called


its amplitude. It dictates the height of the curve. When |a| < 1,
the graphs are shrunk vertically, and when |a| > 1, the graphs are
stretched vertically.

Teaching Notes
Review or teach
the reflection
across the x-axis
when the sign of
the function is
changed.

Now, in Table 3.16, we consider the values of y = sin 2x on [0, 2].

3
2

0.87

x
y

3
2

0.87

0
3
2

0.87

7
6

3
2

5
4

4
3

3
2

0.87

0.87

2
3

5
6

0.87

0.87

5
3

7
4

11
6

23

0.87

3
2

3
2

3
4

3
2

Table 3.16

147
All rights reserved. No part of this material may be reproduced or transmitted in any form or by any means electronic or mechanical including photocopying without written permission from the DepEd Central Office. First Edition, 2016.

PY

Figure 3.17

C
O

Figure 3.17 shows the graphs of y = sin 2x (solid curve) and y = sin x (dashed
curve) over the interval [0, 2]. Notice that, for sin 2x to generate periodic values
similar to [0, 2] for y = sin x, we just need values of x from 0 to . We then
expect the values of sin 2x to repeat every units thereafter. The period of
y = sin 2x is .
2
.
|b|
If 0 < |b| < 1, the graphs are stretched horizontally, and if |b| > 1, the
graphs are shrunk horizontally.

If b 6= 0, then both y = sin bx and y = cos bx have period given by

EP
E

To sketch the graphs of y = a sin bx and y = a cos bx, a, b 6= 0, we may proceed


with the following steps:
(1) Determine the amplitude |a|, and find the period 2
. To draw one cycle
|b|
of the graph (that is, one complete graph for one period), we just need to
.
complete the graph from 0 to 2
|b|

(2) Divide the interval into four equal parts, and get five division points: x1 = 0,
x2 , x3 , x4 , and x5 = 2
, where x3 is the midpoint between x1 and x5 (that
|b|
1
is, 2 (x1 + x5 ) = x3 ), x2 is the midpoint between x1 and x3 , and x4 is the
midpoint between x3 and x5 .

(3) Evaluate the function at each of the five x-values identified in Step 2. The
points will correspond to the highest point, lowest point, and x-intercepts
of the graph.
(4) Plot the points found in Step 3, and join them with a smooth curve similar
to the graph of the basic sine curve.
(5) Extend the graph to the right and to the left, as needed.
148
All rights reserved. No part of this material may be reproduced or transmitted in any form or by any means electronic or mechanical including photocopying without written permission from the DepEd Central Office. First Edition, 2016.

Example 3.3.1. Sketch the graph of one cycle of y = 2 sin 4x.


Solution.

(1) The period is

2
4

= 2 , and the amplitude is 2.

(2) Dividing the interval [0, 2 ] into 4 equal parts, we get the following xcoordinates: 0, 8 , 4 , 3
, and 2 .
8
(3) When x = 0, 4 , and 2 , we get y = 0. On the other hand, when x = 8 , we
.
have y = 2 (the amplitude), and y = 2 when x = 3
8

C
O

PY

(4) Draw a smooth curve by connecting the points. There is no need to proceed
to Step 5 because the problem only asks for one cycle.

EP
E

Example 3.3.2. Sketch the graph of y = 3 cos x2 .


Solution.

(1) The amplitude is | 3| = 3, and the period is

2
1
2

= 4.

(2) We divide the interval [0, 4] into four equal parts, and we get the following
x-values: 0, , 2, 3, and 4.

(3) We have y = 0 when x = and 3, y = 3 when x = 0 and 4, and y = 3


when x = 2.

(4) We trace the points in Step 3 by a smooth curve.


(5) We extend the pattern in Step 4 to the left and to the right.

149
All rights reserved. No part of this material may be reproduced or transmitted in any form or by any means electronic or mechanical including photocopying without written permission from the DepEd Central Office. First Edition, 2016.


Example 3.3.3. Sketch the graph of two cycles of y = 21 sin 2x
.
3


is the same
Solution. Since the sine function is odd, the graph of y = 21 sin 2x
3
as that of y = 12 sin 2x
.
3
(1) The amplitude is 12 , and the period is

2
2
3

= 3.

(2) Dividing the interval [0, 3] into four equal parts, we get the x-coordinates
of the five important points:

(3) We get y = 0 when x = 0,


9
.
4

3
,
2

3
2

+ 3
9
=
.
2
4

PY

0 + 3
3
2
=
,
2
4

3
0 + 3
=
,
2
2

and 3, y = 21 when

3
,
4

and y =

1
2

when

C
O

(4) We trace the points in Step 3 by a smooth curve.

EP
E

(5) We extend the pattern in Step 4 by one more period to the right.

Seatwork/Homework 3.3.2

(1) Sketch the graph of one cycle of y = 21 sin 3x.

Answer:


(2) Sketch the graph of two cycles of y = 2 cos x2 .
Answer:

150
All rights reserved. No part of this material may be reproduced or transmitted in any form or by any means electronic or mechanical including photocopying without written permission from the DepEd Central Office. First Edition, 2016.

(3) Sketch the graph of y = 2 cos 4x.

PY

Answer:


(4) Sketch the graph of one cycle of y = 3 sin x3 .

C
O

Answer:

EP
E

3.3.3. Graphs of y = a sin b(x c) + d and y = a cos b(x c) + d


We first compare the graphs of y = sin x and y = sin x
values and the 5-step procedure discussed earlier.

using a table of

As x runs from 3 to 7
, the value of the expression x 3 runs from 0 to 2. So
3
for one cycle of the graph of y = sin x 3 , we then expect to have the graph of
y = sin x starting from x = 3 . This is confirmed by the values in Table 3.18. We
then apply a similar procedure to complete one cycle of the graph; that is, divide
the interval [ 3 , 7
] into four equal parts, and then determine the key values of
3
x in sketching the graphs as discussed earlier. The one-cycle graph of y = sin x
(dashed curve) and the corresponding one-cycle graph of y = sin x 3 (solid
curve) are shown in Figure 3.19.
x
x

sin x

5
6

4
3

11
6

7
3

3
2

Teaching Notes
Review or teach
the horizontal
translation rule: if
x is replaced by
x h in the
equation, the
graph is translated
|h| units to the
right if h > 0 and
to the left if h < 0.

Table 3.18

151
All rights reserved. No part of this material may be reproduced or transmitted in any form or by any means electronic or mechanical including photocopying without written permission from the DepEd Central Office. First Edition, 2016.

Figure 3.19

PY


Observe that the graph of y = sin x 3 shifts 3 units to the right of
y = sin x. Thus, they have the same period, amplitude, domain, and range.

C
O

The graphs of

y = a sin b(x c) and y = a cos b(x c)

have the same shape as y = a sin bx and y = a cos bx, respectively, but
shifted c units to the right when c > 0 and shifted |c| units to the left
if c < 0. The number c is called the phase shift of the sine or cosine
graph.

EP
E

Example 3.3.4. In the same Cartesian


plane, sketch one cycle of the graphs of

y = 3 sin x and y = 3 sin x + 4 .

Solution. We have sketched the graph


 of y = 3 sin x earlier at the start of the

lesson. We consider y = 3 sin x + 4 . We expect that it has the same shape as


that of y = 3 sin x, but shifted some units.

Here, we have a = 3, b = 1, and c = 4 . From these constants, we get


the amplitude, the period, and the phase shift, and these are 3, 2, and 4 ,
respectively.
One cycle starts at x = 4 and ends at x = 4 + 2 =
the important values of x.
4 +
2

7
4

3
,
4

4 +
2

x
y = 3 sin x +

3
4

,
4

3
4

+
2

7
4

7
.
4

We now compute

5
4

3
4

5
4

7
4

152
All rights reserved. No part of this material may be reproduced or transmitted in any form or by any means electronic or mechanical including photocopying without written permission from the DepEd Central Office. First Edition, 2016.

PY

C
O

While the effect of c in y = a sin b(x c) and y = a cos b(x c) is


a horizontal shift of their graphs from the corresponding graphs of
y = a sin bx and y = a cos bx, the effect of d in the equations y =
a sin b(x c) + d and y = a cos b(x c) + d is a vertical shift. That is,
the graph of y = a sin b(x c) + d has the same amplitude, period, and
phase shift as that of y = a sin b(x c), but shifted d units upward
when d > 0 and |d| units downward when d < 0.

Teaching Notes
Review or teach
the vertical
translation rule: if
the equation
y = f (x) is
changed to
y = f (x) + k, the
graph is translated
|k| units upward if
k > 0 and
downward if k < 0.

Example 3.3.5. Sketch the graph of

EP
E



y = 2 cos 2 x
3.
6

Solution. Here, a = 2, b = 2, c= 6 , and d = 3. We first sketch one cycle of


the graph of y = 2 cos 2 x 6 , and then extend this graph to the left and to
the right, and then move the resulting graph 3 units downward.

The graph of y = 2 cos 2 x 6 has amplitude 2, period , and phase shift

.
6

Start of one cycle:


End of the cycle:

+ 7
2
6
=
,
2
3

+ =

7
6

+ 2
5
3
=
,
2
12
x


y = 2 cos 2 x 6

y = 2 cos 2 x 6 3

2
3

+
2

7
6

11
12

5
12

2
3

11
12

7
6

153
All rights reserved. No part of this material may be reproduced or transmitted in any form or by any means electronic or mechanical including photocopying without written permission from the DepEd Central Office. First Edition, 2016.

PY

C
O

Before we end this sub-lesson, we make the following observation, which will
be used in the discussion on simple harmonic motion (Sub-Lesson 3.3.6).
Different Equations, The Same Graph
1. The graphs of y = sin x and y = sin(x + 2k), k any integer, are
the same.

2. The graphs of y = sin x, y = sin(x + ), y = cos(x 2 ), and


y = cos(x + 2 ) are the same.

EP
E

3. In general, the graphs of

y = a sin b(x c) + d,

y = a sin[b(x c) + + 2k] + d,
y = a cos[b(x c)

+ 2k] + d,

and

y = a cos[b(x c) +

+ 2k] + d,

where k is any integer, are all the same.


Similar observations are true for cosine.

Seatwork/Homework 3.3.3
(1) In the same Cartesian
 plane, sketch one cycle of the graphs of y = 3 cos x

and y = 3 cos x + 3 1.
154
All rights reserved. No part of this material may be reproduced or transmitted in any form or by any means electronic or mechanical including photocopying without written permission from the DepEd Central Office. First Edition, 2016.

PY

Answer:

C
O

1
(2) In the same Cartesian plane,
 sketch one cycle of the graphs of y = 4 sin 2x

1
and y = 2 4 sin 2 x 4 .

EP
E

Answer:

(3) Sketch the graph of y = 2 sin


Answer:


x 2.

155
All rights reserved. No part of this material may be reproduced or transmitted in any form or by any means electronic or mechanical including photocopying without written permission from the DepEd Central Office. First Edition, 2016.

3.3.4. Graphs of Cosecant and Secant Functions


We know that csc x =
graph of y = csc x.

1
sin x

if sin x 6= 0. Using this relationship, we can sketch the

First, we observe that the domain of the cosecant function is


{x R : sin x 6= 0} = {x R : x 6= k, k Z}.

5
6

y = sin x

1
2

1
2

y = csc x

und

7
6

3
2

11
6

12 1 12

2
0

C
O

PY

Table 3.20 shows the key numbers (that is, numbers where y = sin x crosses the
x-axis, attain its maximum and minimum values) and some neighboring points,
where und stands for undefined, while Figure 3.21 shows one cycle of the
graphs of y = sin x (dashed curve) and y = csc x (solid curve). Notice the
asymptotes of the graph y = csc x.

und 2 1 2 und

EP
E

Table 3.20

Figure 3.21

We could also sketch the graph of csc x directly from the graph of y = sin x
by observing the following facts:
(1) If sin x = 1 (or 1), then csc x = 1 (or 1).
(2) At each x-intercept of y = sin x, y = csc x is undefined; but a vertical
asymptote is formed because, when sin x is close to 0, the value of csc x will
have a big magnitude with the same sign as sin x.
156
All rights reserved. No part of this material may be reproduced or transmitted in any form or by any means electronic or mechanical including photocopying without written permission from the DepEd Central Office. First Edition, 2016.

C
O

Figure 3.22

PY

Refer to Figure 3.22 for the graphs of y = sin x (dashed curve) and y = csc x
(solid curve) over a larger interval.

Like the sine and cosecant functions, the cosine and secant functions are also
reciprocals of each other. Therefore, y = sec x has domain
{x R : cos x 6= 0} = {x R : x 6=

k
, k odd integer}.
2

EP
E

Similarly, the graph of y = sec x can be obtained from the graph of y = cos x.
These graphs are shown in Figure 3.23.

Figure 3.23

Example 3.3.6. Sketch the graph of y = 2 csc x2 .


Solution. First, we sketch the graph of y = 2 sin x2 , and use the technique discussed above to sketch the graph of y = 2 csc x2 .
157
All rights reserved. No part of this material may be reproduced or transmitted in any form or by any means electronic or mechanical including photocopying without written permission from the DepEd Central Office. First Edition, 2016.

C
O

PY

The vertical asymptotes of y = 2 csc x2 are the x-intercepts of y = 2 sin x2 :


x = 0, 2, 4, . . .. After setting up the asymptotes, we now sketch the graph
of y = 2 csc x2 as shown below.

EP
E

Example 3.3.7. Sketch the graph of y = 2 sec 2x.

Solution. Sketch the graph of y = cos 2x (note that it has period ), then sketch
the graph of y = sec 2x (as illustrated above), and then move the resulting
graph 2 units upward to obtain the graph of y = 2 sec 2x.

158
All rights reserved. No part of this material may be reproduced or transmitted in any form or by any means electronic or mechanical including photocopying without written permission from the DepEd Central Office. First Edition, 2016.

PY

Seatwork/Homework 3.3.4

EP
E

Answer:

C
O

(1) Sketch the graph of y = sec x on the interval [0, 2].



(2) Sketch the graph of y = 2 csc 4x 1 on the interval 2 , 2 .

Answer:

159
All rights reserved. No part of this material may be reproduced or transmitted in any form or by any means electronic or mechanical including photocopying without written permission from the DepEd Central Office. First Edition, 2016.

3.3.5. Graphs of Tangent and Cotangent Functions


sin x
, where cos x 6= 0. From this definition of the tangent
We know that tan x = cos
x
function, it follows that its domain is the same as that of the secant function,
which is

{x R : cos x 6= 0} = {x R : x 6=

k
, k odd integer}.
2

To sketch the graph of y = tan


 x, it will be enough to know its one-cycle

graph on the open interval 2 , 2 . See Table 3.24 and Figure 3.25.
y = tan x
x

C
O

und

3
3

y = tan x

3
3

und

Table 3.24

EP
E

Teaching Notes
There is also a way
of sketching the
graph of y = tan x
based on the
tangent segment to
the unit circle,
similar to the
construction
described in
sketching the
graph of y = sin x.
But we do not go
anymore into the
details of this
approach.

PY

We note that tan x = 0 when sin x = 0 (that is, when x = k, k any integer), and
that the graph of y = tan x has asymptotes x = k
, k odd integer. Furthermore,
2
by recalling the signs of tangent from Quadrant I to Quadrant IV and its values,
we observe that the tangent function is periodic with period .

Figure 3.25

In the same manner, the domain of y = cot x =

cos x
sin x

is

{x R : sin x 6= 0} = {x R : x 6= k, k Z},
and its period is also . The graph of y = cot x is shown in Figure 3.26.
160
All rights reserved. No part of this material may be reproduced or transmitted in any form or by any means electronic or mechanical including photocopying without written permission from the DepEd Central Office. First Edition, 2016.

PY

Figure 3.26

C
O

In general, to sketch the graphs of y = a tan bx and y = a cot bx, a 6= 0 and


b > 0, we may proceed with the following steps:

(1) Determine the period b . Then
we draw one cycle of the graph on 2b
, 2b


for y = a tan bx, and on 0, b for y = a cot bx.

(2) Determine the two adjacent vertical asymptotes. For y = a tan bx, these

vertical asymptotes are given by x = 2b


. For y = a cot bx, the vertical
asymptotes are given by x = 0 and x = b .

EP
E

(3) Divide the interval formed by the vertical asymptotes in Step 2 into four
equal parts, and get three division points exclusively between the asymptotes.
(4) Evaluate the function at each of these x-values identified in Step 3. The
points will correspond to the signs and x-intercept of the graph.

(5) Plot the points found in Step 3, and join them with a smooth curve approaching to the vertical asymptotes. Extend the graph to the right and to
the left, as needed.

Example 3.3.8. Sketch the graph of y = 21 tan 2x.

Solution. The period of the function is 2 , and the adjacent asymptotes are x =
4 , 3
, . . .. Dividing the interval 4 , 4 into four equal parts, the key x-values
4
are 8 , 0, and 8 .
x
y = 21 tan 2x

8
21

1
2

161
All rights reserved. No part of this material may be reproduced or transmitted in any form or by any means electronic or mechanical including photocopying without written permission from the DepEd Central Office. First Edition, 2016.

PY

C
O

Example 3.3.9. Sketch the graph of y = 2 cot x3 on the interval (0, 3).

Solution. The period of the function is 3, and the adjacent asymptotes are x = 0
and x = 3. We now divide the interval (0, 3) into four equal parts, and the
, 3
, and 9
.
key x-values are 3
4
2
4
x

3
4

9
4

EP
E

y = 2 cot x3

3
2

162
All rights reserved. No part of this material may be reproduced or transmitted in any form or by any means electronic or mechanical including photocopying without written permission from the DepEd Central Office. First Edition, 2016.

Seatwork/Homework 3.3.5
(1) Sketch the graph of y = cot(x) on the interval [, ].

C
O

PY

Answer:

(2) Sketch the graph of y = 2 tan x4 on the interval [2, 2].

EP
E

Answer:

3.3.6. Simple Harmonic Motion


Repetitive or periodic behavior is common in nature. As an example, the timetelling device known as sundial is a result of the predictable rising and setting
of the sun everyday. It consists of a flat plate and a gnomon. As the sun moves
across the sky, the gnomon casts a shadow on the plate, which is calibrated to
tell the time of the day.

163
All rights reserved. No part of this material may be reproduced or transmitted in any form or by any means electronic or mechanical including photocopying without written permission from the DepEd Central Office. First Edition, 2016.

PY
C
O

Sundial, by liz west, 29 March 2007,

https://commons.wikimedia.org/wiki/File:Sundial 2r.jpg. Public Domain.

EP
E

Some motions are also periodic. When a weight is suspended on a spring,


pulled down, and released, the weight oscillates up and down. Neglecting resistance, this oscillatory motion of the weight will continue on and on, and its height
is periodic with respect to time.

t = 0 sec

t = 2.8 sec

164
All rights reserved. No part of this material may be reproduced or transmitted in any form or by any means electronic or mechanical including photocopying without written permission from the DepEd Central Office. First Edition, 2016.

t = 9 sec

PY

t = 6.1 sec

C
O

Periodic motions are usually modeled by either sine or cosine function, and are
called simple harmonic motions. Unimpeded movements of objects like oscillation, vibration, rotation, and motion due to water waves are real-life occurrences
that behave in simple harmonic motion.
Equations of Simple Harmonic Motion

The displacement y (directed height or length) of an object behaving


in a simple harmonic motion with respect to time t is given by one of
the following equations:

EP
E

y = a sin b(t c) + d

or

y = a cos b(t c) + d.

In both equations, we have the following information:

amplitude = |a| = 21 (M m) - the maximum displacement above


and below the rest position or central position or equilibrium, where
M is the maximum height and m is the minimum height;

period = 2
- the time required to complete one cycle (from one
|b|
highest or lowest point to the next);
frequency =

|b|
2

- the number of cycles per unit of time;

c - responsible for the horizontal shift in time; and

d - responsible for the vertical shift in displacement.


Example 3.3.10. A weight is suspended from a spring and is moving up and
down in a simple harmonic motion. At start, the weight is pulled down 5 cm below
the resting position, and then released. After 8 seconds, the weight reaches its
165
All rights reserved. No part of this material may be reproduced or transmitted in any form or by any means electronic or mechanical including photocopying without written permission from the DepEd Central Office. First Edition, 2016.

highest location for the first time. Find the equation of the motion.
Solution. We are given that the weight is located at its lowest position at t = 0;
that is, y = 5 when t = 0. Therefore, the equation is y = 5 cos bt.
Because it took the weight 8 seconds from the lowest point to its immediate
highest point, half the period is 8 seconds.

PY

1 2

= 8 = b =
= y = 5 cos
2
2 b
8
8
? Example 3.3.11. Suppose you ride a Ferris wheel. The lowest point of the
wheel is 3 meters off the ground, and its diameter is 20 m. After it started, the
Ferris wheel revolves at a constant speed, and it takes 32 seconds to bring you
back again to the riding point. After riding for 150 seconds, find your approximate
height above the ground.

C
O

Solution. We ignore first the fixed value of 3 m off the ground, and assume that
the central position passes through the center of the wheel and is parallel to the
ground.

Let t be the time (in seconds) elapsed that you have been riding the Ferris
wheel, and y is he directed distance of your location with respect to the assumed
central position at time t. Because y = 10 when t = 0, the appropriate model
is y = 10 cos bt for t 0.

EP
E

Given that the Ferris wheel takes 32 seconds to move from the lowest point
to the next, the period is 32.

2
= 32 = b =
=
b
16
When t = 150, we get y = 10 cos 150
3.83.
16

y = 10 cos

t
16

Bringing back the original condition given in the problem that the riding point
is 3 m off the ground, after riding for 150 seconds, you are approximately located
3.83 + 13 = 16.83 m off the ground.
2
In the last example, the central position or equilibrium may be vertically
shifted from the ground or sea level (the role of the constant d). In the same way,
the starting point may also be horizontally shifted (the role of the constant c).
Moreover, as observed in Sub-Lesson 3.3.3 (see page 154), to find the function
that describes a particular simple harmonic motion, we can either choose
y = a sin b(t c) + d
or
y = a cos b(t c) + d,

and determine the appropriate values of a, b, c, and d. In fact, we can assume


that a and b are positive numbers, and c is the smallest such nonnegative number.
166
All rights reserved. No part of this material may be reproduced or transmitted in any form or by any means electronic or mechanical including photocopying without written permission from the DepEd Central Office. First Edition, 2016.

Example 3.3.12. A signal buoy in Laguna Bay bobs up and down with the
height h of its transmitter (in feet) above sea level modeled by h(t) = a sin bt + d
at time t (in seconds). During a small squall, its height varies from 1 ft to 9 ft
above sea level, and it takes 3.5 seconds from one 9-ft height to the next. Find
the values of the constants a, b, and d.
Solution. We solve the constants step by step.
The minimum and maximum values of h(t) are 1 ft and 9 ft, respectively.
Thus, the amplitude is a = 12 (M m) = 12 (9 1) = 4.

PY

Because it takes 3.5 seconds from one 9-ft height to the next, the period is
3.5. Thus, we have 2
= 3.5, which gives b = 4
.
b
7

C
O

Because the lowest point is 1 ft above the sea level and the amplitude is 4,
it follows that d = 5.
2

EP
E

Solution.

Example 3.3.13. A variable star is a star whose brightness fluctuates as observed from Earth. The magnitude of visual brightness of one variable star ranges
from 2.0 to 10.1, and it takes 332 days to observe one maximum brightness to
the next. Assuming that the visual brightness of the star can be modeled by the
equation y = a sin b(t c) + d, t in days, and putting t = 0 at a time when the
star is at its maximum brightness, find the constants a, b, c, and d, where a, b > 0
and c the least nonnegative number possible.
10.1 2.0
M m
=
= 4.05
2
2

2
= 332 = b =
b
166
d = a + m = 4.05 + 2.0 = 6.05

a=

For the (ordinary) sine function to start at the highest point at t = 0, the least
possible horizontal movement to the right (positive value) is 3
units.
2
bc =

3
2

c=

3
3
=
= 249
2b
2 166

? Example 3.3.14. The path of a fast-moving particle traces a circle with equation
(x + 7)2 + (y 5)2 = 36.
It starts at point (1, 5), moves clockwise, and passes the point (7, 11) for the
first time after traveling 6 microseconds. Where is the particle after traveling 15
microseconds?

167
All rights reserved. No part of this material may be reproduced or transmitted in any form or by any means electronic or mechanical including photocopying without written permission from the DepEd Central Office. First Edition, 2016.

Solution. As described above, we may choose sine or cosine function. Here, we


choose the sine function to describe both x and y in terms of time t in microseconds; that is, we let
x = a sin b(t c) + d and y = e sin f (t g) + h,
where we appropriately choose the positive values for a, b, e, and f , and the least
nonnegative values for c and g.

PY

The given circle has radius 6 and center (7, 5). Defining the central position
of the values of x as the line x = 7 and that of the values of y as the line y = 5,
we get a = e = 6, d = 7, and h = 5.
From the point (1, 5) to the point (7, 11) (moving clockwise), the particle
has traveled three-fourths of the complete cycle; that is, three-fourths of the
period must be 2.
b=f =

As the particle starts at (1, 5) and moves clockwise, the values of x start
at its highest value (x = 1) and move downward toward its central position
(x = 7) and continue to its lowest value (x = 13). Therefore, the graph of
a sin bt + d has to move 3
= 6 units to the right, and so we get c = 6.
2b
As to the value of g, we observe the values of y start at its central position
(y = 5) and go downward to its lowest value (y = 1). Similar to the argument
used in determining c, the graph of y = e sin f t + h has to move b = 4 units to
the right, implying that g = 4.

EP
E

Teaching Notes
Here, we need an
equation with the
same graph as
y = a sin(bt+ 2 )+d
that will fit in the
equation
y = a sin b(tc)+d,
where c is the least
nonnegative
possible number.
Recall the
observation made
on page 154.

C
O

3 2
3 2

=
=6
4 b
4 f

Hence, We have the following equations of x and y in terms of t:


x = 6 sin 4 (t 6) 7 and y = 6 sin 4 (t 4) + 5.

When t = 15, we get

and

x = 6 sin 4 (15 6) 7 = 7 + 3 2 2.76

y = 6 sin 4 (15 4) + 5 = 5 + 3 2 9.24.

That is, after traveling for 15 microseconds, the particle is located near the point
(2.76, 9.24).
2
Seatwork/Homework 3.3.6
? 1. A weight is suspended from a spring and is moving up and down in a simple
harmonic motion. At start, the weight is pushed up 6 cm above the resting
168
All rights reserved. No part of this material may be reproduced or transmitted in any form or by any means electronic or mechanical including photocopying without written permission from the DepEd Central Office. First Edition, 2016.

position, and then released. After 14 seconds, the weight reaches again to its
highest position. Find the equation of the motion, and locate the weight with
respect to the resting position after 20 seconds since it was released.
Answer: y = 6 cos 7 t or y = 6 sin pi7 (t + 72 ), location of the weight after 20
seconds: about 5.4 cm below the resting position

PY

2. Suppose the lowest point of a Ferris wheel is 1.5 meters off the ground, and its
radius is 15 m. It makes one complete revolution every 30 seconds. Starting at
the lowest point, find a cosine function that gives the height above the ground
of a riding child in terms of the time t in seconds.


Answer: y = 15 cos 15
t 15
+ 16.5
2
Exercises 3.3

C
O

1. Sketch two cycles of the graph (starting from x = 0) of the given function.
Indicate the amplitude, period, phase shift, domain, and range for each function.
(a) y = 4 sin x
Answer: amplitude = 4, period = 2, phase shift = 0, domain = R,
range = [4, 4]

EP
E

(b) y = 3 cos x
Answer: amplitude = 3, period = 2, phase shift = 0, domain = R,
range = [3, 3]
(c) y = cos x4
Answer: amplitude = 1, period = 8, phase shift = 0, domain = R,
range = [1, 1]

(d) y = sin 2x
Answer: amplitude = 1, period = , phase shift = 0, domain = R,
range = [1, 1]

169
All rights reserved. No part of this material may be reproduced or transmitted in any form or by any means electronic or mechanical including photocopying without written permission from the DepEd Central Office. First Edition, 2016.


,
2

phase shift = 0, domain = R,

C
O

PY

(e) y = 2 + sin 4x
Answer: amplitude = 1, period =
range = [1, 3]

(f) y = 1 + cos x
Answer: amplitude = 1, period = 2, phase shift = 0, domain = R,
range = [2, 0]
1
,
2

period =

(g) y = 21 sin 3x
Answer: amplitude =
range = [ 12 , 12 ]

2
,
3

phase shift = 0, domain = R,

EP
E

(h) y = 3 sin(x)
Answer: amplitude = 3, period = 2, phase shift = 0, domain = R,
range = [3, 3]

(i) y = 3 2 cos x2
Answer: amplitude = 2, period = 4, phase shift = 0, domain = R,
range = [1, 5]


(j) y = sin x 4
Answer: amplitude = 1, period = 2, phase shift =
range = [1, 1]

,
4

domain = R,

170
All rights reserved. No part of this material may be reproduced or transmitted in any form or by any means electronic or mechanical including photocopying without written permission from the DepEd Central Office. First Edition, 2016.


(k) y = 2 cos x + 3
Answer: amplitude = 2, period = 2, phase shift = 3 , domain = R,
range = [2, 2]

period = 2, phase shift =

C
O

2
,
3

,
2

domain = R,

,
3

domain = R,

EP
E


(m) y = 2 32 cos x 2
Answer: amplitude =
range = [ 34 , 83 ]

PY

(l) y = 3 sin(x 4)
Answer: amplitude = 3, period = 2, phase shift = 4, domain = R,
range = [3, 3]


(n) y = 4 cos x 3 + 2
Answer: amplitude = 4, period = 2, phase shift =
range = [2, 6]

2. Sketch the graph of the following functions.


(a) y = | sin x|
Answer:

171
All rights reserved. No part of this material may be reproduced or transmitted in any form or by any means electronic or mechanical including photocopying without written permission from the DepEd Central Office. First Edition, 2016.

EP
E

(c) y = |2 sin 2(x + )| 1


Answer:

C
O

PY

(b) y = |4 cos x| + 2
Answer:

3. Sketch the graph of each function over two periods, starting from x = 0.
Indicate the period, phase shift, domain, and range of each function.
(a) y = csc(x)
Answer: period = 2, phase shift = 0, domain = {x|x 6= k, k Z},
range = (, 1] [1, )

(b) y = cot(x)
Answer: period = , phase shift = 0, domain = {x|x 6= k, k Z},
range = R
(c) y = tan x
Answer: period = , phase shift = 0, domain = {x|x 6= (2k + 1) 2 , k
Z}, range = R

172
All rights reserved. No part of this material may be reproduced or transmitted in any form or by any means electronic or mechanical including photocopying without written permission from the DepEd Central Office. First Edition, 2016.

PY

EP
E

C
O

(d) y = sec x
Answer: period = 2, phase shift = 0, domain = {x|x 6= (2k + 1) 2 , k
Z}, range = (, 1] [1, )

(e) y = sec 3x
Answer: period = 2
, phase shift = 0, domain = {x|x 6= (2k + 1) 6 , k
3
Z}, range = (, 1] [1, )
(f) y = 3 csc x
Answer: period = 2, phase shift = 0, domain = {x|x 6= k, k Z},
range = (, 3] [3, )

(g) y = 4 sec 2x
3
Answer: period = 3, phase shift = 0, domain = {x|x 6= (2k + 1) 3
, k
4
Z}, range = (, 4] [4, )

(h) y = tan(x + )

173
All rights reserved. No part of this material may be reproduced or transmitted in any form or by any means electronic or mechanical including photocopying without written permission from the DepEd Central Office. First Edition, 2016.

C
O

PY

Answer: period = , phase shift = , domain = {x|x 6= (2k + 1) 2 , k


Z}, range = R

(i) y = tan x 2
Answer: period = , phase shift = 2 , domain = {x|x 6= k, k Z},
range = R

EP
E


(j) y = cot x + 4
Answer: period = , phase shift = 4 , domain = {x|x 6= (2k1) 4 , k even integer},
range = R
(k) y = 2 3 csc x
Answer: period = 2, phase shift = 0, domain = {x|x 6= k, k Z},
range = (, 1] [5, )

(l) y = 4 + sec 3x
, phase shift = 0, domain = {x|x 6= (2k + 1) 6 , k
Answer: period = 2
3
Z}, range = (, 3] [6, )

(m) y = 2 sec x 3
Answer: period = 2, phase shift = 3 , domain = {x|x 6= (2k + 1) 3
, k
4
Z}, range = (, 2] [2, )
(n) y = 2 3 sec 2x
3
Answer: period = 3, phase shift = 0, domain = {x|x 6= (2k + 1) 3
, k
4
Z}, range = (, 2] [5, )

174
All rights reserved. No part of this material may be reproduced or transmitted in any form or by any means electronic or mechanical including photocopying without written permission from the DepEd Central Office. First Edition, 2016.

PY

3
,
2

domain = {x|x 6= (2k + 1) 2 , k

EP
E

C
O


(o) y = 3 csc x 3
2
Answer: period = 2, phase shift =
Z}, range = (, 3] [3, )

4. Assuming that there is no vertical shift, find a function that describes a simple
harmonic motion with the following properties.
(a) sine function; displacement zero at time t = 0; moving up initially;
amplitude = 6 cm; period = 4 sec
Answer: y = 6 sin 2 t
(b) cosine function; highest point 4 cm above the equilibrium at time t = 0;
period = 10 sec
Answer: y = 5 cos 5 t
175
All rights reserved. No part of this material may be reproduced or transmitted in any form or by any means electronic or mechanical including photocopying without written permission from the DepEd Central Office. First Edition, 2016.

(c) cosine function; lowest point 9 cm below the equilibrium at time t = 0;


(t 52 )
period = 5 sec
Answer: y = 9 cos 2
5
5. A point P moving in a simple harmonic motion makes 10 complete revolutions
every 1 second. The amplitude of the motion is 3 m. Assuming that P is at
its minimum displacement with respect to the equilibrium when t = 0 and
there is a vertical shift of 2 m downward, find a sine function that describes
1
)2
the path traced by P in terms of time t.
Answer: y = 3 sin 20(t 40

PY

? 6. The path of a fast-moving particle (assuming constant speed) traces a circle


with equation
(x 3)2 + (y 4)2 = 25.

It starts at point (3, 1), moves counterclockwise, and passes the point (8, 4)
for the first time after traveling 7 microseconds. Where is the particle after
traveling for 20 microseconds?
Answer: about the point (1.87, 5.11)

C
O

Hint. The coordinates (x, y) of the location of the particle at time t (in mi

(t 7) + 3 and y = 5 sin 14
(t 7) + 4.
croseconds) are given by x = 5 cos 14

EP
E

7. A wooden ball is tied on a string 30 cm long, and is oscillating like a pendulum.


See figure below. It is initially pulled back at 90 angle with the vertical, and
is released with a push so that the ball reaches its maximum height back and
forth. If it reaches its maximum height again after 3 seconds, find its height
10 seconds after it was released.
Answer: 27 cm

Hint. The height h(t) (in cm) of the ball at time t (in seconds) is given by
h(t) = |30 sin 3 (t 23 )| + 12.

8. For what values of k do y = cot x and y = cot(x k) have the same graph?

Answer: any integer

9. For what values of k do y = sec x and y = sec(x k) have the same graph?

Answer: any even integer

176
All rights reserved. No part of this material may be reproduced or transmitted in any form or by any means electronic or mechanical including photocopying without written permission from the DepEd Central Office. First Edition, 2016.

10. Find the least positive value of c such that the graph of y = 2 sin 2(x + c)
coincide with that of y = 2 cos 2x.
Answer: 4
11. Find the largest positive value of c such that the graph of y = 2 cos 3(x c)
coincide with that of y = 2 cos 3(x 2).
Answer: 2 + 3

12. For what values of a do the graphs of y = a cos b(xc) and y = 2 sec 6 (x6)
never intersect for any values of b and c?
Answer: 2 < a < 2

PY

Lesson 3.4. Fundamental Trigonometric Identities


Time Frame: 4 one-hour sessions

C
O

Learning Outcomes of the Lesson

At the end of the lesson, the student is able to:

(1) determine whether an equation is an identity or a conditional equation;


(2) derive the fundamental trigonometric identities;

(3) simplify trigonometric expressions using fundamental trigonometric identities; and

EP
E

(4) prove other trigonometric identities using fundamental trigonometric identities.


Lesson Outline

(1) Domain of an equation

(2) Identity and conditional equation


(3) Fundamental trigonometric identities

(4) Proving trigonometric identities


Introduction

In previous lessons, we have defined trigonometric functions using the unit


circle and also investigated the graphs of the six trigonometric functions. This
lesson builds on the understanding of the different trigonometric functions by
discovery, deriving, and working with trigonometric identities.
3.4.1. Domain of an Expression or Equation
Consider the following expressions:

x2 1,
2x + 1,

x2

x
,
3x 4

x
.
x1

177
All rights reserved. No part of this material may be reproduced or transmitted in any form or by any means electronic or mechanical including photocopying without written permission from the DepEd Central Office. First Edition, 2016.

What are the real values of the variable x that make the expressions defined in
the set of real numbers?
In the first expression, every real value of x when substituted to the expression
makes it defined in the set of real numbers; that is, the value of the expression is
real when x is real.

PY

In the second expression, not every real value of x makes


the expression defined
in R. For example, when x = 0, the expression becomes 1, which is not a real
number.

x2 1 R x2 1 0 x 1 or x 1

Here, for x2 1 to be defined in R, x must be in (, 1] [1, ).


In the third expression, the values of x that make the denominator zero make
the entire expression undefined.

x = 4 or x = 1

C
O

x2 3x 4 = (x 4)(x + 1) = 0

x
is real when x 6= 4 and x 6= 1.
3x 4

In the fourth expression, because the expression x 1 is in the denominator,


x must be greater than 1. Although the value of the entire expression is 0 when
x = 0, we do not include 0 as allowed value of x because part of the expression
is not real when x = 0.
x2

Hence, the expression

EP
E

In the expressions above, the allowed values of the variable x constitute the
domain of the expression.
The domain of an expression (or equation) is the set of all real values of
the variable for which every term (or part) of the expression (equation)
is defined in R.

In the expressions above, the domains of the first, second, third, and fourth
expressions are R, (, 1] [1, ), R \ {1, 4}, and (1, ), respectively.
Example 3.4.1. Determine the domain of the expression/equation.

x+1
x2 1
(a) 3

2
x + 2x 8x
1x

(b) tan sin cos 2


(c) x2
(d) z

1 + x2 =
3

2
x2 1

cos2 z
= 4 sin z 1
1 + sin z
178

All rights reserved. No part of this material may be reproduced or transmitted in any form or by any means electronic or mechanical including photocopying without written permission from the DepEd Central Office. First Edition, 2016.

Solution. (a) x3 + 2x2 8x = x(x + 4)(x 2) = 0


4, or x = 2

x + 1 R x + 1 0 x 1
1x=0

x = 0, x =

x=1

Domain = [1, ) \ {4, 0, 1, 2}


= [1, 0) (0, 1) (1, 2) (2, )

(b) tan sin cos 2 =

Domain = R \

|
{ k
2

k odd integer}

PY

cos = 0

sin
sin cos 2
cos
k
, k odd integer
2

Domain = R \ {1, 1}
(d) 1 + sin z = 0

z=

C
O

(c) The expression 1+x2 is always positive,


and
so
1 + x2 is defined in R. On

3
the other hand, the expression x2 1 is also defined in R, but it cannot
be zero because it is in the denominator. Therefore, x should not be 1
and 1.
3
2

+ 2k, k Z

Domain = R \ { 3
+ 2k|k Z}
2

Seatwork/Homework 3.4.1

EP
E

Find the domain of the expression/equation.


x
x+1
2
2
2
Answer: R \ {1, 1}
x + 1 x + 2x + 1 x 1


1
1
(2) 2 sec2 =
+
Answer: R \ (2k + 1) 2 |k Z
1 sin 1 + sin


2
(3) 2 tan x = 2 cot x + 1
Answer: R \ k
|k

Z
2



1
tan x = sin x
Answer: R \ {1, 1} (2k + 1) 2 |k Z
(4)
1 x2

(1)

3.4.2. Identity and Conditional Equation


Consider the following two groups of equations:
Group A

Group B

(A1) x2 1 = 0

(B1) x2 1 = (x 1)(x + 1)

(A2) (x + 7)2 = x2 + 49
x2 4
= 2x 1
(A3)
x2

(B2) (x + 7)2 = x2 + 14x + 49


x2 4
(B3)
=x+2
x2
179

All rights reserved. No part of this material may be reproduced or transmitted in any form or by any means electronic or mechanical including photocopying without written permission from the DepEd Central Office. First Edition, 2016.

In each equation in Group A, some values of the variable that are in the
domain of the equation do not satisfy the equation (that is, do not make the
equation true). On the other hand, in each equation in Group B, every element
in the domain of the equation satisfies the given equation. The equations in
Group A are called conditional equations, while those in Group B are called
identities.

PY

An identity is an equation that is true for all values of the variable


in the domain of the equation. An equation that is not an identity is
called a conditional equation. (In other words, if some values of the
variable in the domain of the equation do not satisfy the equation,
then the equation is a conditional equation.)

(1) x3 2 = x

C
O

Example 3.4.2. Identify whether the given equation is an identity or a conditional equation. For each conditional equation, provide a value of the variable in
the domain that does not satisfy the equation.


3
2 x2 + 3 2x + 3 4

(2) sin2 = cos2 + 1

EP
E

(3) sin = cos 1

1 x
12 x+x
=
(4)
1x
1+ x

Solution. (1) This is an identity because this is simply factoring of difference of


two cubes.
(2) This is a conditional equation. If = 0, then the left-hand side of the equation
is 0, while the right-hand side is 2.

(3) This is also a conditional equation. If = 0, then both sides of the equation
are equal to 0. But if = , then the left-hand side of the equation is 0,
while the right-hand side is 2.

(4) This is an identity because the right-hand side of the equation is obtained by
rationalizing the denominator of the left-hand side.
2
Seatwork/Homework 3.4.2
Identify whether the given equation is an identity or a conditional equation. For
each conditional equation, provide a value of the variable in the domain that does
not satisfy the equation.
(1) 1 + x +

x2
1
=
1x
1x

Answer: identity
180

All rights reserved. No part of this material may be reproduced or transmitted in any form or by any means electronic or mechanical including photocopying without written permission from the DepEd Central Office. First Edition, 2016.

cos2 sin2
= cos sin
cos + sin
(3) tan = cot

Answer: conditional equation, =

(4) cos2 x = 2 cos x + 3

Answer: conditional equation, x = 0

Answer: identity

(2)

3.4.3. The Fundamental Trigonometric Identities

PY

Recall that if P (x, y) is the terminal point on the unit circle corresponding to ,
then we have
y
1
tan =
sin = y csc =
y
x
1
x
cos = x sec =
cot = .
x
y

C
O

From the definitions, the following reciprocal and quotient identities immediately follow. Note that these identities hold if is taken either as a real number
or as an angle.
Reciprocal Identities
1
sin

sec =

1
cos

cot =

1
tan

csc =

EP
E

Quotient Identities

tan =

sin
cos

cot =

cos
sin

We can use these identities to simplify trigonometric expressions.

Example 3.4.3. Simplify:


tan cos
(1)
sin
Solution.
(2)

(1)

tan cos
=
sin

(2)

cos
cot

sin
cos

cos
=1
sin

cos
cos
= cos = sin
cot
sin

If P (x, y) is the terminal point on the unit circle corresponding to , then


x2 + y 2 = 1. Since sin = y and cos = x, we get
sin2 + cos2 = 1.
181
All rights reserved. No part of this material may be reproduced or transmitted in any form or by any means electronic or mechanical including photocopying without written permission from the DepEd Central Office. First Edition, 2016.

Teaching Notes
The assumption in
the division is that
the divisor is
nonzero.

By dividing both sides of this identity by cos2 and sin2 , respectively, we obtain
tan2 + 1 = sec2

and 1 + cot2 = csc2 .

Pythagorean Identities
sin2 + cos2 = 1
tan2 + 1 = sec2

1 + cot2 = csc2

Example 3.4.4. Simplify:

(2)

1 + tan2
1 + cot2

(1) cos2 + cos2 tan2 = (cos2 )(1 + tan2 )


= cos2 sec2 = 1

1 + tan2
sec2
=
=
1 + cot2
csc2

1
cos2
1
sin2

sin2
= tan2
2
cos

C
O

Solution.

(2)

PY

(1) cos2 + cos2 tan2

In addition to the eight identities presented above, we also have the following
identities.

Even-Odd Identities
cos() = cos

EP
E

sin() = sin

tan() = tan

Teaching Notes
The corresponding
reciprocal
functions follow
the same
Even-Odd
Identities:
csc() = csc
sec() = sec
cot() = cot .

The first two of the negative identities can be obtained from the graphs of
the sine and cosine functions, respectively. (Please review the discussion on page
146.) The third identity can be derived as follows:
tan() =

sin()
sin
=
= tan .
cos()
cos

The reciprocal, quotient, Pythagorean, and even-odd identities constitute


what we call the fundamental trigonometric identities.
We now solve Example 3.2.3 in a different way.

Example 3.4.5. If sin = 34 and cos > 0. Find cos .


Solution. Using the identity sin2 + cos2 = 1 with cos > 0, we have
s
 2
p
3
7
2
cos = 1 sin = 1
=
.
4
4

182
All rights reserved. No part of this material may be reproduced or transmitted in any form or by any means electronic or mechanical including photocopying without written permission from the DepEd Central Office. First Edition, 2016.

Example 3.4.6. If sec = 25 and tan < 0, use the identities to find the values
of the remaining trigonometric functions of .
Solution. Note that lies in QIV.
1
2
=
sec
5

sin = 1 cos2 =

1
5 21
csc =
=
sin
21

 2
2
21
1
=
5
5

PY

cos =

Seatwork/Homework 3.4.3

C
O

521
sin
21
tan =
= 2 =
cos
2
5

2 21
1
=
cot =
tan
21

EP
E

1 + tan x
1 + cot x

1. Use the identities presented in this lesson to simplify each trigonometric expression.
(a)

Solution.

Answer: tan x

1 + tan x
1 + tan x
=
= tan x
1 + cot x
1 + tan1 x

sin
1 + cos
+
Answer: 2 csc
1 + cos
sin
sin
1 + cos
sin2
(1 + cos )(1 + cos )
Solution.
+
=
+
1 + cos
sin
sin (1 + cos )
sin (1 + cos )
2
sin + (1 + 2 cos + cos2 )
=
sin (1 + cos )
2 + 2 cos
2
=
=
= 2 csc
sin (1 + cos )
sin
tan y + cot y
(c)
Answer: 1
sec y csc y
sin y
y
sin2 y+cos2 y
+ cos
tan y + cot y
cos y
sin y
cos y sin y
Solution.
= 1
=
= sin2 y + cos2 y = 1
1
1
sec y csc y

cos y sin y
cos y sin y

(b)

183
All rights reserved. No part of this material may be reproduced or transmitted in any form or by any means electronic or mechanical including photocopying without written permission from the DepEd Central Office. First Edition, 2016.

(d) 1

cos2
1 + sin

Solution. 1

Answer: sin
1 + sin cos2
cos2
=
1 + sin
1 + sin
2
sin + sin
sin (1 + sin )
=
=
= sin
1 + sin
1 + sin

2. Given some initial values, use the identities to find the values of the remaining
trigonometric functions of .
2
5

and sec > 0

Answer: in QI; csc = 52 , cos =


tan = 2 2121 , cot = 221
(b) sec = 38 and tan > 0

1 sin2 =

21
,
5

sec =

PY

(a) sin =

5 21
,
21

(c) tan = 2 and csc < 0

C
O

3
2 = 7 , csc =
Answer:

in
QIII;
cos

=
,
sin

cos
4

2 7
3 7
4 7
,
tan

=
,
cot

=
7
3
14

Answer: in QIII; cot = 12 , sec = 5, cos = 55 , csc = 45 ,


sin = 4 5 5

(d) csc =

3
2

and sec < 0

EP
E

Answer:
in QII; sin = 23 , cos =
tan = 2 5 5 , cot = 25

1 sin2 = 35 , sec = 3 5 5 ,

3.4.4. Proving Trigonometric Identities

We can use the eleven fundamental trigonometric identities to establish other


identities. For example, suppose we want to establish the identity
csc cot =

sin
.
1 + cos

To verify that it is an identity, recall that we need to establish the truth of the
equation for all values of the variable in the domain of the equation. It is not
enough to verify its truth for some selected values of the variable. To prove it, we
use the fundamental trigonometric identities and valid algebraic manipulations
like performing the fundamental operations, factoring, canceling, and multiplying
the numerator and denominator by the same quantity.
Start on the expression on one side of the proposed identity (preferably the
complicated side), use and apply some of the fundamental trigonometric identities
and algebraic manipulations, and arrive at the expression on the other side of the
proposed identity.
184

All rights reserved. No part of this material may be reproduced or transmitted in any form or by any means electronic or mechanical including photocopying without written permission from the DepEd Central Office. First Edition, 2016.

Expression

Explanation

csc cot
cos
1

=
sin
sin

Start on one side.


Apply some reciprocal and
quotient identities.

1 cos
sin
1 cos 1 + cos
=

sin
1 + cos
=

Add the quotients.


Multiply the numerator
and denominator by
1 + cos .

PY

1 cos2
(sin )(1 + cos )
sin2
=
(sin )(1 + cos )

Multiply.

sin
1 + cos

Reduce to lowest terms.

C
O

Apply a Pythagorean
identity.

Upon arriving at the expression of the other side, the identity has been established. There is no unique technique to prove all identities, but familiarity with
the different techniques may help.

Example 3.4.7. Prove: sec x cos x = sin x tan x.

EP
E

Solution.

1
cos x
cos x
1 cos2 x
=
cos x
sin x
sin2 x
=
= sin x
= sin x tan x
cos x
cos x

sec x cos x =

Example 3.4.8. Prove:

1 + sin 1 sin

= 4 sin sec2
1 sin 1 + sin

Solution.
1 + sin 1 sin
(1 + sin )2 (1 sin )2

=
1 sin 1 + sin
(1 sin )(1 + sin )
1 + 2 sin + sin2 1 + 2 sin sin2
=
1 sin2
4 sin
=
= 4 sin sec2
cos2

185
All rights reserved. No part of this material may be reproduced or transmitted in any form or by any means electronic or mechanical including photocopying without written permission from the DepEd Central Office. First Edition, 2016.

Seatwork/Homework 3.4.4
Prove each identity.
1. tan x + cot x = csc x sec x
sin x cos x
+
cos x sin x
sin2 x + cos2 x
1
1
1
=
=
=

= csc x sec x
sin x cos x
sin x cos x
cos x sin x

Answer: tan x + cot x =

1
sec tan
1
1
= 1
Answer:
sin
sec tan
cos
cos

cos
=
1 sin
cos
1 + sin
=

1 sin 1 + sin
(cos )(1 + sin )
=
1 sin2
(cos )(1 + sin )
=
cos2
1 + sin
1
sin
=
=
+
= sec + tan
cos
cos cos
sec y + tan y
= tan y
3.
csc y + 1

EP
E

C
O

PY

2. sec + tan =

sec y + tan y
Answer:
=
csc y + 1

sin y
1
+ cos
cos y
y
1
+
1
sin y

1+sin y
cos y
1+sin y
sin y

sin y
= tan y
cos y

1
1
+
1 cos 1 + cos
1
1
1 + cos + 1 cos
2
Answer:
+
=
=
= 2 csc2
2
1 cos 1 + cos
1 cos
sin2

4. 2 csc2 =

Exercises 3.4
1. Find the domain of the equation.

(a) 3 x + 2 x = 2x

Answer: {x|x 0}

Answer: R
 k

Answer: R \ 2 |k Z

(b) sin3 x = sin x + 1

(c) tan x + cot x = sin x


x+1
(d) 2
+ cos x = csc x
x 1

Answer: R \ {{1, 1} {k|k Z}}


186

All rights reserved. No part of this material may be reproduced or transmitted in any form or by any means electronic or mechanical including photocopying without written permission from the DepEd Central Office. First Edition, 2016.

2. Simplify each expression using the fundamental identities.


(a)

sin2
sec2 1

Answer: cos2

sin2
sin2
sin2
= cos2
=
=
sec2 1
tan2
sin2
cos2
1
1
(b)
+
2
1 + tan x 1 + cot2 x
1
1
1
1
Solution.
+
=
+
2
2
2
1 + tan x 1 + cot x
sec x csc2 x
= cos2 x + sin2 x = 1
Solution.

PY

cos2 x
(c) 1
1 + sin x

sin
cos tan
Solution.

cos2 x
1 sin2 x
=1
1 + sin x
1 + sin x
(1 sin x)(1 + sin x)
=1
1 + sin x
= 1 1 sin x = sin x

Answer: 1

sin
1
= tan
=1
cos tan
tan

(d)

Answer: sin x

C
O

Solution. 1

Answer: 1

EP
E

3. Given some initial information, use the identities to find the values of the
trigonometric functions of .
(a) csc =

5
3

and tan > 0

Answer: in QI; sin = 35 , cos =


cot = 43

(b) tan = 12
and cos < 0
5

1 sin2 = 45 , sec = 54 , tan = 34 ,

5
5
Answer: in QII; cot = 12
, sec = tan2 + 1 = 13
, cos = 13
,
5

12
13
2
sin = 1 cos = 13 , csc = 12

(c) csc = 32 and < x <

3
2

2
5
2
Answer:

in
QIII;
sin

,
cos

sin

, sec =
3

3
3 5
2 5
5
5 , tan = 5 , cot = 2

(d) cot = 57 and

3
2

< < 2

5
,
sec

=
Answer: in QIV; tan =

tan2 + 1 =
7

csc = cot2 + 1 = 574 , sin = 5 7474

74
,
7

cos =

7 74
,
74

187
All rights reserved. No part of this material may be reproduced or transmitted in any form or by any means electronic or mechanical including photocopying without written permission from the DepEd Central Office. First Edition, 2016.

(e) sin = 1
Answer: coterminal with
tan undefined, cot = 0

3
;
2

csc = 1, cos = 0, sec undefined,

(f) cot = 1
Answer: either in QII or QIV

in QII: tan = 1, sin = 22 , csc = 2, cos = 22 , sec = 2

in QIV: tan = 1, sin = 22 , csc = 2, cos = 22 , sec = 2

PY

4. Determine whether the given equation is an identity or a conditional equation.


If it is an identity, prove it; otherwise, provide a value of the variable in the
domain that does not satisfy the equation.
Answer: conditional equation, x = 0

(b) sin3 x = cos x 1

Answer: conditional equation, x =

C
O

(a) sin x cos x = 1

(c) (sin x cos x)2 + (sin x + cos x)2 = 2

Answer: identity

Proof. (sin x cos x)2 + (sin x + cos x)2


= (cos2 x 2 cos x sin x + sin2 x) + (cos2 x + 2 cos x sin x + sin2 x)
=1+1=2
Answer: conditional equation, x =

(e) 2 sin2 x = sec x + cos x

Answer: conditional equation, x =

EP
E

(d) tan(x) cot x = 1

5. Prove the following identities.

(a) sin3 x = sin x sin x cos2 x

Solution. sin3 x = sin2 x sin x = (1 cos x) sin x = sin x sin x cos2 x

(b) sin4 x cos4 x = sin2 x cos2 x


Solution. sin4 xcos4 x = (sin2 xcos2 x)(sin2 x+cos2 x) = sin2 xcos2 x

(c) tan() sin() + cos() = sec()


Solution. tan() sin() + cos() = tan sin + cos
sin2
+ cos
cos
sin2 + cos2
=
cos
1
=
= sec = sec()
cos
=

188
All rights reserved. No part of this material may be reproduced or transmitted in any form or by any means electronic or mechanical including photocopying without written permission from the DepEd Central Office. First Edition, 2016.

(d)

1 + sin u + cos u
1 + cos u
=
1 + sin u cos u
sin u

Solution

1 + sin u + cos u
1 + sin u + cos u 1 + cos u
=

1 + sin u cos u
sin u + 1 cos u 1 + cos u
(1 + sin u + cos u)(1 + cos u)
=
sin u + sin u cos u + 1 cos2 u
(1 + sin u + cos u)(1 + cos u)
=
sin u + sin u cos u + sin2 u
(1 + sin u + cos u)(1 + cos u)
=
(sin u)(1 + cos u + sin u)
1 + cos u
=
sin u

PY

Teaching Notes
Since you need
1 + cos u to retain
in the numerator
at the end, do not
expand the
numerator.

1 sec2 x
in terms of sin x.
sec2 x
1 sec2 x
1
Solution.
=
1 = cos2 x 1 = sin2 x
2
sec x
sec2 x

C
O

6. Express

7. Express tan x sec x in terms of cos x.


sin x
cos x

1
cos x

sin x
cos2 x

1 cos2 x
Answer:
cos2 x

1 cos2 x
=
cos2 x

EP
E

Solution. tan x sec x =

Answer: sin2 x

8. Express all other five trigonometric functions in terms of tan x (allowing in


the expression).
1
1
tan x
; cos x =
; cot x =
; sec x =
Answer: sin x = 2
2
tan x
tan x + 1
tan
x+1

tan2 x + 1
tan2 x + 1; csc x =
tan x
Answer:

1
3

(sec tan )(sec +tan ) = 1 = 3(sec +tan ) = 1 = sec +tan =

1
3

9. If sec tan = 3, what is sec + tan ?

Solution. tan2 + 1 = sec2 = sec2 tan2 = 1

189
All rights reserved. No part of this material may be reproduced or transmitted in any form or by any means electronic or mechanical including photocopying without written permission from the DepEd Central Office. First Edition, 2016.

Lesson 3.5. Sum and Difference Identities


Time Frame: 3 one-hour sessions
Learning Outcomes of the Lesson
At the end of the lesson, the student is able to:
(1) derive trigonometric identities involving sum and difference of two angles;
(2) simplify trigonometric expressions using fundamental trigonometric identities
and sum and difference identities;

PY

(3) prove other trigonometric identities using fundamental trigonometric identities and sum and difference identities; and
(4) solve situational problems involving trigonometric identities.

C
O

Lesson Outline

(1) The sum and difference identities for cosine, sine, and tangent functions
(2) Cofunction identities

(3) More trigonometric identities


Introduction

EP
E

In previous lesson, we introduced the concept of trigonometric identity, presented the fundamental identities, and proved some identities. In this lesson, we
derive the sum and difference identities for cosine, sine, and tangent functions,
establish the cofunction identities, and prove more trigonometric identities.
3.5.1. The Cosine Difference and Sum Identities

Let u and v be any real numbers with 0 < v u < 2. Consider the unit circle
with points A = (1, 0), P1 , P2 , P3 , and u and v with corresponding angles shown
in Figure 3.27. Then P1 P2 = AP3 .
Recall that P1 = P (u) = (cos u, sin u), P2 = P (v) = (cos v, sin v), and P3 =
P (u v) = (cos(u v), sin(u v)), so that
p
P1 P2 = (cos u cos v)2 + (sin u sin v)2 ,

while

AP3 =

p
[cos(u v) 1]2 + [sin(u v) 0]2 .

Equating these two expressions and expanding the squares, we get


(cos u cos v)2 + (sin u sin v)2 = [cos(u v) 1]2 + sin2 (u v)
190
All rights reserved. No part of this material may be reproduced or transmitted in any form or by any means electronic or mechanical including photocopying without written permission from the DepEd Central Office. First Edition, 2016.

PY

C
O

Figure 3.27

cos2 u 2 cos u cos v + cos2 v + sin2 u 2 sin u sin v + sin2 v


= cos2 (u v) 2 cos(u v) + 1 + sin2 (u v)

EP
E

Applying the Pythagorean identity cos2 +sin2 = 1 and simplifying the resulting
equations, we obtain
(cos2 u + sin2 u) + (cos2 v + sin2 v) 2 cos u cos v 2 sin u sin v
= [cos2 (u v) + sin2 (u v)] 2 cos(u v) + 1
1 + 1 2 cos u cos v 2 sin u sin v = 1 2 cos(u v) + 1
cos(u v) = cos u cos v + sin u sin v.

We have thus proved another identity.


Although we assumed at the start that 0 < v u < 2, but because
cos() = cos (one of the even-odd identities), this new identity is true for
any real numbers u and v. As before, the variables can take any real values or
angle measures.
Cosine Difference Identity
cos(A B) = cos A cos B + sin A sin B
Replacing B with B, and applying the even-odd identities, we immediately
get another identity.
191
All rights reserved. No part of this material may be reproduced or transmitted in any form or by any means electronic or mechanical including photocopying without written permission from the DepEd Central Office. First Edition, 2016.

Cosine Sum Identity


cos(A + B) = cos A cos B sin A sin B

Example 3.5.1. Find the exact values of cos 105 and cos 12
.

Solution.

 

= cos

12
4
6

= cos cos + sin sin


4
6
4 6
2
3
2 1
=

2
2 2

2
6+ 2
=
4

EP
E

cos

C
O

PY

cos 105 = cos(60 + 45 )


= cos 60 cos 45 sin 60 sin 45


1
2
3
2
=

2 2 2
2
2 6
=
4

Example 3.5.2. Given cos =


QI, find cos( + ).

3
5

and sin =

12
,
13

where lies in QIV and in

Solution. We will be needing sin and cos .


s
 2

3
4
2
=
sin = 1 cos = 1
5
5
cos =

1 sin =

12
13

2

cos( + ) = cos cos sin sin


 
4 12
3 5
=

5 13
5 13
63
=
65

5
13

192
All rights reserved. No part of this material may be reproduced or transmitted in any form or by any means electronic or mechanical including photocopying without written permission from the DepEd Central Office. First Edition, 2016.

Seatwork/Homework 3.5.1
7
Answer:
1. Find the exact value of cos .
12
 
7

Solution. cos
= cos
+
= cos cos sin sin
12
4
3
3 4 3
4

2 1
2
3
2 6

=
=
2 2
2
2
4
2. Express
cos(5x) cos(2x) + sin(5x) sin(2x)

2
4

Answer: cos(3x)

PY

as a single cosine expression.

EP
E

C
O

Solution. cos(5x) cos(2x) + sin(5x) sin(2x) = cos(5x 2x) = cos(3x)


1
1
and cos = , where lies in QI and in QIII, find
3. Given cos =
3
4

1 2 30
cos( ).
Answer:
12

1
2 2
Solution. cos = and in QI = sin =
3
3

1
15
cos = and in QIII = sin =
4
4
cos ( ) = cos cos + sin sin

 
1
1
2 2
15
1 2 30
=

=
3
4
3
4
12
3.5.2. The Cofunction Identities and the Sine Sum and Difference
Identities

In the Cosine Difference Identity, if we let A = 2 , we get


 


 
cos
B = cos
cos B + sin
sin B
2
2
2
= (0) cos B + (1) sin B
= sin B.
From this identity, if we replace B with 2 B, we have
i


h 
cos

B = sin
B
2
2
2


cos B = sin
B .
2
193

All rights reserved. No part of this material may be reproduced or transmitted in any form or by any means electronic or mechanical including photocopying without written permission from the DepEd Central Office. First Edition, 2016.

As for the tangent function, we have



sin 2 B

tan
B =
2
cos 2 B
cos B
=
sin B
= cot B.


We have just derived another set of identities.

cos




B = sin B
sin
B = cos B
2
2


tan
B = cot B
2

C
O

Teaching Notes
The Cofunction
Identities for the
reciprocal
functions will
follow:

csc 2 B =
sec
B
sec 2 B =
csc
B
cot 2 B =
tan B.

PY

Cofunction Identities

Using the first two cofunction identities, we now derive the identity for sin(A+
B).

i
(A + B)
h2

i
= cos
A B)
 2



= cos
A cos B + sin
A sin B
2
2
= sin A cos B + cos A sin B
h

EP
E

sin(A + B) = cos

Sine Sum Identity

sin(A + B) = sin A cos B + cos A sin B

In the last identity, replacing B with B and applying the even-odd identities
yield
sin(A B) = sin[A + (B)]
= sin A cos(B) + cos A sin(B)
= sin A cos B cos A sin B.

194
All rights reserved. No part of this material may be reproduced or transmitted in any form or by any means electronic or mechanical including photocopying without written permission from the DepEd Central Office. First Edition, 2016.

Sine Difference Identity


sin(A B) = sin A cos B cos A sin B
Example 3.5.3. Find the exact value of sin
Solution.
5
12


+
4
6
 
 
 
 
= sin
cos
+ cos
sin
4
6
6
4
2
3
2 1

=
2
2
2
2

6+ 2
=
4
= sin

C
O

PY

sin

5
12

3
and sin = 21 , where 0 < <
Example 3.5.4. If sin = 13
find sin( + ) and sin( ).

and

< < ,

Solution. We first compute cos and cos .


s

 2
p
4 10
3
2
=
cos = 1 sin = 1
13
13

EP
E

 2
q
1
3
cos = 1 sin2 = 1
=
2
2
sin( + ) = sin cos + cos sin
!

3
4 10 1
3
=

13
2
13
2

4 10 3 3
=
26

sin( ) = sin cos cos sin

!
1 4 10
3 3
=

2
13
2
13

4 10 + 3 3
=
26

195
All rights reserved. No part of this material may be reproduced or transmitted in any form or by any means electronic or mechanical including photocopying without written permission from the DepEd Central Office. First Edition, 2016.

Example 3.5.5. Prove:


sin(x + y) = (1 + cot x tan y) sin x cos y.
Solution.

PY

(1 + cot x tan y) sin x cos y


= sin x cos y + cot x tan y sin x cos y
cos x sin y
= sin x cos y +
sin x cos y
sin x cos y
= sin x cos y + cos x sin y
= sin(x + y)
Seatwork/Homework 3.5.2

C
O



EP
E

1. Find the exact value of sin


.
Answer:
12
 

= sin

= sin cos cos sin


Solution. sin
12
4
6
4
6
4 6
2
3
2 1
=

2
2 2

2
6 2
=
4
2. Find the exact value of sin 20 cos 80 sin 80 cos 20 .

6
4

3
Answer:
2

Solution. sin 20 cos 80 sin 80 cos 20 = sin(20 80 )

sin(x + y)
tan x + tan y
=
.
sin(x y)
tan x tan y

3. Prove:

3
= sin(60 ) = sin 60 =
2

Solution.

sin(x + y)
sin x cos y + cos x sin y
=
sin(x y)
sin x cos y cos x sin y

1
sin x cos y + cos x sin y cos x cos y
=

1
sin x cos y cos x sin y
cos x cos y
sin x cos y
cos x sin y
+
tan x + tan y
cos x cos y cos x cos y
=
=
sin x cos y
cos x sin y
tan x tan y

cos x cos y cos x cos y


196

All rights reserved. No part of this material may be reproduced or transmitted in any form or by any means electronic or mechanical including photocopying without written permission from the DepEd Central Office. First Edition, 2016.

3.5.3. The Tangent Sum and Difference Identities


Recall that tan x is the ratio of sin x over cos x. When we replace x with A + B,
we obtain
sin(A + B)
tan(A + B) =
.
cos(A + B)
Using the sum identities for sine and cosine, and then dividing the numerator
and denominator by cos A cos B, we have
sin A cos B + cos A sin B
cos A cos B sin A sin B
sin A cos B
cos A sin B
+ cos
cos A cos B
A cos B
= cos A cos B
sin A sin B

cos A cos B
cos A cos B
tan A + tan B
=
.
1 tan A tan B

PY

tan(A + B) =

C
O

We have just established the tangent sum identity.

In the above identity, if we replace B with B and use the even-odd identity
tan() = tan , we get

EP
E

tan(A B) = tan[A + (B)]


tan A + tan(B)
=
1 tan A tan(B)
tan A tan B
.
=
1 + tan A tan B

This is the tangent difference identity.


Tangent Sum and Difference Identities
tan A + tan B
1 tan A tan B
tan A tan B
tan(A B) =
1 + tan A tan B

tan(A + B) =

Seatwork/Homework 3.5.3
1. Find the exact values of tan

5
12

, tan

12


, and tan 7
.
12

Answer: 2 + 3, 2 3, 2 + 3

197
All rights reserved. No part of this material may be reproduced or transmitted in any form or by any means electronic or mechanical including photocopying without written permission from the DepEd Central Office. First Edition, 2016.

C
O

PY

 

tan 4 + tan 6
1 + 33
5
= 2+
Solution. tan
= tan
+
=
3
=
12
4
6
1 tan 4 tan 6
1 33

 

1 33
tan 4 tan 6

= 2
tan
3
= tan

=
=

12
4
6
1 + tan 4 tan 6
1 + 33



5
7
= tan
=2+ 3
tan
12
12


2. Express tan 4 + and tan(2 ) in terms of tan .




tan 4 + tan
1 + tan
Solution. tan
+ =
=

4
1 tan 4 tan
1 tan
tan 2 tan
= tan
tan(2 ) =
1 + tan 2 tan
cot A cot B 1
.
3. Prove: cot(A + B) =
cot A + cot B
1
1 tan A tan B
Solution. cot(A + B) =
=
tan(A + B)
tan A + tan B
1 tan A tan B cot A cot B

=
tan A + tan B cot A cot B
cot A cot B 1
=
cot A + cot B
Exercises 3.5

EP
E

1. Find the exact value.

(a) cos 255

(b) tan 12

(c) sin 735

(d) cot 285



(e) cos 9 cos
(f)

2
9

sin

tan 20 + tan 25
1 tan 20 tan 25

sin

2
9

2 6
Answer:
4
Answer: 2 3

6 2
Answer:
4
Answer: 2 + 3
1
Answer:
2
Answer: 1

2. Given some information about a and b, find sin(a+b), cos(ab), and tan(a+b).
(a) sin a = 35 , cos b =
quadrant

12
,
13

a lies in the third quadrant, and b in the first

56
63
56
; cos(a b) = ; tan(a + b) =
65
65
33
5
Solution. cos a = 45 , sin b = 13
Answer: sin(a + b) =

198
All rights reserved. No part of this material may be reproduced or transmitted in any form or by any means electronic or mechanical including photocopying without written permission from the DepEd Central Office. First Edition, 2016.

(b) cos a = 21 , tan b = 32 , 0 < a < 2 , and 2 < b <

3 13 2 39
3 39 2 13
Answer: sin(a + b) =
; cos(a b) =
; tan(a +
26
26

24 13 3
b) =
23

Solution. sin a = 23 , cos b = 2 1313 , sin b = 3 1313

PY

(c) sec a = 25 , cot b = 53 , a in QII, and b in QIII

10 34 3 714
6 34 5 714
Answer: sin(a + b) =
; cos(a b) =
;
170
170

375 + 68 28
tan(a + b) =
489

Solution. cos a = 52 , sin a = 521 , cos b = 3 3434 , sin b = 5 3434

3. Simplify the following expressions.

(b) tan(x + )
(c) sin

3
2

+x

(d) cos(x )

C
O

(a) cos( x)

Answer: tan x

Answer: cos x

Answer: cos x

4. Prove each identity.

Answer: cos x

EP
E

(a) sin(x y) sin(x + y) = sin2 x sin2 y


Solution. sin(x y) sin(x + y)
= (sin x cos y cos x sin y)(sin x cos y + cos x sin y)
= sin2 x cos2 y cos2 x sin2 y
= (sin2 x)(1 sin2 y) (1 sin2 x) sin2 y
= sin2 x sin2 x sin2 y (sin2 y sin2 x sin2 y)
= sin2 x sin2 y

(b) cos(x y) = (cot x + tan y) sin x cos y


cos x sin y
+
sin x cos y
Solution. (cot x + tan y) sin x cos y =
sin x cos y
= cos x cos y + sin x sin y = cos(x y)
csc x csc y
(c) sec(x + y) =
cot x cot y 1
Solution.
sec(x + y) =

1
1
=
cos(x + y)
cos x cos y sin x sin y

1
=

cos x cos y sin x sin y

1
sin x sin y
1
sin x sin y

199
All rights reserved. No part of this material may be reproduced or transmitted in any form or by any means electronic or mechanical including photocopying without written permission from the DepEd Central Office. First Edition, 2016.

=
=

(d)

1
sin x sin y
cos x cos ysin x sin y
sin x sin y

csc x csc y
cot x cot y 1

cos(x + y)
1 tan x tan y
=
cos(x y)
1 + tan x tan y
cos x cos y sin x sin y
cos(x + y)
=
Solution.
cos(x y)
cos x cos y + sin x sin y

cos x cos ysin x sin y


cos x cos y
cos x cos y+sin x sin y
cos x cos y

1 tan x tan y
1 + tan x tan y

C
O

1
cos x cos y
1
cos x cos y

PY

cos x cos y sin x sin y


=

cos x cos y + sin x sin y

5. Let n be an integer. Prove that cos(n + ) = (1)n cos and sin(n + ) =


(1)n sin .
Solution. cos(n) = (1)n and sin(n) = 0 for any integer n.

cos(n + ) = cos(n) cos sin(n) sin = (1)n cos


sin(n + ) = sin(n) cos + cos(n) sin = (1)n sin

EP
E

6. In an alternating current circuit, the instantaneous power P (t) at time t is


given by
P (t) = Im Vm cos sin2 (t) Im Vm sin sin(t) cos(t),

where Im and Vm are the maximum current (in amperes) and voltage (in volts),
respectively. Express this function as a product of two sine functions.

Solution. P (t) = Im Vm sin(t)[cos sin(t) sin cos(t)]


= Im Vm sin(t) sin( t)

? 7. The force F (in pounds) on the back of a person when he or she bends over
sin(+90)
, where W is the
at an acute angle (in degrees) is given by F = 0.6Wsin
12
weight (in pounds) of the person.
(a) Simplify the formula for F .
(b) Find the force on the back of a person whose weight is 154.32 lbs if he
bends an angle of 40 .
(c) How many pounds should a person weigh for his back to endure a force
of 275 lbs if he bends 38 ?
200

All rights reserved. No part of this material may be reproduced or transmitted in any form or by any means electronic or mechanical including photocopying without written permission from the DepEd Central Office. First Edition, 2016.

Solution
0.6W sin( + 90
0.6W cos
=
sin 12
sin 12

0.6(154.32) cos 40
(b) F =
340.46 lbs
sin 12
275 sin 12
F sin 12
=
(c) W =
121.17 lbs
0.6 cos
0.6 cos 38


xy
cos
.
8. (a) Prove: sin x + sin y = 2 sin x+y
2
2
(a) F () =

PY

(b) A particle is moving according to the equation of motion






s(t) = sin 4t +
+ sin 4t +
,
3
6

where s(t) centimeters is the directed distance of the particle from the
origin at t seconds.

Solution

(a) Adding the identities

C
O

(i) Express s(t) in the form s(t) = a sin(bt + c).


(ii) Find the amplitude and frequency of the motion. (Here, frequency is
defined as the reciprocal of the period.)

EP
E

sin(A + B) = sin A cos B + cos A sin B


sin(A B) = sin A cos B cos A sin B,

we get

x+y
2

sin(A + B) + sin(A B) = 2 sin A cos B.


xy
.
2

Then A + B = x and A B = y. Thus, we






xy
x+y
sin x + sin y = 2 sin
cos
.
2
2




(b) (i) s(t) = sin 4t +
+ sin 4t +
6 
 3


4t + 3 + 4t + 6
4t + 3 4t 6
= 2 sin
cos
2
2



= 2 sin 4t +
cos
12

4
2+ 6

=
sin 4t +
4
2



2+ 6

2+ 6

(ii) s(t) =
sin 4t +
=
sin 4 t +
2
4
2
16

2+ 6
4
2
Amplitude =
; frequency =
=
2
2

and B =

Let A =
have

201
All rights reserved. No part of this material may be reproduced or transmitted in any form or by any means electronic or mechanical including photocopying without written permission from the DepEd Central Office. First Edition, 2016.

9. The dual tone multi-frequency is the signal information used in touch-tone


phones to identify which digit you touched on the keypad. It works by adding
a pair of sounds, one with a lower frequency and one with a higher frequency.
Refer to the chart below. For example, the sound created by touching 6 is
produced by adding a 770-hertz sound to a 1477-hertz sound. (Note that
hertz is a unit of frequency and is equal to 1 cycle per second.) This sound
is modeled by the equation
s(t) = sin(2 770t) + sin(2 1477t),

C
O

PY

where t is time in seconds.

EP
E

http://cnx.org/contents/XGjYtByD@4/Lab-6-Analog-to-Digital-Conver

(a) Write the equation of the sound created by touching the * (asterisk) key
as a product of sine and cosine functions.

(b) In (a), what is the maximum value of s(t)?

Solution

(a) s(t) = sin(2 941t) + sin(2 1209t) = 2 sin(2150t) cos(536t)

(b) Max value = 2 occurring at t = 0.75 + k, k nonnegative integer




xy
10. (a) Prove: cos x + cos y = 2 cos x+y
cos
.
2
2

(b) Two atmospheric waves in space produce pressures of F (t) and G(t) pascals at t seconds, where


3
F (t) = 0.04 cos(2t) and G(t) = 0.04 cos 2t
.
4
Express the total pressure P (t) = F (t) + G(t) in the form
P (t) = a cos(bt + c).
202

All rights reserved. No part of this material may be reproduced or transmitted in any form or by any means electronic or mechanical including photocopying without written permission from the DepEd Central Office. First Edition, 2016.

Solution
(a) Adding the identities
cos(A + B) = cos A cos B sin A sin B
cos(A B) = cos A cos B + sin A sin B,
we get
x+y
2

xy
.
2

Then A + B = x and A B = y. Thus, we






xy
x+y
cos
.
cos x + cos y = 2 cos
2
2

and B =

PY

Let A =
have

cos(A + B) + cos(A B) = 2 cos A cos B.

(b) P (t) = F (t) + G(t)

3
4

C
O



3
= 0.04 cos(2t) + 0.04 cos 2t
4


3 
2t + 2t 4
2t 2t +
= 0.04 2 cos
cos
2
2


 
3
3
= 0.08 cos 2t
cos
8
8


q

3
= 0.04 2 2 cos 2t
8

EP
E

11. (a) In the figure, two intersecting lines have equations y = m1 x + b1 and
y = m2 x + b2 , respectively. Let be the acute angle between them, as
shown. Prove that
m2 m1
tan =
.
1 + m1 m2

(b) Two non-vertical lines intersect at the point (3, 2), and one angle between them measures 30 . If one line is 2y = x + 7, find the equation of
the other line.
203
All rights reserved. No part of this material may be reproduced or transmitted in any form or by any means electronic or mechanical including photocopying without written permission from the DepEd Central Office. First Edition, 2016.

Solution

PY

(a) Let and be the angles between each line and the (positive side) x-axis,
as shown in the following diagram:

Then m1 = tan and m2 = tan , so that

tan tan
m2 m1
=
.
1 + tan tan
1 + m1 m2

C
O

tan = tan( ) =
(b) Solve for m2 in the equation

tan 30 =

m2 12
,
1 + 21 m2

EP
E

and then use the point-slope form of the equation of the line to get

5 3+8
(x + 3) + 2.
y=
11

12. The length s() of the shadow cast by a vertical pole when the angle of the
sun with the horizontal is given by
s() =

h sin(90 )
,
sin

where h is the height of the pole.

204
All rights reserved. No part of this material may be reproduced or transmitted in any form or by any means electronic or mechanical including photocopying without written permission from the DepEd Central Office. First Edition, 2016.

(a) Express s() as a single trigonometric expression.


(b) At what angle will give the shortest shadow of the pole? Longest
shadow?
Answer: (a) s() = h cot ; (b) Shortest shadow occurs at = 90 . But the
length of the shadow increases when the value of approaches 0 ; that is, no
maximum length for the shadow.
13. In 4ABC, prove that

PY

tan A + tan B + tan C = tan A tan B tan C.


Solution

EP
E

C
O

A + B + C = 180 = tan(A + B + C) = tan 180 = 0


tan A + tan(B + C)
= 0 = tan A + tan(B + C) = 0
1 tan A tan(B + C)
tan B + tan C
=0
tan A +
1 tan B tan C
tan A tan A tan B tan C + tan B + tan C
=0
1 tan B tan C
= tan A tan A tan B tan C + tan B + tan C = 0
tan A + tan B + tan C = tan A tan B tan C

Lesson 3.6. Double-Angle and Half-Angle Identities


Time Frame: 2 one-hour sessions
Learning Outcomes of the Lesson
At the end of the lesson, the student is able to:
(1) derive the double-angle and half-angle identities;
(2) simplify trigonometric expressions using known identities;
(3) prove other trigonometric identities using known identities; and
(4) solve situational problems involving trigonometric identities.

205
All rights reserved. No part of this material may be reproduced or transmitted in any form or by any means electronic or mechanical including photocopying without written permission from the DepEd Central Office. First Edition, 2016.

Lesson Outline
(1) The double-angle and half-angle identities for cosine, sine, and tangent
(2) More trigonometric identities
Introduction

3.6.1. Double-Angle Identities


Recall the sum identities for sine and cosine.

PY

Trigonometric identities simplify the computations of trigonometric expressions. In this lesson, we continue on establishing more trigonometric
identities.

In particular, we derive the formulas for f (2) and f 12 , where f is the sine,
cosine, or tangent function.

C
O

sin(A + B) = sin A cos B + cos A sin B

cos(A + B) = cos A cos B sin A sin B


When A = B, these identities becomes

sin 2A = sin A cos A + cos A sin A = 2 sin A cos A


and

EP
E

cos 2A = cos A cos A sin A sin A = cos2 A sin2 A.


Double-Angle Identities for Sine and Cosine
sin 2A = 2 sin A cos A

cos 2A = cos2 A sin2 A

The double-identity for cosine has other forms. We use the Pythagorean
identity sin2 + cos2 = 1.
cos 2A = cos2 A sin2 A
= cos2 A (1 cos2 A)
= 2 cos2 A 1
cos 2A = cos2 A sin2 A
= (1 sin2 A) sin2 A
= 1 2 sin2 A

206
All rights reserved. No part of this material may be reproduced or transmitted in any form or by any means electronic or mechanical including photocopying without written permission from the DepEd Central Office. First Edition, 2016.

Other Double-Angle Identities for Cosine


cos 2A = 2 cos2 A 1
Example 3.6.1. Given sin t =

3
5

and

cos 2A = 1 2 sin2 A

< t < , find sin 2t and cos 2t.

cos 2t = 1 2 sin2 t
 2
3
=12
5
7
=
25

C
O

sin 2t = 2 sin t cos t


  
3
4
=2

5
5
24
=
25

PY

Solution. We first find cos t using the Pythagorean identity. Since t lies in QII,
we have
s
 2
p
4
3
2
= .
cos t = 1 sin t = 1
5
5

In the last example, we may compute cos 2t using one of the other two doubleangle identities for cosine. For the sake of answering the curious minds, we include
the computations here.

EP
E

cos 2t = cos2 t sin2 t


 2  2
3
4
=

5
5
7
=
25

cos 2t = 2 cos2 t 1
 2
4
=2
1
5
7
=
25

In the three cosine double-angle identities, which formula to use depends on


the convenience, what is given, and what is asked.
Example 3.6.2. Derive an identity for sin 3x in terms of sin x.

Solution. We use the sum identity for sine, the double-angle identities for sine
and cosine, and the Pythagorean identity.
sin 3x = sin(2x + x)
= sin 2x cos x + cos 2x sin x
= (2 sin x cos x) cos x + (1 2 sin2 x) sin x
= 2 sin x cos2 x + sin x 2 sin3 x
= 2(sin x)(1 sin2 x) + sin x 2 sin3 x
= 3 sin x 4 sin3 x

207
All rights reserved. No part of this material may be reproduced or transmitted in any form or by any means electronic or mechanical including photocopying without written permission from the DepEd Central Office. First Edition, 2016.

For the double-angle formula for tangent, we recall the tangent sum identity:
tan(A + B) =

tan A + tan B
.
1 tan A tan B

When A = B, we obtain
tan(A + A) =

tan A + tan A
2 tan A
=
.
1 tan A tan A
1 tan2 A

Tangent Double-Angle Identity


2 tan A
1 tan2 A

PY

tan 2A =

C
O

Example 3.6.3. If tan = 13 and sec > 0, find sin 2, cos 2, and tan 2.
Solution. We can compute immediately tan 2.


2 13
3
2 tan
=
tan 2 =
2 =
2
1
1 tan
4
1 3

EP
E

From the given information, we deduce that lies in QIV. Using one Pythagorean
identity, we compute cos through sec . (We may also use the technique discussed in Lesson 3.2 by solving for x, y, and r.) Then we proceed to find cos 2.
s
 2
p
1
10
2
sec = 1 + tan = 1 +
=
3
3

1
1
3 10
cos =
= =
10
sec
10
3
!2
3
10
4
cos 2 = 2 cos2 1 = 2
1=
10
5

tan 2 =

sin 2
3
= sin 2 = tan 2 cos 2 =
cos 2
5

Seatwork/Homework 3.6.1
1. If cos =

2
3

and

3
2

< < 2, find sin 2, cos 2, and tan 2.

4 5
1
Answer: sin 2 =
, cos 2 = , tan 2 = 4 5
9
9
208

All rights reserved. No part of this material may be reproduced or transmitted in any form or by any means electronic or mechanical including photocopying without written permission from the DepEd Central Office. First Edition, 2016.

2. Express tan 3 in terms of tan .

Answer: tan 3 =

2 tan
= sin 2.
1 + tan2
2 tan
2 tan
= 2 sin cos = sin 2
=
Solution.
1 + tan2
sec2

3 tan tan3
1 3 tan2

3. Prove:

3.6.2. Half-Angle Identities


Recall two of the three double-angle identities for cosine:
cos 2A = 1 2 sin2 A.

PY

cos 2A = 2 cos2 A 1 and

From these identities, we obtain two useful identities expressing sin2 A and cos2 A
in terms of cos 2A as follows:
1 + cos 2A
2

sin2 A =

and

1 cos 2A
.
2

C
O

cos2 A =

Some Useful Identities


1 + cos 2A
2

sin2 A =

cos2 A =

EP
E

From these identities, replacing A with


1 + cos 2
A
cos
=
2
2
2

and

A
,
2

A
2

1 cos 2A
2

we get
=

1 + cos A
2


1 cos 2 A2
A
1 cos A
sin
=
=
.
2
2
2
These are the half-angle identities for sine and cosine.

Half-Angle Identities for Sine and Cosine


 
 
A
1 + cos A
A
1 cos A
2
2
cos
=
sin
=
2
2
2
2

Because of the square in the formulas, we get


r
r
A
1 + cos A
A
1 cos A
cos =
and sin =
.
2
2
2
2
The appropriate signs of cos A2 and sin A2 depend on which quadrant

A
2

lies.

209
All rights reserved. No part of this material may be reproduced or transmitted in any form or by any means electronic or mechanical including photocopying without written permission from the DepEd Central Office. First Edition, 2016.

Example 3.6.4. Find the exact values of sin 22.5 and cos 22.5 .

Solution.

PY

Solution. Clearly, 22.5 lies in QI (and so sin 22.5 and cos 22.5 are both positive), and 22.5 is the half-angle of 45 .
s

p
r

cos
45
2 2

2
=
=
sin 22.5 =
2
2
2
s

p
r

1
+
1
+
cos
45
2+ 2

2
=
=
2
cos 22.5 =
2
2
2
 

tan + sin
2
Example 3.6.5. Prove: cos
=
.
2
2 tan
 

1 + cos
cos
=
2
2


1 + cos tan
=
2
tan
tan + cos tan
=
2 tan
sin
tan + cos cos

=
2 tan
tan + sin
=
2 tan

EP
E

C
O

We now derive the first version of the half-angle formula for tangent.

tan

sin A2
A
=
2
cos A2
sin A2
=
cos A2

2 sin A2
2 sin A2

A

2 sin2 2
=
2 sin A2 cos A2
A
2 1cos
2 
=
sin 2 A2
1 cos A
=
sin A

There is another version of the tangent half-angle formula, and we can derive
it from the first version.
A
1 cos A
tan =
2
sin A
210
All rights reserved. No part of this material may be reproduced or transmitted in any form or by any means electronic or mechanical including photocopying without written permission from the DepEd Central Office. First Edition, 2016.

=
=
=
=



1 cos A 1 + cos A
sin A
1 + cos A
2
1 cos A
(sin A)(1 + cos A)
sin2 A
(sin A)(1 + cos A)
sin A
1 + cos A

A
1 cos A
=
2
sin A

sin A2
A
tan =
2
cos A2

A
sin A
=
2
1 + cos A
 
A
1 cos A
tan2
=
2
1 + cos A
tan

C
O

tan

PY

Tangent Half-Angle Identities

.
Example 3.6.6. Find the exact value of tan 12

1 cos 6
1

=
tan
=

1
12
sin 6
2

3
2

=2

Solution.

EP
E

Example 3.6.7. If sin = 25 , cot > 0, and 0 < 2, find sin 2 , cos 2 , and
tan 2 .

. It follows
Solution. Since sin < 0 and cot > 0, we conclude the < < 3
2
that

3
< <
,
2
2
4
which means that 2 lies in QII.
s

 2
p
2
21
cos = 1 sin2 = 1
=
5
5
v
 
u
p
r

u 1 21
5

1 cos t
50 + 10 21
sin =
=
=
2
2
2
10
v
 
u
p
r

u 1 + 21
t
5
1 + cos
50 10 21

cos =
=
=
2
2
2
10
 

1 521

1 cos
5 + 21
tan =
=
=
2
2
sin
2
25
211

All rights reserved. No part of this material may be reproduced or transmitted in any form or by any means electronic or mechanical including photocopying without written permission from the DepEd Central Office. First Edition, 2016.

Seatwork/Homework 3.6.2
1. Find the exact value of tan 8 .
2. If cos =

3
5

and

3
2

Answer:

< < 2, find sin 2 , cos 2 , and tan 2 .


Answer: sin 2 =

5
,
5

21

cos 2 = 2 5 5 , tan 2 = 12

 
A
2 2 cos A
3. Prove: sec
=
.
2
sin2 A
A
1
2
2(1 cos A)
2 2 cos A
=
=
=
Solution. sec2 =
A
2
2
2
1 + cos A
1 cos A
sin2 A
cos 2

PY

Exercises 3.6

(a) cos = 41 and


(b) sec =

5
2

C
O

1. Given some information about , find sin 2, cos 2, and tan 2.


<<

Answer: sin 2 =

and sin > 0

Answer: sin 2 =

3
5

cos 2 = 87 , tan 2 =

15
7

cos 2 = 17
, tan 2 = 4 1721
25

< < 2
Answer: sin 2 = 45 , cos 2 = 35 , tan 2 =

4
3

and tan < 0

EP
E

(d) sin =

3
2

4 21
,
25

15
,
8

(c) tan = 2 and

Answer: sin 2 = 24
, cos 2 =
25

7
,
25

tan 2 = 24
7

2. Given the same information as in Item (1), where 0 < 2, find sin 2 , cos 2 ,
and tan 2 .
Answer:

(a) sin 2 =

(b) sin 2 =
(c) sin 2 =
(d) sin 2 =

6
15

,
tan
=
4
2
3

cos 2 = 1070 , tan 2 = 721

5010 5
50+10 5

, cos 2 =
,
10
10

3 10
, cos 2 = 1010 , tan 2 = 3
10
10
,
4

30
,
10

cos 2 =

tan 2 =

1 5
2

3. Express each expression as one trigonometric expression, but do not find the
exact value.
(a) 2 sin 10 cos 10
r

1 cos 7
6
(b)
2

Answer: sin 20
Answer: sin 7
12
212

All rights reserved. No part of this material may be reproduced or transmitted in any form or by any means electronic or mechanical including photocopying without written permission from the DepEd Central Office. First Edition, 2016.

(c) 1 2 sin2
1 + cos 8
(d)
2

3
10

Answer: cos 3
5
Answer: cos2 4

PY

4. Prove each identity.



(a) tan2 2 = (csc cot )2

1 cos
1 cos 1 cos
Solution. tan2 =
=

2
1 + cos
1 + cos 1 cos
(1 cos )2
=
1 cos2

2
1 cos
=
= (csc cot )2
sin

EP
E

C
O

(b) tan 2 + cot 2 = 2 csc

1 cos
sin

+
Solution. tan + cot =
2
2
sin
1 cos
(1 cos )2 + sin2
=
(sin )(1 cos )
1 2 cos + cos2 + sin2
=
(sin )(1 cos )
2 2 cos
2
=
=
= 2 csc
(sin )(1 cos )
sin

(c) sec2 2 = (csc2 )(2 2 cos )
1
2

Solution. sec2 =
=
2
1 + cos
cos2 2
2
1 cos
=

1 + cos 1 cos
2 2 cos
=
= (csc2 )(2 2 cos )
sin2

5. If a = 2 tan , express sin 2 and cos 2 in terms of a.


Answer: sin 2 =

4a
4 a2
,
cos
2
=
4 + a2
4 + a2

Solution. sin 2 = 2 sin cos


= 2 tan cos2 =

2 a2
2 tan
2 tan
4a
=
=
=

2
2
2
sec
1 + tan
4 + a2
1 + a2

cos 2 = 2 cos2 1
2
2
2
4 a2
=

1
=

1
=

1
=
2
sec2
1 + tan2
4 + a2
1 + a2
213

All rights reserved. No part of this material may be reproduced or transmitted in any form or by any means electronic or mechanical including photocopying without written permission from the DepEd Central Office. First Edition, 2016.

6. Find the exact value of cos 36 cos 72 .

Answer:

1
2

Solution

=
=
=
=

C
O

cos 36 cos 72 2(cos 36 + cos 72 )

1
2(cos 36 + cos 72 )
2 cos2 36 2 cos2 72
2(cos 36 + cos 72 )
(2 cos2 36 1) (2 cos2 72 1)
2(cos 36 + cos 72 )

cos 72 cos 144


Half-Angle Identity
2(cos 36 + cos 72 )
cos 72 + cos 36
cos(180 ) = cos
2(cos 36 + cos 72 )
1
2

PY

cos 36 cos 72 =

? 7. The range R of a projectile fired at an acute angle with the horizontal and
with an initial velocity of v meters per second is given by
R=

v2
sin(2),
g

where g is the acceleration due to gravity, which is 9.81 m/sec2 near the Earths
surface.

EP
E

(a) An archer targets an object 100 meters away from her position. If she
positions her arrow at an angle of 32 and releases the arrow at the speed
of 30 m/sec, will she hit her target?
Answer: No
2
30
Solution. R =
sin(2 32 ) 82.46 < 100
9.81
Answer: v 25.86 m/sec
(b) If sin = 52 , solve for v when R = 50.

2
21
Solution. sin = and acute angle = cos =
5
5
v2
2
21
50 =
2
= v 25.86
9.81
5
5
(c) Given v, find the value of that gives the largest possible range. At this
v2
, what is the range?
Answer: = 45 , largest R =
g
Solution. To reach the largest R, sin(2) must be 1.

8. The figure shows a laser scanner projection system. The optical angle , throw
distance D, and projected image width W are related by the equation
D=

W
2

csc cot

214
All rights reserved. No part of this material may be reproduced or transmitted in any form or by any means electronic or mechanical including photocopying without written permission from the DepEd Central Office. First Edition, 2016.

Answer: W = 2D tan 2

PY

Solve for W in terms D and 2 .

C
O

https://pangolin.com/userhelp/scanangles.htm

9. The slope of a mountain makes an angle of 45 with the horizontal. At the


base of the mountain, a cannon is fired at an angle with the horizontal, where
45 < < 90 , and with initial velocity of v m/sec. Neglecting air resistance,
the distance R (in meters) it drops on the slope of the mountain from the base
is given by

2 2v 2
(sin cos ) cos ,
R=
g

EP
E

where g is the acceleration due to gravity in m/sec2 . Express this formula for
R in terms of 2.

2 2v 2
Solution. R =
(sin cos ) cos
g
2
2v
=
(2 sin cos 2 cos2 )
g
2
2v
=
[sin 2 (cos 2 + 1)]
g
2
2v
=
(sin 2 cos 2 1)
g

215
All rights reserved. No part of this material may be reproduced or transmitted in any form or by any means electronic or mechanical including photocopying without written permission from the DepEd Central Office. First Edition, 2016.

Lesson 3.7. Inverse Trigonometric Functions


Time Frame: 3 one-hour sessions
Learning Outcomes of the Lesson
At the end of the lesson, the student is able to:
(1) graph the six basic inverse trigonometric functions;
(2) illustrate the domain and range of the inverse trigonometric functions;
(3) evaluate inverse trigonometric expressions; and

Lesson Outline

PY

(4) solve situational problems involving inverse trigonometric functions.

(1) Definitions of the six inverse trigonometric functions

C
O

(2) Graphs of inverse trigonometric functions

(3) Domain and range of inverse trigonometric functions


(4) Evaluation of inverse trigonometric expressions
Introduction

EP
E

In the previous lessons on functions (algebraic and trigonometric), we computed for the value of a function at a number in its domain. Now, given a value
in the range of the function, we reverse this process by finding a number in the
domain whose function value is the given one. Observe that, in this process,
the function involved may or may not give a unique number in the domain. For
example, each of the functions f (x) = x2 and g(x) = cos x do not give a unique
number in their respective domains for some values of each function. Given
f (x) = 1, the function gives x = 1. If g(x) = 1, then x = 2k, where k is an
integer. Because of this possibility, in order for the reverse process to produce a
function, we restrict this process to one-to-one functions or at least restrict the
domain of a non-one-to-one function to make it one-to-one so that the process
works. Loosely speaking, a function that reverses what a given function f does
is called its inverse function, and is usually denoted by f 1 .

Teaching Notes
The concept of
inverse function
was studied in
General
Mathematics
course.

More formally, two functions f and g are inverse functions if


g(f (x)) = x for any x in the domain of f ,
and
f (g(x)) = x for any x in the domain of g.
We denote the inverse function of a function f by f 1 . The graphs of a function
and its inverse function are symmetric with respect to the line y = x.

216
All rights reserved. No part of this material may be reproduced or transmitted in any form or by any means electronic or mechanical including photocopying without written permission from the DepEd Central Office. First Edition, 2016.

In this lesson, we first restrict the domain of each trigonometric function


because each of them is not one-to-one. We then define each respective inverse
function and evaluate the values of each inverse trigonometric function.
3.7.1. Inverse Sine Function

PY

All the trigonometric functions that we consider are periodic over their entire
domains. This means that all trigonometric functions are not one-to-one if we
consider their whole domains, which implies that they have no inverses over those
sets. But there is a way to make each of the trigonometric functions one-to-one.
This is done by restricting their respective domains. The restrictions will give us
well-defined inverse trigonometric functions.

C
O

The domain of the sine function is the set R of real numbers, and its range is
the closed interval [1, 1]. As observed in the previous lessons, the sine function
is not one-to-one, and the first step is to restrict its domain (by agreeing what the
convention is) with the following conditions: (1) the sine function is one-to-one
in that restricted domain, and (2) the range remains the same.

The inverse of the (restricted) sine function


 f (x)
 = sin x, where the
domain is restricted to the closed interval 2 , 2 , is called the inverse
sine function or arcsine function, denoted by f 1 (x) = sin1 x or
1
f 1 (x) = arcsin x. Here,
 the domain of f (x) = arcsin x is [1, 1],

and its range is 2 , 2 . Thus,

EP
E

y = sin1 x or y = arcsin x

if and only if

sin y = x,

where 1 x 1 and 2 y 2 .

Throughout the lesson, we interchangeably use sin1 x and arcsin x to mean


the inverse sine function.
Example 3.7.1. Find the exact value of each expression.
(1) sin1 21
(3) arcsin 0

(2) arcsin(1)
(4) sin1 12

Solution. (1) Let = sin1 21 . This is equivalent to sin = 12 . This


 means that

we are looking for the number in the closed interval 2 , 2 whose sine is
1
. We get = 6 . Thus, we have sin1 21 = 6 .
2



(2) arcsin(1) = 2 because sin 2 = 1 and 2 2 , 2 .
217

All rights reserved. No part of this material may be reproduced or transmitted in any form or by any means electronic or mechanical including photocopying without written permission from the DepEd Central Office. First Edition, 2016.

(3) arcsin 0 = 0

(4) sin1 12 = 6

As emphasized
1, sin1 x is that
 in the last example, as long as 1 x1
number y 2 , 2 such that sin y = x. If |x| > 1, then sin x is not defined in
R.

PY

We can sometimes find the exact value of sin1 x (that is, we can find a value
in terms of ), but if no such special value exists, then we leave it in the form
sin1 x. For example, as shown above, sin1 12 is equal to 6 . However, as studied
in Lesson 3.2, no special number satisfies sin = 23 , so we leave sin1 32 as is.
Example 3.7.2. Find the exact value of each expression.
(1) sin sin1 12
(3) arcsin(sin )



(2) arcsin sin 3


(4) sin sin1 12
1
2

C
O


Solution. (1) sin sin1 21 = sin 6 =


(2) arcsin sin 3 = arcsin 23 = 3

(3) arcsin(sin ) = arcsin 0 = 0




(4) sin sin1 21 = sin 6 = 12

EP
E

From the last example, we have the following observations:

1. sin(arcsin x) = x for any x [1, 1]; and






2. arcsin(sin ) = if and only if  2 , 2 , and if 6 2 , 2 , then
arcsin(sin ) = , where 2 , 2 such that sin = sin .

To sketch the graph of y = sin1 x, Table 3.28 presents the tables of values
for y = sin x and y = sin1 x. Recall that the graphs of y = sin x and y = sin1 x
are symmetric with respect to the line y = x. This means that if a point (a, b) is
on y = sin x, then (b, a) is on y = sin1 x.
y = sin x

y = sin

x
y

3
2

3
2

2
2

2
2

6
12

21

1
2

1
2

2
2

3
2

3
2

2
2

Table 3.28

218
All rights reserved. No part of this material may be reproduced or transmitted in any form or by any means electronic or mechanical including photocopying without written permission from the DepEd Central Office. First Edition, 2016.

(a) y = sin x

C
O

PY

The graph (solid thick curve) of the restricted sine function y = sin x is shown
in Figure 3.29(a), while the graph of inverse sine function y = arcsin x is shown
in Figure 3.29(b).

(b) y = sin1 x

Figure 3.29

EP
E

Example 3.7.3. Sketch the graph of y = sin1 (x + 1).


Solution 1. In this solution, we use translation of graphs.

Because y = sin1 (x + 1) is equivalent to y = sin1 [x (1)], the graph of


y = sin1 (x + 1) is 1-unit to the left of y = sin1 x. The graph below shows
y = sin1 (x + 1) (solid line) and y = sin1 x (dashed line).

219
All rights reserved. No part of this material may be reproduced or transmitted in any form or by any means electronic or mechanical including photocopying without written permission from the DepEd Central Office. First Edition, 2016.

Solution 2. In this solution, we graph first the corresponding sine function, and
then use the symmetry with respect to y = x to graph the inverse function.
y = sin1 (x + 1) sin y = x + 1 x = sin y 1

PY
C
O
EP
E

Teaching Notes
Keep in mind that,
because of the
restriction in the
domain, we have
the following:
sin(sin1 x) = x
for all x [1, 1].
But sin1 (sin x) is
not always x. We
have
sin1 (sin x) = x
only if
2 x 2 .

The graph below shows the process of graphing of y = sin1 (x + 1) from y =


sin x 1 with 2 x 2 , and then reflecting it with respect to y = x.

Seatwork/Homework 3.7.1

1. Find the exact value of each expression.


Answer:

(a) sin1 1
 
(b) arcsin 22
 
(c) arcsin 23
 
(d) sin1 23


1 2
(e) sin sin
2


(f) arcsin sin 3

(g) arcsin cos 3



(h) sin1 sin 4
3

Answer: 2
Answer:

Answer: 3
Answer:


2
2

Answer:

Answer:

Answer: 3
220

All rights reserved. No part of this material may be reproduced or transmitted in any form or by any means electronic or mechanical including photocopying without written permission from the DepEd Central Office. First Edition, 2016.

2. Sketch the graph of each equation.

C
O

PY

(a) y = sin1 (x 2)

EP
E

(b) y = sin1 (2x)

221
All rights reserved. No part of this material may be reproduced or transmitted in any form or by any means electronic or mechanical including photocopying without written permission from the DepEd Central Office. First Edition, 2016.

3.7.2. Inverse Cosine Function


The development of the other inverse trigonometric functions is similar to that
of the inverse sine function.
y = cos1 x or y = arccos x
means
cos y = x,
where 1 x 1 and 0 y .

PY

Teaching Notes
Observe that this
definition of
cos1 x is
equivalent to
cos1 x =

sin1 x.
2

EP
E

C
O

The graph (solid thick curve) of the restricted cosine function y = cos x is
shown in Figure 3.30(a), while the graph of inverse cosine function y = arccos x
is shown in Figure 3.30(b).

(b) y = cos1 x

(a) y = cos x

Figure 3.30

Example 3.7.4. Find the exact value of each expression. 


(1) cos1 0
(4) cos1 cos 3
4
 

(2) arccos 23
(5) arccos cos 7
6

 
(3) cos cos1 23

Solution. (1) cos1 0 =


 
(2) arccos 23 = 5
6


(6) sin cos1

because cos 2 = 0 and

2
2

[0, ].

222
All rights reserved. No part of this material may be reproduced or transmitted in any form or by any means electronic or mechanical including photocopying without written permission from the DepEd Central Office. First Edition, 2016.

 


= 23 because 23 [1, 1]
(3) cos cos1 23

(4) cos1 cos 3
=
4

because 3
[0, ].
4
 

(5) arccos cos 7
=
arccos
23 = 5
6
6


(6) sin cos

2
2

3
4

2
2


Example 3.7.5. Simplify: sin arcsin 23 + arccos 21 .

C
O

PY

Solution. We know that arccos 12 = 3 . Using the Sine Sum Identity, we have

sin arcsin 32 + arccos 12

= sin arcsin 23 + 3


= sin arcsin 23 cos 3 + cos arcsin 23 sin 3

= 23 21 + cos arcsin 23 23


= 13 + 23 cos arcsin 23 .


We compute cos arcsin 23 . Let = arcsin 23 . By definition, sin = 23 , where
lies in QI. Using the Pythagorean identity, we have
p


cos arcsin 32 = cos = 1 sin2 = 35 .

EP
E

Going back to the original computations above, we have



sin arcsin 32 + arccos 12 = 13 + 23 cos arcsin 32
=

Example 3.7.6. Simplify: sin 2 cos1 45

3
1
+
3
2

2+ 15
.
6



5
3


Solution. Let = cos1 45 . Then cos = 54 . Because cos < 0 and range
of inverse
cosine function is [0, ], we know that must be within the interval


, . Using the Pythagorean Identity, we get sin = 53 .


2
Using the Sine Double-Angle Identity, we have

sin 2 cos1 54 = sin 2
= 2 sin cos

= 2 35 54
.
= 24
25

223
All rights reserved. No part of this material may be reproduced or transmitted in any form or by any means electronic or mechanical including photocopying without written permission from the DepEd Central Office. First Edition, 2016.

Example 3.7.7. Sketch the graph of y = 41 cos1 (2x).


Solution.
y=

1
1
cos1 (2x) 4y = cos1 (2x) x = cos(4y)
4
2

EP
E

C
O

PY

We graph first y = 12 cos(4x). The domain of this graph comes from the restriction
of cosine as follows:

0 4x = 0 x .
4
Then reflect this graph with respect to y = x, and we finally obtain the graph of
y = 41 cos1 (2x) (solid line).

In the last example, we may also use the following technique. In graphing
y = 14 cos1 (2x), the horizontal length of cos1 x is reduced to half, while the
vertical height is reduced to quarter. This comparison technique is shown in
the graph below with the graph of y = cos1 x in dashed line and the graph of
y = 41 cos1 (2x) in solid line.

224
All rights reserved. No part of this material may be reproduced or transmitted in any form or by any means electronic or mechanical including photocopying without written permission from the DepEd Central Office. First Edition, 2016.

Seatwork/Homework 3.7.2
1. Find the exact value of each expression.
(a) cos1 (1)
 
(b) arccos 22

(c) arccos sin 5
2

Answer:
Answer:

Answer: 0


2

Answer: 52

(d) cos cos1 5


(f) arccos cos


3

Answer: 0

PY

(e) cos1 tan

3
4

4

13
3

Answer:

2. Simplify each expression.

4 2 5
sin
Answer:
(a) cos
9
2
=
and
in
Solution. Let = cos1 32 . This implies that cos
3

5
QI. Using a Pythagorean
identity, one gets sin = 3 . Similarly, let

1
1
= sin
3 .This implies that sin = 13 and in QIV. Then,
we get cos = 2 3 2 .
We now use Cosine Difference Identity, and the given and computed
values to simplify the expression.

 
1
1
1 2
sin
cos cos

= cos( )
3
3
!
! 

 
2
2 2
5
1
4 2 5
= cos cos + sin sin =
+

=
.
3
3
3
3
9
13



EP
E

C
O

cos1 32

17 3 4 21
(b) tan arcsin
+ arccos
Answer:
9

1

Solution. Note that arcsin 2 = 6 . Second, we let = arccos 52 ,


which means that cos = 25 and is in QI. We get tan = 221 .

 



1
2
tan arcsin
+ arccos
= tan +
2
5
6
tan( 6 ) + tan
=
1 (tan( 6 ))(tan )

12

2
5

3
21
+
3
2

( 3 3 )( 221 )

17 3 4 21
=
9
225
All rights reserved. No part of this material may be reproduced or transmitted in any form or by any means electronic or mechanical including photocopying without written permission from the DepEd Central Office. First Edition, 2016.

3. Sketch the graph of each equation.

EP
E

(b) y = 12 cos1 (x + 2)

C
O

PY

(a) y = 2 cos1 (3x)

3.7.3. Inverse Tangent Function and the Remaining Inverse


Trigonometric Functions

The inverse tangent function is similarly defined as inverse sine and inverse cosine
functions.
y = tan1 x or y = arctan x
means
tan y = x,
where x R and 2 < y < 2 .

226
All rights reserved. No part of this material may be reproduced or transmitted in any form or by any means electronic or mechanical including photocopying without written permission from the DepEd Central Office. First Edition, 2016.

(b) y = tan1 x

C
O

(a) y = tan x

PY

The graph (solid thick curve) of the restricted function y = tan x is shown
in Figure 3.31(a), while the graph of inverse function y = arctan x is shown in
Figure 3.31(b).

Figure 3.31

EP
E

Example 3.7.8. Find the exact value of each expression.



(4) tan1 tan 6
(1) tan1 1


(2) arctan 3
(5) tan1 tan 7
6


5
1
(6) arctan tan 19
(3) tan tan
2
6


Solution. Note the range of arctan is the open interval 2 , 2 .
(1) tan1 1 =


(2) arctan 3 = 3

(3) tan tan1 52 = 25


(4) tan1 tan 6 = 6 because 6 2 , 2 .

(5) Here, note that 7
6 2 , 2 . Use the idea of reference angle, we know that
6
tan 7
= tan 6 .
6



7

1
1
tan
tan
= tan
tan
=
6
6
6
(6) Here, we cannot use the idea of reference angle, but the idea can help in a
way. The number (or angle) 19
is in QII, wherein tangent is negative, and
6
227

All rights reserved. No part of this material may be reproduced or transmitted in any form or by any means electronic or mechanical including photocopying without written permission from the DepEd Central Office. First Edition, 2016.

its reference angle is 6 .






 
19
= arctan tan
arctan tan
6
6

=
6
Example 3.7.9. Findthe exact value of each expression.

(2) tan sin1 53 tan1 14
(1) sin 2 tan1 38

PY


Solution. (1) Let = tan1 38 . Then tan = 83 . Following the notations in
Lesson 3.2 and the definition of inverse tangent
know that lies
p function, we
2
2
in QIV, and x = 3 and y = 8. We get r = 3 + (8) = 73.

C
O

Applying the Sine Double-Angle Identity (page 206) gives



 
8
1
sin 2 tan
= sin 2

3
= 2 sin cos
y x
=2
r r  

8
3

= 2
73
73
48
= .
73

EP
E

(2) Using the Tangent Difference Identity, we obtain




1 3
1 1
tan sin
tan
5
4


1 3
tan sin 5 tan tan1 41


=
1 + tan sin1 35 tan tan1 41

tan sin1 53 14
 .
=
1 + tan sin1 35 41

We are left to compute tan sin1 35 . We proceed as in (1) above. Let
= sin1 35 . Then sin = 53 . From the definition of inverse sine function and
the notations used
in Lesson 3.2, we know that lies in QI, and y = 3 and
r = 5. We get x = 52 32 = 4, so that tan = xy = 34 .



tan sin1 53 14
1 3
1 1

tan
=
tan sin
5
4
1 + tan sin1 35 41
41
1 + 34 14
8
=
19
=

3
4

228
All rights reserved. No part of this material may be reproduced or transmitted in any form or by any means electronic or mechanical including photocopying without written permission from the DepEd Central Office. First Edition, 2016.

PY

? Example 3.7.10. A student is


viewing a painting in a museum.
Standing 6 ft from the painting,
the eye level of the student is 5 ft
above the ground. If the painting is 10 ft tall, and its base is
4 ft above the ground, find the
viewing angle subtended by the
painting at the eyes of the student.
Solution. Let be the viewing angle, and let = + as shown below.
We observe that
1
6

and

9
tan = .
6

C
O

tan =

Using the Tangent Sum Identity, we have

tan + tan
1 tan tan
1
+9
= 6 169
1 6 6
20
= .
9
Using a calculator, the viewing angle is = tan1

EP
E

tan = tan( + ) =

20
9

65.8 .

We now define the remaining inverse trigonometric functions.


Define

tan1 x.
2
It follows that the domain of y = cot1 x is R and its range is (0, ).

cot1 x =

y = sec1 x or y = arcsec x

2
Teaching Notes
Keep in mind that
the domain
restrictions are
conventions we set.
Other books and
sources might have
different domain
restrictions. The
restrictions we
made aim to make
calculus
computations
easier in the future.

means
sec y = x,
   3 
where |x| 1 and y 0, 2 , 2 .

229
All rights reserved. No part of this material may be reproduced or transmitted in any form or by any means electronic or mechanical including photocopying without written permission from the DepEd Central Office. First Edition, 2016.

Define

sec1 x.
2
This means that thedomainof y = csc1 x is (, 1] [1, ) and
its range is , 2 0, 2 .
csc1 x =

C
O

PY

The graphs of these last three inverse trigonometric functions are shown in
Figures 3.32, 3.33, and 3.34, respectively.

(a) y = cot x

(b) y = cot1 x

EP
E

Figure 3.32

(b) y = sec1 x

(a) y = sec x
Figure 3.33
230

All rights reserved. No part of this material may be reproduced or transmitted in any form or by any means electronic or mechanical including photocopying without written permission from the DepEd Central Office. First Edition, 2016.

Figure 3.34

PY

(b) y = csc1 x

(a) y = csc x

[1, 1]

EP
E

sin1 x

Domain

Function

C
O

Observe that the process in getting the value of an inverse function is the
same to all inverse functions. That is, y = f 1 (x) is the same as f (y) = x. We
need to remember the range of each inverse trigonometric function. Table 3.35
summarizes all the information about the six inverse trigonometric functions.
Range

Graph

 
2, 2

Figure
3.29(b)

[1, 1]

[0, ]

tan1 x

2 , 2

cot1 x

(0, )

sec1 x

{x : |x| 1}

   3 
0, 2 , 2

csc1 x

{x : |x| 1}

cos1 x



, 2 0, 2

Figure
3.30(b)
Figure
3.31(b)
Figure
3.32(b)
Figure
3.33(b)
Figure
3.34(b)

Table 3.35

Example 3.7.11. Find the exact value of each expression.



1
(1) sec1 (2)
(3)
cot

3
 

 

2 3
1
3
1
1
(2) csc
3
(4) sin sec
2 csc
233
231

All rights reserved. No part of this material may be reproduced or transmitted in any form or by any means electronic or mechanical including photocopying without written permission from the DepEd Central Office. First Edition, 2016.

Solution. (1) sec1 (2) =


 
1
(2) csc
2 3 3 = 2
3

(3) cot1 3 =

4
3

because sec 4
= 2 and
3

4
3



, 3
2

5
6

 

3
1
.
Let

=
sec

. Then
(4) From (2), we know that csc1 2 3 3 = 2
3
2

sec = 32 . From defined range of inverse secant function and the notations
in Lesson
and r = 3 and x = 2. Solving for y, we get
p 3.2, lies in QIII,

2
2
y = 3 (2) = 5. It follows that sin = 35 and cos = 23 .

!
3
2

EP
E

C
O

PY

We now use the Sine Sum Identity.


!!



 
2
2
3
3
1
1
csc

= sin
sin sec

2
3
3


2
= sin +
3
2
2
= sin cos
+ cos sin
3
3
!   
2
5
1
=

+
3
2
3

52 3
=
6

Seatwork/Homework 3.7.3

1. Find the exact value of each expression.


(a) sec1 (1)

Answer:
Answer: 4

(b) arctan(1)

(c) arccsc csc


5

Answer:

(d) cot (cot1 (10))



(e) sec1 tan 3
4

(f) csc csc1 38

Answer: 10
Answer: 0

Answer: 83

2. Simplify each expression.

6 10 + 210
Answer:
50
Solution. Let = arcsec 52 and = arccot 3. These imply that sec =
5
and cot = 3, where
and are
both in QI. Weobtain the following:
2

21
3 10
2
cos = 5 , sin = 5 , cos = 10 , and sin = 1010 .


(a) cos arcsec 52 arccot 3

232
All rights reserved. No part of this material may be reproduced or transmitted in any form or by any means electronic or mechanical including photocopying without written permission from the DepEd Central Office. First Edition, 2016.

Using Cosine Difference Identity and the above values, we simplify the
expression as follows:
5
arccot 3) = cos cos + sin sin
2
!
! !
 
2
3 10
21
10
=
+
5
10
5
10

6 10 + 210
=
50

 

1
5
7
(b) tan tan1
+ tan1
Answer:
2
3
11
Solution



 

1
1 5
1
+
tan
tan
tan
tan

1
5
2
3 

tan tan1
+ tan1
=
2
3
1 tan tan1 21 tan tan1 53

PY

cos(arcsec

Exercises 3.7

EP
E

1. Find the exact value of each expression.



(a) sin1 21
(b) cos1 0

(c) tan1 3

Answer: 6
Answer:

Answer: 2
Answer: 4
3
3
Answer: 4

Answer:

(d) csc1 1

(e) sec1 (2)

(f) cot1 (1)


1
2

Answer: Undefined

(g) csc1

12 + 35
7
 5 =
1
11
1 2 3

C
O
=

? 2. Find the value of each expression using a calculator. Round your answer to
two decimal places.
(a) sin1 (1/3)

Answer: 0.34

(b) cos1 (2/5)

Answer: 1.98

Answer: 1.56

(d) csc (11/9)

Answer: 0.96

(e) sec1 (20/3)

Answer: 1.72

(f) cot1 (5/7)

Answer: 0.95

(c) tan (100)

233
All rights reserved. No part of this material may be reproduced or transmitted in any form or by any means electronic or mechanical including photocopying without written permission from the DepEd Central Office. First Edition, 2016.

3. Simplify each expression.



(a) cos1 cos 3

(b) csc1 tan 6

(c) tan1 tan 5
4

(d) sin1 cos 4

(e) cos1 csc 3

Answer:

Answer: undefined
Answer:
Answer:
Answer:

4. Simplify each expression.


5

(d) cos tan1 34 + cos1 13



(e) sin 2 sin1 12 3 tan1

5. Solve for t in terms of x.

3
3

Answer: 33
65
Answer: 12

Answer: t = tan1

(b) x = 2 tan(t + 1)
(c) x = 21 cos(2t + 1)

EP
E

(d) x = 2 23 sec(1 t)
1
2

Answer: undefined

Answer: t = 13 sin1 x

(a) x = sin 3t

(e) x =

Answer: 1

C
O

(c) tan (2 tan1 (1))

Answer: 1

PY


(a) sin sin1 12 + cos1 21

 

1
1
(b) cos tan
3 + sin
23

cot(2 3t)

6. Sketch the graph of each function.

x
2

cos1 (2x)1
2

1 2
Answer: t = 1 sec
(2

x)
3

2
1
1 1
Answer: t = 3 3 cot

x
2

Answer: t =

(a) y = cos1 (x + 1)

234
All rights reserved. No part of this material may be reproduced or transmitted in any form or by any means electronic or mechanical including photocopying without written permission from the DepEd Central Office. First Edition, 2016.

PY

(b) y = sin1 (x 2)

EP
E

C
O

(c) y = sin1 2x

x
2

(d) y = cos1

235
All rights reserved. No part of this material may be reproduced or transmitted in any form or by any means electronic or mechanical including photocopying without written permission from the DepEd Central Office. First Edition, 2016.

C
O

EP
E

(f) y = 21 sin1 (2x)

PY

(e) y = 2 cos1 (x 1)

(g) y = 2 sin1 (2x + 2)

236
All rights reserved. No part of this material may be reproduced or transmitted in any form or by any means electronic or mechanical including photocopying without written permission from the DepEd Central Office. First Edition, 2016.

C
O

PY

(h) y = 2 cos1 (2x 1)

7. Solve for x in the equation sin1 (x2 2x) = 2 .

8. Solve for x in the equation tan1 (4x2 + 5x 7) = 4 .

Answer: x = 1
Answer: 2, 43

EP
E

9. A woman is standing x ft from a wall with a billboard nailed on it. The


billboard is 15 ft tall, and its base is 6 ft above the eye level of the woman.
Find the viewing angle subtended on the eyes of the woman from the base to
15x
the top of the billboard.
Answer: tan1 2
x + 126
? 10. During a leap year, the number of hours of daylight in a city can be modeled
by D(t) = 12 + 2.4 sin(0.017t 1.377), where t is the day of the year (that is,
t = 1 means January 1, t = 60 is February 29, and so on).
(a) Give one day of that year whose number of hours of daylight is about
14.4.
Answer: 173 days, so the day would be June 21

(b) Find another day of that year whose number of hours of daylight is the
same as that of February 29.
Answer: 287 days, so the day would be October 13

? 11. After getting a job, a man started saving a percentage of his annual income,
which can be modeled by
P (t) = 2.5 cos(0.157t) + 5.2,
where P (t) is the percentage of his annual income that he was able to save on
year t after he got a job.
(a) What percentage of his annual income did he save on the second year?
Answer: 7.58% of his annual income
237

All rights reserved. No part of this material may be reproduced or transmitted in any form or by any means electronic or mechanical including photocopying without written permission from the DepEd Central Office. First Edition, 2016.

(b) On what year right after getting a job did he save the least?
Answer: 20 years after getting a job
(c) On what year right after getting a job did he save the most? When would
it happen again?
Answer: 40 years after getting a job
(d) If he got his job at the age of 20, how much will he save on the year of
his retirement (that is, when he is 60)?
Answer: 7.7% of his annual
income
12. Prove each identity.

(b) sin (tan1 2x) =

2x
,
1+4x2
1

C
O

PY

1+x
(a) cos (tan1 x) = 1+x
2 , x R
Solution. Let = tan1 x, where x is any real number. This implies that
tan = x. One can think of a right triangle, with acute angle whose
opposite side is x and adjacent side as 1. Solving for the hypotenuse,

adjacent side
1
1+x2
= 1+x
we get 1 + x2 . Thus cos =
. But
2 =
1+x2
hypotenuse

1+x2
= tan1 x, therefore, cos(tan1 x) = 1+x
2 .

xR

EP
E

Solution. Let = tan 2x, where x is any real number. This implies that
tan = 2x. One can think of a right triangle, with acute angle whose
opposite side is 2x and adjacent side as 1. Solving for the hypotenuse, we

opposite side
2x
1
= 1+4x
2x,
get 1 + 4x2 . Thus sin =
2 . But = tan
hypotenuse
2x
therefore, sin(tan1 2x) = 1+4x
2.

(c) tan1 x + tan1 x1 = 2 , x > 0


Solution. To prove the identity, one can prove an equivalent identity;
that is, to show that sin(tan1 x + tan1 x1 ) = 1, x > 0.
Let = tan1 x and = tan1 x1 . The same techniques as above are
1
x
1
, sin = 1+x
applied to get the following: sin = 1+x
2 , cos =
2,
1+x2
x

and cos = 1+x2 .


We now do the following manipulations:
1
sin(tan1 x + tan1 )
x
= sin( + ) = sin cos + cos sin


 


x
x
1
1

=
+
1 + x2
1 + x2
1 + x2
1 + x2
x2 + 1
=
= 1.
1 + x2

It follows that tan1 x + tan1

1
x

= 2 .

238
All rights reserved. No part of this material may be reproduced or transmitted in any form or by any means electronic or mechanical including photocopying without written permission from the DepEd Central Office. First Edition, 2016.

(d) sin1 x + cos1 x = 2 , x [1, 1]


Solution. Same as in (c), we show that sin(sin1 x + cos1 x) = 1, where
x [1, 1].
Let = sin1 x and = cos1 x. It follows that sin = x and
cos = x.
Using fundamental identities, we obtain the following: cos = 1 x2 =
sin .
Then, we do the following manipulations:

C
O

Therefore, sin1 x + cos1 x = 2 .

sin( + )
sin cos + cos sin

x(x) + ( 1 x2 )( 1 x2 )
x2 + 1 x2
1.

PY

sin(sin1 x + cos1 x) =
=
=
=
=

Lesson 3.8. Trigonometric Equations

Time Frame: 3 one-hour sessions

EP
E

Learning Outcomes of the Lesson

At the end of the lesson, the student is able to:


(1) solve trigonometric equations; and
(2) solve situational problems involving trigonometric equations.
Lesson Outline

(1) Definition of a trigonometric equation


(2) Solution to a trigonometric equation
(3) Techniques of solving a trigonometric equation
Introduction
We have studied equations in Lesson 3.4. We differentiated an identity from
a conditional equation. Recall that an identity is an equation that is true for all
values of the variable in the domain of the equation, while a conditional equation
is an equation that is not an identity.
In this lesson, we mostly study conditional trigonometric equations. Though
not explicitly, we have started it in the preceding lesson. For example, the equation sin x = 12 has the unique solution x = sin1 12 = 6 in the closed interval
239
All rights reserved. No part of this material may be reproduced or transmitted in any form or by any means electronic or mechanical including photocopying without written permission from the DepEd Central Office. First Edition, 2016.


2 , 2 . However, if we consider the entire domain (not the restricted domain)
of the sine function, which is the set R of real numbers, there are solutions (other
than 6 ) of the equation sin x = 21 . This current lesson explores the techniques of
solving (conditional) trigonometric equations.
We divide the lesson into two groups of equations: the ones using a basic way
of solving, and those using more advanced techniques.
3.8.1. Solutions of a Trigonometric Equation

PY

C
O

1
2

(2) tan x = 1

(3) 3 sec x = 2 3

(4) 3| cot x| = 1

(1) sin x =

(5) sec2 x tan2 x = 1

(6) sin x + cos x = 0

(7) cos2 x = cos 2x + sin2 x


(8) sin x + cos 2x = 0

(9) 2 sin x + tan x 2 cos x = 2

(10) sin2 x + cos2 x = 2


(11) sin 2x = sin x

(12) 2 tan x + 4 sin x = 2 + sec x

Solution. Note that the choices (except 2) are numbers within the interval [0, ].
To quickly determine which numbers among the choices are solutions to a particular equation, we use some distinctive properties of the possible solutions.

Teaching Notes
In the process of
showing that a
number is a
solution of an
equation, note that
we cannot assume
yet that it is a
solution. This
means that we
cannot use the
equality sign yet in
the process.

Technically, the basic method to show that a particular number is a solution


of an equation is to substitute the number to the variable and see if the equation
becomes true. However, we may use our knowledge gained from the previous
lessons to do a quicker verification process by not doing the manual substitution
and checking. We use this technique in the example.


3 5
,
,
,
,
2
are
Example 3.8.1. Which numbers in the set 0, 6 , 4 , 3 , 2 , 2
3
4
6
solutions to the following equations?

EP
E

Teaching Notes
The word
solution has two
meanings in our
discussion. One is
a process of solving
a problem, and the
other is a number
that makes an
equation true. The
intended meaning
depends on the
context of its
usage.

Any equation that involves trigonometric expressions is called a trigonometric


equation. Recall that a solution or a root of an equation is a number in the domain
of the equation that, when substituted to the variable, makes the equation true.
The set of all solutions of an equation is called the solution set of the equation.

Teaching Notes
In the actual
classroom
discussion, you
may only choose
some of these
items.

(1) The sine function is positive on (0, ). From Lesson 3.2, we recall that 6 is
an obvious solution. We may imagine the graph of y = sin x. We may also
use the idea of reference angle. Thus, among the choices, only 6 and 5
are
6
1
the only solutions of sin x = 2 .
(2) Since tan x = 1 > 0, any
 solution of the equation among the choices must

be in the interval 0, 2 (that is, in QI). Again, among the choices, the only
solution to tan x = 1 is 4 .
240

All rights reserved. No part of this material may be reproduced or transmitted in any form or by any means electronic or mechanical including photocopying without written permission from the DepEd Central Office. First Edition, 2016.

(3) Here, the given equation is equivalent to sec x = 2 3 3 . Among the choices,

.
the only solution of the equation 3 sec x = 2 3 is 5
6
(4) Eliminating
the absolute
value sign, the given equation is equivalent tocot x =

3
3
or cot x = 3 . Among the choices, the only solution of cot x = 33 is 3 ,
3

.
Thus,
the
only
solutions
of
while the other equation has 2
3| cot x| = 1
3
from the given set are 3 and 2
.
3

PY

(5) The given equation is one of the Pythagorean Identities (page 182). It means
that all numbers in the domain of the equation are solutions. The domain
of the equation is R \ {x : cos x = 0}. Thus, all except 2 are solutions of
sec2 x tan2 x = 1.

Teaching Notes
For convenience in
showing or finding
a solution of an
equation, we may
use an equivalent
equation. By
definition, the
solutions of the
equivalent equation
are exactly the
same as the
solutions of the
original equation.

C
O

(6) For the sum of sin x and cos x to be 0, they must have equal absolute values
satisfies these properties, and
but different signs. Among the choices, only 3
4
it is the only solution of sin x + cos x = 0.
(7) This equations is one of the Double-Angle Identities for Cosine. This means
that all numbers in the domain of the equation are its solutions. Because the
domain of the given equation is R, all numbers in the given set are solutions
of cos2 x = cos 2x + sin2 x.

(8) We substitute each number in the choices to the expression on the left-side
of the equation, and select those numbers that give resulting values equal to
1.

EP
E

x = 0: sin 0 + cos 2(0) = 0 + 1 = 1


x = 6 : sin 6 + cos 2( 6 ) =

x = 4 : sin 4 + cos 2( 4 ) =
x = 3 : sin 3 + cos 2( 3 ) =

1
+ 12 = 1
2

2
2
+
0
=
2
2

3
31
1

=
2
2
2

x = 2 : sin 2 + cos 2( 2 ) = 1 1 = 0
2
:
3

sin 2
+ cos 2( 2
)=
3
3

x=

3
:
4
5
:
6

sin 3
+ cos 2( 3
)=
4
4

x=

x=

sin 5
+ cos 2( 5
)=
6
6

3
31
1

=
2
2
2

2
+ 0 = 22
2
1
+ 12 = 1
2

x = : sin + cos 2 = 0 + 1 = 1

Teaching Notes
If one side of an
equation is
constant and if the
non-constant
expression is a bit
complicated, the
basic method of
showing that a
number is a
solution of the
equation is more
appropriate; that
is, to start on the
non-constant side,
then substitute the
number to the
variable, simplify
the expression, and
end on the value of
the constant on the
other side.

x = 2: sin 2 + cos 2(2) = 0 + 1 = 1


From these values, the only solution of sin x + cos 2x = 0 among the choices
is 2 .
(9) We again substitute the numbers in the given set one by one, and see which
resulting values are equal to 1.
241
All rights reserved. No part of this material may be reproduced or transmitted in any form or by any means electronic or mechanical including photocopying without written permission from the DepEd Central Office. First Edition, 2016.

x = 0: 2 sin 0 + tan 0 2 cos 0 = 2

x = 6 : 2 sin 6 + tan 6 2 cos 6 =

32 3
3

x = 4 : 2 sin 4 + tan 4 2 cos 4 = 1

x = 3 : 2 sin 3 + tan 3 2 cos 3 = 2 3 1

x = 2 : Since tan 2 is undefined, this value of x cannot be a solution of the


equation.
x=

2
:
3

x=

5
:
6

2 sin 2
+ tan 2
2 cos 2
=1
3
3
3

: 2 sin 3
+ tan 3
2 cos 3
=2 21
x = 3
4
4
4
4
x = : 2 sin + tan 2 cos = 2

3+2 3
3

x = 2: 2 sin 2 + tan 2 2 cos 2 = 2

PY

2 sin 5
+ tan 5
2 cos 5
=
6
6
6

C
O

Thus, the only solution of 2 sin x + tan x 2 cos x = 2 from the given set is .
(10) This equation has no solution because one of the Pythagorean Identities says
sin2 x + cos2 x = 1.

x = 0: sin 2(0) = 0; sin 0 = 0

3
;
2

sin 6 =

1
2

EP
E

x = 6 : sin 2( 6 ) =

x = 4 : sin 2( 4 ) = 1; sin 4 =
x = 3 : sin 2( 3 ) =

3
;
2

2
2

sin 3 =

3
2

x = 2 : sin 2( 2 ) = 0; sin 2 = 1
x=

3
:
4

sin 2( 3
) = 1; sin 3
=
4
4

x=

5
:
6

sin 2( 5
)=
6

If both sides of an
equation are both
non-constant
expressions, one
method of showing
that a number is a
solution of the
equation is to
substitute the
number to both
expressions (but
never join them
with equality sign
because they are
not yet equal
logically), and
check if the
resulting values are
equal.

(11) We substitute each number in the given set to the expression of each side of
the equation, and see which resulting values are equal.
Teaching Notes

3
;
2

sin 3 =

2
2
1
2

x = : sin 2 = 0; sin = 0
x = 2: sin 2(2) = 0; sin 2 = 0
Thus, among the numbers in the given set, the solutions of sin 2x = sin x are
0, 3 , , and 2.

242
All rights reserved. No part of this material may be reproduced or transmitted in any form or by any means electronic or mechanical including photocopying without written permission from the DepEd Central Office. First Edition, 2016.

(12) We employ the same technique used in the previous item.


x=0:

2 tan 0 + 4 sin 0 = 0
2 + sec 0 = 3

2 tan 6 + 4 sin 6 =
2 + sec 6 =

x=

x=

2 3+6
3

2 3+6
3

2 tan 4 + 4 sin 4 = 2 2 + 2

2 + sec 4 = 2 + 2

2 tan 3 + 4 sin 3 = 4 3
2 + sec 3 = 4

x=

2
3

Both tan 2 and sec 2 are undefined.


: 2 tan 2
+ 4 sin 2
=0
3
3
2 + sec 2
=0
3

3
4

x=

5
6

: 2 tan 3
+ 4 sin 3
=2 22
4
4

2 + sec 3
=
2

2
4
: 2 tan 5
+ 4 sin 5
=
6
6
2 + sec 5
=
6

2 tan + 4 sin = 0

EP
E

x=:

62 3
3

62 3
3

x=

C
O

x=

PY

x=

2 + sec = 1

x = 2 : 2 tan 2 + 4 sin 2 = 0
2 + sec 2 = 3

After checking the equal values, the solutions of 2 tan x + 4 sin x = 2 + sec x
among the given choices are 6 , 2
, and 5
.
2
3
6

Seatwork/Homework 3.8.1



In each equation, list down its solutions from the set 3 , 4 , 6 , 4 , 2
, , 3
.
3
2
(1)

3 sec = 2

Answer:

(2) (sin x)(tan x + 1) = 0

Answer: 4 ,

(3) 2 + cos = 1 + 2 sin2

Answer: 3 ,

(4) cos tan2 = 3 cos

Answer:

2 3
, 2
3

243
All rights reserved. No part of this material may be reproduced or transmitted in any form or by any means electronic or mechanical including photocopying without written permission from the DepEd Central Office. First Edition, 2016.

3.8.2. Equations with One Term


From the preceding discussion, you may observe that there may be more solutions
of a given equation outside the given set. We now find all solutions of a given
equation.
We will start with a group of equations having straightforward techniques
in finding their solutions. These simple techniques involve at least one of the
following ideas:

PY

(1) equivalent equations (that is, equations that have the same solutions as the
original equation);
(2) periodicity of the trigonometric function involved;
(3) inverse trigonometric function;

C
O

(4) values of the trigonometric function involved on the interval [0, ] or [0, 2]
(depending on the periodicity of the function); and
(5) Zero-Factor Law: ab = 0 if and only if a = 0 or b = 0.

To solve an equation means to find all solutions of the equation. Here,


unless stated as angles measured in degrees, we mean solutions of the equation
that are real numbers (or equivalently, angles measured in radians).

EP
E

Example 3.8.2. Solve the equation 2 cos x 1 = 0.


Solution. The given equation is equivalent to
1
cos x = .
2

On the interval [0, 2], there are only two solutions of the last equation, and these
are x = 3 (this is in QI) and x = 5
(in QIV).
3
Because the period of cosine function is 2, the complete solutions of the
equation are x = 3 + k(2) and x = 5
+ k(2) for all integers k.
2
3

In the preceding example, by saying that the complete solutions are x =


+ k(2) and x = 5
+ k(2) for all integers k, we mean that any integral
3
value of k will produce a solution to the given equation. For example, when
k = 3, x = 3 + 3(2) = 19
is a solution of the equation. When k = 2,
3
5
7
x = 3 + (2)(2) = 3 is another solution of 2 cos x 1 = 0. The family of
solutions x = 3 + k(2) can be equivalently enumerated as x = 19
+ 2k, while
3
5
7
the family x = 3 + k(2) can also be stated as x = 3 + 2k.

Teaching Notes
Any particular
solution in a family
of solutions can be
used as a seed
solution to
produce the other
solutions in the
family.

Example 3.8.3. Solve: (1 + cos )(tan 1) = 0.


244

All rights reserved. No part of this material may be reproduced or transmitted in any form or by any means electronic or mechanical including photocopying without written permission from the DepEd Central Office. First Edition, 2016.

Solution. By the Zero-Factor Law, the given equation is equivalent to


1 + cos = 0

tan 1 = 0

or

cos = 1

tan = 1

= + 2k, k Z

+ k, k Z.

Therefore, the solutions of the equation are = + 2k and =


k Z.

+ k for all
2

PY

Example 3.8.4. Find all values of x in the interval [2, 2] that satisfy the
equation (sin x 1)(sin x + 1) = 0.
Solution.
sin x = 1
x=

or

or

sin x + 1 = 0
sin x = 1

C
O

sin x 1 = 0
3
2

Solutions:

x=

,
2

3
,
2

3
2

3
,
2

or

Example 3.8.5. Solve: cos x = 0.1.

EP
E

Solution. There is no special number whose cosine is 0.1. However, because


0.1 [1, 1], there is a number whose cosine is 0.1. In fact, in any one-period
interval, with cos x = 0.1 > 0, we expect two solutions: one in QI and another in
QIV. We use the inverse cosine function.
From Lesson 3.7, one particular solution of cos x = 0.1 in QI is x = cos1 0.1.
We can use this solution to get a particular solution in QIV, and this is x =
2 cos1 0.1, which is equivalent to x = cos1 0.1.

From the above particular solutions, we can produce all solutions of cos x =
0.1, and these are x = cos1 0.1+2k and x = cos1 0.1+2k for all k Z. 2
Example 3.8.6. Solve: 3 tan + 5 = 0.
Solution.
3 tan + 5 = 0

tan = 35

We expect only one solution in any one-period interval.



tan = 53 = = tan1 53 + k, k Z

? Example 3.8.7. The voltage V (in volts) coming from an electricity distributing company is fluctuating according to the function V (t) = 200 + 170 sin(120t)
at time t in seconds.
245

All rights reserved. No part of this material may be reproduced or transmitted in any form or by any means electronic or mechanical including photocopying without written permission from the DepEd Central Office. First Edition, 2016.

(1) Determine the first time it takes to reach 300 volts.


(2) For what values of t does the voltage reach its maximum value?
Solution. (1) We solve for the least positive value of t such that V (t) = 300.
200 + 170 sin(120t) = 300
100
sin(120t) =
170
120t = sin1

sin1 100
170
0.00167 seconds
120

PY

t=

100
170

C
O

(2) The maximum value of V (t) happens when and only when the maximum
value of sin(120t) is reached. We know that the maximum value of sin(120t)
is 1, and it follows that the maximum value of V (t) is 370 volts. Thus, we
need to solve for all values of t such that sin(120t) = 1.

EP
E

sin(120t) = 1

120t = + 2k, k nonnegative integer


2

+ 2k
t= 2
120
1
+ 2k
t= 2
0.00417 + 0.017k
120

This means that the voltage is maximum when t 0.00417 + 0.017k for each
nonnegative integer k.
2

Seatwork/Homework 3.8.2

1. Solve each equation.

Answer: 4 + k, k Z

(a) tan x = 1

(b) sin x =

1
2

Answer:

+ 2k,

5
6

+ 2k, k Z

Answer: 2k, 4 + k, k Z

(c) (cos x 1)(tan x + 1) = 0

2. Find all values of the variable in the interval [2, 2] that satisfy the given
equation.
Answer: 2 , 3
, 0, , 2, , 2
2
Answer: 2
, 4
, 4
, 2
3
3
3
3

(a) (sin + 1)(tan ) = 0


(b) sec + 2 = 0
246

All rights reserved. No part of this material may be reproduced or transmitted in any form or by any means electronic or mechanical including photocopying without written permission from the DepEd Central Office. First Edition, 2016.

3.8.3. Equations with Two or More Terms


We will now consider a group of equations having multi-step techniques of finding
their solutions. Coupled with the straightforward techniques we learned in the
preceding discussion, these more advanced techniques involve factoring of expressions and trigonometric identities. The primary goal is to reduce a given equation
into equivalent one-term equations.
Example 3.8.8. Solve: 2 cos x tan x = 2 cos x.

PY

Solution.

2 cos x = 0
cos x = 0

C
O

2 cos x tan x = 2 cos x


2 cos x tan x 2 cos x = 0
(2 cos x)(tan x 1) = 0

tan x 1 = 0

or

tan x = 1

x = 4 + k,
kZ

x = 2 + 2k or
x = 3
+ 2k,
2
kZ
Solutions:

+ 2k,

3
2

+ 2k,

+ k, k Z

Teaching Notes
The method for
solving
trigonometric
equations follows
the usual way of
solving nonlinear
equations; that is,
transform the
equation so that
one side is 0, and
then factor.

EP
E

Example 3.8.9. Solve for x [0, 2): sin 2x = sin x.


Solution.

sin 2x = sin x

sin 2x sin x = 0

2 sin x cos x sin x = 0

Sine Double-Angle Identity

(sin x)(2 cos x 1) = 0


sin x = 0

2 cos x 1 = 0

or

x = 0 or x =

cos x =
x=

Solutions: 0, ,

5
, 3
3

1
2

or x =

5
3

247
All rights reserved. No part of this material may be reproduced or transmitted in any form or by any means electronic or mechanical including photocopying without written permission from the DepEd Central Office. First Edition, 2016.

Tips in Solving Trigonometric Equations


(1) If the equation contains only one trigonometric term, isolate that
term, and solve for the variable.
(2) If the equation is quadratic in form, we may use factoring, finding
square roots, or the quadratic formula.
(3) Rewrite the equation to have 0 on one side, and then factor (if
appropriate) the expression on the other side.

PY

(4) If the equation contains more than one trigonometric function,


try to express everything in terms of one trigonometric function.
Here, identities are useful.

C
O

(5) If half or multiple angles are present, express them in terms of a


trigonometric expression of a single angle, except when all angles
involved have the same multiplicity wherein, in this case, retain
the angle. Half-angle and double-angle identities are useful in
simplification.
Example 3.8.10. Solve for x [0, 2): 2 cos2 x = 1 + sin x.

Solution.

2 cos2 x = 1 + sin x

EP
E

2(1 sin2 x) = 1 + sin x

Pythagorean Identity

2 sin2 x + sin x 1 = 0

(2 sin x 1)(sin x + 1) = 0

2 sin x 1 = 0
sin x =

x=

Factoring

or

sin x + 1 = 0

1
2

or x =

sin x = 1

5
6

x=

Solutions:

3
2

5 3
, 6, 2
6

Example 3.8.11. Solve for x [0, 2) in the equation 3 cos2 x + 2 sin x = 2.


Solution.
3 cos2 x + 2 sin x = 2
3(1 sin2 x) + 2 sin x = 2

Pythagorean Identity

(3 sin x + 1)(sin x 1) = 0

Factoring

248
All rights reserved. No part of this material may be reproduced or transmitted in any form or by any means electronic or mechanical including photocopying without written permission from the DepEd Central Office. First Edition, 2016.

3 sin x + 1 = 0

sin x 1 = 0

or

sin x = 13

sin x = 1

x = sin1 ( 31 ) + 2
or
x = sin1 ( 13 )

x=

Solutions: 2 sin1 ( 31 )+, + sin1 ( 13 ),

C
O

PY

One part of the last solution needs further explanation. In the equation
) (which
sin x = 31 , we expect two solutions in the interval [0, 2): one in (, 3
2
3
is QIII), and another in ( 2 , 2) (which is QIV). Since no special number satisfies
sin x = 13 , we use inverse sine function. Because the range of sin1 is [ 2 , 2 ], we
know that 2 < sin1 ( 13 ) < 0. From this value, to get the solution in ( 3
, 2),
2
1
1
we simply add 2 to this value, resulting to x = sin ( 3 ) + 2. On the other
hand, to get the solution in (, 3
), we simply add sin1 ( 31 ) to , resulting to
2
x = sin1 ( 13 ).
Example 3.8.12. Solve: sin2 x + 5 cos2

Teaching Notes
Using the reference
angle of sin1 ( 31 ),
we get two
solutions (QIII and
QIV), and these
are + sin1 ( 13 )
and 2 sin1 ( 31 ).

Solution.

x
= 2.
2

Teaching Notes
Using the
Odd-Even Identity,
sin1 ( 31 ) =
sin1 ( 13 ).

x
sin2 x + 5 cos2 = 2
2 

1 + cos x
2
sin x + 5
=2
2

EP
E

Cosine Half-Angle Identity

2 sin2 x + 5 cos x + 1 = 0

2(1 cos2 x) + 5 cos x + 1 = 0 Pythagorean Identity


2 cos2 x 5 cos x 3 = 0

(2 cos x + 1)(cos x 3) = 0
2 cos x + 1 = 0

cos x 3 = 0

or

cos x = 12

cos x = 3

x = 2
+ 2k or
3
4
x = 3 + 2k,
kZ
Solutions:

2
3

no solution

+ 2k,

4
3

+ 2k, k Z

Example 3.8.13. Solve for x [0, 2) in the equation tan 2x 2 cos x = 0.


249
All rights reserved. No part of this material may be reproduced or transmitted in any form or by any means electronic or mechanical including photocopying without written permission from the DepEd Central Office. First Edition, 2016.

Solution.
tan 2x 2 cos x = 0
sin 2x
2 cos x = 0
cos 2x
sin 2x 2 cos x cos 2x = 0
Apply the Double-Angle Identities for Sine and Cosine, and then factor.

2 cos x = 0
x = 2 or
x = 3
2

2 sin x 1 = 0
sin x =

1
2

or

sin x + 1 = 0
sin x = 1

C
O

cos x = 0

or

PY

2 sin x cos x 2(cos x)(1 2 sin2 x) = 0


(2 cos x)(2 sin2 x + sin x 1) = 0
(2 cos x)(2 sin x 1)(sin x + 1) = 0

x = 6 or
x = 5
6

x=

3
2

These values of x should be checked in the original equation because tan 2x may
not be defined. Upon checking, this is not the case for each value of x obtained.
, 6 , 5
, and 3
.
2
The solutions are 2 , 3
2
6
2

EP
E

? Example 3.8.14. A weight is suspended from a spring and vibrating vertically


according to the equation

f (t) = 20 cos 45 t 56 ,

where f (t) centimeters is the directed distance of the weight from its central
position at t seconds, and the positive distance means above its central position.

(1) At what time is the displacement of the weight 5 cm below its central
position for the first time?
(2) For what values of t does the weight reach its farthest point below its central
position?

Solution. (1) We find the least positive value of t such that f (t) = 5.

20 cos 45 t 56 = 5

cos 45 t 56 = 41
There are two families of solutions for this equation.

250
All rights reserved. No part of this material may be reproduced or transmitted in any form or by any means electronic or mechanical including photocopying without written permission from the DepEd Central Office. First Edition, 2016.

t=

5
6

5
= cos1 14
6
cos1 ( 14 )+2k
4

+ 2k, k Z

In this family of solutions, the least positive value of t happens when


k = 0, and this is

5 cos1 41 + 2(0)
t= +
1.5589.
4
6

5
4

t=

5
6

5
= 2 cos1
6
2cos1 ( 14 )+2k
4


14 + 2k, k Z

PY

C
O

Here, the least positive value of t happens when k = 1, and this is



5 2 cos1 41 + 2(1)
t= +
0.1078.
4
6

5
Therefore, the first time that the displacement of the weight is 5 cm below
its central position is at about 0.1078 seconds.

EP
E

(2) The minimum value of f (t) happens when and only when the minimum

value of cos 45 t 56 is reached. The minimum value of cos 54 t 56 is
1, which implies that the farthest point the weight can reach below its
central position
 is 20 cm. Thus, we need to solve for all values of t such that
cos 54 t 65 = 1.

cos 45 t 56 = 1

5
4

= cos1 (1) + 2k, k 0


5
6

4
t 56 = + 2k
5
5
6

+2k
4

25
12

+ 52 k

t=

Therefore, the weight reaches its farthest point (which is 20 cm) below its
25
central position at t = 12
+ 52 k for every integer k 0.
2

Seatwork/Homework 3.8.3
1. Solve each equation.
+ 2k, k Z

(a) 2 sin2 = sin + 1

Answer:

(b) tan2 x + tan x = 6

Answer: tan1 (3) + k, tan1 2 + k, k Z

(c) sin x = 1 + cos x

7
6

+ 2k,

11
6

Answer:

+ 2k,

+ 2k, + 2k, k Z

251
All rights reserved. No part of this material may be reproduced or transmitted in any form or by any means electronic or mechanical including photocopying without written permission from the DepEd Central Office. First Edition, 2016.

2. Find the solutions in the interval [0, 2).


, , 5
Answer: 2 , 3
2 6 6
Answer: 0, , sin1 13 , sin1 13

(a) sin 2 = cos


(b) 3 cos2 x + sin x = 3
Exercises 3.8
1. Solve each equation.

Answer: 6 + 2k,

(a) 2 sin x + 1 = 0

PY

(c) tan x + 1 = 0

Answer:

(e) tan 4x 1 = 0
(f) sec x 1 = 0

(g) sec2 x + 6 tan x + 4 = 0


(h) cos 2x + 3 = 5 cos x

+ k, k Z

1
(4k
16

+ 1), k Z

Answer: k, k Z

Answer: 4 + k, tan1 (5) + k, k Z


Answer: 3 + 2k,

Answer:

2
3

+ 2k,

4
3

+ 2k, k Z

+ 2k, 2k, k Z

+ 3 = 7 tan x2

Answer: 2 tan1 13 + 2k, 2 tan1 23 + 2k, k Z

x
2

=1

C
O

Answer:

(j) 6 sec2

3
4

Answer: 16 (2k 1), k Z

(d) cos 3x = 0

x
2

+ 2k, k Z

Answer: k, k Z

(b) sin x tan x = 0

(i) cos2 x + sin2

7
6

EP
E

2. Find the solutions in the interval [0, 2).


(a) 4 sin2 x 1 = 0

5 7 11
, 6, 6, 6
6
Answer: 3 , 5
3
5
Answer: 4 , 4
Answer: 6 , 3 , 7
, 4
6
3
2 4 5
Answer: 3 , 3 , 3 , 3
Answer: 7
, 11
6
6
3
Answer: 2
, cos1 34 , 2 cos1 34
Answer: 0, 3 , 5
3
11
Answer: 6

Answer:

(b) 2 cos2 x + 3 cos x 2 = 0

(c) tan x cot x = 0

(d) 2 sin 2x = 3

(e) sec2 x 4 = 0

(f) 2 sin2 x 5 sin x = 3

(g) tan x + sec x = 0

(h) 2 sin 2x = 3 sin x

Answer: 0,

(i) tan2 x = 1 + sec x

(j) tan x + 3 = sec x


3. Find the solutions in the interval [0 , 180 ).
(a) sin x cos x = 0

(b) cot 4x 1 = 0

Answer: 45
Answer: 11.25 , 56.25 , 101.25 , 146.25
252

All rights reserved. No part of this material may be reproduced or transmitted in any form or by any means electronic or mechanical including photocopying without written permission from the DepEd Central Office. First Edition, 2016.

Answer: 0 , 120

(c) 3 cos 2x 3 cos x = 0

? (d) 6 sec2 x + 3 = 7 tan x


? (e) tan2 x + tan x = 6

Answer: 161.6 , 56.3


Answer: 63.4 , 108.4

? 4. A weight is suspended from a spring and vibrating vertically according to the


equation
f (t) = 25.2 sin(3.8t 2.1),

PY

where f (t) centimeters is the directed distance of the weight from its central position at t seconds, and the positive distance means above its central
position.
(a) Find the times when the weight is at its central position.
Solution. We solve the equation f (t) = 0.

C
O

25.2 sin(3.8t 2.1) = 0


sin(3.8t 2.1) = 0
3.8t 2.1 = k k nonnegative integer
2.1 + k
t =
3.8
t 0.55 + 0.83k

EP
E

Therefore, the weight is at its central position at t 0.55 + 0.83k seconds


(where k is a nonnegative integer). In other words, it is at central position
when t = 0.55 s, t = 1.38 s, t = 2.21 s, etc.
(b) For what values of t does the weight reach its farthest point below its
central position?
Solution. The weight reaches it farthest point below the central position
when sin(3.8t 2.1) = 1. Solving for t, we get

sin(3.8t 2.1) = 1
3
3.8t 2.1 =
+ 2k where k is a whole number
2
3+4k
+ 2.1
2
t =
where k is a whole number
3.8
(3 + 4k) + 4.2
t =
where k is a whole number
7.6
t 1.79 + 1.65k where k is a whole number

Therefore, the weight reaches it farthest point below the central position
at t 1.79 + 1.65k seconds (where k is a whole number). For instance,
at t = 1.79 s, t = 3.44 s, t = 5.09 s, etc.

253
All rights reserved. No part of this material may be reproduced or transmitted in any form or by any means electronic or mechanical including photocopying without written permission from the DepEd Central Office. First Edition, 2016.

? 5. The finance department of a car company conducted a study of their weekly


sales in the past years, and came out with the following approximating function:
s(t) = 12.18 cos(0.88t 7.25) + 20.40, t 0,
where s(t) represents weekly car sales in million pesos at week t (t = 0 represents the start of the study).
(a) Find the weekly sales at the start of the study.
Solution.
12.18 cos(0.88(0) 7.25) + 20.40
12.18 cos(7.25) + 20.40
(12.18)(0.5679) + 20.40
27.32

PY

s(0) =
=
=
=

C
O

The weekly sales of the car company at the start of the study is approximately 27.32 million pesos.

(b) Find the projected maximum and minimum weekly sales of the company.
Solution. The projected maximum and minimum weekly sales of the
company are attained when the cosine values are 1 and 1, respectively.
Thus, the maximum weekly sales is 12.18 + 20.40 = 32.58 million pesos,
and the minimum weekly sales is 12.18 + 20.40 = 8.22 million pesos.

EP
E

(c) If the company were able to reach its maximum sales this week, when
will the next projected maximum weekly sales and upcoming projected
minimum weekly sales be?
Solution. The next projected maximum weekly sales will be attained after
2
7.14. Hence, if the company were able
one period. That is, P = 0.88
to reach its maximum sales this week, then the next projected maximum
weekly sales will be after about 7 weeks.

On the other hand, the upcoming minimum weekly sales is projected after
2
3.57. Hence, if the company were
half the period. That is, 12 P = 12 0.88
able to reach its maximum sales this week, then the upcoming projected
minimum weekly sales will be after about 3.5 weeks.

(d) After the start of the study, when did the company experience a weekly
sales of only 10 million for the first time?
Solution. Here, we want to solve s(t) = 10 for the least nonnegative value
of t.
12.18 cos(0.88t 7.25) + 20.40 = 10
520
cos(0.88t 7.25) =
609
254
All rights reserved. No part of this material may be reproduced or transmitted in any form or by any means electronic or mechanical including photocopying without written permission from the DepEd Central Office. First Edition, 2016.

We get
0.88t 7.25 = cos



520

+ 2k, k Z
609

t 11.19 + 7.14k

or
1

0.88t 7.25 = cos

520

609

+ 2k

t 5.29 + 7.14k.

PY

Among these solutions, the least nonnegative value of t is t = 11.19 +


7.14(1) 4.05. Thus, about 4 weeks after the start of the study, the
company experienced a weekly sales of only 10 million for the first time.

C
O

? 6. After many years in business, the financial analyst of a shoe company projected
that the monthly costs of producing their products and monthly revenues from
the sales of their products are fluctuating according to the following formulas:
C(t) = 2.6 + 0.58 sin(0.52t 7.25)
and

R(t) = 2.6 + 1.82 cos(0.52t 7.25),

EP
E

where C(t) and R(t) are the costs and revenues in million of pesos at month t
(t = 0 represents January 2010). About how many months after January 2010
did the company experience a zero profit for the first time?
Solution. The profit is zero when the revenue is the same as the cost.

2.6 + 0.58 sin(0.52t 7.25) = 2.6 + 1.82 cos(0.52t 7.25)


91
tan(0.52t 7.25) =
  29
91
+ k, k Z
0.52t 7.25 = tan1
29

7.25 + tan1 91
+ k
29
t=
16.37 + 6.04k
0.52

The company experienced zero profit for the first time about 16.37+6.04(2)
4.29 or 4 months after January 2010.
7. If x be a real number such that
cos 2x =

2
3

and

1
cos x sin x = ,
2

what is cos x + sin x?


255
All rights reserved. No part of this material may be reproduced or transmitted in any form or by any means electronic or mechanical including photocopying without written permission from the DepEd Central Office. First Edition, 2016.

Solution.
cos 2x =
cos2 x sin2 x =
(cos x + sin x)(cos x sin x) =
 
1
=
(cos x + sin x)
2

PY

cos x + sin x =

2
3
2
3
2
3
2
3
4
3

8. Solve for x [, ): 16 sin4 x + 1 = 8 sin2 x.


Solution.

8 sin2 x
0
0
0
1
sin x =
2
5
x = ,
6
6
=
=
=
=

EP
E

C
O

16 sin4 x + 1
16 sin4 x 8 sin2 x + 1
(4 sin2 x 1)2
4 sin2 x 1

9. Find the smallest positive value of that satisfies the equation




 3

+ + sin
= .
sin
3
3
8
Solution.

3
3
3
8

3
sin cos + cos sin + sin cos cos sin =
3
3
3
3
8

3
2 sin cos =
3
8

3
3 cos =
8
sin

+ + sin

= cos1

!
3
8

256
All rights reserved. No part of this material may be reproduced or transmitted in any form or by any means electronic or mechanical including photocopying without written permission from the DepEd Central Office. First Edition, 2016.

10. In 4ABC, angles A and B (in degrees) satisfy


3 sin A + cos B = 3 and

sin B + 3 cos A = 2.

Prove that C = 30 .
Solution.

3 sin A + cos B = 3
3 cos A + sin B = 2

PY

Square both equations:


(
9 sin2 A + 6 sin A cos B + cos2 B = 9
9 cos2 A + 6 cos A sin B + sin2 B = 4
Add and solve:

C
O

9 + 6 sin(A + B) + 1 = 13
6 sin(A + B) = 3
1
sin(A + B) =
2
A + B = 30 , 150

EP
E

If A + B = 150 , then C = 30 . If A + B = 30 , then A < 30 , and


3 sin A + cos B < 3 21 + 1 = 52 < 3, contradicting the first equation. Therefore,
we only have C = 30 .

Lesson 3.9. Polar Coordinate System

Time Frame: 3 one-hour sessions

Learning Outcomes of the Lesson


At the end of the lesson, the student is able to:
(1) locate points in polar coordinate system;
(2) convert the coordinates of a point from rectangular to polar system and vice
versa; and
(3) solve situational problems involving polar coordinate system.
Lesson Outline
(1) Polar coordinate system: pole and polar axis
257

All rights reserved. No part of this material may be reproduced or transmitted in any form or by any means electronic or mechanical including photocopying without written permission from the DepEd Central Office. First Edition, 2016.

(2) Polar coordinates of a point and its location


(3) Conversion from polar to rectangular coordinates, and vice versa
(4) Simple graphs and applications
Introduction

EP
E

C
O

PY

Two-dimensional coordinate systems are used to describe a point in a plane.


We previously used the Cartesian or rectangular coordinate system to locate a
point in the plane. That point is denoted by (x, y), where x is the signed distance of the point from the y-axis, and y is the signed distance of the point
from the x-axis. We sketched the graphs of equations (lines, circles, parabolas,
ellipses, and hyperbolas) and functions (polynomial, rational, exponential, logarithmic, trigonometric, and inverse trigonometric) in the Cartesian coordinate
plane. However, it is often convenient to locate a point based on its distance
from a fixed point and its angle with respect to a fixed ray. Not all equations
can be graphed easily using Cartesian coordinates. In this lesson, we also use
another coordinate system, which can be presented in dartboard-like plane as
shown below.

3.9.1. Polar Coordinates of a Point


We now introduce the polar coordinate system. It is composed of a fixed point
called the pole (which is the origin in the Cartesian coordinate system) and a
fixed ray called the polar axis (which is the nonnegative x-axis).
258
All rights reserved. No part of this material may be reproduced or transmitted in any form or by any means electronic or mechanical including photocopying without written permission from the DepEd Central Office. First Edition, 2016.

C
O

PY

In the polar coordinate system, a point is described by the ordered pair (r, ).
The radial coordinate r refers to the directed distance of the point from the pole.
The angular coordinate refers to a directed angle (usually in radians) from the
polar axis to the segment joining the point and the pole.

EP
E

Because a point in polar coordinate system is described by an order pair of


radial coordinate and angular coordinate, it will be more convenient to geometrically present the system in a polar plane, which serves just like the Cartesian
plane. In the polar plane shown below, instead of rectangular grids in the Cartesian plane, we have concentric circles with common center at the pole to identify
easily the distance from the pole (radial coordinate) and angular rays emanating
from the pole to show the angles from the polar axis (angular coordinate).

259
All rights reserved. No part of this material may be reproduced or transmitted in any form or by any means electronic or mechanical including photocopying without written permission from the DepEd Central Office. First Edition, 2016.

Example 3.9.1. Plot the following points in one polar plane: A(3, 3 ), B(1, 5
),
6
7
19
7
17
17
5
C(2, 6 ), D(4, 12 ), E(3, ), F (4, 6 ), G(2.5, 4 ), H(4, 6 ), and I(3, 3 ).

EP
E

C
O

PY

Solution.

As seen in the last example, unlike in Cartesian plane where a point has a
unique Cartesian coordinate representation, a point in polar plane have infinitely
many polar coordinate representations. For example, the coordinates (3, 4) in
the Cartesian plane refer to exactly one point in the plane, and this particular
point has no rectangular coordinate representations other than (3, 4). However,
the coordinates (3, 3 ) in the polar plane also refer to exactly one point, but
this point has other polar coordinate representations. For example, the polar
coordinates (3, 5
), (3, 7
), (3, 13
), and (3, 19
) all refer to the same point as
3
3
3
3

that of (3, 3 ).
The polar coordinates (r, + 2k), where k Z, represent the same
point as that of (r, ).
In polar coordinate system, it is possible for the coordinates (r, ) to have
a negative value of r. In this case, the point is |r| units from the pole in the
opposite direction of the terminal side of , as shown in Figure 3.36.
260

All rights reserved. No part of this material may be reproduced or transmitted in any form or by any means electronic or mechanical including photocopying without written permission from the DepEd Central Office. First Edition, 2016.

PY

Figure 3.36

C
O

), B(4, 11
),
Example 3.9.2. Plot the following points in one polar plane: A(3, 4
3
6
C(2, ), and D(3.5, 7
).
4

EP
E

Solution. As described above, a polar point with negative radial coordinate lies
on the opposite ray of the terminal side of .

261
All rights reserved. No part of this material may be reproduced or transmitted in any form or by any means electronic or mechanical including photocopying without written permission from the DepEd Central Office. First Edition, 2016.

Points in Polar Coordinates


1. For any , the polar coordinates (0, ) represent the pole.
2. A point with polar coordinates (r, ) can also be represented by
(r, + 2k) or (r, + + 2k) for any integer k.

Seatwork/Homework 3.9.1

A(2, 2 )

B(3, 7
)
4

G(3, 2
)
3

H(4, 12
)

F (1, 41
)
12

I(2, 11
)
2

EP
E

Answer:

E(4, 4 )

C(4, 6 )

C
O

D(2, 4
)
3

PY

1. Plot the following points in one polar plane:

2. Give the polar coordinates (r, ) with indicated properties that represent the
same point as the given polar coordinates.
262
All rights reserved. No part of this material may be reproduced or transmitted in any form or by any means electronic or mechanical including photocopying without written permission from the DepEd Central Office. First Edition, 2016.

(a)
(b)
(c)
(d)
(e)
(f)

(2, ); r > 0, 2 < 0


); r < 0, 0 < 2
(5, 3
4
4
(5, 3 ); r < 0, 2 < 0
(1, 0); r < 0, 0 < 2
(2, sin1 (0.6)); r < 0, 2 < 0
(3, cos1 (0.4)); r > 0, 0 < 2

Answer: (2, )
Answer: (5, 7
)
4
2
Answer: (5, 3 )
Answer: (1, )
Answer: (2, sin1 (0.6) )
Answer: (3, cos1 (0.4) + )

3.9.2. From Polar to Rectangular, and Vice Versa

PY

We now have two ways to describe points on a plane whether to use the Cartesian coordinates (x, y) or the polar coordinates (r, ). We now derive the conversion from one of these coordinate systems to the other.

EP
E

C
O

We superimpose the Cartesian and polar planes, as shown in the following


diagram.

Figure 3.37

Suppose a point P is represented by the polar coordinates (r, ). From Lesson


3.2 (in particular, the boxed definition on page 139), we know that
x = r cos

and

y = r sin .

Conversion from Polar to Rectangular Coordinates

x = r cos
(r, )
(x, y)
y = r sin

Given one polar coordinate representation (r, ), there is only one


rectangular coordinate representation (x, y) corresponding to it.
263
All rights reserved. No part of this material may be reproduced or transmitted in any form or by any means electronic or mechanical including photocopying without written permission from the DepEd Central Office. First Edition, 2016.

Example 3.9.3. Convert the polar coordinates (5, ) and (3, 6 ) to Cartesian
coordinates.
Solution.

x = 3 cos = 3 3
6
2
(3, 6 )
y = 3 sin = 3
2

( 3 2 3 , 32 )

PY

(5, 0)

As explained on page 260 (right after Example 3.9.1), we expect that there
are infinitely many polar coordinate representations that correspond to just one
given rectangular coordinate representation. Although we can actually determine
all of them, we only need to know one of them and we can choose r 0.

C
O

Teaching Notes
One can also easily
convert the polar
coordinates (5, )
to its
corresponding
rectangular
coordinates (5, 0)
by simply plotting
the point.

x = 5 cos = 5
(5, )
y = 5 sin = 0

Suppose a point P is represented by the rectangular coordinates (x, y). Referring back to Figure 3.37, the equation of the circle is
p
x2 + y 2 = r 2
=
r = x2 + y 2 .

We now determine . If x = y = 0, then r = 0 and the point is the pole. The


pole has coordinates (0, ), where is any real number.

EP
E

If x = 0 and y 6= 0, then we may choose to be either 2 or


equivalents) depending on whether y > 0 or y < 0, respectively.

3
2

(or their

Now, suppose x 6= 0. From the boxed definition again on page 139, we know
that
y
tan = ,
x
where is an angle in standard position whose terminal side passes through the
point (x, y).
Conversion from Rectangular to Polar Coordinates
(x, y) = (0, 0) (r, ) = (0, ), R

(y, )
if y > 0
2
(0, y) (r, ) =
(|y|, 3 ) if y < 0
y6=0
2

(x, 0) if x > 0
(x, 0) (r, ) =
(|x|, ) if x < 0
x6=0
264

All rights reserved. No part of this material may be reproduced or transmitted in any form or by any means electronic or mechanical including photocopying without written permission from the DepEd Central Office. First Edition, 2016.

(x, y)
x6=0, y6=0

(r, )
r=

x2 + y 2

tan =

y
x

same quadrant as (x, y)


Given one rectangular coordinate representation (x, y), there are
many polar coordinate representations (r, ) corresponding to it. The
above computations just give one of them.

PY

Example 3.9.4. Convert each Cartesian coordinates to polar coordinates (r, ),


where r 0.
(4) (6, 2)
(1) (4, 0)
(5) (3, 6)

(3) (3, 3)

C
O

(2) (4, 4)

(6) (12, 8)

Solution. (1) (4, 0) (4, )

tan =

y
x

(2) The point (4, 4) is in QI.


p

r = x2 + y 2 = 42 + 42 = 4 2
4
4

= 1 =

4 2, 4

EP
E

(4, 4)

(3) (3, 3) in QIII


q

r = (3)2 + ( 3)2 = 2 3

7
6

tan = 33 = 33 = =


(3, 3) 2 3, 7
6

Teaching Notes
Plotting the points
on the
superimposed
Cartesian and
polar planes is a
quicker approach
in converting
rectangular
coordinates to
polar.

(4) (6, 2) in QIV


p

r = 62 + (2)2 = 2 10
2
6

= 13 = = tan1 13


(6, 2) 2 10, tan1 13
tan =

(5) (3, 6) in QII


p

r = (3)2 + 62 = 3 5
6
3

= 2 = = + tan1 (2)


(3, 6) 3 5, + tan1 (2)

tan =

Teaching Notes
Recall that
tan1 (2) is in
QIV.

265

All rights reserved. No part of this material may be reproduced or transmitted in any form or by any means electronic or mechanical including photocopying without written permission from the DepEd Central Office. First Edition, 2016.

Teaching Notes
We may also use
= tan1 23 .

(6) (12, 8) in QIII


p

r = (12)2 + (8)2 = 4 13
tan =

8
12

2
3

(12, 8)

= = + tan1


4 13, + tan1 23

2
3

Seatwork/Homework 3.9.2

1. Convert each polar coordinates to Cartesian coordinates.


(a) (2, )

(d)
(e)

(f) (3, 2
)
3

(g) (1, sin1 ( 31 ))


(h) (2, tan1 34 )

PY

(c)

Answer: (0, 6)

Answer: (2, 3)

Answer: (2 3, 2)

C
O

(b)

Answer: (2, 0)

Answer: (2 2, 2 2)

)
(4, 3
4
)
(6, 3
2
(2, 2
)
3
(4, 7
)
6

Answer: ( 32 , 3 2 3 )

Answer: ( 2 3 2 , 31 )
Answer: ( 65 , 58 )

(a) (0, 6)

EP
E

(b) (3, 3)

(c) (3 3, 3)

(d) (1, 3)

2. Convert each Cartesian coordinates to polar coordinates (r, ), where r 0.


Answer: (6, 2 )

Answer: (3 2, 4 )

Answer: (6, 5
)
6

Answer: (2, 4
)
3

Answer: ( 17, tan1 4)

Answer: (2 5, + tan1 (2))

Answer: (2 10, + tan1 31 )

Answer: ( 2, 3
)
4

(e) (1, 4)

(f) (2, 4)

(g) (6, 2)

(h) (1, 1)

3.9.3. Basic Polar Graphs and Applications


From the preceding session, we learned how to convert polar coordinates of a
point to rectangular and vice versa using the following conversion formulas:
y
r2 = x2 + y 2 , tan = , x = r cos , and y = r sin .
x
Because a graph is composed of points, we can identify the graphs of some equations in terms of r and .
266
All rights reserved. No part of this material may be reproduced or transmitted in any form or by any means electronic or mechanical including photocopying without written permission from the DepEd Central Office. First Edition, 2016.

Graph of a Polar Equation


The polar graph of an equation involving r and is the set of all
points with polar coordinates (r, ) that satisfy the equation.
As a quick illustration, the polar graph of the equation r = 2 2 sin consists
of all points (r, ) that satisfy the equation. Some of these points are (2, 0), (1, 6 ),
(0, 2 ), (2, ), and (4, 3
).
2

PY

Example 3.9.5. Identify the polar graph of r = 2, and sketch its graph in the
polar plane.

EP
E

C
O

Solution. Squaring the equation, we get r2 = 4. Because r2 = x2 + y 2 , we have


x2 + y 2 = 4, which is a circle of radius 2 and with center at the origin. Therefore,
the graph of r = 2 is a circle of radius 2 with center at the pole, as shown below.

In the previous example, instead of using the conversion formula r2 = x2 + y 2 ,


we may also identify the graph of r = 2 by observing that its graph consists of
points (2, ) for all . In other words, the graph consists of all points with radial
distance 2 from the pole as rotates around the polar plane. Therefore, the
graph of r = 2 is indeed a circle of radius 2 as shown.

Example 3.9.6. Identify and sketch the polar graph of = 5


.
4
Solution. The graph of = 5
consists of all points (r, 5
) for r R. If
4
4
5
r > 0, then points (r, 4 ) determine a ray from the pole with angle 5
from
4
the polar axis. If r = 0, then (0, 5
)
is
the
pole.
If
r
<
0,
then
the
points
4
(r, 5
)
determine
a
ray
in
opposite
direction
to that of r > 0. Therefore, the
4
267
All rights reserved. No part of this material may be reproduced or transmitted in any form or by any means electronic or mechanical including photocopying without written permission from the DepEd Central Office. First Edition, 2016.

C
O

PY

graph of = 5
is a line passing through the pole and with angle 5
with
4
4
respect to the polar axis, as shown below.

Example 3.9.7. Identify (and describe) the graph of the equation r = 4 sin .
Solution.

EP
E

r
r2
x2 + y 2
x2 + y 2 4y
x2 + (y 2)2

= 4 sin
= 4r sin
= 4y
=0
=4

Therefore, the graph of r = 4 sin is a circle of radius 2 and with center at (2, 2 ).

268
All rights reserved. No part of this material may be reproduced or transmitted in any form or by any means electronic or mechanical including photocopying without written permission from the DepEd Central Office. First Edition, 2016.

? Example 3.9.8. Sketch the graph of r = 2 2 sin .


Solution. We construct a table of values.
0

7
6

0.59 0.27
5
4

4
3

3
2

3
4

5
6

0.27 0.59
5
3

7
4

11
6

3.73 3.41

EP
E

C
O

3.41 3.73

2
3

PY

This heart-shaped curve is called a cardioid.

? Example 3.9.9. The sound-pickup capability of a certain brand of microphone


is described by the polar equation r = 4 cos , where |r| gives the sensitivity of
the microphone to a sound coming from an angle (in radians).
(1) Identify and sketch the graph of the polar equation.
(2) Sound coming from what angle [0, ] is the microphone most sensitive
to? Least sensitive?

Solution. (1)

r
r2
x2 + y 2
x2 + 4x + y 2
(x + 2)2 + y 2

= 4 cos
= 4r cos
= 4x
=0
=4
269

All rights reserved. No part of this material may be reproduced or transmitted in any form or by any means electronic or mechanical including photocopying without written permission from the DepEd Central Office. First Edition, 2016.

C
O

PY

This is a circle of radius 2 and with center at (2, ).

(2) We construct a table of values.

2
3

4 3.46

2.83

3
4

5
6

2.83 3.46

EP
E

From the table, the microphone is most sensitive to sounds coming from
angles = 0 and = , and least sensitive to sound coming from an angle
2
= 2 .

Seatwork/Homework 3.9.3

1. Identify (and describe) the graph of each polar equation.


(a) = 2
3
Answer: Line passing through the pole with angle
polar axis

2
3

with respect to the

(b) r = 3
Answer: Circle with center at the pole and of radius 3
(c) r = 2 sin
Answer: Circle of radius 1 and with center at (1, 2 )

270
All rights reserved. No part of this material may be reproduced or transmitted in any form or by any means electronic or mechanical including photocopying without written permission from the DepEd Central Office. First Edition, 2016.

(d) r = 3 cos
Answer: Circle of radius 1.5 and with center at (1.5, 0)
(e) r = 2 + 2 cos

Answer: A cardioid

2. Sketch the graph of each polar equation.

EP
E

C
O

PY

(a) r = 3

(b) r = 2 sin

271
All rights reserved. No part of this material may be reproduced or transmitted in any form or by any means electronic or mechanical including photocopying without written permission from the DepEd Central Office. First Edition, 2016.

EP
E

(d) r = 4 cos

C
O

PY

(c) r = 2 + 2 sin

3. The sound-pickup capability of a certain brand of microphone is described by


the polar equation
r = 1.5(1 + cos ),
272
All rights reserved. No part of this material may be reproduced or transmitted in any form or by any means electronic or mechanical including photocopying without written permission from the DepEd Central Office. First Edition, 2016.

where |r| gives the sensitivity of the microphone of a sound coming from an
angle (in radians).

C
O

PY

(a) Identify and sketch the graph of the polar equation.


Answer: A cardioid

Exercises 3.9

(b) Sound coming from what angle [0, 2) is the microphone most sensitive to? Least sensitive?
Answer: Most sensitive at = 0; least sensitive at =

EP
E

1. Plot the following points in one polar plane:


A(2, 2 )

D(3, 2
)
3

)
G(4, 8
3

B(1, 7
)
3
E(4, 4 )

H(2, 11
)
12

C(2, 4 )
F (3, 7
)
12
I(1, 15
)
2

Answer:

273
All rights reserved. No part of this material may be reproduced or transmitted in any form or by any means electronic or mechanical including photocopying without written permission from the DepEd Central Office. First Edition, 2016.

2. Give the polar coordinates (r, ) with indicated properties that represent the
same point as the given polar coordinates.
(a)
(b)
(c)
(d)
(e)

(3, 2); r > 0, 0 < 2


(10, 4
); r < 0, 0 < 2
3
3
(4, 2 ); r < 0, 2 < 0
(1, ); r < 0, 0 < 2
(2, cos1 32 ); r > 0, 2 < 0

Answer: (3, )
Answer: (10, 5
)
3

Answer: (4, 2 )
Answer: (1, )
Answer: (2, + cos1 32 )

PY

3. Convert each polar coordinates to Cartesian coordinates.

(c) (2, 2
)
3
(d) (5, 3)
(e) (8, 11
)
6

Answer: (4, 0)

Answer: (2 2, 2 2)

Answer: (1, 3)
Answer: (5, 0)

Answer: (4 3, 4)

C
O

(a) (4, )
)
(b) (4, 7
4

(a) (0, 6)
(b) (5, 5)

EP
E

(c) (2, 6)

4. Convert each Cartesian coordinates to polar coordinates (r, ), where r 0


and 0 2.

(d) (1, 4)

(e) (1, 3)

Answer: (6, 3
)
2
5
Answer: (5 2, 4 )

Answer: (2 10, + tan1 (3))

Answer: ( 17, 2 + tan1 (4))


Answer: (2, 5
)
3

5. Identify and sketch the graph of each polar equation.

(a) = 3
Answer: A line passing through the pole and with angle 3 with respect
to the polar axis

274
All rights reserved. No part of this material may be reproduced or transmitted in any form or by any means electronic or mechanical including photocopying without written permission from the DepEd Central Office. First Edition, 2016.

C
O

PY

(b) r = 3 sin
Answer: A circle tangent to the x-axis with center at (0, 1.5)

EP
E

(c) r = cos
Answer: A circle tangent to the y-axis with center at (0.5, 0)

(d) r = 2 2 cos
Answer: A cardioid

275
All rights reserved. No part of this material may be reproduced or transmitted in any form or by any means electronic or mechanical including photocopying without written permission from the DepEd Central Office. First Edition, 2016.

PY

(e) r = 1 + sin
Answer: A cardioid

EP
E

Answer:

C
O

6. The graph of the polar equation r = 2 cos 2 is a four-petaled rose. Sketch its
graph.

? 7. A comet travels on an elliptical orbit that can be described by the polar


equation
1.164
r=
1 + 0.967 sin
with respect to the sun at the pole. Find the closest distance between the sun
and the comet.
Answer: Closest distance occurs when sin = 1, so r =

1.164
1.967

0.59 units.

? 8. Polar equations are also used by scientists and engineers to model motion of
satellites orbiting the Earth. One satellite follows the path
r=

36210
,
6 cos
276

All rights reserved. No part of this material may be reproduced or transmitted in any form or by any means electronic or mechanical including photocopying without written permission from the DepEd Central Office. First Edition, 2016.

where r is the distance in kilometers between the center of the Earth and the
satellite, and is the angular measurement in radians with respect to a fixed
predetermined axis.
(a) At what value of [0, 2) is the satellite closest to Earth, and what is
the closest distance?
Answer: The satellite is closest to Earth when cos = 1, and this occurs
36210
5182.86
when = . The closest distance is, therefore, r = 6(1)
kilometers.

PY

(b) How far away from Earth can the satellite reach?
Answer: The satellite can reach as far as r = 36210
7242 km away from
61
the Earth.
9. The graph of the polar equation

15
3 2 cos
is a conic section. Identify and find its equation in rectangular coordinate
system.
Answer: Ellipse, 5x2 + 9y 2 60x 225 = 0

C
O

r=

Solution

EP
E

3r 2r cos = 15
p
x
3 x2 + y 2 2r = 15
r
 p
2
3 x2 + y 2 = (2x + 15)2

5x2 + 9y 2 60x 225 = 0 an ellipse

10. The graph of the polar equation

6
3 + 3 sin
is a parabola. Find its equation in rectangular coordinate system.
r=

Answer: y = 14 x2 + 1

11. For what values of [0, 2) will the graphs of r = 4 cos and r cos = 1
intersect?
Answer: 3 , 2
, 4
, 5
3
3
3

12. Convert the polar equation


r=
into Cartesian equation.

2 sin 2
sin3

cos3

Answer: x3 = y 3 + 4xy

277
All rights reserved. No part of this material may be reproduced or transmitted in any form or by any means electronic or mechanical including photocopying without written permission from the DepEd Central Office. First Edition, 2016.

PY

C
O

Answers to

All Exercises
in Supplementary Problems

and

EP
E

All Exercises in Topic Tests

All rights reserved. No part of this material may be reproduced or transmitted in any form or by any means electronic or mechanical including photocopying without written permission from the DepEd Central Office. First Edition, 2016.

All rights reserved. No part of this material may be reproduced or transmitted in any form or by any means electronic or mechanical including photocopying without written permission from the DepEd Central Office. First Edition, 2016.

6. center (5, 8), r = 10



3
3. center 4,
,r=1
4

EP
E

4. center (2, 2), r = 4

5. center (7, 6), r = 11

1
2

2. center (0, 0), r = 5

1. center (0, 0), r =

Supplementary Problems 1.1 (page 17)

279

5
3


5 7
7
,
,r=
2 2
4

PY


1 4
, , r = 2
3 3

C
O
9. center

8. center

7. center (2, 4), r =

All rights reserved. No part of this material may be reproduced or transmitted in any form or by any means electronic or mechanical including photocopying without written permission from the DepEd Central Office. First Edition, 2016.

21. (x + 2)2 + (y 3)2 = 12

20. (x 1)2 + (y 1)2 = 5

19. (x + 10)2 + (y 7)2 = 36

18. (x 1.5)2 + (y 1)2 = 18

y 2x + 8 = 0.

Finally, the radius of the circle can be obtained by computing the distance
between the center and any of the points A, B, or C. The radius is 5; therefore,
the circles equation is (x 3)2 + (y + 2)2 = 25

The center of the circle is on both perpendicular bisectors; that is, the center is
the intersection of these two lines. By solving the system involving y 2x+8 =
0 and y 7x + 23 = 0, we see that the center is (3, 2).

2y + 3x + 5 = 0.

25. (x + 5)2 + (y + 1)2 = 8

280

The intersection of 2y + 3x + 5 = 0 and the tangent line is the point of


tangency. By solving a system,
p we see that the point of tangency
is (1, 1).
The radius of the circle is (5 (1))2 + (1 (1))2 = 13. Therefore,
the equation of the circle is(x + 3)2 + (y 2)2 = 13.

3
y 2 = (x + 3)
2

24. The tangent line 2x 3y = 1 has the equivalent equation y = 32 x 31 , which


implies that its slope is 2/3. Then the equation of the line that contains the
circles center and is perpendicular to the tangent line is

23. (x 2.5)2 + (y 0.5)2 = 14.5

28. We set up a coordinate system by making the circles center the origin. Since
the circle has radius 11/2 = 5.5, its equation is x2 + y 2 = 5.52 .

27. Set up a Cartesian coordinate system by assigning C as the origin. Then the
circle on the left end has radius 100 and has equation x2 + y 2 = 10000. A
radius of the circle on the right end can be drawn from C to the upper right
corner
of the figure;
this radius has length (by the Pythagorean theorem)

3002 + 1002 = 100000. Then the circle on the right end has equation x2 +
y 2 = 100000. We want the
at y = 50. In this case, the

length of the segment


2
left endpoint
has x =
10000 50 = 7500 and the
right endpoint
has

x = 100000 502 = 97500. Then the total length is 97500 ( 7500)


= 50 3 + 50 39 m 398.85 m.

26. The equation x + 3y = 9 is equivalent to y = 13 x + 3, which implies that


the slope of the tangent line is 1/3. Thus, the equation of the line containing
the point of tangency (3, 4) and is perpendicular to the tangent line is y
4 = 3(x 3). This line contains the center, which has x coordinate 4.
Then
the center is k = 4 3(4 3) = 1. The radius
p the y coordinate of
is (3 4)2 + (4 1)2 = 10. Therefore, the circles equation is (x 4)2 +
(y 1)2 = 10.

1. vertex (0, 0), focus (9, 0), directrix x = 9, axis y = 0

Supplementary Problems 1.2 (page 31)

PY

From the conditions of the problem, the surface of the water is at y = 1 so


the base of the boat will be at y = 2. The left and right endpoints can be
computed as follows:

x2 + 4 = 5.52 x = 26.25

Then the boats maximum possible width is 26.25 ( 26.25), which is


approximately 10.25 ft.

C
O

Similarly, the slope of BC is 1/7 and its midpoint is (3.5, 1.5). Then the
perpendicular bisector to BC has equation


7
3
y 7x + 23 = 0.
y =7 x
2
2

y 2 = 2(x 5)

22. Let A(3, 3), B(7, 1), and C(0, 2). The slope of AB is 1/2 and its midpoint
is (5, 2). Then the perpendicular bisector to AB has equation

15. (x 15)2 + (y + 7)2 = 9

17. (x + 2)2 + (y 3.5)2 = 31.25

16. (x 15)2 + (y + 7)2 = 49

EP
E

14. (x 15)2 + (y + 7)2 = 225

13. (x 15)2 + (y + 7)2 = 49

12. (x + 8)2 + (y 4)2 = 20

11. (x 17)2 + (y 5)2 = 144

10. x2 + y 2 = 75

All rights reserved. No part of this material may be reproduced or transmitted in any form or by any means electronic or mechanical including photocopying without written permission from the DepEd Central Office. First Edition, 2016.

4. vertex (1, 1), focus



1
3
, 1 , directrix x = , axis y = 1
2
2

3. vertex (1, 7), focus (2, 7), directrix x = 0, axis y = 7

EP
E

2. vertex (0, 0), focus (0, 5), directrix y = 5, axis x = 0


3,


3
11
, directrix y = , axis x = 3
2
2

PY

(3x 2)2 = 84y 112


 
2
2
3 x
= 84y 112
3

2
2
9 x
= 84y 112
3

2
28
112
2
= y
x
3
3
9

2


2
28
4
x
=
y
3
3
3




2 11
2
2 4
vertex
,
, focus
,
, directrix y = 1, axis x =
3 3
3 3
3

C
O
281

6. The equation can be simplified as follows:

5. vertex (3, 2), focus

All rights reserved. No part of this material may be reproduced or transmitted in any form or by any means electronic or mechanical including photocopying without written permission from the DepEd Central Office. First Edition, 2016.

15. 4.17 cm

25
2.08 ft
12

EP
E

14.

13. (y 8)2 = 8(x + 3)

12. x2 = 8(y 7)

1.

(0, 0)

covertices:

W1 (0, 2), W2 (0, 2)

F1 (2, 0), F2 (2, 0)

vertices: V1 (2 2, 0), V2 (2 2, 0)

foci:

center:

282

Since the rectangular object is 25 m wide, it will span the interval from x =
3

12.5 to x = 12.5. The height of the arch at these endpoints is y = 32


1197
2
12.5 + 24 =
9.35 m.
128

Supplementary Problems 1.3 (page 45)

3.

2.

4.

F1 (0, 1), F2 (0, 5)

(0, 2)

F1 (1

(1, 1)

3, 1), F2 (1 +

W1 (4, 2), W2 (4, 2)

F1 (5, 2 6 2), F2 (5, 2 + 6 2)

3, 1)

covertices:

W1 (12, 2), W2 (2, 2)

vertices: V1 (5, 9), V2 (5, 13)

foci:

center:

(5, 2)

W1 (1, 0), W2 (1, 2)

PY

covertices:

vertices: V1 (1, 1), V2 (3, 1)

foci:

center:

covertices:

vertices: V1 (0, 3), V2 (0, 7)

foci:

center:

C
O

18. Set up a coordinate system by assigning the midpoint of the archs base as
the origin. Then the parabolic arch has vertex at (0, 24), opens downward,
and passes through the point (16, 24). Its equation is of the form x2 =
4c(y 24). By substituting x = 16 and y = 0, we get c = 8/3 and the
(y 24).
equation is x2 = 32
3

17. 3.75 cm

16. Note that the lowest point of the rope is the vertex of the parabola. This
occurs midway between the two posts. Set up a coordinate system by making
the vertex (0, 0). Then the parabola has equation of the form x2 = 4cy and
passes through the points (4.5, 8) and (4.5, 8). Substituting either of these
points to the equation tells us that c = 4.52 /32. Therefore, the equation of
4.52
the parabola is x2 =
y.
8
The x coordinates of the points 2 m from the posts are 2.5 and 2.5. Substitut200
ing either of these values to the equation tells us that the height is
2.47
81
m

11. (y 9) = 80(x 4)

10. (y 3) = 34(x + 10)

9. (x + 10)2 = 34(y 3)

8. (x + 10)2 = 16(y + 5)

7. (y 11)2 = 36(x 7)

All rights reserved. No part of this material may be reproduced or transmitted in any form or by any means electronic or mechanical including photocopying without written permission from the DepEd Central Office. First Edition, 2016.

F1 (1, 5), F2 (13, 5)

(7, 5)

W1 (4, 5), W2 (12, 5)

(x 2)2 (y 8)2
+
=1
49
16

covertices:

8.

(x + 3)2 (y + 7)2
+
=1
36
100

EP
E

F1 (4, 1), F2 (4, 11)

(4, 5)

W1 (7, 13), W2 (7, 3)

vertices: V1 (4, 5), V2 (4, 15)

foci:

center:

covertices:

vertices: V1 (3, 5), V2 (17, 5)

foci:

center:

16. Set up a coordinate system with the center of the ellipse at the origin. Then
a = 50, b = 30, and c = 40. Since the radiation source and the target area will
be located at the foci, they should be placed at the coordinates (40, 0) and
(40, 0). These positions are 10 cm away from an end of the reflector.

15. Set up a coordinate system with the center of the ellipse at the origin. Then
a = 60 and b = 20. We want the length of the segment with endpoints (on the
2
y2
+ 20
ellipse) having x = 45 (or 45). The y coordinates are given by 45
2 = 1,
602
q

2
45
2
or y = 20 1 602 13.23. Hence, the desired width is 26.46 ft.

14. Set up a coordinate system with the center of the ellipse at theorigin. Then
c = 50 and a = 56. It follows that the height at the center is b = 562 502
25.22 m.

283

12. Set up a coordinate system with the center of the ellipse at the origin. Then
y2
x2
the equation is of the form 20
2 + b2 = 1. The point (18, 2) is on the ellipse so
182
22
we have 202 + b2 = 1. The height at the center is b 4.59 ft.

11. Since the major axis is vertical, the center has the same x coordinate as the
focus and the same y coordinate as the covertex; that is, the center is (9, 10).
(x + 9)2
Then c = 5, b = 10, and a2 = 125. Therefore, the equation is
+
100
2
(y 10)
= 1.
125

10. The center is at (12, 18) or (3, 7). Since the major axis should be longer
than the minor axis, the center should be at (3, 7) with a = 11 and b = 9.
(x + 3)2 (y + 7)2
Moreover, the ellipse is vertical. Hence, the equation is
+
= 1.
81
121

y2
(x + 211)2
+
= 1.
14 776 336 14 731 815

13. Recall that the unit is 100 km. The vertices of the ellipse are at (3633, 0) and
(4055, 0). Then the center of the ellipse is at (211, 0). Then a = 3844 and
c = 211. It follows that b2 = 14731815. The equation is

2.

1.

F1 ( 181, 0), F2 ( 181, 0)

F1 (0, 2), F2 (0, 2)

(0, 0)

asymptotes: y = x

vertices: V1 (0, 1), V2 (0, 1)

foci:

center:

vertices: V1 (10, 0), V2 (10, 0)


9
asymptotes: y = x
10

PY

foci:

center: (0, 0)

Supplementary Problems 1.4 (page 59)

C
O

9. The center is (9, 10) and c = 12.


We see that the given point (9, 15) is
a covertex, so b = 5. Then a = 52 + 122 = 13. Therefore, the equation is
(x + 9)2 (y 10)2
+
= 1.
169
25

7.

6.

5.

All rights reserved. No part of this material may be reproduced or transmitted in any form or by any means electronic or mechanical including photocopying without written permission from the DepEd Central Office. First Edition, 2016.

F1 (8, 4), F2 (8, 16)

(8, 6)

EP
E

vertices: V1 (8, 2), V2 (8, 14)


4
asymptotes: y 6 = (x 8)
3

foci:

center:

4.

(0, 5)

foci: F1 ( 19, 5), F2 ( 19, 5)

vertices: V1 ( 15, 5), V2 ( 15, 5)


2
asymptotes: y 5 = x
15

center:

3.

284

F2 (3, 3 +

F1 (3, 3

(3, 3)

15),

15)

(2, 4)

(x 6)2 (y + 2)2

=1
9
72
8.

9.

(x + 10)2 (y + 4)2

=1
81
256

(x 11)2 (y 9)2

= 1.
36
64

10. The intersection (11, 9) of the two asymptotes is the center of the hyperbola.
Then the hyperbola is horizontal and a = 6. Since the slopes of the asymptotes
are 34 , we have ab = 43 and b = 8. Therefore, the equation is

y2
(x + 7)2

=1
144
145
7.

PY

F1 (13, 4), F2 (9, 4)

vertices: V1 (2 105, 4), V2 (2 + 105, 4)


4
asymptotes: y 4 =
(x + 2)
105

foci:

center:

6),

V2 (3, 3 + 6)

6
asymptotes: y + 3 =
(x + 3)
3

vertices: V1 (3, 3

foci:

center:

C
O

6.

5.

All rights reserved. No part of this material may be reproduced or transmitted in any form or by any means electronic or mechanical including photocopying without written permission from the DepEd Central Office. First Edition, 2016.

b
y k = (x h).
a

Thus, the plane is approximately 5.71 miles to the west of station B.

285

14. Situate the two stations on the Cartesian plane so that A(75, 0) and B (75, 0).
Let P represent the plane and let its coordinates be (h, 60). Since the signal
from B arrives earlier than the signal from b, it should follow that h > 0.
Moreover, P should lie on the the right branch of the hyperbola whose equation
x2
y2
is given by 2 2 = 1, where 2a = P A P B = 0.2 (480) = 96 a = 48,
b

a
and b = 752 482 = 9 41.
s

602
h2
602
2 = h =
1+
482 69.29
2
48
3321
9 41

8. circle

7. empty set

6. hyperbola

5. parabola

If the focus of the parabola is (1, 0), then its vertex is (1, 0.5) and its
equation is (x + 1)2 = 2(y 0.5). Similarly, if the focus of the parabola is
(3, 0), then its vertex is (3, 0.5) and its equation is (x 3)2 = 2(y 0.5).

10. The standard equation of the hyperbola is

(x 1)2
y 2 = 1; its foci are (1, 0)
3
and (3, 0) and the top side of its auxiliary rectangle is on the line y = 1.

9. The standard equation of the ellipse is

(y 2)2
(x 5)2
+
= 1; so its foci
36
100
are (5, 10) and (5, 6) while its vertices are (5, 12) and (5, 8). The equations
of the circles are (x 5)2 + (y 10)2 = 4, (x 5)2 + (y 10)2 = 324,
(x 5)2 + (y + 6)2 = 4, and (x 5)2 + (y + 6)2 = 324.

4. point

3. parabola

2. ellipse

1. pair of intersecting lines

Supplementary Problems 1.5 (page 66)

(a) is a circle if

Its graph

r+2
.
r1
r+2
> 0; that is, when r (, 2) (1, +).
r1

(x + 7)2 + (y 3)2 =

13. The equation simplifies to

12. The standard equation of the ellipse is

PY

x2 (y 5)2
+
= 1; so its center is (0, 5)
17
9

and its vertical tangent lines are x = 17 and x = 17.





If the parabolas directix is x = 17, then its vertex is 217 , 5 and its


equation is (y 5)2 = 2 17 x + 217 . Similarly, if the parabolas direc


17
tix is x = 17, then its vertex is
, 5 and its equation is (y 5)2 =
2


2 17 x 217 .

11. The standard equation of the hyperbola is

(y + 5)2 (x + 9)2

= 1. Its auxil25
25
iary rectangle has corners (14, 0), (4, 0), (4, 10), (14, 10). The equation of the circle is (x + 9)2 + (y + 5)2 = 50.

C
O

13. The midpoint (9, 1) of the two given corners is the center of the hyperbola.
Since the transverse axis is horizontal, a = 7 and b = 2. Therefore, the
equation is
(x 9)2 (y 1)2

= 1.
49
4

(y 4)2 x2

= 1.
16
9

12. Note that (3, 8) is not on the given asymptote. Then two other corners of the
auxiliary rectangle are on the given asymptote; one has x coordinate 3 and one
has y coordinate 8. These points are (3, 8) and (3, 0). Their midpoint (0, 4)
is the center of the hyperbola; then a = 4 and b = 3. Hence, the equation is

a
5
(y 8)2 (x + 4)2

=
= a = 5 and b = 12 =

= 1.
2
12
25
144
169 a

11. The intersection (4, 8) of the two asymptotes is the center of the hyperbola.
Then the hyperbola is vertical and c = 13. Since the slopes of the asymptotes
5
5
, we have ab = 12
.
are 12

Since c = 13, we have a2 + b2 = 169 and b = 169 a2 . It follows that

(y k)2 (x h)2
a

= 0 y k = (x h).
a2
b2
b
Hence, the slopes of the asymptotes are a/b. This is used in the following
item.

In contrast, the asymptotes of a vertical hyperbola are given by the equation

EP
E

(x h)2 (y k)2

=0
a2
b2

Note: The slopes of the asymptotes of a horizontal hyperbola are b/a. In


fact, the asymptotes are given by the equation

All rights reserved. No part of this material may be reproduced or transmitted in any form or by any means electronic or mechanical including photocopying without written permission from the DepEd Central Office. First Edition, 2016.

EP
E

(x 4)2
y2
+
= 1. Its graph is
m+7
2m

(b)


 

4
9
4 89
,
, ,
3 20
3 20

1. (a) (1, 6)

Supplementary Problems 1.6 (page 77)

(e) the empty set if m + 7 < 0 and 2m < 0; that is, when m < 7.

(d) a hyperbola if 2m(m + 7) < 0; that is, when 7 < m < 0.

(c) a vertical ellipse if 2m > m + 7 > 0; that is, when m > 7.

(b) a horizontal ellipse if m + 7 > 2m > 0; that is, when 0 < m < 7.

(a) a circle if m + 7 = 2m; that is, when m = 7.

14. The equation simplifies to

r+2
= 0; that is, when r = 2.
r1
r+2
(c) is the empty set if
< 0; that is, when r (2, 1).
r1

(b) is a point if

286

PY
(e) No solution






3
1,
, 1 15, 1 , 1 + 15, 1
2

C
O

(d)

(c) (1, 6), (1, 2)

All rights reserved. No part of this material may be reproduced or transmitted in any form or by any means electronic or mechanical including photocopying without written permission from the DepEd Central Office. First Edition, 2016.

EP
E

x2 + (y 6)2 = 36

x2 = 4ky,

Thus k (, 0] (3, +).

If k 0, the system will always have a unique solution.

287

If k > 0, the system might have one or two solutions. To ensure that the
solution is unique, we set the discriminant to be nonpositive: 4k 12 0
k 3.

We now consider two cases.

Substituting the second equation in the first equation yields y 2 +(4k12)y = 0.


Note that y = 0 is already a root.

where the first equation is a circle above the x-axis, tangent to the x-axis at
x = 0, and the second equation is a parabola facing upward or downward
depending on k.

5. We have the system

PY

(0, 0)

F1 (0, 3 2), F2 (0, 3 2)

y2
x2
+
= 1. This is a vertical ellipse.
7
25

vertices: V1 (0, 5), V2 (0, 5)

covertices: W1 ( 7, 0), W2 ( 7, 0)

foci:

center:

2. (a) The equation is equivalent to

Hence, the graph is the point (8, 0).

(x 8)2 y 2
+
= 0.
2
7

(d) Since the coefficients of x2 and y 2 have the same sign and are unequal,
the graph is an ellipse, a point, or the empty set. Completing the squares,
we see that the equation is equivalent to

Hence, the graph is a horizontal hyperbola with center at (7, 3).

(x 7)2 (y + 3)2

= 1.
4
3

(c) Since the coefficients of x2 and y 2 are of opposite signs, the graph is a
hyperbola or a pair of intersecting lines. Completing the squares, we see
that the equation is equivalent to

(b) By inspection, the graph is a parabola. Completing the squares, we see


that the equation is equivalent to (x + 2)2 = 14(y + 4). Hence, the graph
has vertex at (2, 4) and is opening upward.

Hence, the graph is a circle with center (0.5, 1.5) and radius 2.

1. (a) Since the coefficients of x2 and y 2 are equal, the graph is a circle, a point,
or the empty set. Completing the squares, we see that the equation is
equivalent to
2 
2

1
3
x
+ y+
= 4.
2
2

C
O

giving x = 4 or x = 5. Thus, the solutions are (4, 5), (4, 5), (5, 4), and
(5, 4).

. Substituting this into the first


4. Using the second equation, we get y = 20
x
equation, we have
 2
20
x2 +
= 41 (x2 16)(x2 25) = 0,
x

3. Let (x, y) be the ordered pair that satisfies the conditions. The resulting
system of equations is

x2 = 2y 2 +
8
x2 + y 2 = 5

16

 
 



1 1
1 1
1 1
1 1
,
, , and , .
Solving yields
, ,
,
2 4
2 4
2 4
2 4

has a solution. Equating the first and second equation to each other yields
t 16.2 seconds (note that t > 0).

2. Note that this is equivalent to solving for t such that the system

d(t) = 150 + 75t 1.2t2


d(t) = 4t2

Topic Test 1 for Unit 1 (page 78)

All rights reserved. No part of this material may be reproduced or transmitted in any form or by any means electronic or mechanical including photocopying without written permission from the DepEd Central Office. First Edition, 2016.

F1 (1, 14), F2 (1, 6)

EP
E

1)

11
2

+ 1, 52 .

x2 + (y + 1)2 = 4(x2 6y + 9)

Manipulating this equation gives us

6. Let (x, y) be the coordinates of the point. This point satisfies


p
x2 + (y + 1)2 = 2|x 3|.

288

To determine if the cabinet can be pushed through the doorway, we determine


the height of the doorway when x = 0.25 (or 0.25). We solve for y from the
2
2
equation 0.25
+ (y2)
= 1. Solving for the y coordinate, we see that the height
0.52
0.32
is 2.2598 m. Hence, the cabinet cannot be pushed through the doorway.

5. Set up a coordinate system by making the center of the doors base the origin.
Then the ellipse has center (0, 2) with a = 1/2 and b = 0.3; then its is equation
2
x2
+ (y2)
= 1.
0.52
0.32

(3, 0),

When y = 0, the corresponding x values are 3 and 1. When y = 5/2,


the corresponding
x values
are 211 + 1. Therefore, the solutions are (1, 0),



4. Multiplying the first equation by 2, we get 2(x 1)2 + 2(y + 1)2 = 10. By
subtracting the second equation from this new equation, we get the equation
2(y + 1)2 + 8 = 10 y. This has solutions y = 0 and y = 5/2.

1. (a) By inspection, the graph is a parabola. Completing the squares, we see


that the equation is equivalent to (y 5)2 = 8(x 5). Hence, the graph
has vertex at (5, 5) and is opening to the left.
(b) Since the coefficients of x2 and y 2 are equal, the graph is a circle, a point,
or the empty set. Completing the squares, we see that the equation is
equivalent to (x + 5)2 + (y + 9)2 = 4. Hence, the graph is the empty
set.
(c) Since the coefficients of x2 and y 2 have the same sign and are unequal,
the graph is an ellipse, a point, or the empty set. Completing the squares,
we see that the equation is equivalent to

Topic Test 2 for Unit 1 (page 79)

3(x2 8x + 16) + (y + 1)2 = 36 48


3(x 4)2 + (y + 1)2 = 12
(x 4)2 (y + 1)2

= 1.
4
12
Therefore, the point traces a horizontal hyperbola with center at (4, 1).

F1 (8 2, 0),

F2 (8 2, 0)

(0, 0)

x2 y 2

= 1. This is a horizontal hyperbola.


64 64

asymptotes: y = x

vertices: V1 (8, 0), V2 (8, 0)

foci:

center:

2. (a) The equation is equivalent to

(y 4)2 (x 6)2

= 0.
11
17
Hence, the graph is a pair of intersecting lines given by the equations
y 4 = 11
(x 6).
17

PY

(x + 2)2 (y 1)2
+
= 1.
4
9
Hence, the graph is a vertical ellipse with center (2, 1).
(d) Since the coefficients of x2 and y 2 are of opposite signs, the graph is a
hyperbola or a pair of intersecting lines. Completing the squares, we see
that the equation is equivalent to

C
O

(b) The intersection (2, 5) of the two asymptotes is the center of the
hyperbola. Then the hyperbola is horizontal and a = 5. Using the slopes
. It follows that b = 12 and the
of the asymptotes, we have ab = 12
4
(x + 2)2 (y + 5)2
equation is

= 1.
25
144

3. (a) The parabola opens to the right and has focal distance c = 6. Its equation
is (y 3)2 = 24(x + 1).

asymptotes: y + 4 =

43 (x

(y + 4)2 (x 1)2

= 1. This is a vertical
64
36

vertices: F1 (1, 12), F2 (1, 6)

foci:

center: C(1, 4)

(b) The equation is equivalent to


hyperbola.

All rights reserved. No part of this material may be reproduced or transmitted in any form or by any means electronic or mechanical including photocopying without written permission from the DepEd Central Office. First Edition, 2016.

W1 (3, 0), W2 (3, 4)

289

5. Set up a coordinate system so that the opening of the hose (the parabolas
vertex) is at (0, 3) and that the water flows towards the positive x-axis. Then
the x-axis (y = 0) corresponds to the ground; it follows the parabola passes
through the point (2, 0). Hence, the equation of the parabola is x2 = 43 (y3).

When y = 0, the corresponding x


values are 5 and 1. When y = 9, the
corresponding x
valuesare 3 2 10. Therefore, the solutions are (5, 0),
(1, 0), 3 2 10, 9 .

4. Completing the squares, we see that the first equation is equivalent to 9(x +
3)2 = 4y 2 + 36. On the other hand, the second equation is equivalent to
9(x + 3)2 = 36y + 36. Subtracting the second equation from the first, we get
4y 2 36y = 0, which has solutions y = 0 and y = 9.

(x + 5)2 (y 3)2
+
= 1.
16
81

(b) Since the ellipse has vertical or horizontal major axis, the center is at
either (1, 12) or (5, 3). Since the major axis is longer than the minor
axis, the center must be at (5, 3). Then the ellipse is vertical with a = 9
and b = 4. Its equation is

3. (a) The parabola opens downward and has focal distance c = 5. Its equation
is (x 7)2 = 20(y + 7).

covertices:

F1 (3 3 5, 2), F2 (3 + 3 5, 2)

vertices: F1 (10, 2), F2 (4, 2)

foci:

center: C(3, 2)

PY

100[2a1 + (100 1)d]


= 2a1 + 99d
2
= (a1 + 2d) + (a1 + 5d) + (a1 + 98d)
= 33a1 + (2 + 5 + + 98)d
33[2(2) + (33 1)(3)]
= 33a1 +
d
2
= 33(a1 + 50d)

n (2(17) + (n 1)3)
= 30705 3n2 + 31n 61410 = 0
2
Using the quadratic formula and noting that n must be a whole number, we
have n = 138.
sn =

5016 = a3 + a6 + + a99

4. 15000 = a1 + a2 + a3 + + a100 =

3.

2. a4 = a1 r41 =

2
8
a6
4
2
; a6 = a1 r61 =

r = . We then
= r2 =
9
81
a
9
3
4
 3
3
2
2
compute for a1 by taking a4 = a1
= a1 = .
3
9
4

1. a3 = a1 +(3 1)d = 35; a10 = a1 + (10 1)d = 77 d = 6, a1 = 23 a5 = 47.

Supplementary Problems 2.1 (page 85)

Therefore, the point traces a vertical ellipse with center at (2, 4).

4
(x 2)2 + y 2 = (y 2 10y + 25)
9
9(x 2)2 + 5(y 2 + 8y) = 100
9(x 2)2 + 5(y + 4)2 = 100 + 80
(x 2)2 (y + 4)2
+
= 1.
20
36

Manipulating this equation gives us

p
2
(x 2)2 + y 2 = |y 5|.
3

6. Let (x, y) be the coordinates of the point. This point satisfies

If Nikko stands on a 1.5-ft stool and the vertex remains at (0, 3), the line
y = 1.5 will correspond to the ground.
will strike the
q Hence, the water

ground when y = 1.5. This gives x = 43 (1.5 3) = 6. Therefore, the

water will travel 6 2 ft further before striking the ground.

C
O

(x + 3)2 (y 2)2
+
= 1. This is a horizontal
49
4

EP
E

(b) The equation is equivalent to


ellipse.

All rights reserved. No part of this material may be reproduced or transmitted in any form or by any means electronic or mechanical including photocopying without written permission from the DepEd Central Office. First Edition, 2016.

EP
E

2a + 99d = 300
1
a + 50d = 152,

s4 =

290

4 (2a1 + (4 1)d)
= 80 2a1 + 3d = 40.
2
Since the sum of the first two numbers are one-third of the sum of the last two
1
numbers, we have (a1 + a2 ) = a3 + a4 4a1 + 14d = 0. Combining yields
3
d = 10, and thus a1 = 5, a2 = 15, a3 = 25, a4 = 35.

9. We have

yielding a2 = 1 or a2 = 2, respectively.

a3
a7
2 + 2d
2 + 6d
=

=
2d2 + 6d = 0 d = 3 or 0,
a2
a3
2+d
2 + 2d

(c)

(b)

1. (a)
i=3

i=1

i=2

5
X

x2 x4 x6 x8 x10
+
+
+
+
2
4
8
16
32

(1)i xi1 = x x2 + x3 x4

2i




3 3
10
i
=
+ ... +
= 26 3
2
2
2
2

5
X
x2i

10
X

Supplementary Problems 2.2 (page 95)

i=3

i(i 5) =

i=1

i=1

150
X

2=4

150(151)
+ 2(150) = 45, 600
2

a3n

(1)i+1 (x + 7 2i)i

(i2 5i) 1(1 5) 2(2 5)

i+

PY

i=1

150
X

120
X

(4i + 2) = 4
120
X

i=1

150
X

i=1

27
X

i=1

i=1

i=1

i2

i=1

130
X

9
130(131)[2(130) + 1]
+ 9(130) = 741 975
6

=4

i=1
130
X

120(121)(2(120) + 1)
=
+ 10 = 583 230
6
130
130
X
X
(c)
(2i 3)(2i + 3) =
(4i2 9)

(b)

3. (a)

10

1
22 32
102 X n2
+ 3 + 3 + ... + 3 =
3
3
4
5
11
(n + 2)3
i=1
(c) a3 + a6 + a9 + . . . + a81 =

(b)

2. (a) (x + 5) (x + 3)2 + (x + 1)3 (x 1)4 =

4
X

11. Note that this is a geometric series with common ratio 2n 1. Thus, the sum
will have a finite value if |2n 1| < 1 1 < 2n 1 < 1 0 < n < 1.
Thus, n (0, 1).

10. Note that c1 = a and c5 = b = ar51 = ar4 in our geometric sequence.


Furthermore, the product c2 c3 c4 = (ar) (ar2 ) (ar3 ) = a3 r6 = 27, yielding
2
ar2 = 3. However, ab = a (ar4 ) = (ar2 ) = 9.

C
O

8. We have a1 = 2, a2 = a1 + d, a3 = a1 + 2d, a7 = a1 + 6d. Since these three form


a geometric sequence,

0.123
41
=
.
0.123 =
1 0.001
333

which is an infinite geometric series with r = 0.001. Thus,

0.123123 . . . = 0.123 + 0.000123 + 0.000000123 +


= 0.123 + 0.123(0.001) + 0.123(0.001)2 + ,

7. Note that

c1
9/5
1/5
25
b1

= .
s=
1 rb 1 rc
1 3/5 1 2/5
6

where bn and cn are both geometric series with |r| < 1. Thus, the infinite sum
s is given by

6. If an is the nth term in the series, then


 n1
 n1
3n+1 2n1
9 3
1 3
an =
=

= bn cn ,
5n
5 5
5 2

a1
a1 (1 r3 )
5. We have s = 108 =
and s3 = 112 =
= 108 (1 r3 )
1r
1r
1
r = a1 = 144.
27

we get a = 48 and d = 4.

Solving the system

All rights reserved. No part of this material may be reproduced or transmitted in any form or by any means electronic or mechanical including photocopying without written permission from the DepEd Central Office. First Edition, 2016.

i=1

(i 1) i


=

i=1

EP
E

i=1

1
X

(i + 1) = (1 + 1) =

Assume:

Part 2.

i=1

k
X

(i + 1) =

k(k + 3)
.
2

The formula is true for n = 1.

2. Part 1.

1(1 + 3)
2

k+2 k+3
= 2 k + k+1
2
2
k+3
= 2 k+1 .
2

k+1
k
X
X
i
k+1
i
=
+ k+1
i
i
2
2
2
i=1
i=1

k+1
X
i
k+3
To show:
= 2 k+1 .
i
2
2
i=1

k
X
i
k+2
Assume:
=2 k .
i
2
2
i=1

Part 2.

The formula is true for n = 1.

1
1+2
=2 1
2
2

Supplementary Problems 2.3 (page 108)

1. Part 1.

291

i=1

k+1
X

i=1

k+1
X

k+1
X

i(i!) =
i=1

i=1

k
X

To show:

Assume:

Part 2.

i=1

2(1) 1 = 12

(2i 1) = (k + 1)2 .

(2i 1) = k 2 .
k
X

i=1

k
X

i(i!) + (k + 1)[(k + 1)!]


= (k + 1)! 1 + (k + 1)[(k + 1)!]
= (k + 2)(k + 1)! 1
= (k + 2)! 1.

PY

i=1

i(i!) = (k + 2)! 1.

i(i!) = (k + 1)! 1.

k+1
X

i=1

k
X

1(1!) = (1 + 1)! 1

The formula is true for n = 1.

4. Part 1.

To show:

Assume:

Part 2.

(i + 1) + [(k + 2)]

k(k + 3)
(k + 2)
2
2
k + 5k + 4
=
2
(k + 1)(k + 4)
=
.
2

i=1

k
X

(k + 1)(k + 4)
.
2

(i + 1) =

(i + 1) =

The formula is true for n = 1.

3. Part 1.

To show:

C
O

1
= (30 + 3 20) = 35 2
2

a1 + a2
s
a1 a2 X ai
+
=
6=
= .
b1
b2
b
b
+
b
t
i
1
2
i=1

i=1

200 s
i=
2

f (i)

200
X

i=1

50
X

(1 2i)

g(i) + 3

D
200
X

i=1

50
X

6. No. For example, for n = 2, we have

5. s =

200
X


50
X
g(i) + 3f (i)
1

=
2
2
i=1

4. We have

All rights reserved. No part of this material may be reproduced or transmitted in any form or by any means electronic or mechanical including photocopying without written permission from the DepEd Central Office. First Edition, 2016.

1
1
1
= =
2
2
2(1)

(1)1 12 =

i 2

(1)i i2 =

k+1
X

i=1

k
X

(1)k k(k + 1)
.
2

(1)1 1(1 + 1)
2

i=1

k+1
X
i=1

k
X

(1)i i2 + (1)k+1 (k + 1)2

(1)k k(k + 1)
=
+ (1)k+1 (k + 1)2
2

(1)i i2 =

(1)k+1 (k + 1)(k + 2)
To show:
(1) i =
.
2
i=1

Assume:

Part 2.

The formula is true for n = 1.

6. Part 1.







1
1
1
1
1
=P 1
2
k+1
k+1
k
1
=

2k k + 1
1
=
.
2(k + 1)


 


 

1
1
1
1
1
Assume: P = 1
.
1
1
1
=
2
3
k1
k
2k



 

1
1
1
1
To show: 1
.
1
1
=
2
k
k+1
2(k + 1)

Part 2.

(2i 1) + 2(k + 1) 1

EP
E

D
i=1
2

k
X

= k + 2k + 1
= (k + 1)2 .

(2i 1) =

The formula is true for n = 2.

5. Part 1.

i=1

k+1
X

111+2 + 122(1)+1 = 3059 = 133(23)


The number is divisible by 133 for n = 1.

8. Part 1.


43(k+1)+1 +23(k+1)+1 +1 = 6443k+1 +823k+1 +1 = 5643k+1 +8 43k+1 + 23k+1 + 1
7

Prove: 43(k+1)+1 + 23(k+1)+1 + 1 is divisible by 21.

Assume: 43k+1 + 23k+1 + 1 is divisible by 21.

Part 2.

PY

52(1)+1 21+2 + 31+2 22(1)+1 = 1216 = 19(64)

111 6 = 5


11k+1 6 = 11 11k 11 6 + 11 6 6 = 11 11k 6 + 60

Prove: 11k+1 6 is divisible by 5.

Assume: 11k 6 is divisible by 5.

Part 2.

The number is divisible by 5 for n = 1.

10. Part 1.

52(k+1)+1 2(k+1)+2 + 3(k+1)+2 22(k+1)+1 = 50 52k+1 2k+2 + 12 3k+2 22k+1 =


12 (52n+1 2n+2 + 3n+2 22n+1 ) + 38 52n+1 2n+2

Prove: 52(k+1)+1 2(k+1)+2 + 3(k+1)+2 22(k+1)+1 is divisible by 19.

Assume: 52k+1 2k+2 + 3k+2 22k+1 is divisible by 19.

Part 2.

The number is divisible by 19 for n = 1.

9. Part 1.

Prove: 11(k+1)+2 + 122(k+1)+1 is divisible by 133.



11(k+1)+2 + 122(k+1)+1 = 11 11k+2 + 122k+1 + 133 122k+1

Assume: 11k+2 + 122k+1 is divisible by 133.

Part 2.

C
O
292

43(1)+1 + 23(1)+1 + 1 = 273 = 7(39)


The number is divisible by 7 for n = 1.

7. Part 1.

(1)k (k + 1)
(k 2(k + 1))
2
(1)k+1 (k + 1)(k + 2)
.
=
2
=

All rights reserved. No part of this material may be reproduced or transmitted in any form or by any means electronic or mechanical including photocopying without written permission from the DepEd Central Office. First Edition, 2016.

EP
E

52 = 25 < 25 = 32

Assume:

Part 2

1
=1
1

k
X
1
1
2
3
i
k
i=1

13. Part 1. 1 2

Thus, (k + 1)2 < 2k+1 .

2(k + 1) + 1 = 2k + 1 + 2 < 2k + 2 < 2k + 2k = 2k+1 .

Prove: 2(k + 1) + 1 < 2k+1

Assume: 2k + 1 < 2k

Part 2

Part 1. 2(5) + 1 = 11 < 25 = 32

k+1
X
1
1
2
i3
k+1
i=1

Prove: ak+1 < ak+2

Assume: ak < ak+1

Part 2

293

< ak+1
< 2ak+1
p
< 2ak+1
< ak+2 .

PY

1. (a) (2x 3y)5 = 32x5 240x4 y + 720x3 y 2 1080x2 y 3 + 810xy 4 243y 5



4
x
2
8
1
8
32
(b)
2
= x11/2 + 16x8 + x3 + x2 x1/2
3
x
3
3
81
27
4

(c) (1 + x) = 4x3/2 + x2 + 6x + 4 x + 1
 
 
9 5 3 9 6
2. (a) 24
x,2
x
5
6
  8
1 10 p
(b) 6
2 2 q2
 
21 2 80 84
(c)
xy ,y
20
 
20 5/3
(d)
x
10
 
15 8k45 605k
(e) (k + 1)st term, 2153k
x
y
, so set 60 5k = 0 k = 12 and
k
 
15 51
get 221
x
12
 
13 4k26
(f) (k + 1)st term, (1)k 2k13
x
, so set 4k 26 = 2 k = 7 and
k
 
13 2
get 26
x
7

Supplementary Problems 2.4 (page 119)

ak
2ak

2ak
ak+1

14. Part 1. a41 = 4 < a42 = 8, hence a1 < a2 .

k+1
X
1
1
1
(k + 1)3 k
2 +
= 2
. Note that 0 < (k + 1)2
3
3
i
k
(k + 1)
(k + 1)3
i=1
(k + 1)3 k
(k + 1)2
1
.
(k + 1)2 < (k + 1)3 k, thus 2
<2
=2
3
(k + 1)
(k + 1)3
k+1

Prove:

C
O

To finish the proof, we must use induction again to prove that 2k + 1 < 2k for
k 5.

(k + 1)2 = k 2 + 2k + 1 < 2k + (2k + 1).

Prove: (k + 1)2 < 2k+1

Assume: k 2 < 2k

Part 2

The inequality is true for n = 5.

12. Part 1.

Assume:

10k 5
+ + 4k+2 is divisible by 3.
3
3
10k+1 5
Prove:
+ + 4k+3 is divisible by 3.
3
3
 k

5
10k+1 5 k+3
10k 5
10
5
+ +4
= 10
+ +44k+2 = 10
+ + 4k+2 64n+2 9
3
3
3
3
3
3
3

Part 2.

101 5
+ + 41+2 = 1029 = 3(343)
3
3
The number is divisible by 3 for n = 1.

11. Part 1.

All rights reserved. No part of this material may be reproduced or transmitted in any form or by any means electronic or mechanical including photocopying without written permission from the DepEd Central Office. First Edition, 2016.
19  
X
19
(3)k = (1 3)19 = (2)19 .
k
k=0

(b) O

Thus,

i=1

i=1

i2 + 3i + 2

50
X
2i3 + 9i2 + 13i + 6

i2 + 3i + 2

50
X
2i3 + 9i2 + 13i + 6

a2 = 10, and a3 = 15.

i=1

(2i + 3) = 2700.

6. a1 = 10, 000, r = 1.04, 60 20 = 40


1 (1.04)40
s40 = 10, 000
499, 675.83 pesos
1 (1.04)

294

(b) O

i=1

30
X

i=1

50
X

30

2i2 5i 3 = 79, 472


i2 2i + 1 X i 1
435
=
=
4
2
2
i=1

i=1

50
X

PY

(2i + 1)(i 3) =

(b) For n = k + 1: 3(n+1) + 7(n+1)1 + 8 = 7 (3n + 7n1 + 8) 4 3n 6 8,


where 4 3n is divisible by 12 for n 1, and 6 8 = 48 = 12(4).

1 + 4 + 7 + + (3(k + 1) 2)
k(3k 1)
3k 2 + 5k + 2
(k + 1)(3k + 2)
=
+ (3k + 1) =
=
2
2
2

5. (a) For n = k + 1:

 
 k  
 
8
1
8 244k
8
8k
4.
(x3 )
=
x
= 24 4k = 0 = k = 6 =
= 28
k
x
k
6

(b)

3. (a)

2. We have a1 + a2 = 2a1 + d = 9 and a1 + a2 + a3 = 3a1 + 2d = 9 yielding


a1 = 9, d = 9. Using sn = 126, we get n = 7.

1. (a) G, r = 4/5

Topic Test 2 for Unit 2 (page 122)


(c) A, d = 5/2

1
1
1
+
+ +
13 35
(2(k + 1) 1)(2(k + 1) + 1)
k
1
2k 2 + 3k + 1
k+1
=
+
=
=
2k + 1 (2k + 1)(2k + 3)
(2k + 1) (2k + 3)
2k + 3

5. For n = k + 1:

 k
 
1
8 162k

x
= 16 2k = 8 = k = 4
k
2
   4
1
8 8
35
=
x
= x8
4
2
8
 
 
28
28 19 27 19
2819
(b) k = 19 =
(n3 )
(3m)19 =
3 n m
19
19

4. (a)

C
O

a2 + 2
a3 + 1
=
. Thus, a1 = 5,
a2 + 2
a1 + 4

(c) O

(2i + 3)(i + 1)(i + 2)


= 2i + 3.
(i + 1)(i + 2)

50
X

2. First, a3 = a2 + 5 = a1 + 10. Also,

1. (a) G, r = 3/2

Topic Test 1 for Unit 2 (page 121)

5. In sigma notation, we have

3. We have

4 34!
3 34!
=
k = 14.
k!(34 k)!
(k + 1)!(34 (k + 1))!

Simplifying yields

34k k

 


34
34
3
= 434(k+1) 3k+1
k
k+1

EP
E

 
 
6
6 k
x , so set k = 3 and get (2)3
3
k
 
704k
k 30k 30
y
, so set 70 4k = 2 k = 18
(h) (k + 1)st term, (1) 2
k
 
30
and get 212
18
1
 
8 4 k
1
7
x 2 , so set 4 k = k = 1 and get
(i) (k + 1)st term, (3)k
k
2
2
 
8
3
1
1
 
 
6 3 k
1
3
6
(j) (k + 1)st term, 2k
x 2 , so set 3 k = k = 3 and get 23
k
2
2
3


4
X 10
3. Approximating yields (2.1)10
210k (0.1)k = 1667.904, which has
k
k=0
an approximate error of 0.08.
 
34
X
34 34k
434k 3k
4. Given (4x + 3)34 =
x
, we get k such that
k
k=0

(g) (k + 1)st term, (2)k

All rights reserved. No part of this material may be reproduced or transmitted in any form or by any means electronic or mechanical including photocopying without written permission from the DepEd Central Office. First Edition, 2016.

 2
9 7
189
=
cm2
2
6
16

s
6
= = 1 rad; 1 rad = 1
r
6
 


180

8
cm
3

1
1
14. 2r = r = = 2; A = r2 = r2 (2) = r2
2
2

13.

180

= 54

57.30

3
2

3
3
12. r = cm; s = cm; =  5  =
rad;
rad =
2
3
5
10
10
10
3

11. r = 6 in; s = 6 in; =

10. =

5
, A = 32 in2
4
v

u 2(32)
16
16 5
u
r = u  = =
in
5
t 5
5
4

8. = 108 =

3
1
3
135
; r = 15 cm; A = (15)2
=
cm2
5
2
5
2
s

5
2(15)
6
6
2

; A = 15 in ; r =
= =
in
9. = 150 =
5
6

7
9
1
; r = cm; A =
6
2
2

EP
E

107

9(2) = rad
6
6

7. =

6.

5. 2110 5(360 ) = 310

6
6
rev = rev
5
5


360
= 432
1 rev


11
11
2
22
2.
rev =
rev
=
rad
5
5
1 rev
5
 
  6
6
24
3. 216 = 216
=
rad; s = 4
=
cm
180
5
5
5


  11
6 11
33
4. 99 = 99
=
rad; s =
=
cm
180
20
20
10

1.

Supplementary Problems 3.1 (page 133)

1 2
(6)
2

3
10



54
cm2
5

Total length = 15 + 2(2) = (15 + 4) cm


 

1
1 2
(6)2 (3)(6 3) = (12 9 3) cm2
19. Asegment = Asector Atriangle =
2 3
2

18. The total length of the path that the point traces is the sum of the sides of
the equilateral triangle and the three arcs.

4. The tangent function is positive in QI and QIII. Cosine is negative in QII and
QIII. Therefore, the angle is in QIII.
!

 
5

5
3 1
5.
is in QII. The reference angle is , and therefore P
=
,
.
6
6
6
2 2

3. The secant function is positive in QI and QIV. The cotangent function is


negative in QII and QIV. Therefore, the angle is in QIV.

17
17
7
17
is coterminal with
+ 4 =
, and
terminates in QIII.
6
6
6
6

33
33

33
is coterminal with
8 = , and
terminates in QI.
4
4
4
4
2.

1.

Supplementary Problems 3.2 (page 143)

PY

20. Angle A measures 60 , thus triangle AOC is equilateral. Central angle BOC
measures 120 . The area of the shaded region is the sum of the areas of triangle
AOC and sector BOC.

 

4 2 1 2
Area =
(6) +
(6)2 = (9 3 + 12) cm2
3
2 3

C
O
295

3
10

Area of shaded region = 2area of sector AOE = 2

17. r = 6 cm; = 54 =

16. Area AECB = Area sector BDC Area sector ADE


 
 
1
5
1
5
25
= (8)2
(2)2
=
cm2
2
12
2
12
2

15. = 20 = rad; A = 800 cm2


9
v


u 2(800)
120
120
120   40
cm; s =
=
cm
r=u
t   = =

9
3

All rights reserved. No part of this material may be reproduced or transmitted in any form or by any means electronic or mechanical including photocopying without written permission from the DepEd Central Office. First Edition, 2016.

EP
E

34
sec =
5

11. csc = 4 and not in QIII = in QIV


4
csc =
= r = 4, y = 1
1
p

x = (4)2 (1)2 = 15, is in Quadrant IV, x = 15

x
15
r
4 15
y
1
cos = =
sec = =
sin = =
r
4
x
15
r
4

r
y
15
x
csc = = 4
tan = =
cot = = 15
y
x
15
y
p

2
2
12. x = 7, y = 1, r = (7) + (1) = 5 2

x
7 2
r
5 2
y
2
cos = =
sec = =
sin = =
r
10
x
7
r
10

r
y
1
x
csc = = 5 2
tan = =
cot = = 7
y
x
7
y
p

2
2
13. x = 2, y = 4 = r = (2) + (4) = 2 5

x
5
r
y
2 5
cos = =
sec = = 5
sin = =
r
5
x
r
5
r
y
x
1
csc = = 52
tan = = 2
cot = =
y
x
y
2

296

3
1
10; cos = , sin =
10
10

and

3
2

< < 2

11
6

in QII, so its reference angle is 4 .



 
2
35
cos
= cos
=
4
4
2

3
4

4
k

|a|
2

= 8

= 2 = k = 8

= 3 = |a| = 6; P =

1
4

b
3

= 4 = b = 23 ; |a| + b =

4. min value = 8; max value = 2



5. y = 3 sin 34 8
+ 2
5 = 13
9
3
2

3.

2.

1. P =

15
2

13
25
109

+ sec
+ sec
+ + sec
= 10 sec
= 10
6
6
6
6
6
Supplementary Problems 3.3 (page 170)

sec

20. All the given angles are coterminal with

PY

19. f (x) = sin 2x + cos 2x + sec 2x + csc 2x + tan 2x + cot 2x

 

7
2
2
f
=
+
+ 2 2 1 1 = 2
8
2
2

.
6

2 3
20 3
=
3
3

sin 30 + sin 60 + sin 90 + + sin 510 + sin 540

1
3
3 1
+1+
+ +0=2+ 3
= sin 390 + sin 420 + + sin 540 = +
2
2
2
2

18. 540 = 360 + 180 ; sin 30 + sin 60 + sin 90 + + sin 360 = 0

3
2

is coterminal with

=
17. cos = sin 2
3

35
4

15. x = 2, y = 6, r =

16.

(3)2 + (1)2 =

1
8
4
9

=
=
10 10
10
5

10
(2)2 + (6)2 = 2 10; sec = 10, csc =
3
10
80
sec2 csc2 = 10
=
9
9

cos2 sin2 =

14. x = 3, y = 1, r =

C
O

!
1 3
,
.
2 2



11

=
3

r
2
2 3
9. csc = 2, cos < 0; r = 2, y = 1, x = 3; sec = = =
x
3
3
p
4
10. cot = 3 , sin < 0 = x = 4, y = 3, r = (4)2 + (3)2 = 5
4
r
5
y
3
x
sec = =
sin = =
cos = =
r
5
x
4
r
5
r
5
y
3
x
4
csc = =
tan = =
cot = =
y
3
x
4
y
3

3
8. tan = , is in QIII
5

3 = 17 4 13
9
2
3

13
sec =
3

2
7. tan = , cos > 0
3

13

sec + tan
3
=
sec tan
13
+
3

11

6.
is in QI. The reference angle is , and therefore P
3
3

All rights reserved. No part of this material may be reproduced or transmitted in any form or by any means electronic or mechanical including photocopying without written permission from the DepEd Central Office. First Edition, 2016.

+4

+ 2k, k Z

12. Asymptotes: x =

,
2

domain =


x|x 6=


(d) P = , phase shift = 6 , domain = x|x 6=
(, 1] [3, )

(c) P = 4
, phase
3 
 shift =
, 23 12 ,


(b) P = , phase shift = 6 , domain = x|x 6=

k
,
2

2k
,
3

12

297


k Z , range =


k Z , range =

(c)

(d)

1
15. y = 8 cos 10
(t 10); at t = 10, y 4.32 (that is, the mass is located about
4.32 cm below the resting position)

14. d = 4 sin t

PY

1
1
sin x
=
=
1
cos x
1 cos x
1 cos x

sin x sin x
sin x

4. (1 cos2 A)(1 + cot2 A) = 1 cos2 A + cot2 A cos2 A cot2 A


 2 
 2 
cos A
cos A
2
= sin2 A +

cos
A
sin2 A
sin2 A
4
2
4
sin A + cos A cos A
=
sin2 A
(sin4 A cos4 A) + cos2 A
=
sin2 A
(sin2 A + cos2 A)(sin2 A cos2 A) + cos2 A
=
sin2 A
(sin2 A cos2 A) + cos2 A
sin2 A
=
=
=1
2
sin A
sin2 A

cos2 A
sin A + sin2 A cos2 A
sin A + 1
=
=
=1
1 + sin A
1 + sin A
1 + sin A

1
=
csc x cot x
3. sin A +

2.

sin x
tan x sin x
cos
x sin x = sin x 1 1 = sec x 1
1.
=
sin x
sin x
sin x
cos x sin x

Supplementary Problems 3.4 (page 179)

C
O


+ k, k Z , range = R

13. (a) P = 8, phase shift = 4 , domain = R, range = [3, 1]

+ 2k, k Z

2
3

11. Asymptotes: x =

+ 1 + 3 = 2 sin 2 x +

3
2

EP
E

92 3
3

10. y = 2 sin 2 x +

9. y = 3 sec 2(x ) 3

8. range = (, 3] [3, )

7
6

+3=


7. domain = R; range = 43 , 83

6. y = 2 cot 43

All rights reserved. No part of this material may be reproduced or transmitted in any form or by any means electronic or mechanical including photocopying without written permission from the DepEd Central Office. First Edition, 2016.

EP
E

1
1
cos x + sin x
+
csc x + sec x
x cos x = cos x + sin x
= sin x cos x = sin
cos x sin x
cot x + tan x
cos2 x + sin2 x
+
sin x cos x
sin x cos x

cos
cot
cot
a
=
=
=
sin
csc2
1 + cot2
1 + a2
r
1

1
sin
1 cos2
a2 1
sec2
10. sin cos =
=
=
=
sec
sec
sec
a2

13.

cos a
=
sec a + tan a

cos a
cos a
1 sin a
=
=
= 1 sin a
1
sin a
1 + sin a
1 + sin a
+
cos a cos a

1 + sin a 1 sin a
(1 + sin a)2 (1 sin a)2
12.

=
1 sin a 1 + sin a
(1 sin a)(1 + sin a)
(1 + 2 sin a + sin2 a) (1 2 sin a + sin2 a)
=
(1 sin2 a)
4 sin a
=
2
cos
 a 

sin a
1
=4
= 4 tan a sec a
cos a
cos a

1
1 + sin a
+1
csc a + 1
1 + sin a
sin
a
=
11.
= sin a =
1
1 sin a
csc a 1
1 sin a
1
sin a
sin a

9. sin cos = sin2

sin x + sin x cos x


sin x
sin x
tan x + sin x
cos x
=
= cos x
1
cos x
1 + cos x
csc x + cot x
+
sin x sin x
sin x
sin x(1 + cos x)
1 cos2 x
sin2 x
cos x
=
=
=
1 + cos x
cos x
cos x
sin x

1
1
8.
=
= cos2 x = 1 sin2 x
1 + tan2 x
sec2 x

7.

sin x cos x
sin2 x cos2 x

tan x cot x
cos
x
sin
x
x sin x = sin2 x cos2 x
=
6.
= cos
sin x cos x
tan x + cot x
sin2 x + cos2 x
+
cos x sin x
cos x sin x

5.

298

PY

tan2 + sec + 1
sec + sec2
=
sin cos
tan + cot
+
cos sin
(sec )(1 + sec )
=
sin2 + cos2
sin cos
= (sec )(1 + sec )(sin cos )
= (sin )(1 + sec ) = sin + tan
cos sin

cot sin sec


19.
= sin cos
1
1
sec csc

cos sin
cos2 sin2
=
cos sin = cos2 sin2
sin cos

18.

tan
=
17.
1 tan2

sin
sin
cos
cos

=
sin2
cos2 sin2
1
2
cos2
sin


sin
cos2
=
cos cos2 sin2
sin cos
sin cos
=
=
cos2 (1 cos2 )
2 cos2 1

sin3 cos3
(sin cos )(sin2 + sin cos + cos2 )
=
sin cos
sin cos
= sin2 + cos2 + sin cos = 1 + sin cos

1
1
1 + cos a + 1 cos a
+
=
1 cos a 1 + cos a
(1 cos a)(1 + cos a)
2
2
=
=
= 2 csc2 a
1 cos2 a
sin2 a

csc a + sec a tan a


csc a
sec a tan a
=
+
2a
csc2 a
csc2 a
csc

1
sin a
1
cos a cos a
=
+
1
csc a
sin2 a
sin a
= sin a +
(sin2 a)
cos2 a
= (sin a)(1 + tan2 a)


sin a
sec a = tan a sec a
= sin a sec2 a =
cos a

C
O

16.

15.

14.

All rights reserved. No part of this material may be reproduced or transmitted in any form or by any means electronic or mechanical including photocopying without written permission from the DepEd Central Office. First Edition, 2016.

sin
sin

11
6


+ k B

+ k B

+ k cos B cos

+ k cos B + sin

+ k cos B

+ k sin B

11
6


+ k sin B

+ k sin B

6. tan 1875 = tan 75 = tan(45 + 30 )


tan 45 + tan 30
=

1 tan
45 tan 30
3
1+

3 = 2 + 3
=
3
1
3

7. cot = 7, csc = 10, and and are acute

3 2
7 2
2
10
= cos =
, sin =
, sin =
, cos =
10
10
10
10

5. sin 105 cos 15 = sin(90 + 15 ) cos 15 = cos 15 cos 15 = 0




 1 + tan 5 tan 5
5
5 5
6
4
4. cot
= cot

=
5
5
12
6
4
tan
tan
4
6
3
1
(1)

= 2 + 3
= 3
3
1

= cot B

sin
cos

4
1
= and in QIV
3
2
 sin

+ k B =
3. tan A = tan
2
cos

2. sin = cos

EP
E

2
3
1. cos = sin
=
and in QIV
3
2

Supplementary Problems 3.5 (page 188)

20. tan2 sec2 sec2 + 1 = tan2 sec2 + (1 sec2 )


= tan2 sec2 tan2
= (tan2 )(sec2 1) = tan4
!
=

2 5
5

11. sin A =

PY

4
3
and A in QII = cos A =
5
5
3
4
cos B = and B in QIV = sin B =
5
5
     
4
4
3
3
7
(a) sin(A B) =

=
5
5
5
5
25
     
3
4
4
3
24
(b) cos(A B) =
+
=
5
5
5
5
25
7
7
(c) tan(A B) = 25 =
24
24

25
cos(A B) < 0 and sin(A B) > 0 = A B in QII

3
6
12. csc A = 3 and A in QI = sin A =
, cos A =
3
3

2
2
sec B = 2 and B in QIV = sin B =
, cos B =
2
2

! !
!
!
3
2
6
2
6+2 3
(a) sin(A B) =

=
3
2
3
2
6

9. 3 sin x = 2 = sin x =

2
3
sin(x ) + sin(x + )
= sin x cos cos x sin + sin x cos + cos x sin
 
2
4
= 2 sin x cos = 2
(1) =
3
3





10. cos x +
+ cos
x = sin(x) + sin x = sin x + sin x = 0
2
2

8. sin =

C
O
299

10
10

1
15
3
8
, sin = , and in QIV = cos = , cos =
17
2
17
2
cos( + ) = cos cos sin sin


  !  
1
8
8 + 15 3
15
3

=
=
17
2
2
17
34

cos( + ) = cos cos sin sin


!
!
!
7 2
2
3 2
=

10
10
10

All rights reserved. No part of this material may be reproduced or transmitted in any form or by any means electronic or mechanical including photocopying without written permission from the DepEd Central Office. First Edition, 2016.

B in QI

EP
E

tan x + tan y
1
=
1 tan x tan y
3
3 tan x + 3 tan y = 1 tan x tan y
 
 
1
1
3 tan x + 3
= 1 tan x
2
2
6 tan x + 3 = 2 tan x
1
tan x =
7

A + B = 45

1
tan A =
4
3
tan B =
5

1
1
16. Given: tan y = , tan(x + y) =
2
3

1 3
+
4 5  = 1
tan(A + B) =
1 3
1
4 5

csc B =

34
,
3

15. csc A = 17, A in QI

13. Given: sin =

23


+ tan
9
36 = tan + 23 = tan 3 = 1

23
9
36
4
1 tan tan
9
36
tan

20. cot 2 =

300

3. sin A =

12
5
12
, A is in QII = cos A = , tan A =
13
13
5
5
3
4
3
cos B = , B is in QIV = sin B = , cos B = , tan A =
3
5
5
4
(a) cos(A B) = cos A cos B + sin A sin B

    
5
4
12
3
56
=
+

=
13
5
13
5
65

PY

  5
1. r = 6 cm, = 37.5 = 37.5
=
rad
180
24
 
5
5
(a) s = 6
=
cm
24
4
 
1
5
15
(b) A = (6)2
cm2
=
2
24
4
p

2. x = 1, y = 2, r = (1)2 + (2)2 = 5

2 5 5
10 3 5
sin + cos + tan =
+
+2=
5
5
5

Topic Test 1 for Unit 3 (page 190)

1
1
=
tan 2
tan( + )
1
=
tan + tan
1 tan2
1 tan2
cot tan
cot2 1
=
=
=
2 tan
2
2 cot

19. sin 2 = sin( + ) = sin cos + cos sin = 2 sin cos

18. sin(A + B + C) = sin(A + (B + C))


= sin A cos(B + C) + cos A sin(B + C)
= (sin A)(cos B cos C sin B sin C)
+ (cos A)(sin B cos C + cos B sin C)
= sin A cos B cos C + cos A sin B cos C
+ cos A cos B sin C sin A sin B sin C

17.

C
O

4
5
and cos =
5
13
sin( + ) + sin( ) = sin cos + cos sin + sin cos cos sin
  
4
5
8
= 2 sin cos = 2
=
5
13
13

3
5
2
14. cos = , sin = , in QII = cos =
4
3
3
cos( + ) + cos( ) = cos cos sin sin + cos cos + sin sin
= 2 cos cos
! 

5
3
5
=2
=
3
4
2

! !
!
!

2 3 6
6
2
3
2
(b) cos(A B) =
+

=
3
2
3
2
6

6+2 3
=2 2+3
(c) tan(A B) =
2 3 6
cos(A B) > 0 and sin(A B) > 0 = A B in QI

All rights reserved. No part of this material may be reproduced or transmitted in any form or by any means electronic or mechanical including photocopying without written permission from the DepEd Central Office. First Edition, 2016.

EP
E

 

1. Asector = cm2 , = 30 = 30
= rad
3
180
6
 

1   2
=
r = r = 2 cm = 2
= cm
3
2 6
6
3

Topic Test 2 for Unit 3 (page 191)

301

(8)2 + (6)2 = 10

= sin(20 35) = sin(45 30) =

2 6
4

 
tan + tan
7
4
3 = 1 + 3 = 2 3
= tan
+
=
5. tan

12
4
3
1 3
1 tan tan
4
3

4. sin 160 cos 35 sin 70 cos 55


= sin 20 cos 35 cos 20 sin 35

8
3. sin A =
17




15 8
7
A + cos
A = cos A + sin A =
+
=
sin
2
2
17
17
17

(sin + cos ) = sin + 2 sin cos + cos2


  
6
8
1
= 1 + 2 sin cos = 1 + 2
=
10
10
25

2. x = 8, y = 6, r =

8.

7.

(sin x cos x)2 =

 2
1
1
= sin2 x 2 sin x cos x + cos2 x =
3
9
1
8
4
= 1 2 sin x cos x = = 2 sin x cos x = = sin x cos x =
9
9
9

sin x
= sin x cos x
sec x
1
sin x cos x =
3

tan2 x
tan2 x
tan x
=
=
= sin x cos x
3
tan x + tan x
tan x(1 + tan2 x)
sec2 x

PY

3
4
4
6. cos A = , A is in QIII = sin A = , tan A =
5
5
3
24
24
7
tan B = , B is in QIII = sin B = , cos B = ,
7
25
25
(a) sin(A + B) = sin A cos B + cos A sin B
 
  

4
7
3
24
4
=

=
5
25
5
25
5
 
4 24
1
1 tan A tan B
3
3 7
  =
(b) cot(A + B) =
=
4
24
tan A + tan B
4
+
3
7

C
O

7. Connect the three diagonals of the hexagon. In doing!this, the hexagon is

1 3
divided into 6 equilateral triangles. Hence, B
,
. Same coordinates
2 2
for C, E and F , except that they will just vary in signs depending on the
quadrant.


x 
1
2
8. y = 2 sin
+
1 = y = 2 sin
x+
1
2 3
2
3
2
, Amplitude = 2, Range = [3, 1]
P = 4, Phase Shift =
3

6. cos6 x + sin6 x = (cos2 x)3 + (sin2 x)3


= (cos2 x + sin2 x)(cos4 x cos2 x sin2 x + sin4 x)
= cos4 x cos2 x sin2 x + sin4 x
= cos4 x cos2 x(1 cos2 x) + (1 cos2 x)2
= cos4 x cos2 x + cos4 x + 1 2 cos2 x + cos4 x
= 3 cos4 x 3 cos2 x 1

tan 57 + tan 78
= tan(57 + 78) = tan 135 = 1
1 tan 57 tan 78
p

cos x tan x + sin x


sin x
= cos x +
= 2 cos x = 2 1 sin2 x = 2 1 a2
5.
tan x
tan x

4.

 
12
3
+
tan A + tan B
33
5
4
  =
(b) tan(A B) =
=
12
3
1 + tan A tan B
56
1+

5
4

All rights reserved. No part of this material may be reproduced or transmitted in any form or by any means electronic or mechanical including photocopying without written permission from the DepEd Central Office. First Edition, 2016.

7
9

4 2
9

(d) sec 2 =

(c) tan 2 =

1
8

= 2+4 2 and cos2



2+ 2
=
2

5
8

10. cos 8

9. sin2

5
8

2 2
4

and cos2 15 =

1 cos y
1 cos y sin y
1
sin y
sin y
=
sin y
sin y + 1 + cos y
+1
1 + cos y
1 + cos y
1 cos2 y sin y sin y cos y
=
sin2 y + sin y + sin y cos y
sin2 y sin y sin y cos y
=
sin2 y + sin y + sin y cos y
sin y 1 cos y
=
.
sin y + 1 + cos y

4 tan (1 tan2 )
1 tan2 + tan4

tan 12 y 1
11.
=
tan 12 y + 1

6. tan 4 =

5. tan x =

1 5
2

2 3
4

1/2

2/2

3/2

302

PY

6. (a) (1 4 2)/3

(b) 35/17

(c) /6

(b) /6

5. (a) 3/4

(d)

(d) does not exist

(f) does not exist

(c) (2 + 3)/4

(e) 5/6

(d) 2

(c) The xintercept of f is when x + 1 = 0, or equivalently x = 1; and the


yintercept of f is y = /4.

(b) The range of f is (/2, /2).

4. (a) The domain of f is R.

(b) 119/169

(c) 33/65

(d) (15 + 8 3)/34

(f) 0.2533

3. (a) 24/25

(e)

(c) 1

(d) /4

(g) 1/2

(e) does not exist

(f) 15/15

(b) /4

2. (a) /6

1. (a)
(b)
(c)
(d)

Supplementary Problems 3.7 (page 219)

1 + sec1 t
1
1 + cos t
csc t(sec t + 1)
=
= cot t(sec t + 1)
=
1 =
1
sin t
sec t
tan 2 t
csc t

2+ 3
13. (a) cos 105 =
(b) tan 22.5 = 2 1
2
12. cot 12 t =

C
O

2+ 3
4

(f) cot 2 =

9 2
8

7 2
8

7. cot 4 = 1/(tan 4) = 7/24

9
7

4 2
7

4. sin 4 = 2(sin cos3 sin3 cos ) 8. sin2 15 =

3. cos(2t) =

2. csc 46 1.3902 and sec 46 1.4395

(b) cos 2 =

1. (a) sin 2 =

(e) csc 2 =

EP
E

Supplementary Problems 3.6 (page 200)

18

x
1

+ 2 = tan
x
+2
3
3
6

P = 3, phase shift =
6

9. y = tan

All rights reserved. No part of this material may be reproduced or transmitted in any form or by any means electronic or mechanical including photocopying without written permission from the DepEd Central Office. First Edition, 2016.

EP
E

7. {/2, /6}

6. {(2k + 1)/2 | k Z}

5. {/8, 3/8, 9/8, 11/8}

3. (a) (3, 0)

(b) (3 2/2, 3 2/2)

2. Polar plane with plotted points:

(b) (0, 2), (0, /4)

1. (a) (13, 7/3), (13, 13/3)

(c) (0, 5/2)

(d) (4 3, 4)

(c) (15, 7/4), (15, 23/4)

Supplementary Problems 3.9 (page 252)

10. The ball is thrown by the pitcher at an angle of elevation of 62.73 .

9. The bullet should be fired with an angle of = 60 .

8. {0, /2, , 3/2} {/6, /3, 2/3, 5/6, 7/6, 4/3, 5/3, 11/6}

4. {/6, 5/6, 7/6, 11/6}

3. {2k/3 | k Z}

2.

1. {(2k + 1)/2 | k Z}

Supplementary Problems 3.8 (page 236)

7. Vertex angle should be /3.

303

PY

2
0

119
169
120
169

(f) cot(2) = 119


120

(e) csc(2) = 169


120

2 tan x
.
1 tan2 x

4 6 2

(b) tan 7.5 =


6 2


40
9
(b) cos sin1
=
41
41

(d) sec(2) =

169
119

(c) tan(2) = 120


119

3


103

4. (a) tan1 cot


=
6
3

3. (a) tan 15 = 2

2. Hint: Use the double-angle identity for tangent tan(2x) =

(b) sin(2) =

1. (a) cos(2) =

Topic Test 3 for Unit 3 (page 253)

6. = tan1 (800/600) 53.13 , that is, the angle of elevation of the helicopter
from the truck driver is 53.13

C
O

(b)

r 4

0 /6 /4 /3 /2 2/3 3/4 5/6 7/6 5/4 4/3 3/2

5. (a) r = 4 cos 2

(c) (8.24, tan1 ) (8.24, 72.34 )

(d) (14 2, /4)

(b) (25, tan1 (4/3)) (25, 53.13 )

4. (a) (41, tan1 (40/9) + ) (41, 102.68 )

All rights reserved. No part of this material may be reproduced or transmitted in any form or by any means electronic or mechanical including photocopying without written permission from the DepEd Central Office. First Edition, 2016.



3
0, ,
2 2

5/6

7/6

/2

3/2

304

(c) tan2 24 0.1983

(b) sin2 24 0.1655

2. (a) cos2 24 0.8346

336
527

(c) tan(2) =

(b) sin(2) = 336


625

4 6 2

(b) tan(/24) =
6 2

4
5/6

2 3

/3

/2

2/3

(c) (x, y) = (1, 0)

(f) (x, y) = ( 15
, 152 3 )
2

(e) (x, y) = (, 0)

8
(f) (r, ) = (17, tan1 ( 15
))

(c) (r, ) = (5, 2 )

9. (a) (x, y) = (2 3, 2)

(b) (x, y) = (50 2, 50 2)

(d) (x, y) = (5, 0)

(d) (r, ) = (5, )

(e) (r, ) = ( 2, 4 )

3/4

2 2

8. (a) (r, ) = (35, tan1 ( 43 ))

(b) (r, ) = (100 2, 5


)
4

7/6
5/4 4/3 3/2

2 3 2 2
1
2

/6
/4

2+ 3 2+ 2

PY

7. r = 2 + 2 cos



1
3
3 16 3
4. cos cos1 + cos1
=
7
5
35




5 7 13 15 5
3 4 5
, , ,
6.
, ,
,
, ,
5.
4 4 3 3
8 8 8
8 4 4

3. (a) tan(/12) = 2

C
O

527
1. (a) cos(2) = 625

( 9 2 3 , 92 )

(e) (x, y) = (2, 0)

(d) (x, y) = (5, 0)

(f) (r, ) = (13, tan1

12
)
5

(d) (r, ) = (7, 0)

(e) (r, ) = (2 2, 4 )

(f) (x, y) =

2/3
3/4

2 3 2 2

5 7 11
0, , 2, , , ,
6 6 6 6

5/4
4/3

2 + 2 2 + 3

Topic Test 4 for Unit 3 (page 254)

(c) (x, y) = (1, 0)

(b) (x, y) =

9. (a) (x, y) =

( 32 , 3 2 3 )

( 452 2 , 452 2 )

(c) (r, ) = (7, 2 )

EP
E

6.

/4
/3

2 2 2 3

8. (a) (r, ) = (70, tan1 43 )

(b) (r, ) = (100 2, 4 )

/6

7. r = 2 2 sin

5.

References

[1] R.N. Aufmann, V.C. Barker, and R.D. Nation, College Trigonometry, Houghton
Mifflin Company, 2008.
[2] E.A. Cabral, M.L.A.N. De Las Pe
nas, E.P. De Lara-Tuprio, F.F. Francisco,
I.J.L. Garces, R.M. Marcelo, and J.F. Sarmiento, Precalculus, Ateneo de
Manila University Press, 2010.

PY

[3] R. Larson, Precalculus with Limits, Brooks/Cole, Cengage Learning, 2014.


[4] L. Leithold, College Algebra and Trigonometry, Addison Wesley Longman
Inc., 1989, reprinted by Pearson Education Asia Pte. Ltd., 2002.

C
O

[5] M.L. Lial, J. Hornsby, and D.I. Schneider, College Algebra and Trigonometry
and Precalculus, Addison-Wesley Educational Publisher, Inc., 2001.
[6] J. Stewart, L. Redlin, and S. Watson, Precalculus: Mathematics for Calculus,
Brooks/Cole, Cengage Learning, 2012.
[7] M. Sullivan, Algebra & Trigonometry, Pearson Education, Inc., 2012.

EP
E

[8] C. Young, Algebra and Trigonometry, John Wiley & Sons, Inc., 2013.

305
All rights reserved. No part of this material may be reproduced or transmitted in any form or by any means electronic or mechanical including photocopying without written permission from the DepEd Central Office. First Edition, 2016.

Vous aimerez peut-être aussi